Download as pdf or txt
Download as pdf or txt
You are on page 1of 310

Serial Page

No. TOPICS No.

Contents 1. Basic Mathematics used in Physics

& Vectors
01

2. Units, Dimensions & Measurement 21

3. Kinematics 30

4. Laws of motion & Friction 50


PHYSICS

5. Work, Energy & Power 71


CLASS-XI

6. Circular Motion 89

7. Collisions & Centre of Mass 100

8. Rotational Motion 113

9. Gravitation 135

10. Properties of Matter & Fluid Mechanics 147

11. Thermal Physics 170

12. Oscillations 202

13. Wave Motion & Doppler's Effect 220

14. Solution 243


E
Pre-Medical : Physics
BASIC MATHEMATICS USED IN PHYSICS AND VECTORS

ALGEBRA
Quadratic Equation and Its Solution
An algebraic equation of second order (highest power of the variable is equal to 2) is called a quadratic equation.
Equation ax2 + bx + c = 0 is the general quadratic equation. The solution was given by Indian mathematician shri
Dhara Charya.
The general solution (Roots) of the above quadratic equation or value of variable x is

- b ± b2 - 4ac - b + b2 - 4ac - b - b2 - 4ac


x= Þ x1 = and x2 =
2a 2a 2a

x1 & x2 are the roots of the equation

-b coefficient of x
Sum of the roots x1+x2 = =–
a coefficient of x 2

c coefficient of constant term


Product of the roots x1 x2 = =
a coefficient of x 2
Arithmetic Progression (A.P.)
In this sequence every number is obtained by adding a certain constant value (Positive or Negative) in the preceding
number, called common difference.
In general, arithmetic progression can be written as, a,(a + d), (a + 2d) ........
where a is the value of 1st term, d is common difference
l value of nth term of A.P. is tn = a + (n – 1) d

n
l The sum of n terms of A.P. is Sn = {2a + (n - 1)d}
2
Geometric Progression
In this sequence, every number can be obtained by multiplying its preceding number by a fixed number called
common ratio
l It can be written as, a, ar, ar2 .........
where a is the value of 1st term, r is common ratio
l value of nth term of GP is tn = arn – 1

a(1 - r n )
l Sum of n terms is given by Sn =
1-r

a
l For sum of infinite terms (r < 1) S¥ =
1-r

Binomial Theorem
For x << 1
l (1 + x) n = 1 + nx where n may be a fraction
l (1 – x)n = 1 – nx
l (1 – x)–n = 1 + nx
l (1 + x)–n = 1 – nx

E 1
Pre-Medical : Physics
TRIGONOMETRY
Angle
One radian is the angle subtended at the centre of a circle by an arc of the circle,
r
whose length is equal to the radius of the circle. l =r
q = 1 rad
o
180
1 rad = » 57° 17' 45" » 57.3°
p

æ 1 ö'
o
p æ 1 ö
1° = radian , 1' = ç ÷ , 1'' = ç ÷
180 è 60 ø è 60 ø
The T-ratios of a few standard angles ranging from 00 to 1800

Angle (q) 0° 30° 45° 60° 90° 120° 135° 150° 180°

1 1 3 3 1 1
sin q 0 1 2 0
2 2 2 2 2

3 1 1 1 1 3
cos q 1 0 – - - –1
2 2 2 2 2 2

1 1
tan q 0 1 3 ¥ – 3 –1 - 0
3 3

FACTS :

sin (90° + q) = cos q sin (180° – q) = sin q sin (–q) = – sin q sin (90° – q) = cos q
cos (90° + q) = – sin q cos (180° – q) =– cos q cos (–q) = cos q cos (90° – q) = sin q
tan (90°+ q) =– cot q tan (180° – q) =– tan q tan (–q) = – tan q tan (90° –q) = cotq

sin (180° + q) = – sin q sin (270° – q) = – cos q sin (270° + q) = – cos q sin (360° – q) = – sin q
cos (180° + q) = – cos q cos(270° – q)= – sinq cos (270° + q) = sin q cos (360° – q) = cos q
tan (180° +q) = tan q tan (270° – q) = cot q tan (270° + q) = – cot q tan (360° – q) = – tan q

CALCULUS
Differential Calculus
d
l If c = constant, (c) = 0
dx
dy d du
l y = c u, where c is a constant and u is a function of x, = (cu) = c
dx dx dx
dy d du dv dw
l y = u ± v ± w, where u, v and w are function of x, ( u ± v ± w) =
dx ± dx ± dx
=
dx dx
dy d dv du
l y = u v where u and v are functions of x, = (uv) = u +v
dx dx dx dx
du dv
u dy d u
v -u
l y=
v
, where u and v are functions of x, = = dx 2 dx
dx dx v v
dy d n
l y = xn, n real number, = (x ) = nxn–1
dx dx

2 E
Pre-Medical : Physics
Formulae for differential coefficients of trigonometric, logarithmic and exponential functions

d d
1. (sin x) = cos x 5. (sec x) = sec x tan x
dx dx

d d
2. (cos x) = – sin x 6. (cosec x)=– cosec x cot x
dx dx

d d 1
3. (tan x) = sec2 x 7. (logex) =
dx dx x

d d
4. (cot x) = – cosec2 x 8. (ex) = ex
dx dx
Integration
Few basic formulae of integration
Following are a few basic formulae of integration :

x n +1
1. ò x n dx =
n +1
+ c, (not valid for n ¹ – 1) 2. ò sin xdx = - cos x + c

1
3. ò cos xdx = sin x + c 4. ò x
dx = loge x + c (c is constant of integration)

1 1
5. ò ex dx = ex + c 6. ò ax + b
dx = log e (ax + b) + c
a

l Definite Integrals
When a function is integrated between a lower limit and an upper limit, it is called a definite integral.
b b
ò a
f ' (x) dx = f (x ) a = f(b) – f(a)

Without any limit integration is called indefinite integral

Graphs : Some Standard graphs and their equations

y y A y
-c

Straight line c
x

x+
m

q
=m
=

O x
y

y = mx y
c Straight line
Straight line
c
q x q x
O O
y
y y

y = - mx + c x = ky2 x = -ky2
Straight line x
x
O O
Parabola Parabola
q
x
O

E 3
Pre-Medical : Physics

y y
y y=sinx y=cosx
y=1–e–kx
1
x x

l Maxima and Minima (Use of Differential Calculus)


For maxima or minima, consider a function y = f(x),

dy
Step - 1 Differentiate given function y with respect to x and equate it with zero. =0
dx

dy
Step - 2 Again differentiate with respect to x and verify whether it has a positive value or negative value.
dx

y
d 2y
At maxima <0 Maxima
dx 2

Minima
d 2y
At minima >0
dx 2
x1 x2 x
2
d y d æ dy ö
= ç ÷ is called second derivative
dx 2 dx è dx ø

Area of graph (Use of Integral Calculus)


Let y = f(x) is a function, then area of graph with x axis is given by

x2
A= òx1
y.dx

x1 x2 x
dx

4 E
Pre-Medical : Physics
VECTORS
Scalar Quantities : A physical quantity which can be described completely by its magnitude only and does not
require a direction is called a scalar.
Ex : Distance, mass, time, speed, density, volume, temperature, current etc.
Vector Quantities : A physical quantity which requires magnitude and a particular direction, when it is expressed.
i.e. Displacement, velocity, acceleration, force etc.
Representation of vector
A vector is represented by a line headed with an arrow.
® ®
Magnitude of A =|A|or A Length of the arrow

®
r ) A
Direction of A, A= r Tip of the arrow
|A|

TYPES OF VECTOR
l Parallel Vectors :- A
Those vectors which have same direction are called parallel vectors. B
Angle between two parallel vectors is always 0°

l Equal Vectors
Vectors which have equal magnitude and same direction are called
equal vectors. A
r r
A =B B

l Anti–parallel Vectors :
A
Those vectors which have opposite direction are called anti–parallel vector.
B
Angle between two anti–parallel vectors is always 180°

l Negative (or Opposite) Vectors


Vectors which have equal magnitude but opposite direction
are called negative vectors of each other. A B
uuur uuur
AB and BA are negative vectors A B
uuur uuur
AB = -BA

c
l Co-initial vector
b
Co-initial vectors are those vectors which have the same initial point.
r r r a
In figure a,b and c are co-initial vectors.

l Collinear Vectors :
The vectors lying in the same line are known as collinear vectors.
Angle between collinear vectors is either 0° or 180°.
Example.
(i) (q = 0° ) (ii) ( q = 0° )
(iii) (q = 180° ) (iv) (q = 180° )

E 5
Pre-Medical : Physics
l Coplanar Vectors
Vectors located in the same plane are called coplanar vectors.
Note :- Two vectors are always coplanar.

®
l Concurrent vectors a® b

Those vectors which pass through a common point are called concurrent vectors

r r
®
r c
In figure a,b and c are concurrent vectors.
l Null or Zero Vector
A vector having zero magnitude is called null vector.
r r r
Note : Sum of two vectors is always a vector so, ( A ) + ( - A ) = 0
r
0 is a zero vector or null vector..

l Unit Vector
A vector having unit magnitude is called unit vector. It is used to specify direction. A unit vector is represented
by  ( Read as a cap or A hat or A caret).
r
r A æ Vector ö
Unit vector in the direction of A is  = r ç unit vector= Magnitude of the vector ÷
A è ø
r r
ˆ = AA
A = AA ˆ

A unit vector is used to specify the direction of a vector.

Y
Base Vectors

In an XYZ co-ordinate frame there are three unit vectors $i , $j and ^


j ^
k$ , these are used to indicate X, Y and Z directions respectively.
i
These three unit vectors are mutually perpendicular to each other. ^ X
Z k

· Polar Vector
Vectors which have initial point or a point of application are called Q
n t
polar vectors.
e me
c
p la s
Ex. : Displacement, force etc. dis
P initial point
l Axial Vector
These vectors are used in rotational motion to define rotational effects.
Direction of these vectors is always along the axis of rotation in Axis
accordance with right hand screw rule or right hand thumb rule.
uur r ,
Ex. : Infinitesimal angular displacement ( dq ) , Angular velocity ( w )
r r and Torque r
Angular momentum ( J ) , Angular acceleration ( a ) ( t)
Addition of two vectors
Vector addition can be performed by using following methods
(i) Graphical methods (ii) Analytical methods
Addition of two vectors is quite different from simple algebraic sum of two numbers.

6 E
Pre-Medical : Physics
Triangle low of Addition of Two Vectors
If two vectors are represented by two sides of a triangle in same order then their sum or 'resultant vector' is given by
the third side of the triangle taken in opposite order of the first two vectors.

Parallelogram Law of Addition of Two Vectors (Analytical Method)


If two vectors are represented by two adjacent sides of a parallelogram D C
which are directed away from their common point then their sum
(i.e. resultant vector) is given by the diagonal of the parallelogram B A+
B
R=
passing away through that common point. b
a
® ® ® ® ® ® ® q
A B
AB + AD = AC = R or A + B = R A
From the geometry,

R= (A + Bcos q)2 + (B sin q)2 Þ R = A 2 + B2 + 2AB cos q

Bsin q
R
B
B sin q A sin q a q
tan a = and tan b =
A + B cos q B + A cos q A Bcos q

Note : l Resultant of two vectors will be maximum when they are parallel i.e. angle between them is zero.
® ®
Rmax =| A + B |max = A2 + B2 + 2AB cos 0 = (A + B)2 = A + B
l Resultant of two vectors will be minimum when they are antiparallel i.e. angle between them is 180°.
® ®
Rmin =| A + B | = A2 + B2 + 2ABcos180o = (A - B)2 = A ~ B (Bigger — smaller)

® ®
or Rmin =| A |~| B |

l Vector addition is commutative ® ® ® ®


A+ B = B+ A
® ® ® ® ® ®
l Vector addition is associative A + ( B + C ) = ( A + B ) + C

Subtraction of two vectors


® ® ® ® ® ®
Let A and B are two vectors. Their difference i.e. A – B can be treated as sum of the vector A and vector (– B ).

® ® ® ® B
A - B = A + (- B )

® ® ® ®
To subtract B from A , invert the direction of B and add to vector A . q
A
® ®
a
|A- B|= A 2 + B2 - 2AB cos q
r r –B
A-

Where q is angle between A and B


B

–B
B sin q
tana =
A - B cos q
E 7
Pre-Medical : Physics
® ®
FACTS : If two vectors have equal magnitude, i.e. |A|=|B|= a and q is the angle between them, then

® ® q
2 2 2
|A - B| = a + a - 2a cos q = 2a sin .
2

Resolution of vectors into rectangular components

® ® ® ® ®
OA =OB + OC or a = a x $i + a y $j or a = a cos a $i + a sin a $j

a cos a = projection of 'a' on x-axis or component of 'a' along x-axis.


a sin a = projection of 'a' on y-axis or component of 'a' along y-axis.

Similiary in three dimensions


Y
® ® ® ® C
OA = (O B+ OD) + EA ay

® ® ® ® A
OA = OB + OD + OC a
ax B
® O
a = a x $i + a y $j + a z k$
X

D az
E
2 2 2 Z
a = a x + a y + az

Directional Cosines
r
Let a makes angle : a with x axis, b with y axis and g with z axis
Y
a
cos a = x ax = a cos a A
a
ay b
cos b = ay = a cos b a ax
a
g X
a Z
cos g = z az = a cos g
a
cos a, cos b and cos g are directional cosines of the vector.
cos2 a + cos2 b + cos2 g = 1
sin2 a + sin2 b + sin2 g = 2
Multiplication and Division of a Vector by a Scalar
r
r r r r A C E
If there is a vector A and a scalar K and if B = KA and C =
K
where K > 0 then 45° B
D O
(a) In multiplication of a vector by a scalar the magnitude becomes
K times while the direction remains same.
r r A = 10N
So that angle between A and B is zero.
(b) In division of a vector by a scalar,the magnitude becomes (1/K) times
r r
and the direction remains same. So that angle between A and C is zero.
Note : A scalar or a vector, can not be divided by a vector.

8 E
Pre-Medical : Physics
Scalar Product or Dot Product of Two Vectors
® ®
Definition : There are two vectors A and B having angle q between them then their scalar product is written as
® ®
A . B = AB cos q
Properties :
l Dot product is always a scalar, which is positive if angle between the vectors is acute (i.e.q < 90°) and negative
if angle between them is obtuse (i.e. 90° < q £ 180°).
r r r r
l Dot product is commutative A × B = B × A
r r r r r r r
l Dot product is distributive A × ( B + C ) = A × B + A × C
r r
l According to definition A × B = AB cos q

r r B
æ A×Bö
-1 A.B = AB cos q
the angle between the vectors q = cos ç ÷
è AB ø q
X
A
l Scalar product of two vectors will be maximum when cos q = max =1, i.e. q =0°,
r r
i.e, vectors are parallel. ( A × B )max = AB
r r
l Scalar product of two vectors will be zero when cos q = 0, i.e. q =90° so ( A × B ) = 0 if the scalar product of
two nonzero vectors is zero then vectors are orthogonal or perpendicular to each other.

l In case of orthogonal unit vectors ˆi,jˆ and kˆ : ˆi.jˆ = ˆj.k=k.i


ˆ ˆ ˆ =1 × 1 × cos 90°=0

l The scalar product of a vector by itself is termed as self dot product and is given by
r r r r r
A.A = A A cos 0° = A 2 Þ A = A × A

l In case of unit vector nˆ : n.n


ˆ ˆ = 1 ´ 1 ´ cos 0° = 1 so n.n ˆ ˆ =1
ˆ ˆ = ˆi .iˆ = ˆj.jˆ = k.k
r r
l ˆ
In terms of components : A × B = (A xˆi + A y ˆj + A z k).(B ˆ = (A B + A B + A B )
ˆi + B ˆj + B k)
x y z x x y y z z

r r
Projection of A on B
®
r r r r A
r r æ A.B ö A.B r ˆ
(i) In scalar form : Projection of A on B=A cos q = A ç ÷= = A.B
è AB ø B q ®
B
Acosq
r r
r r æ A.B ö ˆ r
(ii) ˆ
In vector form : Projection of A on B= ( A cos q ) B = çç ÷÷ B = ( A.B
ˆ )B
ˆ
è B ø

VECTOR PRODUCT OR CROSS PRODUCT OF TWO VECTORS


® ®
Definition: A and B are two vectors, then their vector Ù
C=A×B n
® ® Ù
product A ´ B = AB sin q n n
ˆ

Right Hand Thumb Rule


A B
® ® q
Place the vector A and B tail to tail.

E 9
Pre-Medical : Physics
® ®
Now place stretched fingers and thumb of right hand perpendicular to the plane of A and B such that the fingers
® ®
are along the vector A . If the fingers are now closed through smaller angle so as to go towards B , then the thumb
® ® ®
gives the direction of A ´ B i. e. C
Properties
® ® ® ®
(i) It is non-commutative A ´ B ¹ B ´ A

(ii) The vector product is distributive when the order of the vectors is strictly maintained, i.e.

® ® ® ® ® ® ®
A ´( B + C ) = A´ B + A´ C

® ®
(iii) If A and B are parallel to each other then their cross product A ´ B = 0

(iv) i.e. vector product is maximum if the vectors are orthogonal (perpendicular).

® ®
|A ´ B|max = ABsin90o = AB

Q R
® ®
(v) \ Area of parallelogram OPRQ = 2[area of D OPQ] = | A ´ B|
B

base ´ height (OP) (NQ) 1 ® ® q


Area of DPOQ = = = | A ´ B|
2 2 2 O N P
A
Examples of Vector Product

® ® ® ® ® ®
(i) Torque t = r ´ F (ii) Angular momentum J = r ´ p

® ® ® ® ® ®
(iii) Velocity v = w ´ r (iv) Accelaration a = a ´ r

Illustration 2x2 + 5x – 12 = 0. Find the roots of x.

Solution By comparison with the standard quadratic equation a = 2, b = 5 and c = –12

-5 ± (5)2 - 4 ´ 2 ´ (-12) -5 ± 121 -5 ± 11 +6 -16 3


x= = = = , or x = , – 4
2´ 2 4 4 4 4 2
Illustration Find the value of
(i) cos (–60°) (ii) tan 210° (iii) sin 300° (iv) cos 120°

1
Solution (i) cos (–60°) = cos 60° =
2
1
(ii) tan 210° = tan (180° + 30°) = tan 30° =
3

3
(iii) sin 300° = sin (270° + 30°) = – cos 30° = –
2
1
(iv) cos 120° = cos (180° – 60°) = – cos60° = –
2
10 E
Pre-Medical : Physics
Illustration A circular arc is of length p cm. Find angle subtended by it at the centre in radian and degree.

6cm

q p cm

6cm

s p cm p 180° p 180°
Solution q == =rad=30° As 1 rad = So q = ´ = 30°
r 6 cm 6 p 6 p

Illustration Find the approximate values of (i) sin1° (ii) tan2° (iii) cos1°

æ p ö æ p ö p æ p ö æ pö p
Solution (i) sin1° = sin çè1° ´ ÷ = sin ç ÷ » (ii) tan2° = tan ç 2° ´ ÷ = tan ç ÷ »
180° ø è 180 ø 180 è 180° ø è 90 ø 90

(iii) cos1° » 1

dy
Illustration Find , when
dx
(i) y= x (ii) y = x5 + x4 + 7 (iii) y = x2 + 4x –1/2 – 3x–2

dy d d 1/ 2 1 1 -1/ 2 1
Solution (i) y= x = ( x) = (x ) = x1/ 2-1 = x =
dx dx dx 2 2 2 x
(ii) y = x5 + x4 + 7

dy d 5 d 5 d 4 d
= (x + x4 +7) = (x ) + (x ) + (7) = 5x4 + 4x3 + 0 = 5x4 + 4x3
dx dx dx dx dx
(iii) y = x2 + 4x–1/2 – 3x–2

dy d 2 d 2 d d
= (x + 4x–1/2 – 3x–2) = (x ) + (4x–1/2) – (3x–2)
dx dx dx dx dx
d 2 d –1/2 d –2 1
= (x ) + 4 (x ) –3 (x ) = 2x + 4 (- ) x–3/2 –3(–2)x–3 = 2x – 2x–3/2 + 6x–3
dx dx dx 2
Illustration The minimum value of y = 5x2 – 2x + 1 is

1 2 4 3
(1) (2) (3) (4)
5 5 5 5

dy 1 1 d2 y
Solution For maximum/minimum value = 0 Þ 5 ( 2x ) - 2 (1) + 0 = 0 Þ x = . Now at x= , =10
dx 5 5 dx2

1
which is positive so y has minimum value at x = .
5

2
æ1ö æ1ö 4
Therefore y min = 5 ç ÷ - 2 ç ÷ + 1 =
5
è ø 5
è ø 5

E 11
Pre-Medical : Physics
Illustration Integrate w.r.t. x. : (i) x11/2 (ii) x–7 (iii) xp/q

x11/2+1 2 13/2
Solution (i) ò x11/ 2dx =
11
+c=
13
x +c
+1
2

x -7+1 1 -6
(ii) ò x -7dx =
-7 + 1
+c = – x +c
6
p
+1
p
xq q
(iii) ò x dx = +c= x(p + q) / q + c
q
p p +q
+1
q

1
Illustration Evaluate ò ( x2 - cos x + ) dx
x
1
Solution = ò x 2dx - ò cos xdx + ò
x
dx

x 2+ 1 x3
= – sin x + loge x + c = – sin x + loge x + c
2+1 3
5
Illustration Evaluate the integral : ò1
x2 dx

5
é x3 ù 1 3
[ ] 1 124
5 5 1
ò
2
Solution x dx = ê ú = x = 3 3
= (125 – 1) =
1 3
ë û1 3 1 3 ((5) - (1) ) 3 3

Illustration Determine the average value of y = 2x +3 in the interval 0 £ x £ 1.


(1) 1 (2) 5 (3) 3 (4) 4

ò ydx1
é æ x2 ö ù
1

Solution y av = 0
= ò (2x + 3 ) dx = ê2 ç ÷ + 3x ú = 12 + 3 (1) - 02 - 3 (0 ) = 1 + 3 = 4
1-0 0 ë è 2ø û0

Illustration Two vectors having equal magnitude of 5 units, have an angle of 60° between them. Find the magnitude
of their resultant vector and its angle from one of the vectors.
Solution a = b = 5 unit and q = 60°

R = a2 + a2 + 2a.a cos 60o = a 3 = 5 3 unit

3
o
a sin 60
2 1 o
tan a = = = = tan30
a + a cos 60
o 3 3 \ a = 30o
2
Illustration Two forces each numerically equal to 10 dynes are acting as shown in the figure, then find resultant of
these two vectors.
ne

Solution The angle q between the two vectors is 120°, not 60°.
dy
10

\ R = (10)2 + (10)2 + 210 60° 10 dyne


( )(10)(cos 120o ) = 100 + 100 - 100 = 10 dyne

12 E
Pre-Medical : Physics
®
Illustration Find out the angle made by A = $i + $j + k$ vector from X, Y and Z axes respectively..

2 2 2
Solution Given Ax = Ay = Az = 1 so A = A x + A y + Az = 1 + 1 + 1 = 3

Ax = 1 a = cos -1
1
cos a = or
A 3 3

Ay 1 1
cos b = = or b = cos -1
A 3 3

Az = 1 g = cos -1
1
cos g = or
A 3 3
®
Illustration Determine that vector which when added to the resultant of P = 2$i + 7$j - 10k$ and
®
Q = $i + 2$j + 3k$ gives a unit vector along X-axis.
® ® ®
Solution Resultant R = P + Q = (2$i + 7$j - 10k$ ) + ($i + 2$j + 3k$ ) = 3$i + 9$j - 7k$
®
But R + required vector = $i
®
or required vector = $i – R = $i - (3$i + 9$j - 7k$ ) = -2$i - 9$j + 7k$
Illustration The sum of three vectors shown in figure, is zero.
C
®
(i) What is the magnitude of vector OB ? B
D 45°
® O
(ii) What is the magnitude of vector OC ?
Solution OC cos 45° = OB and OC sin 45° = OA = 10 A = 10N
Þ OC = 10 2N so OB = 10N

® ® ® ® ® ®
Illustration If | A + B| = | A - B|, then find the angle between A and B .

Solution Q ® ® ® ®
| A + B| = | A - B|

A2 + B2 + 2AB cos q = A2 + B2 - 2AB cos q


or A2 + B2 + 2AB cos q = A2 + B2 – 2AB cos q or cos q = 0 \ q = 90°
® ® ® ®
Illustration If A = 4$i + nj$ - 2k$ and B = 2$i + 3$j + k$ , then find the value of n so that A ^ B .

®®
Solution Dot product of two mutually perpendicular vectors is zero A . B = 0
ˆ ˆ + 3ˆj + k)
\ (4iˆ + njˆ - 2k).(2i ˆ = 0 Þ (4 ´ 2) + (n ´ 3) + (-2 ´ 1) = 0 Þ 3n = – 6 r n = – 2
® ® ® ® ® ® r r r
Illustration If F = (4 i - 10 j ) and r = (5 i - 3 j ) , then calculate torque t = r ´ F ( )
® ®
Solution Here r = 5 ˆi - 3 ˆj + 0kˆ and F = 4 ˆi - 10 ˆj + 0 kˆ

ˆi ˆj kˆ
® ® ® = 5 -3 0 ˆ -50 + 12) = -38kˆ
\ t= r´F = ˆi(0 - 0) - ˆj(0 - 0) + k(
4 -10 0

E 13
Pre-Medical : Physics
Illustration ˆ and (iˆ - ˆj + 2k)
Find a unit vector perpendicular to both the vectors (2iˆ + 3ˆj + k) ˆ .
® ®
Solution Let A = 2iˆ + 3jˆ + kˆ and B = ˆi - ˆj + 2kˆ

r ® ®
r A´ B
unit vector perpendicular to both A and B is n̂ = ® ®
| A´ B |

ˆi ˆj kˆ
® ®= 2 3 ˆ -2 - 3) = 7iˆ - 3jˆ - 5kˆ
1 = ˆi(6 + 1) - ˆj(4 - 1) + k(
A´ B
1 -1 2

\ ® ® 2 2 2 unit
| A ´ B | = 7 + ( -3) + (-5) = 83
1
\ n̂ = (7iˆ - 3ˆj - 5k)
83
Illustration Find the sum of given Arithmetic Progression 4 + 8 + 12 +.........+ 64
(1) 464 (2) 540 (3) 544 (4) 646

n 16
Solution Here a = 4, d = 4, n = 16 So, sum = [First term + last term] = [4 + 64] = 8(68) = 544
2 2
Illustration Find the sum of given series 1 + 2 + 4 + 8 +..............+ 256
(1) 510 (2) 511 (3) 512 (4) 513

(1)(1 - 29 )
Solution Here a = 1, r = 2, n = 9 (Q 256 = 28). So S9 = = 29 – 1 = 512 – 1 = 511
(1 - 2)

1 1 1
Illustration Find 1 + + + + ....upto ¥ .
2 4 8
(1) ¥ (2) 1 (3) 2 (4) 1.925

1 a 1
Solution Here, a = 1, r = So, S¥ = = =2
2 1-r 1
1-
2

Illustration A dog wants to catch a cat. The dog follows the path whose equation is y–x = 0 while the cat
follows the path whose equation is x2 + y2 = 8. The coordinates of possible points of catching the
cat are :
(1) (2, –2) (2) (2, 2) (3) (–2, 2) (4) (–2, –2)
Solution Let catching point be (x1, y1) then, y1–x1=0 and x12 + y12 = 8
Therefore, 2x12 = 8 Þ x12 =4 Þ x1=±2;
So possible points are (2, 2) and (–2, –2).

4
Illustration Calculate the unshaded area.
3
Solution Shaded area = Area of ellipse = pab
2
Here a = 6 – 4 = 2 and b = 4 – 3 = 1
1
r Area = p × 2 × 1 = 2p units
2 4 6

14 E
Pre-Medical : Physics
BASIC MATHEMATICS USED IN PHYSICS AND VECTORS EXERCISE

1 1 1 dy
1. Find sum of 1 + + + - - - -¥ 9. If y = x sin x then find
2 4 8 dx
1 3 2
(1) x sin x + cos x
(1) 2 (2) (3) (4) (2) sin x + x cos x
2 2 3
(3) x cos x
(4) cos x
1
2. Find ò 2x - 3 dx
ò cos
2
10. Find qdq
-2
(2x - 3) log e (2x - 3)
(1) +c (2) +c
2 2 q sin 2q sin 2q
(1) + +c (2) +c
(2x - 3)
2 4 4
(3) log e (2x - 3) + c (4) 2e
q cos 2q q sin 2q
(3) - +c (4) -
-2 4 4 2 4
æ hö
3. Find ç 1 + ÷ if h << R 11. The greatest value of the function –5 sinq + 12 cosq
è Rø
is
h2 2h 2h (1) 12 (2) 13 (3) 7 (4) 17
(1) 1 + - (2) 1 +
R2 R R
dy
2h h 12. Find if y = sin(4x – 3)
(3) 1 - (4) 1 - dx
R R
(1) - cos(4x - 3)
4. Find sum and product of roots of equation
2x2 – 3x + 5 = 0 cos(4x - 3)
(2)
4
3 5 2 5
(1) , (2) ,
2 2 3 2 sin(4x - 3)
(3)
5 2 4
(3) , (4) 3, 5
2 5 (4) 4 cos(4x - 3)

Find the value of ( 0.996 )


1/ 4
5. 13. Graph of equation 3x – 2y + 4 = 0 is–
y y
(1) 0.992 (2) 0.996
(3) 0.249 (4) 0.999
(1) (2)
6. ò sin 4x dx is– x
x
(1) -4 cos 4x + c (2) -4 sin 4x + c
y y
cos 4x tan 4x
(3) - +c (4) +c
4 4

7. Magnetic flux is given by f = 2t2 – 3t + 4, then find (3) x (4) x


æ df ö
induced emf at t = 2 çèQ e = - ÷ø
dt
(1) –5 unit (2) 5 unit 14. The graph between x and y is given in the figure then,
(3) 6 unit (4) 3 unit x and y are related as–
8. The value of sin 480° is– (1) y 2 µ x y
1 3 (2) y µ x 2
(1) (2)
2 2 (3) y µ x parabola

1 3 1
(3) - (4) - (4) y µ
2 2 x
x
E 15
Pre-Medical : Physics
15. In which part slope of graph is continuously increasing– 19. The slope of graph as shown in figure at points 1,
2 and 3 is m1, m2 and m3 respectively then
y
y
3
A E
2
D 1
B x
C x
o
(1) m1 > m2 > m3 (2) m1 < m2 < m3
(1) only in OA (2) only in AB (3) m1 = m2 = m3 (4) m1 = m3 > m2
20. Frequency f of a simple pendulum depends on its
(3) in BCD (4) only at E
length l and acceleration g due to gravity according
16. Velocity of a body is given by v = 4t2 – 2t then the
correct a-t graph is– 1 g
to the following equation f = . Graph
2p l
a a
between which of the following quantities is a straight
line ?
(1) t (2) t (1) f on the ordinate and l on the abscissa
(2) f on the ordinate and Öl on the abscissa
(3) f2 on the ordinate and l on the abscissa
(4) f2 on the ordinate and 1/l on the abscissa
a a 21. In the given figure, each box represents a
function machine. A function machine illustrates
what it does with the input.
(3) t (4) t
Input (x) Double the Square root Output (z)
input and of
add three the input
17. For the given graph the point of local minima is/are–
Which of the following statements is correct ?
(1) z=2x+3 (2) z=2(x+3)
y
(3) z = 2x + 3 (4) z = 2 ( x + 3)
x2 x3 x5
x 22. If velocity of a particle is given by v = (2t + 3) m/s,
x1 x4
then average velocity in interval 0 £ t £ 1s is :

7 9
(1) m/s (2) m/s
(1) only at x3 (2) only at x2 2 2
(3) at both x3 and x5 (4) only at x5 (3) 4 m/s (4) 5 m/s
23. The kinetic energy of a particle of mass m moving
18. The graph of equation y = 2x – 4x2 is–
y y
1
with speed v is given by K = mv 2 . If the kinetic
2
(1) x (2) x
energy of a particle moving along x-axis varies with
x as K(x) = 9 – x2. then the region in which particle
y y lies is :

x
(1) x ³ 9 (2) –3 £ x £ 3
(3) (4)
(3) 0 £ x £ 9 (4) –¥ < x < ¥
x

16 E
Pre-Medical : Physics
24. If 3 cosq + 4 sinq = A sin(q + a), then values of A and 31. A vector of length l is turned through the angle q
a are about its tail. What is the change in the position
vector of its head ?
-1 æ 3 ö -1 æ 3 ö (1) l cos (q/2) (2) 2l sin (q/2)
(1) 5, cos ç ÷ (2) 5, sin ç ÷
è5ø è5ø (3) 2l cos (q/2) (4) l sin(q/2)
32. The sum of magnitudes of two forces acting at a
æ4ö
-1 point is 16N. If the resultant force is 8N and its
(3) 7, sin ç ÷ (4) None of these
è5ø
direction is perpendicular to smaller force, then the
25. What is the maximum number of rectangular forces are :
components into which a vector can be split in (1) 6N & 10N (2) 8N & 8N
space ? (3) 4N & 12N (4) 2N & 14N
(1) 2 (2) 3 (3) 4 (4) Infinite r r r
33. Given that P = Q = R. If P + Q = R then the angle
26. Which of the following sets of concurrent forces may
r r r r r r
be in equilibrium ? between P & R is q1. If P+Q+R = 0 then the angle
(1) F1 = 3N, F2 = 5N, F3 = 1N r r
between P & R is q2. What is the relation between
(2) F1 = 3N, F2 = 5N, F3 = 9N
q1 and q2 ?
(3) F1 = 3N, F2 = 5N, F3 = 6N
(4) F1 = 3N, F2 = 5N, F3 =15N q2
(1) q1 = q2 (2) q1 =
27. How many minimum number of coplanar vectors 2
having different magnitudes can be added to give
(3) q1 = 2q2 (4) None of the above
zero resultant ?
(1) 2 (2) 3 (3) 4 (4) 5 34. Two billiard balls are rolling on a flat table. One has
28. What is the maximum number of components into the velocity components vx = 1 m/s, vy = 3 m/s
which a vector can be split ? and the other has components v'x=2 m/s, v'y=2m/s. If
(1) 2 (2) 3 (3) 4 (4) Infinite both the balls start moving from the same point, the
r angle between their paths is
29. In vector diagram shown in figure where ( R ) is the
(1) 30° (2) 45° (3) 60° (4) 15°
r r
resultant of vectors ( A ) and ( B ). 35. Two vectors, both equal in magnitude, have their
resultant equal in magnitude of the either vector. The
B angle between the vectors is :-
R
(1) 90° (2) 120° (3) 180° (4) zero
q
36. The length of a second's hand in a watch is 1 cm.
A
The change in velocity in 15 sec is :-
B p
If R = , then value of angle q is : (1) zero (2) cm / s
2 30 2
(1) 30o (2) 45o (3) 60o (4) 75o
r r p p
30. Six vectors, a through f have the magnitudes and (3) cm / s (4) 2cm / s
30 30
directions indicated in the figure. Which of the
following statements may be true ? r r
37. The angle between two vectors A and B is q.

b r q
a c Resultant of these vectors R makes an angle with
2
r
d e f A which of the following is true :-
r
r r r r r r r B
(1) b - er = f (2) b + cr = f (1) A = 2B (2) A =
2
r r r r
(3) d - cr = f (4) d - er = f r r
(3) |A||B|
= (4) AB = 1

E 17
Pre-Medical : Physics
r r r r r r
38. Two vectors A and B are such that A + B = C 45. The components of a = 2iˆ + 3jˆ along the direction
r r r r r
and |A|+|B||C|. = Then the vectors A and B of vector (iˆ + ˆj) is :-
are :-
(1) parallel (2) perpendicular 1ˆ ˆ
(1) ˆi + ˆj (2) (i + j)
(3) anti-parallel (4) null vectors 2
r r r r
39. Four forces P, 2P, 3P and 4P act along sides of a 5 ˆ ˆ 5 ˆ ˆ
(3) (i + j) (4) (i - j)
square taken in order. Their resultant is :- 2 2
r
(1) 2 2P (2) 2P 46. The angle that the vector A = 2$i + 3$j makes with

(3) P / 2 (4) zero y-axis is :


r r r r r r (1) tan-1 (3/2) (2) tan-1 (2/3)
40. If |v 1 + v 2 | = |v 1 - v 2 | and v 1 and v 2 are finite,
(3) sin-1 (2/3) (4) cos-1 (3/2)
then :-
r r 47. The direction cosines of a vector ˆi + ˆj + 2 kˆ are :-
(1) v 1 is parallel to v 2
r r
(2) v1 = v 2 1 1 1 1 1
r r (1) , ,1 (2) , ,
(3) |v 1 | = |v 2 | 2 2 2 2 2
r r
(4) v 1 and v 2 are mutually perpendicular

41. ˆ and
The vector that is added to (iˆ - 5ˆj + 2k) 1 1 1 1 1 1
(3) , , (4) , ,
2 2 2 2 2 2
ˆ to give a unit vector along the x-axis
(3iˆ + 6ˆj - 7k)
48. The magnitude of resultant of three unit vectors can
is:-
be :-
(1) 3iˆ + ˆj + 5kˆ (2) ˆi + 3jˆ + 5kˆ (1) zero (2) 1
(3) 3 (4) All of these
(3) -3iˆ - ˆj + 5kˆ (4) 3iˆ + ˆj - 5kˆ
49. The following sets of three vectors act on a body,
r r whose resultance can be zero. These are :-
42. The two vectors A and B that are parallel to each
other are :- (1) 10, 10, 10 (2) 10, 10, 20
r r (3) 10, 20, 20 (4) All of these
(1) A = 3iˆ + 6ˆj + 9kˆ B = ˆi + 2jˆ + 3kˆ
50. Which of the following vectors is/are not perpendicular
r r
(2) A = 3iˆ - 6jˆ + 9kˆ B = ˆi + 2jˆ + 3kˆ to the vector 5kˆ ?
r r
(3) A = 2iˆ + 6jˆ - 9kˆ B = ˆi + 2jˆ - 3kˆ (1) 4iˆ + 3jˆ (2) 6iˆ
r r (3) 7kˆ (4) 3iˆ + 4jˆ
(4) A = 2iˆ + 3jˆ + 3kˆ B = ˆi - 2jˆ - 3kˆ
r r 51. A bird moves with velocity 20 m/s in a direction making
43. If A = 3iˆ + 4ˆj and B = 7iˆ + 24jˆ , the vector having
an angle of 60° with the eastern line and 60° with
r r
the same magnitude as B and parallel to A is :- vertical upward. The velocity vector in rectangular
(1) 15i + 20j (2) 3i + 4j form.
(3) 7i + 24j (4) none of these (1) 10iˆ + 10jˆ + 10kˆ (2) 10iˆ + 10 2jˆ + 10kˆ
r
44. A vector A makes an angle 240° with the positive x-
(3) -20iˆ + 10 2jˆ + 20kˆ (4) 10 2iˆ + 10jˆ + kˆ
axis, its components along x-axis and y-axis are :-

A A A 52. ˆ is perpendicular to the


If a vector (2iˆ + 3jˆ + 8k)
(1) A and 3 (2) - and 3
2 2 2
ˆ , then the value of a is :
vector (4ˆj - 4iˆ + a k)
A A A
(3) - and - 3 (4) - and 3A
2 2 2 (1) –1 (2) 1/2 (3) –1/2 (4) 1
18 E
Pre-Medical : Physics
r r r r r r
53. The vector P = aiˆ + ajˆ + 3kˆ and Q = aiˆ - 2jˆ - kˆ are 59. The angle between (A ´ B) and (A + B) is :-

perpendicular to each other. The positive value of p p


a is : (1) 0 (2) (3) (4) p
4 2
(1) 3 (2) 2 (3) 1 (4) zero
r
60. Let A = ˆi A cos q + ˆj A sin q , be any vector. Another
54. If vectors
r r
r r wt wt vector B which is normal to A is :
A = cos wtiˆ + sin wtjˆ and B = cos ˆi - sin ˆj
2 2
are functions of time, then the value of t at which (1) ˆiB cos q + ˆjB sin q (2) ˆiB sin q + ˆjB cos q
they are orthogonal to each other is :
(3) ˆiB sinq - ˆjB cosq (4) ˆiA cosq - ˆjA sinq
p
(1) t = 0 (2) t = r r r r r r
w
61. If A ´ B = 3 A . B , then the value of A - B
p p
(3) t = (4) t = is :
2w 3w

55. The vector sum of two forces is perpendicular to F


(1) G A
ABI 1/ 2

their vector difference. In that case, the force :


H
2
+ B2 + J
3K
(1) Are equal to each other.
(2) Are equal to each other in magnitude. (2) (A2 + B2 – AB)1/2
(3) Are not equal to each other in magnitude.
1/2
(3) (A2 + B2 + 3 AB)
(4) Cannot be predicted.

56. If ˆi, ˆj and k̂ represent unit vectors along the x, y (4) (A2+B2+AB)1/2

and z-axes respectively, then the angle q between 62. Area of a parallelogram, whose diagonals are

the vectors ( ˆi + ˆj + kˆ ) and ( ˆi + ˆj ) is equal to : 3iˆ + ˆj - 2kˆ and ˆi - 3jˆ + 4kˆ will be :

-1 æ 1 ö æ 2ö (1) 14 unit (2) 5 3 unit


(1) sin ç
-1
÷ (2) sin ç ÷
è 3ø è 3ø
(3) 10 3 unit (4) 20 3 unit
r r r
æ 1 ö
-1 63. The resultant of A and B is perpendicular to A .
(3) cos ç ÷ (4) 90° r r
è 3ø
What is the angle between A and B ?
r r
57. A vector A points vertically upward and B points
-1 æ A ö -1 æ Aö
r r (1) cos ç ÷ (2) cos ç - ÷
towards north. The vector product A ´ B is èBø è Bø
(1) zero
(2) along west -1 æ A ö -1 æ Aö
(3) sin ç ÷ (4) sin ç - ÷
(3) along east èBø è Bø
(4) vertically downward r
r r r r r r 64. What is the angle between A and the resultant of
58. If A ´ B = 0 and B ´ C = 0 , then the angle between r r
r r (A+B )
ˆ and A - B
(ˆ ?
)
A and C may be :
-1 æ A ö
p (1) 0° (2) tan ç ÷
(1) zero (2) èBø
4

p -1 æ B ö -1 æ A - B ö
(3) (4) None (3) tan ç ÷ (4) tan ç ÷
2 èAø èA+Bø

E 19
Pre-Medical : Physics
65. What is the projection of 3iˆ + 4kˆ on the y-axis ?
(1) 3 (2) 4
(3) 5 (4) zero
r r r r
66. Given |A1 |= 2,|A 2 |= 3 and |A1 + A2 |= 3 . Find
r r r r
the value or |(A1 + 2A 2 ) × (3A1 - 4A 2 )| :-
(1) 64 (2) 60 (3) 62 (4) 61

ANSWER KEY
Que. 1 2 3 4 5 6 7 8 9 10 11 12 13 14 15
Ans. 1 2 3 1 4 3 1 2 2 1 2 4 1 2 3
Que. 16 17 18 19 20 21 22 23 24 25 26 27 28 29 30
Ans. 1 3 2 2 4 3 3 2 2 2 3 2 4 2 3
Que. 31 32 33 34 35 36 37 38 39 40 41 42 43 44 45
Ans. 2 1 2 4 2 4 3 1 1 4 3 1 1 3 3
Que. 46 47 48 49 50 51 52 53 54 55 56 57 58 59 60
Ans. 2 3 4 4 3 2 3 1 4 2 1 2 1 3 3
Que. 61 62 63 64 65 66
Ans. 2 2 2 1 4 1

20 E
Pre-Medical : Physics
UNITS, DIMENSIONS AND MEASUREMENTS
The system of Internationally accepted units contain three classes of units :
(i) Fundamental units (ii) Derived units (iii) Supplementary units
(1) Fundamental units
Physical Quantity Units Symbol Dimensions
Length metre m L
Mass kilogram kg M
Time Second s T
Electric Current Ampere A A
Temperature Kelvin K K or q
Luminous Intensity candela cd Cd
Amount of Substance mole mol mol
(2) Derived units : The units of derived quantities or the units that can be expressed in terms of the base units are
called derived units.
unit of distance metre
e.g. unit of speed = = = ms -1
unit of time second
Some derived units are named in honour of great scientists. e.g. unit of force - newton (N), unit of frequency - hertz
(Hz), etc.
(3) Supplementary Units : There are two supplementary units -
Physical Quantity Units Dimensions
Plane angle Radian Dimensionless
Solid angle Steradian Dimensionless
(4) Dimensions : Dimensions of a physical quantity are the powers (or exponents) to which the base quantities are
raised to represent that quantity.
SETS OF QUANTITIES HAVING SAME DIMENSIONS
S .No. Quantities D imensions
S tra in, refra ct ive in dex, r elat ive d ensity, angle , solid a ng le,
p ha se , d ista n ce g r a d i e nt , re la ti ve pe rm ea b ilit y, r e la ti ve
1. [M 0 L 0 T 0 ]
permittivity, angle o f contact, Reynold s nu mber, coefficient of
friction, m echanical equivalent o f heat, electric susceptibility, et c.
2. Mass and inertia [M 1 L 0 T 0 ]

3. Momentu m and im pulse. [M 1 L 1 T – 1 ]


4. Thr ust, force, weight, tension, energy gradient. [M 1 L 1 T – 2 ]
Pressure, stress, Young's modulus, b ulk mo dulus, shear mod ulus,
5. [M 1 L – 1 T – 2 ]
m odu lu s of rigidity, energy density.

6. An gular momentu m and Planck's con stant (h ). [ M 1 L 2 T –1 ]


7. Acceleration, g and gravi tational field intensity. [ M 0 L 1 T –2 ]
Surface tension, free surface energy (energy per unit area),
8. [ M 1 L 0 T –2 ]
force gradien t, spring constant.

9. Latent heat capacity an d gravitational potential. [ M 0 L 2 T –2 ]

1 0. Therm al c apacity, B oltzm ann constant, entr opy. [ M L 2 T –2 K – 1 ]


Work, tor qu e, internal energ y, potential energy , kinetic energy,
V2
m om ent of force, (q 2 /C), (L I2 ), (qV ), (V 2 C), (I 2 R t), t , (V It),
1 1. R [M 1 L 2 T – 2 ]

(PV ), (RT), (mL ), (m c DT)


Frequency, an gular frequ ency, ang ular velo city, velocity
1 2. R 1 1 [M 0 L 0 T – 1 ]
gradient, radioactivity , ,
L RC L C

ælö æmö æLö


12 12

1 3. ç ÷ ,ç ÷ , ç ÷ , (RC), ( LC ) , time [ M 0 L0 T 1 ]
ègø èk ø èRø

1 4. (V I), (I 2 R), (V2 /R ), P ower [ M L 2 T– 3 ]

E 21
Pre-Medical : Physics
USES OF DIMENSIONAL EQUATIONS
(i) Conversion of one system of units into another.
(ii) To check the dimensional correctness of a given physical relation.
(iii) Deriving a correct relationship between different physical quantities.

SIGNIFICANT FIGURES OR DIGITS


The significant figures (SF) in a measurement are the figures or digits that are known with certainity plus one that is
uncertain. Significant figures in a measured value of a physical quantity tell the number of digits in which we have
confidence. Larger the number of significant figures obtained in a measurement, greater is its accuracy and vice
versa.
Rules to find out the number of significant figures
Rule I : All the non-zero digits are significant e.g. 1984 has 4 SF.
Rule II : All the zeros between two non-zero digits are significant. e.g. 10806 has 5 SF.
Rule III : All the zeros to the left of first non-zero digit are not significant. e.g.00108 has 3 SF.
Rule IV : If the number is less than 1, zeros on the right of the decimal point but to the left of the first non-
zero digit are not significant. e.g. 0.002308 has 4 SF.
Rule V : The trailing zeroes (zeroes to the right of the last non-zero digit) in a number with a
decimal point are significant. e.g. 01.080 has 4 SF.
Rule VI : The trailing zeroes in a number without a decimal point are not significant e.g. 010100
has 3 SF. But if the number comes from some actual measurement then the trailing
zeroes become significant. e.g. m = 100 kg has 3 SF.
Rule VII : When the number is expressed in exponential form, the exponential term does not
affect the number of S.F. For example in x = 12.3 = 1.23 × 101 = .123 × 102
= 0.0123 × 103 = 123 × 10 – 1 each term has 3 SF only.
Rules for arithmetical operations with significant figures
Rule I : In addition or subtraction the number of decimal places in the result should be equal to the number of
decimal places of that term in the operation which contain lesser number of decimal places. e.g.
12.587 – 12.5 = 0.087 = 0.1 (Q second term contain lesser i.e. one decimal place)
Rule II : In multiplication or division, the number of SF in the product or quotient is same as the smallest
number of SF in any of the factors. e.g. 5.0 × 0.125 = 0.625 = 0.62
Rounding Off To represent the result of any computation containing more than one uncertain digit, it is
rounded off to appropriate number of significant figures.
Rules for rounding off the numbers :
Rule I : If the digit to be rounded off is more than 5, then the preceding digit is increased by one.
e.g. 6.87» 6.9
Rule II : If the digit to be rounded off is less than 5, then the preceding digit is unaffected and is left
unchanged. e.g. 3.94 » 3.9
Rule III : If the digit to be rounded off is 5 then the preceding digit is increased by one if it is odd and is
left unchanged if it is even. e.g. 14.35 » 14.4 and 14.45 » 14.4
Order of Magnitude
Order of magnitude of a quantity is the power of 10 required to represent that quantity. This power is determined
after rounding off the value of the quantity properly. For rounding off, the last digit is simply ignored if it is less than
5 and, is increased by one if it is 5 or more than 5.

22 E
Pre-Medical : Physics
ERRORS IN MEASUREMENT
Definition
The difference between the true value and the measured value of a quantity is known as the error of measurement.
Error = True value ~ Measured value
Classification of Errors

Systematic or Controllable Errors Random Errors


· Due to the known causes like imperfect · Due to unknown causes.
design of instruments, imperfect technique
and carelessness in taking observation.
· It can be either positive or negative. · Can vary in magnitude and sign.
· It can be elliminated · Can be minimised by taking several
observation.
1
Note :- If the number of observations is made n times then the random error reduces to ( ) times.
n
Accuracy and Precision : The accuracy of a measurement is a measure of how closed quantity. Precision tells
us to what resolution or limit the quantity is measured. Accuracy can be checked by relative error while precision can
be checked by least count.

Calculation of Errors in Mathematical Operations


Rule I : In addition / subtraction process, If X = A + B or X = A – B
the maximum absolute error in addition / subtraction process DX = DA + DB

DX
Maximum percentage error = ´ 100
X
DX
The result will be written as (X ± DX) (in terms of absolute error) or (X ± ´ 100 %) (in terms of percentage error)
X
Rule II : In multiplication / division process, then the maximum fractional or relative error in the product or division

DX é DA DB ù
If X=A× B or X = A/B then = ±ê +
X ë A B úû

Rule III : The maximum fractional error in a quantity raised to a power (n) is n times the fractional error in the
quantity itself, i.e.

DX DA
If X = An then = n( )
X A

DX é æ DA ö æ DBö æ D Cö ù
If X = ApBq Cr then = pç ÷ + q çè ÷ + r çè ÷
X êë è A ø Bø C ø úû

A p Bq DX é æ DA ö æ DBö æ D Cö ù
If X= then = pç ÷ + q çè ÷ + r çè ÷
Cr X êë è A ø Bø C ø úû

E 23
Pre-Medical : Physics

Vernier Callipers Least count = 1MSD – 1 VSD (MSD ® main scale division, VSD ® Vernier scale division)

0 1 2 3 4 5 6 14 15

Ex. A vernier scale has 10 parts, which are equal to 9 parts of main scale having each path equal to 1 mm then
9
least count = 1 mm – mm = 0.1 mm [Q 9 MSD = 10 VSD]
10
Screw Gauge
Spindle Circular (Head) scale

Ratchet
pitch

0
Least count =

5
total no. of divisions

10
Linear (Pitch)
on circular scale Scale Thimble
Sleeve

Ex. The distance moved by spindle of a screw gauge for each turn of head is 1mm. The edge of the humble
1mm
is provided with a angular scale carrying 100 equal divisions. The least count = = 0.01 mm
100
Zero Error
Main scale Main scale Main scale
0 1 0 1 0 1

0 5 10 0 5 10 0 5 10
Vernier scale Vernier scale Vernier scale
without zero error with positive zero error with negative zero error
(i) (ii)
The zero error is always subtracted from the reading to get the corrected value.
If the zero error is positive, its value is calculated as we take any normal reading.
Negative zero error = – [Total no. of vsd – vsd coinciding] ×L.C.
Zero Error in Screw Gauge
If there is no object between the jaws (i.e. jaws are in contact), the screwgauge should give zero reading. But
due to extra material on jaws, even if there is no object, it gives some excess reading. This excess reading is
called Zero error.
Negative Zero Error Positive Zero Error
(3 division error) i.e., –0.003 cm (2 division error) i.e., +0.002 cm
Circular scale
Circular scale

10
zero of the circular 15
5 scale is above the 10
0 0 zero of main scale Zero of the circular
0 5
95 scale is below the
0
Main scale 90 zero of main scale
reference line Main scale 95
85 reference line 90

24 E
Pre-Medical : Physics
Illustration The length, breadth and thickness of a metal sheet are 4.234 m, 1.005 m and 2.01 cm respectively.
Give the area and volume of the sheet to correct number of significant figures.
Solution length (l) = 4.234 m breadth (b) = 1.005 m
thickness (t) = 2.01 cm = 2.01 × 10–2 m
Therefore area of the sheet = 2 ( l × b + b × t + t × l )
= 2 ( 4.234 × 1.005 + 1.005 × 0.0201 + 0.0201 × 4.234) m2 = 8.7209478 m2
Since area can contain a maximum of 3 SF (Rule II of article 4.2) therefore, rounding off,
we get Area = 8.72 m2
Like wise volume = l × b × t = 4.234 × 1.005 × 0.0201 m3 = 0.0855289 m3
Since volume can contain 3 SF, therefore, rounding off, we get
Volume = 0.0855 m3
Illustration The inital and final temperatures of water as recorded by an observer are (40.6 ± 0.2)°C and
(78.3 ± 0.3)°C. Calculate the rise in temperature with proper error limits.
Solution Given q1 = (40.6 ± 0.2)°C and q2 = (78.3 ± 0.3)°C
Rise in temp. q = q2 – q1 = 78.3 – 40.6 = 37.7°C.
Dq = ± (Dq1 + Dq2) = ± (0.2 + 0.3) = ± 0.5°C
\ rise in temperature = (37.7 ± 0.5)°C
Illustration The length and breadth of a rectangle are (5 ± 0.1) cm and (3 ± 0.2) cm. Calculate area of
the rectangle with error limits.
Solution Given l = (5 ± 0.1) cm and b = (3 ± 0.2) cm

DA Dl Db 0.1 0.2
Area A = l × b = 5 × 3 = 15 cm2 Þ = ±( + ) = ±( + ) = ± (0.02 + 0.06)
A l b 5 3
or DA = ±0.08 × 15 = ±1.2 \ Area = (15 ± 1.2) sq. cm2
Illustration A body travels uniformly a distance (16 ± 0.2) m in a time (4.0 ± 0.2) s. Calculate its velocity with
error limits. What is the percentage error in velocity ?
Solution Given distance s = (16 ± 0.2) m and time t = (4.0 ± 0.2) s
s 16 Dv Ds Dt 0.2 0.2
velocity v = = = 4 ms–1 Þ = ±( + ) = ±( + ) =±(0.0125 + 0.05)
t 4 v s t 16 4
or Dv = 0.0625 × v = ± 0.0625× 4 = ± 0.25 \ v = (4 ± 0.25) ms–1

Dv 0.25
percentage error in velocity = × 100 = + ´ 100 = 6.25%
v 4
Illustration A thin copper wire of length L increase in length by 2% when heated from T 1 to T2. If a copper cube
having side 10 L is heated from T1 to T2 what will be the percentage change in
(i) area of one face of the cube and. (ii) volume of the cube.
Solution (i) Area A = 10 L × 10 L = 100 L2.
DA DL
Percentage change in area = × 100 = 2 × × 100 = 2 × 2% = 4%
A L

DA D100 DL DL DL
(Q = +2 = 0+2 =2 )
A 100 L L L
(ii) Volume V = 10 L × 10 L × 10 L = 1000 L3
DV DL
Percentage change in volume = × 100 = 3 = 3 × 2% = 6%
V L
Conclusion (i) Constants do not have any error in them.
(ii) The maximum percentage change will be observed in volume, lesser in area and the least
(minimum) change will be observed in length or radius.

E 25
Pre-Medical : Physics
F
Illustration If a = sin bt , find dimensions of a and b. Here v = velocity, F = force and t = time.
v2
Solution Here sin bt and bt must be dimensionless

é1 ù ù é F ù é MLT ù
-2
éF
So [bt ] = 1 Þ [b ] = ê ú = [ T ] ; [ a ] = ê 2 sin bt ú = ê 2 ú = ê 2 -2 ú = [ ML ]
-1 -1

ëtû ëv û ëv û ë L T û

Illustration Following observations were taken with a vernier callipers while measuring the length of a cylinder.
3.29 cm, 3.28 cm, 3.29 cm, 3.31 cm, 3.28 cm, 3.27 cm, 3.29 cm, 3.30 cm
Then find
(a) Most accurate length of the cylinder. (b) Absolute error in each observation.
(c) Mean absolute error (d) Relative error
(e) Percentage error
Express the result in terms of absolute error and percentage error.

Solution (a) Most accurate length of the cylinder will be the mean length l ()
3.29 + 3.28 + 3.29 + 3.31 + 3.28 + 3.27 + 3.29 + 3.30
l= = 3.28875 cm or l = 3.29 cm
8
(b) Absolute error in the first reading = 3.29 – 3.29 = 0.00 cm
Absolute error in the second reading = 3.29 – 3.28 = 0.01 cm
Absolute error in the third reading = 3.29 – 3.29 = 0.00 cm
Absolute error in the forth reading = 3.29 – 3.31 = –0.02 cm
Absolute error in the fifth reading = 3.29 – 3.28 = 0.01 cm
Absolute error in the sixth reading = 3.29 – 3.27 = 0.02 cm
Absolute error in the seventh reading = 3.29 – 3.29 = 0.00 cm
Absolute error in the last reading = 3.29 – 3.30 = –0.01 cm
0.00 + 0.01 + 0.00 + 0.02 + 0.01 + 0.02 + 0.00 + 0.01
(c) Mean absolute error = Dl = = 0.01 cm
8
Dl 0.01
(d) Relative error in length = = = 0.0030395 = 0.003
l 3.29
Dl
(e) Percentage error = × 100 = 0.003 × 100 = 0.3%
l
So length l = 3.29 cm ± 0.01 cm (in terms of absolute error )
or l = 3.29 cm ± 0.30% (in terms percentage error )
Illustration One cm on the main scale of vernier callipers is divided into ten equal parts. If 20 divisions of vernier
scale coincide with 8 small divisions of the main scale. What will be the least count of callipers ?

8 ö
Solution 20 division of vernier scale = 8 division of main scale Þ 1 VSD= æç ÷ MSD = 0.4 MSD
è 20 ø
Least count = 1 MSD – 1 VSD = 1 MSD – 0.4 MSD = 0.6 MSD

1
= 0.6 × 0.1 cm = 0.06 cm (Q 1 MSD= cm = 0.1 cm)
10
Illustration A spherometer has 100 equal divisions marked along the periphery of its disc, and one full rotation of
the disc advances on the main scale by 0.01 cm. Find the least count of the system.
Solution Given Pitch = 0.01 cm

Pitch 0.01
\ Least count = = cm = 10–4 cm.
Total no. of divisions on the the circular scale 100

26 E
Pre-Medical : Physics
UNITS, DIMENSIONS AND MEASUREMENTS EXERCISE
1. If the time period (T) of vibration of a liquid drop 8. If dimension of critical velocity nc, of liquid flowing
depends on surface tension (S), radius (r) of the drop through a tube is expressed as (hx ry rz), where h,
and density (r) of liquid, then the expression of T is r and r the coefficient of viscosity of liquid, density
rr 3 r1 / 2 r 3 of liquid and radius of the tube respectively, then
(1) T = k (2) T = k the values of x, y and z are given by :
S S
(1) 1, 1, 1 (2) 1, –1, –1
rr 3 (3) –1, –1, 1 (4) –1, –1, –1
(3) T = k (4) none of these
S1 / 2
9. A physical quantity of the dimensions of length that
r A 2ˆ
2. The electric field is given by E = 3 ˆi + Byjˆ + Cz k . e2
x can be formed out of c, G and is [c is velocity
4 pe0
The SI units of A, B anc C are respectively.
of light, G is universal constant of gravitation and
N - m3
(1) , V/m2, N/m2-C e is charge] :-
C
1/ 2 1/2
(2) V-m2, V/m, N/m2-C 2 é e2 ù 1 é e2 ù
(3) V/m2, V/m, N-C/m2 (1) c êG ú (2) 2 ê ú
ë 4pe0 û c ë G 4pe0 û
(4) V/m, N-m3/C, N-C/m
3. What are the dimensions of A/B in the relation 1/2
1 e2 1é e2 ù
F = A x + Bx , where F is the force, x is the dis-
2
(3) c G 4pe (4) 2 êG ú
0 c ë 4pe0 û
tance and t is time?
(1) L1/2 (2) L3/2 (3) L–3/2 (4) L–1/2 10. Which one of the following does not have the same
4. The potential energy of a particle is given by the dimensions
x (1) work and energy
expression U(x) = -ax + b sin . A dimensionless
g (2) angle and strain
combination of the constants a,b and g is :- (3) relative density and refractive index
(4) plank constant and energy
a a2 g ag 11. If unit of length and force incresed 4 times. The unit
(1) (2) (3) (4)
bg ba ab b of energy.
1 (1) is increased by 4 times
5. The dimensions of is that of :- (2) is increased by 16 times
e0µ 0
(3) is increased by 8 times
(1) velocity (2) time (4) remains unchanged
(3) capacitance (4) distance 12. The density of a material in CGS system is 8 g/cm 3.
6. The dimensional formula for Planck's constant h and In a system of a unit in which unit of length is 5 cm
gravitational constant G respectively are :- and unit of mass is 20 g. The density of material is :-
(1) [ML3T–2], [M–1L2T–3] (1) 8 (2) 20 (3) 50 (4) 80
13. In a new system the unit of mass is a kg, unit of
(2) [ML2T–1], [M–1L3T–2]
length is b m and unit of times of g s. The value of 1J
(3) [ML3T–2], [M–1L2T2] in this new system is :-
(4) [MLT–3], [M–1L3T–3]
g2 ga
7. If energy (E), velocity (V) and time (T) are chosen as (1) (2) (3) abg (4) ag2/b2
ab2 b2
the fundamental quantities, the dimensional formula
of surface tension will be : 14. Which of the following has the highest number of
significant figures ?
(1) [EV–1T–2] (2) [EV–2T–2]
(1) 0.007 m2 (2) 2.64 × 1024 kg
(3) [E–2V–1T–3] (4) [EV–2T–1]
(3) 0.0006032 m2 (4) 6.3200 J

E 27
Pre-Medical : Physics
15. The density of a material in CGS system of units is 21. A quantity is represented by X = Ma Lb Tc. The
4 g/cm3. In a system of units in which unit of length percentage error in measurement of M, L and T
is 10 cm and unit of mass is 100 g, the value of are a%, b % and g % respectively. The percentage
density of material will be :- error in X would be
(1) 0.04 (2) 0.4 (3) 40 (4) 400 (1) (a a + bb + gc) %
1 (2) (aa – bb + gc) %
16. The least count of a stop watch is s . The time of
5 (3) (a a – bb– gc) %
20 oscillations of a pendulum is measured to be 25s. (4) None of these
What is the maximum percentage error in this
measurement? 22. The period of oscillation of a simple pendulum in
(1) 8% (2) 1% (3) 0.8% (4) 16% an experiment is recorded as 2.63s, 2.56s, 2.42s,
17. The refractive index of water measured by the relation 2.71s and 2.80s respectively. The average absolute
error is
real depth
m= is found to have values of 1.34, (1) 0.1s (2) 0.11s
apparent depth
(3) 0.01s (4) 1.0s
1.38, 1.32 and 1.36; the mean value of refractive
index with percentage error :- 23. The length of a cylinder is measured with a metre
(1) 1.35 ± 1.48% (2) 1.35 ± 0% rod having least count 0.1 cm. Its diameter is
(3) 1.36 ± 6% (4) 1.36 ± 0% measured with vernier callipers having least count
18. Choose the incorrect statement out of the following :- 0.01 cm. Given the length is 5.0 cm. and radius is
(1) Every measurement made by any measuring 2.00 cm. The percentage error in the calculated
instrument has some error. value of volume will be –

(2) Every calculated physical quantity that is based (1) 2% (2) 1%


on measured values has some error. (3) 2.5% (4) 4%
24. A physical quantity e is calculated by using the
(3) A measurement can have more accuracy but
less precision and vice versa. xy2
fo rmula e= where x, y an d z are
(4) The percentage error is different from relative 10z1 / 3
error.
experimentally measured quantities. If the fractional
19. A student measures the distance traversed in free percentage error in the measurements of x,y and z
fall of a body, initially at rest in a given time. He uses are 2%, 1% and 3% respectively, then the
this data to estimate g, the acceleration due to fractional percentage error in e will be :-
gravity. If the maximum percentage errors in
(1) 0.5 % (2) 5 % (3) 6 % (4) 7 %
measurement of the distance and the time are e1
and e2 respectively, the percentage error in the 25. A wire has a mass (0.3 ± 0.003) g, radius
estimation of g is :- (0.5 ± 0.005) mm and length (6 ± 0.06) cm. The
(1) e1 + 2e2 (2) e1 + e2 maximum percentage error in the measurement
of its density is–
(3) e1 – 2e2 (4) e2 – e1
(1) 1 (2) 2 (3) 3 (4) 4
20. If a set of defective weights are used by a student
to find the mass of an object using a physical 26. The resistance of a metal is given by R = V/I, where
balance, a large number of reading will reduce :- V is potential difference and I is the current. In a
circuit the potential difference across resitance is
(1) random error
V =(10 ± 0.5)V and current in resistance,
(2) random as well as systematic error I = (2 ± 0.2)A. The value of resitance in W with
(3) systematic error percentage error is :-
(4) neither random nor systematic error (1) 5 ± 10% (2) 5 ± 15%
(3) 5 ± 20% (4) 5 ± 25%

28 E
Pre-Medical : Physics
27. The period of oscillation of a simple pendulum is given 29. A student measured the diameter of a small steel
l ball using a screw gauge of least count 0.001 cm.
by T = 2p , where l is about 100 cm and is known The main scale reading is 5 mm and zero of circular
g
to have 1 mm accuracy. The period is about 2 s. The scale division coincides with 25 divisions above the
time of 100 oscillations is measured by a stop watch reference level. If screw gauge has a zero error of
of least count 0.1 s. The percentage error in g is :- – 0.004 cm, the correct diameter of the ball is :-
(1) 0.1% (2) 1% (3) 0.2% (4) 0.8%
28. In a vernier callipers, one main scale division is (1) 0.521 cm (2) 0.525 cm
x cm and n divisions of the vernier scale coincide (3) 0.053 cm (4) 0.529 cm
with (n –1) divisions of the main scale. The least
count (in cm) of the callipers is :-

æ n -1ö nx x x
(1) ç ÷ x (2) (3) (4)
è n ø (n - 1) n (n - 1)

ANSWER KEY
Que. 1 2 3 4 5 6 7 8 9 10 11 12 13 14 15
Ans. 1 1 2 4 1 2 2 2 4 4 2 3 1 4 3
Que. 16 17 18 19 20 21 22 23 24 25 26 27 28 29
Ans. 3 1 4 1 1 1 2 3 2 4 2 3 3 4

E 29
Pre-Medical : Physics

KINEMATICS
w Distance and Displacement
C
Total length of path (ACB) covered by the particle, in definite time interval
is called distance. Displacement vector or displacement is the minimum A B
distance (AB) and directed from initial position to final position.
w Displacement in terms of position vector
r r r A
From DOAB Dr = rB - rA
Dr
r r
rB = x2 $i + y2$j + z2k$ and rA = x1$i + y1$j + z1k$ rA
B
r $
D rr = (x2 - x1 )$i + (y2 - y1 )$j + (z2 - z1 )k O
rB
r
w Average velocity = Displacement = vr av =
Dr
Time interval Dt

Distance travelled
w Average speed =
Time interval
w For uniform motion (Constant velocity)
Average speed = |Average velocity| = |Instantaneous velocity| = Instantaneous speed
r
w Velocity vr = dr = d xiˆ + yjˆ + zkˆ = dx ˆi + dy ˆj + dz kˆ = v xˆi + v yˆj + v z kˆ
( )
dt dt dt dt dt
r
change in velocity r Dv
w Average Acceleration = = a av =
total time taken Dt
w Acceleration
r
r dv d dv
a= =
dt dt x
( dv
dt dt
) dv
v ˆi + v yˆj + v z kˆ = x ˆi + y ˆj + z kˆ = a xˆi + a yˆj + a z kˆ
dt
Important points about 1D motion
w Distance ³|displacement| and Average speed ³ |average velocity|
w If distance >|displacement| this implies
The body must have retardation during the motion and atleast at one point in path, velocity is zero.
w Speed increase if acceleration and velocity both are positive or negative
(i.e. both have same sign)

dv dv
w In 1-D motion a = =v
dt dx
a
w Graphical analysis in Motion
v2 t2 t2
dv
• a=
dt
Þ ò dv = ò adt
v1 t1
Þ v 2 - v1 = ò adt
t1

Þ Change in velocity
t1 t
t2
= Area between acceleration curve
shaded area = change in velocity
and time axis, from t1 to t2.
v
x2 t2 t2
dx
• v=
dt
Þ ò dx = ò vdt Þ x
x1 t1
2 - x1 = ò vdt
t1

Þ Change in position = displacement


= area between velocity curve and t
t1 t2
time axis, from t1 to t2. shaded area = displacement

30 E
Pre-Medical : Physics
Important point about graphical analysis of motion

æ dx ö
w Instantaneous velocity is the slope of position time curve at a point. èç v = ÷
dt ø

æ dv ö
w Slope of velocity-time curve (at a point) = instantaneous acceleration çè a = ø÷
dt

é Dx = vdt ù
w v-t curve area gives displacement. ë ò û
é Dv = adt ù
w a-t curve area gives change in velocity. ë ò û
Differentiation Differentiation
Displacement Velocity Acceleration
Integration Integration

Different Cases v–t graph s–t graph


v v=constant s vt
s=
1. Uniform motion
t t 2
v s s =½ at
2. Uniformly accelerated at
motion with u = 0 at v=
t=0 t t

v s
3. Uniformly accelerated s = ut+½ at
2
t
+a
with u ¹ 0 at t = 0 u v =u
t t
4. Uniformly accelerated v s 2
s =s0+ut+½ at
motion with u ¹ 0 =u
+a
t
u v
and s = s0 at t = 0 t t

v s s=ut – ½ at2
5. Uniformly retarded u v=
motion till velocity u –a
t
becomes zero t t0
t
t0 s
v
6. Uniformly retarded
u
then accelerated in t0
t
opposite direction t0
t

Motion with constant acceleration : Equations of motion


r In vector form : r r
r r r r r r r æ u + vö r 1r r 1r
D r = r2 - r1 = s = ç t = ut + at2 = vt - at2
v = u + at è 2 ÷ø 2 2
r
rr r r a
v2=u2+ 2a.s snth = u + (2n - 1) [ S n th ® displacement in nth second]
2
r In scalar form (for one dimensional motion) :
æ u + vö 1 1
v = u + at s=ç t = ut + at2 = vt - at 2
è 2 ÷ø 2 2
a
v2 = u2 + 2as s nth = u + (2n - 1)
2
w RELATIVE MOTION
There is no meaning of motion without reference or observer. If reference is not mentioned then we take the
ground as a reference of motion. Generally velocity or displacement of the particle w.r.t. ground is called actual
velocity or actual displacement of the body. If we describe the motion of a particle w.r.t. and object which is also
r
moving w.r.t. ground then velocity of particle w.r.t. ground is its actual velocity ( v act ) and velocity of particle w.r.t.
r
moving object is its relative velocity ( v rel. ) and the velocity of moving object (w.r.t. ground) is the reference
r r r r r r r
velocity ( v ref. ) then v rel = v act - v ref v actual = v relative + v reference
E 31
Pre-Medical : Physics
w Relative velocity of Rain w.r.t. the Moving Man :
r uuuur Vertically up
A man walking west with velocity v m , represented by OA .
r q
Let the rain be falling vertically downwards with velocity v r , A vm O –vm C
W E
uuur q
represented by OB as shown in figure.
r r r vr vrm
The relative velocity of rain w.r.t. man v rm = v r - v m
uuuur
will be represented by diagonal OD of rectangle OBDC. B D

\ v rm = v 2r + v 2m + 2v r v m cos 90° = v 2r + v 2m
r
If q is the angle which v rm makes with the vertical direction then

BD v m æv ö
tan q = = Þ q = tan-1 ç m ÷
OB v r è vr ø
w Swimming into the River
A man can swim with velocity vr , i.e. it is the velocity of man w.r.t. still water. If water is also flowing with velocity
r r r r
v R then velocity of man relative to ground v m = v + v R

• If the swimming is in the direction of flow of water or along the downstream then
v
vR vm = v + vR

• If the swimming is in the direction opposite to the flow of water or along the upstream then
v
vR vm = v– vR
r
• If man is crossing the river as shown in the figure i.e. vr and v R not collinear then use the vector algebra
r r r
v m = v + v R (assuming v > vR)

vR

v
vm

B
• For shortest path :

vR
d
vm
q
v

A
For minimum displacement
v
To reach at B, vsin q = vR Þ sin q = v
R

d
Time of crossing (t) =
v cos q
v
Note : If vR > v then for minimum drifting sin q =
vR
• For minimum time
B C
then d Drift = v R t min
t min = (BC)
vR v
d
v vm
q

A
(for minimum time)

32 E
Pre-Medical : Physics
MOTION UNDER GRAVITY
If a body is thrown vertically up with a velocity u in the uniform gravitational field (neglecting air resistance) then

u2
(i) Maximum height attained H =
2g H
u u
(ii) Time of ascent = time of descent = g

2u
(iii) Total time of flight = g

(iv) Velocity of fall at the point of projection = u (downwards)


(v) Gallileo's law of odd numbers : For a freely falling body ratio of successive distance covered in equal
time internval 't'
S1 : S2 : S3 : ....Sn = 1: 3: 5 : ....: 2n–1
• At any point on its path the body will have same speed
for upward journey and downward journey.
• If a body thrown upwards crosses a point in time t1 t2
H
t1 & t2 respectively then height of point h = ½ gt1t2 h

1
Maximum height H = g(t1 + t2)2
8

• A body is thrown upward, downward & horizontally v 1


v
3
with same speed takes time t1, t2 & t3 respectively to reach the ground then t 3 = t1 t2 v 2

& height from where the particle was throw is H = 1 gt1 t2


2
w PROJECTILE MOTION y
r Horizontal Motion u ucosq
H
u cosq = ux q
x
ax = 0 R
x = uxt = (u cosq)t u

r Vertical Motion :
1 1
vy = uy – gt where uy = u sinq; y = uyt – gt2= usinqt – gt2
2 2
r
Net acceleration = a = a xˆi + a yˆj = -gjˆ

r At any instant : vx = ucosq, vy = usinq – gt


r For projectile motion :
• A body crosses two points at same height in time t1 and t2 the points are at distance x and y from starting
point then
(a) x1 + x2 = R
A B
(b) t1 + t2 = T
u h
q
(c) h = 1/2 gt1t2
x1
x2
(d) Average velocity from A to B is ucosq
• If a person can throw a ball to a maximum distance 'x' then the maximum height to which he can throw
the ball will be (x/2)

E 33
Pre-Medical : Physics
r Velocity of particle at time t :
r
v = v xˆi + v yˆj = u xˆi + (u y - gt)ˆj = u cos qˆi + (u sin q - gt)ˆj

vy u y - gt u sin q - gt
If angle of velocity vr from horizontal is a, then
gt
tan a = = = = tan q -
vx ux u cos q u cos q

r At highest point : vy=0, vx=ucosq


2u y 2u sin q
r Time of flight : T= =
g g

2u 2 sin q cos q u 2 sin 2q 2u x u y


r Horizontal range : R = ( u cos q ) T = = =
g g g

It is same for q and (90° – q) and maximum for q = 45°

u2y u 2 sin2 q 1 2
r Maximum height H= = = gT
2g 2g 8

H 1
r = tan q
R 4

gx2 æ xö
r Equation of trajectory y = x tan q - = x tanq ç 1 - ÷
2u cos2 q
2
è Rø

w Horizontal projection from some height u

2h
r Time of flight T= h
g
2h
r Horizontal range R = uT = u
g R

æ gt ö
r Angle of velocity at any instant with horizontal q = tan -1 ç ÷
èuø
KEY POINTS :

• A positive acceleration can be associated with a "slowing down" of the body because the origin and the positive
direction of motion are a matter of choice.
x

• The x-t graph for a particle undergoing rectilinear motion, cannot be as

shown in figure because infinitesimal changes in velocity are physically possible


t
only in infinitesimal time.
• In oblique projection of a projectile the speed gradually decreases up to the highest point and then increases
because the tangential acceleration opposes the motion till the particle reaches the highest point, and then
it favours the motion of the particle.

• In free fall, the initial velocity of a body may not be zero.

• A body can have acceleration even if its velocity is zero at an instant.


• Average velocity of a body may be equal to its instantaneous velocity.

• The trajectory of an object moving under constant acceleration can be straight line or parabola.
• The path of one projectile as seen from another projectile is a straight line as relative acceleration of one
projectile w.r.t. another projectile is zero.

34 E
Pre-Medical : Physics
Illustration A car moves from O to D along the path A
O 8km N
OABCD shown in fig. 4km W E
D
S
1km
What is distance travelled and its net displacement ? 4km
C B
uuuur uuur uuur uuur
Solution Distance = |OA|+|AB|+|BC|+|CD|

= 8 + 4 + 4 +1 = 17 km
N
uuuur uuur uuur uuur
Displacement = OA + AB + BC + CD

= 8iˆ + (–4j)
ˆ + ( -4i)
ˆ + ˆj = 4iˆ - 3jˆ W 37°
E

(4i–3j)
Þ|displacement| = (4)2 + (3)2 = 5
So, Displacement = 5 km, 37° S of E S

Illustration On an open ground a motorist follows a track that turns to his left by an angle of 60° after every
500 m. Starting from a given turn, specify the displacement of the motorist at the third, sixth and
eighth turn. Compare the magnitude of displacement with the total path length covered by the
motorist in each case.
IV 60°
C III
Solution At III turn 60°
60° 500m
uuuur uuur uuur uuur 60°
|Displacement| = |OA|+|AB|+|BC|+|OC| V
B II, VIII
= 500 cos 60° + 500 + 500 cos 60° 60° 500m

60° A 60°
1 1 VI
= 500 × + 500 + 500 × = 1000 m O 500m I VII
2 2

So |Displacement|= 1000 m from O to C

|Displacement| 1000 2
Distance = 500 + 500 + 500 = 1500 m \ = =
Distance 1500 3

At VI turn
Q initial and final positions are same so |displacement| = 0 and distance = 500 × 6 = 3000 m
|Displacement| 0
\ = =0
Distance 3000
At VIII turn
60° ö 3
|Displacement| = 2(500)cos æç ÷ = 1000 × cos 30° = 1000 ´ 2 = 500 3 m
è 2 ø
|Displacement| 500 3 3
Distance = 500 × 8 = 4000 m \ = =
Distance 4000 8
Illustration If a particle travels the first half distance with speed v1 and second half distance with speed v2.Find
its average speed during the journey.
A s O s B
s+s 2s 2v1v 2 v1 v2
Solution vav = = =
t1 + t 2 s s v1 + v 2 s s
+ t1 = v t2 = v
v1 v 2 1 2

Note :- Here vav is the harmonic mean of two speeds.


E 35
Pre-Medical : Physics
Illustration If a particle travels with speed v1 during first half time interval and with speed v 2 during second half
time interval. Find its average speed during its journey.
s1 s2
Solution s1 = v1t and s2 = v2t t v1 t v2
A O B
Total distance = s1 + s2 = (v1 + v2)t

total time = t + t = 2t then vav =


s1 + s 2
=
b =
g
v1 + v 2 t v1 + v 2
t+t 2t 2

Note :- here vav is arithmetic mean of two speeds.


Illustration The displacement of a point moving along a straight line is given by
s = 4t2 + 5t – 6
Here s is in cm and t is in seconds calculate
(i) Initial speed of particle
(ii) Speed at t = 4s

ds
Solution (i) Speed, v = = 8t + 5 Initial speed (i.e at t = 0), v = 5 cm/s
dt

(ii) At t = 4s, v = 8(4) + 5 = 37 cm/s


Illustration A particle moves in a straight line with a uniform acceleration a. Initial velocity of the particle is
zero. Find the average velocity of the particle in first 's' distance.

1 2 s2 1 s as
Solution Q s= at \ = as Average velocity = =
2 t2 2 t 2

Illustration A car starting from rest, accelerates at the rate f through a distance S, then continues at constant

f
speed for time t and then comes to rest with retardation . If the total distance travelled is 15S
2
then calculate the value of S in term of f and t.
Solution Let constant speed be vm

1
for time t1; vm = ft1 and S = f t12
2

f
for time t2 0 = vm – t Þ t2 = 2t1
2 2

1 FG f IJ t FG f IJ e4t j = f t
2 2
S3 =
H 2K =
H 4K = 2S
2
1 1
2 2

Therefore S + vmt + 2S = 15S Þ vmt = 12S Þ ft1t = 12S

FG 2S IJ 1
2 144S
2
ft 2
Þ f
HfK t = 12 S Þ 2Sf =
t
2 Þ S =
72

36 E
Pre-Medical : Physics
Illustration A ball is thrown upwards from the top of a tower 40 m high with a
velocity of 10 m/s, find the time when it strikes the ground +ve
(g = 10 m/s ) 2 u=10m/s 2
a=–10m/s
Solution In the problem u = + 10 m/s, a = – 10 m/s 2

and s = –40m (at the point where ball strikes the ground)
h
1 2
Substituting in s = ut + at
2
– 40 = 10t – 5t2 or 5t2 – 10t – 40 = 0 or t2 – 2t – 8 = 0
Solving this we have t = 4 s and – 2s.
Taking the positive value t = 4s.
Illustration A block slides down a smooth inclined plane when released from the top, while another falls freely
from the same point. Which one of them will strike the ground: (a)earlier (b) with greater speed?
Solution In case of sliding motion on the inclined plane.

h h
= sin q Þ s= , a = g sinq
s sin q

2s 2 h 1 2h t s
ts = = ´ = = F h
a g sin q sin q sin q g sin q
q

h
vs = 2as = 2g sin q ´ = 2gh
sin q

2h
In case of free fall tF = and vF = 2gh = vs
g

(a) Q sinq <1, tF < ts , i.e.,falling body reaches the ground first.
(b) vF = vs, i.e.,both reach the ground with same speed.
Special Note : (not same velocity, as for falling body direction is vertical while for sliding
body along the plane downwards).
Illustration A Juggler throws balls into air. He throws one ball whenever the previous one is at its highest point.
How high do the balls rise if he throws n balls each second ? Acceleration due to gravity is g.

1
Solution Juggler throws n balls in one second so time interval between two consecutive throws is t = s
n

1
each ball takes s to reach maximum height
n
1s
t=
2
hmax n
1 1 æ1ö
So hmax = ´ gt 2 = ´ gç ÷
2 2 ènø

g
hmax =
2n2

E 37
Pre-Medical : Physics
Illustration A particle is thrown vertically upwards from the surface of the earth. Let TP be the time taken by
the particle to travel from a point P above the earth to its highest point and back to the point P.
Similarly, let TQ be the time taken by the particle to travel from another point Q above the earth
to its highest point and back to the same point Q. If the distance between the points P and Q is H,
find the expression for acceleration due to gravity in terms of T P, TQ and H.

2(h + H) Highest
Solution Time taken from point P to point P TP = 2 point
g
h

2h Q
Time taken from point Q to point Q TQ = 2 H
g P

8H
8(h + H) 8h
Þ TP2 = & TQ2 = Þ TP2 = TQ2 + 8H Þ g = T2 - T 2
g g g P Q

Illustration A boy stands 78.4 m away from a building and throws

a ball which just enters a window at maximum height 39.2m

above the ground. Calculate the velocity of projection of

the ball.
u 2 sin 2 q
Solution Maximum height = = 39.2 m ... (i)
2g
u 2 sin 2q 2u 2 sin q cos q
Range = = = 2 × 78.4 ... (ii)
g g
from equation (i) divided by equation (ii) tanq = 1 Þ q = 45°
u 2 sin 90°
from equation (ii) range = = 2 × 78.4 Þ u = 2 ´ 78.4 ´ 9.8 = 39.2 m/s
g
Illustration A particle thrown over a triangle from one end of a horizontal base falls on the other end of the
base after grazing the vertex. If a and b are the base angles of triangle and angle of projection is
q, then prove that tan q = tan a + tan b .
Solution From triangle y = x tan a and y = (R – x) tan b
u y
q
y y yR a b
tan a + tan b = + =
x R-x x(R - x)
x R-x
é xù yR
Q y = x tan q ê1 - ú Þ tan q =
ë R û x(R - x)
\ tan q = tan a + tan b
Illustration A ball is thrown from the ground to clear a wall 3 m high at a distance of 6 m and falls 18 m away
from the wall, the angle of projection of ball is :-
æ 3ö æ 2ö æ 1ö æ 3ö
(A) tan–1 çè ÷ø (B) tan–1 çè ÷ø (C) tan–1 çè ÷ø (D) tan–1 çè ÷ø
2 3 2 4
é 1ù 2
From equation of trajectory, y = xtanq éê1 - ùú Þ 3 = 6 tanq
x
Solution êë1 - 4 úû Þ tanq = 3
y
ë Rû

3m
q x
6m 1 8m

38 E
Pre-Medical : Physics
Illustration A ball rolls off the top of a stair way with a horizontal velocity u. If each step has height h and width
b the ball will just hit the edge of n th step. Find the value of n.

Solution If the ball hits the nth step, the horizontal and vertical distances traversed are nb and nh respectively.
Let t be the time taken by the ball for these horizontal and vertical displacements. Velocity along
horizontal direction = u (remains constant) and initial vertical velocity = zero.

1 2 u
\ nb = ut and nh = 0 + gt h 1
2
2
Eliminating t from the equation b
nth step
2
1 æ nb ö 2hu 2
nh = g Þ n=
2 çè u ÷ø gb2 R

Illustration Three boys A, B and C are situated at the vertices of an equilateral triangle of side d at t = 0. Each
of the boys move with constant speed v. A always moves towards B , B towards C and C towards
A. When and where will they meet each other ?
Solution By symmetry they will meet at the centroid of the triangle. A
v
Approaching velocity of A and

°
60
B towards each other is v + v cos 60° and they cover distance d
O

os
vc
when they meet. So that time taken, is given by 60°
B C
d d 2d
\ t= = =
v + v cos 60 o v 3v
v+
2

Illustration A man at rest observes the rain falling vertically. When he walks at 4 km/h, he has to hold his umbrella
at an angle of 53° from the vertical. Find the velocity of raindrops.
r r r
Solution Assigning usual symbols vm , v r and vr /m to velocity of man,

velocity of rain and velocity of rain relative to man, we can express r


v r/m
r Vertical
their relationship by the following equation r vm
r r r vr 53°
v r = v m + v r/ m
The above equation suggests that a standstill man observes vm = 4

r r
37°
velocity v r of rain vr 53°
r
vr / m
relative to the ground and while he is moving with
r
velocity vm , he observes

r
velocity of rain relative to himself vr /m . It is a common intuitive fact that umbrella must be held

r
against vr /m for optimum protection from rain. According to these facts, directions of the velocity

vectors are shown in the adjoining figure.


Therefore vr=vm tan37°=3 km/h

E 39
Pre-Medical : Physics
Illustration A boat can be rowed at 5 m/s in still water. It is used to cross a 200 m wide river from south bank to
the north bank. The river current has uniform velocity of 3 m/s due east.
(a) In which direction must it be steered to cross the river perpendicular to current?
(b) How long will it take to cross the river in a direction perpendicular to the river flow?
(c) In which direction must the boat be steered to cross the river in minimum time? How far will
it drift?
Solution (a) To cross the river perpendicular to current i.e. along shortest path

u 3
sin q = = Þ q = 37°
v 5 N
(b) Time taken by boat,
d=200m u=3 m/s W E
v=5 m/s
d 200 q
t= = = 50s
v cos q 5 ´ 4 S
5
(c) To cross the river in minimum time, q = 0°

d 200
Therefore t min = = s = 40s
v 5
Drift = u(tmin) = 3(40)m = 120 m

40 E
Pre-Medical : Physics
KINEMATICS EXERCISE
1. If a car covers 2/5th of the total distance with v1 speed 6. A particle experiences a constant acceleration for 20
and 3/5th distance with v2, then average speed is :- second after starting from rest. If it travels a distance
s1 in 10 second and distance s2 in the next 10 sec,
1 v1 + v 2
(1) v1 v 2 (2) then :-
2 2
s2
2v 1 v 2 5v1 v 2 (1) s1 = s2 (2) s1 =
(3) v + v (4) 3v + 2v 3
1 2 1 2

s2 s2
(3) s1 = (4) s1 =
2. A particle moving in a straight line covers half the 2 4
distance with speed of 3m/s. The other half of the
distance covered in two equal time intervals with speed 7. A point moves with uniform acceleration and v 1, v2
of 4.5 m/s and 7.5 m/s respectively. The average speed and v3 denote the average velocities in three successive
of the particle during this motion is :- intervals of time t1, t2 and t3. Which of the following
(1) 4.0 m/s (2) 5.0 m/s relations is correct :-
(3) 5.5 m/s (4) 4.8 m/s (1) v1 – v2 : v2 – v3 = t1 – t2 : t2 + t3
(2) v1 – v2 : v2 – v3 = t1 + t2 : t2 + t3
3. A particle located at x = 0 at time t = 0, starts (3) v1 – v2 : v2 – v3 = t1 – t2 : t1 – t3
moving along the positive x–direction with a velocity (4) v1 – v2 : v2 – v3 = t1 – t2 : t2 – t3

'v' which varies as v = a x , then velocity of particle 8. Speed of two identical cars are u and 4u at a specific
varies with time as : (a is a constant) instant. The ratio of the respective distances in which
(1) v µ t (2) v µ t2 the two cars are stopped from that instant is :-
(1) 1 : 1 (2) 1 : 4
(3) v µ t (4) v = constant (3) 1 : 8 (4) 1 : 16
4. A train accelerates from rest at a constant rate a
9. A particle is dropped vertically from rest from a height.
for distance x1 and time t1. After that is retards
The time takes by it to fall through successive of 1
at constant rate b for distance x2 and time t2 and meter each will then be :-
comes to the rest. Which of the following relations
is correct ? 2
(1) all equal, being equal to second
g

x1 a t1 x1 b t1 (2) in the ratio of square roots of integers 1, 2, 3,...


(1) = = (2) = = (3) in the ratio of the dirrence in the square roots of
x2 b t2 x2 a t2
integers ( 1 - 0 ),( 2 - 1),( 3 - 2),( 4 - 3 )

x1 a t2 x1 b t2 1 1 1 1
(3) = = (4) = = (4) in the ratio : : :
x2 b t1 x2 a t1 1 2 3 4

5. Which of of the following represents uniformly 10. Two ball A and B of same mass are thrown from the
acceleated motion :- top of the building. A thrown upward with velocity v
and B, thrown down with velocity v, then :-
t-a t-a (1) velocity A is more than B at the ground
(1) x = (2) x =
b b (2) velocity of B is more than A at the ground
(3) both A and B strike the ground with same velocity
x-a (4) none of these
(3) t = (4) x = t + a
b

E 41
Pre-Medical : Physics
11. A stone is thrown vertically upward. On its way up it 17. An elevator is accelerating upward at a rate of
passes point A with speed of v, and point B, 3m 6 ft/sec2 when a bolt from its ceiling falls to the
highter than A, with speed V/2 The maximum height floor of the lift (Distance = 9.5 feet). The time (in
reached by stone above point B is :- seconds) taken by the falling bolt to hit the floor is
(1) 1 m (2) 2 m (take g = 32 ft/ sec2)
(3) 3 m (4) 5 m 1
(1) 2 (2)
2
12. A body is projected vertically upwards. If t1 and t2 be
the times at which it is at hight h above the projection 1
(3) 2 2 (4)
while ascending and descending respectively, then h 2 2
is :-
18. The velocity versus time curve of a moving point is as
1 given below. The maximum acceleration is :-
(1) gt1 t 2 (2) gt1t2
2
60

v(cm/s) ®
(3) 2gt1t2 (4) 2hg
40
13. A body A starts from rest with an accelertion a1. After
2 seconds, another body B starts from rest with an
20
acceleration a2. If they travel equal distances in the
5th seconds, after start of A, then the ratio a1 : a2 is
equal to :-
(1) 5 : 9 (2) 5 : 7
0 10 20 30 40 50 60 70
(3) 9 : 5 (4) 9 : 7
t(second) ®
14. Two balls are dropped to the ground from different
(1) 1 cm/s2 (2) 2 cm/s2
heights. One ball is dropped 2s after the other but
(3) 3 cm/s2 (4) 4 cm/s2
they both strike the ground at the same time. If the
first ball takes 5s to reach the ground, then the 19. The graph of displacement v/s time is, then its
difference in initial heights is (g = 10 ms ) :-
–2 corresponding velocity time graph will be :-
(1) 20 m (2) 80 m (3) 170 m (4) 40 m s

15. The initial velocity of particle is u and the acceleration


at the time t is at, a being a constant. Then the v at
the time t is given by :-
(1) v = u t
(2) v = u + at v
v
(3) v = u + at2

1 2 t
(4) v = u + at
2 (1) (2)
t
16. The displacement-time graph for two particles A and
B are straight lines inclined at angles of 30° and 60°
v v
with the time axis. The ratio of velocities of VA : VB
is :-

(1) 1 : 2 (2) 1 : 3 (3) (4) t

t
(3) 3 :1 (4) 1 : 3

42 E
Pre-Medical : Physics
20. The velocity-time graph of a body is shown in figure.
t1 + t2 t1 – t 2
The slope of the line is 'm'. The distance travels by (1) t = (2) t =
2 2
body in time T s :-
t1
v (3) t = t1 t2 (4) t = t2
v0
25. A particle of unit mass undergoes one-dimensional
motion such that its velocity varies according to
T t
v(x) = bx–4n
2 2
mv 0 v 0
(1) (2)
2T 2T where b and n are constants and x is the position

v 20 of the particle. The acceleration of the particle as


(3) 2mv20 (4)
2m a function of x, is given by :
21. Position of a particle moving along x–axis is given
(1) –4nb2x–8n–1 (2) –4b2x–8n+1
by x = 2 + 8t – 4t2. The distance travelled by the
particle from t = 0 to t = 2 is : (3) –4nb2e–8n+1 (4) –4nb2x–8n–1
(1) 0 (2) 8 (3) 12 (4) 16
22. A ball is thrown upwards. Its height varies with time 26. A particle moves a distance y in time t according
as shown in figure. If the acceleration due to gravity to equation y = (t + 10)–1. The acceleration of
is 7.5 m/s2, then the height h is particle is proportional to :-
(1) (velocity)2/3 (2) (velocity)3/2
Height (m)
(3) (distance)2 (4) (distance)–2

27. A body starting from rest is moving under a constant


h
acceleration up to 40 sec. If it moves S1 distance in
first 20 sec., and S2 distance in next 20 sec. then S2
will be equal to :
1 2 5 6 time (s)
(1) S1 (2) 2S1 (3) 3S1 (4) 4S1
(1) 10 m (2) 15 m (3) 20 m (4) 25 m
28. A particle moves along a straight line OX. At a time
23. A body is projected upwards with a velocity u. It
t (in seconds) the distance x (in metres) of the particle
passes through a certain point above the ground
after t1 second. The time after which the body passes from O is given by x = 50 + 12t – t3. How long
through the same point during the return journey would the particle travel before coming to rest ?
is: (1) 66 m (2) 56 m (3) 26 m (4) 16 m
u 2 u 29. A particle shows distance-time curve as given in
(1) ( - t1 ) (2) 2( - t1 )
g g this figure.The maximum instantaneous velocity of

u2 u2 the particle is around the point :-


(3) 3( - t1 ) (4) 3( - t1 )
g g2
24. A body is thrown vertically upwards from the top A D
Distance

of a tower. It reaches the ground in t1 seconds. If it S


C
is thrown vertically downwards from A with the same
speed it reaches the ground in t2, seconds. If it is A B
t Time
allowed to fall freely from A, then the time it takes
to reach the ground is given by : (1) D (2) A (3) B (4) C

E 43
Pre-Medical : Physics
30. If a body starts from rest and travels 120cm in the
6th second then what is the acceleration ?

velocity
(1) 0.526 m/s2 (2) 0.218 m/s2
(3) 0.109 m/s2 (4) 0.056 m/s2 (1) T 2T t

31. A ballon rises from ground with an acceleration of


2 m/s2 after 10 second, a stone is released from the
ballon the stone will
(1) have a displacement of 150 m

velocity
(2) cover a distance of 140 m in reaching the ground T 2T
(2) t
(3) reach the ground in 4 second
(4) begin to move downward after being released

32. A person walks along an east-west street and a graph


of his displacement from home is shown in figure.
His average velocity for the whole time interval is:

velocity
T 2T
(3) t

40

20
x (meter)

15 18 19 21
0
3 6 9 12 t (sec)
velocity

–20
2T
–40 (4) T t

(1) 0 (2) 23 ms–1 35. Acceleration-time graph of a body is shown. The


(3) 8.4 ms–1 (4) None of above corresponding velocity-time graph is :

33. A rocket is launched upward from the earth is


a
surface whose velocity time graphs shown in figure.
Then maximum height attained by the rocket is:

t
A
1000
v(ms–1)

B 140 v v
0 t(s)
20 40 60 80 100 120
C (1) (2)
(1) 1 km (2) 10 km t t

(3) 100 km (4) 60 km


v v
34. A ball is dropped from the certain height on the
surface of glass. It collides elastically and comes back
(3) (4)
to its initial position. If this process it repeated then
t t
the velocity time graph is :
(Take downward direction as positive)

44 E
Pre-Medical : Physics
36. A particle starts from rest. Its acceleration at time 42. A boat which has a speed of 5 km/h, in still water
t = 0 is 5 m/s2 which varies with time as shown in crosses a river of width 1 km along the shortest possible
the figure. The maximum speed of the particle will path in 15 minute. The velocity of the river water in
be : km/h is :-
a 5 m/s
2
(1) 7.5 m/s (1) 1 (2) 3 (3) 4 (4) 41

(2) 15 m/s
43. A man is running up hill with a velocity (2i$ + 3j)
$ m/ s
(3) 30 m/s
w.r.t. ground. He feels that the rain drops are falling
0 t
(4) 37.5 m/s 6s vertically with velocity 4 m/s. If he runs down hill with
same speed, find vrm.
37. Two trains A and B, each of length 100m, are running
on parallel tracks. One overtakes the other in 20 s (1) 2 2 m/s (2) 2 3 m/s
and one crosses the other in 10 s. The velocity of
(3) 2 5 m/s (4) 2 10 m/s
trains are :-
(1) 5 m/s, 5 m/s (2) 10 m/s, 15 m/s
44. A man is going east in a car with a velocity of
(3) 15 m/s, 5 m/s (4) 15 m/s, 30 m/s
20 km/hr, a train appears to move towards north to
38. A ball A is thrown up vertically with a speed u and at
him with a velocity of 20 3 km/hr. What is the
the same instant another ball B is relased from a
actual velocity and direction of motion of train ?
height h. At time t, the speed of A relative of B is :-
(1) 40 m/s, 60° N of W (2) 40 m/s, 60° W of N
(1) u (2) 2u
(3) 40 m/s, 60° N of E (4) 40 m/s, 60° E of N
(3) u – 2gt (4) (u 2 - gt) 45. Four persons P, Q, R and S of same mass travel with
same speed u along a square of side 'd' such that
39. A stone is dropped from a hight h, simultaneously each one always faces the other. After what time will
another stone is thrown up from the ground which they meet each other ?
reaches at a height 4h, the two stones cross each u
P Q
other after time :- u

h h d
(1) (2) (3) 8hg (4) 2hg u
2g 8g
S u R

40. A car moves on a straight track from station A to the


d 2d 2d
station B, with an acceleration a = (b – cx), where b (1) (2) (3) (4) d 3u
u 3u u
and c are contants and x is the distance from station A.
The maximum velocity between the two stations is :- 46. Six persons of same mass travel with same speed
b c u along a regular hexagon of side 'd' such that each
b
(1) (2) b/c (3) (4) one always faces the other. After how what will they
c a c
meet each other ?
41. A car, start from rest, accelerates at the rate 'f' through
u u
a distance s, then continues at constant speed for
time t and then decelerates at the rate
f/2 come to rest. If the total distance traversed is 5 s, u u d
then :-
u u
1 2 1 2
(1) s = ft (2) s = ft
4 2
d 2d 2d
1 2 (1) (2) (3) (4) d 3u
(3) s = ft (4) s = ft u 3u u
6
E 45
Pre-Medical : Physics
47. A boy is running on a levelled road with velocity (v) 52. A ship A is moving Eastward with a speed of
with a long hollow tube in his hand. Water is falling 10 km h–1 and a ship B 100 km North of A, is
vertically downwards with velocity (u). At what angle moving Sorthward with a speed of 10 km h –1. The
to the vertical, should ve incline the tube so that the time after which the distance between them
water drops enters without touching its side : becomes shortest, is :-

-1 æ vö -1 æ vö
(1) tan çè u ÷ø (2) sin ç ÷ (1) 5 h (2) 5 2 h (3) 10 2 h (4) 0 h
è uø

æ vö 53. Two particles A and B, move with constant velocities


æ uö -1
r r
-1
(3) tan çè v ÷ø (4) cos çè u ÷ø v1 and v 2 . At the initial moment their position vectors
48. A river 2 km wide is flows at the rate of 2km/h. A r r
are r1 and r2 respectively. The condition for particle
boatman who can row a boat at a speed of
A and B for their collision is :-
4 km/h in still water, goes a distance of 2 km
upstream and then comes back. The time taken by r r r r
(1) r1 - r2 = v1 - v 2
him to complete his journey is
r r r r
(1) 60 min (2) 70 min (3) 80 min (4) 90 min r1 - r2 v 2 - v1
r
(2) r - rr = r r
49. Two towns A and B are connected by a regular 1 2 v 2 - v1
bus service with a bus leaving in either direction
r r r r
every T minutes. A man cycling with a speed of (3) r1 × v1 = r2 × v 2
20 km h–1 in the direction A to B notices that a
r r r r
bus goes past him every 18 min in the direction (4) r1 ´ v1 = r2 ´ v 2
of his motion, and every 6 min in the opposite
direction. What is the period T of the bus service 54. Two cars A and B start from a point at the same
and with what speed (assumed constant) do the buses time in a straight line and their positions are
represented by xA(t) = at + bt2 and xB(t) = ft + t2.
ply on the road?
(1) T = 9 min., Speed = 40 km h–1 At what time do the cars have the same velocity?
(2) T = 10 min., Speed = 40 km h–1 a+f f-a
(1) (2)
(3) T = 9 min., Speed = 20 km h–1 2(1 + b) 2(1 - b)
(4) T = 10 min., Speed = 20 km h–1 a-f a+f
(3) (4)
1+ b 2(b - 1)
50. Rain drops are falling with velocity ( 2iˆ – 4ˆj ) m/s.
55. Priya reached the metro station and found that the
What should be the velocity of a man so that rain escalator was not working. She walked down the
drops hit him with speed 5m/s ? stationary escalator in time t1. On other days, if she
(1) –iˆ (2) 5iˆ remains stationary on the moving escalator, then the
escalator takes her down in time t2. The time taken
(3) 2iˆ (4) Both (1) & (2) by her to walk down on the moving escalator will
51. A man runs at a speed of 4.0 m/s to overtake a be
standing bus. When he is 6.0 m behind the door (at t1 t2 t1 t 2 t1 + t2
(1) t - t (2) t + t (3) t1 – t2 (4)
t = 0), the bus moves forward and continues with a 2 1 2 1 2
constant acceleration of 1.2 m/s2. The man shall
56. A stone is just released from the window of a train
access the door at time t equal to moving along a horizontal striaght track. The stone
(1) 5.2 s will hit the ground following :-
(2) 4.3 s (1) straight path
(2) circular path
(3) 2.3 s
(3) parabolic path
(4) the man shall never access the door (4) hyperbolic path

46 E
Pre-Medical : Physics
57. At the top of the trajectory of a projectile, the 63. A particle is projected from the ground with a velocity
acceleration is :- of 25 m/s. After 2 second, it just clears a wall 5 m
(1) maximum (2) minimum height. Then angle of projection of particle is
(3) zero (4) g [g = 10 m/s2]
(1) 30° (2) 45° (3) 60° (4) 75°
58. The range of a projectile which is the launched at an
angle of 15° with the horizontal is 1.5 km. What is 64. The horizontal and vertical components of the velocity
the range of the projectile if it is projected of an angle of a projectile are 10 m/s and 20 m/s, respectively.
45° to the horizontal ? The horizontal range of the projectile will be [g = 10
(1) 1.5 km (2) 3 km m/s2] :-
(3) 6 km (4) 0.75 km (1) 5 m (2) 10 m (3) 20 m (4) 40 m
59. An aeroplane is flying horizontally with a velocity of
65. A cart is moving horizontally along a straight line with
600 km/h at a height of 1960 m. When it is vertically
constant speed 30 m/s. A projectile is to be fired from
at a point A on the ground, a bomb is released from
the moving cart in such a way what it will return to
it. The bomb strikes the ground at point B. The
the cart after the cart has moved 80 m. At what
distance AB is :-
speed (relative to the cart) must the projectile be fired
(1) 1200 m (2) 0.33 km
(Take g = 10 m/s2) :-
(3) 3.33 km (4) 33 km
(1) 10 m/s (2) 10 8 m/s
60. A body is projected with velocity v 1 from point A. At
the same time another body is projected vertically 40
upwards with velocity v2. The point B lies vertically (3) m/s (4) None of these
3
below the highest point. For both the bodies to collide
v2 66. For an object thrown at 45° to horizontal, the
v1 should be :- maximum height (H) and horizontal range (R) are
related as :-
v1 v2
(1) R = 16 H (2) R = 8 H
30°
A B (3) R = 4 H (4) R = 2 H

67. A ball is thrown up at an angle 45° with the horizontal.


3 Then the total change of momentum by the instant it
(1) 2 (2) (3) 0.5 (4) 1
2 returns to ground is :-

61. A boy throws a ball with a velocity u at an angle q mv


with the horizontal. At the same instant he starts (1) zero (2) 2 mv (3) 2 mv (4) 2
running with uniform velocity to catch the ball before
it hits the ground. To achieve this he should run with 68. A stone projected with a velocity u at an angle q with
a velocity of :- the horizontal reaches maximum height H1. When it
(1) u cos q (2) u sin q
æp ö
(3) u tan q (4) u2 tan q is projected with velocity u at an angle ç - q ÷ with
è 2 ø

62. A cricketer can throw a ball to a maximum horizontal the horizontal, it reaches maximum height H2. The
distance of d. How high above the ground can the relation between the horizontal range R of the
cricketer throw the same ball? projectile. H1 and H2 is :-

d (1) R = 4 H1H2 (2) R = 4(H1 – H2)


(1) (2) d
2
H12
5d (3) R = 4(H1 + H2) (4) R =
(3) 2d (4) H22
2
E 47
Pre-Medical : Physics
69. A projectile is thrown with an initial velocity of that the boy jumps horizontally, then calculate
minimum velocity with which he has to jump to land
v = a $i + b $j . If range of the projectile is double
®
safely on building B.
the maximum height attained by it then :
(1) a = 2 b (2) b = a
(3) b = 2a (4) b = 4a
25 m
70. Two stones are projected with the same speed but
making different angles with the horizontal. Their A B 5m
4m
ranges are equal. If the angle of projection of one

p (1) 6 m/s (2) 8 m/s


is and its maximum height is y1, then the (3) 4 m/s (4) 2 m/s
3
75. A ball was thrown from height H and the ball hit the
maximum height of the other will be :
(1) 3y1 (2) 2y1 floor with velocity 10(iˆ - ˆj) after 1.5 sec of its pro-

y1 y1 jection. Find initial speed of ball.


(3) (4) y
2 3
71. A particle is projected with a velocity v, so that its x

range on a horizontal plane is twice the greatest H


height attained. If g is acceleration due to gravity,
then its range is :
v = 10 ( i – j )
4v 2 4g
(1) (2) (1) 10 5 m/s (2) 5 5 m/s
5g 5v 2
(3) 15 m/s (4) 30 m/s
4v3 4v 76. Two particles are separated by a horizontal distance
(3) (4) x as shown in figure. They are projected as shown
5g2 5g2
in figure with different initial speeds. The time after
72. A projectile can have the same range R for two which the horizontal distance between them
angles of projection. If t1 and t2 be the times of becomes zero is :
flight in the two cases, then :– u
3 u
(1) t1 t 2 µ R 2 (2) t1 t2 µ R

1 1 30° 60°
(3) t1 t 2 µ (4) t1 t2 µ x
R R2

73. A number of bullets are fired in all possible directions x u


with the same initial velocity u. The maximum area (1) (2)
u 2x
of ground covered by bullets is :–
2 2 2u
æ 2u 2 ö æuö (3) (4) none of these
(1) p ç ÷ (2) 3p ç ÷ x
è g ø ègø

æ u ö
2
æ u2 ö
2
77. A particle has initial velocity (3iˆ + 4ˆj) and has
(3) 5p ç ÷ (4) p ç ÷
è 2g ø è g ø
acceleration (0.4iˆ + 0.3j)
ˆ . Its speed after 10s is :-
74. A boy wants to jump from building A to building B.
Height of building A is 25 m and that of building B (1) 7 unit (2) 7 2 unit
is 5m. Distance between buildings is 4m. Assume (3) 8.5 unit (4) 10 unit
48 E
Pre-Medical : Physics
78. A body starts from rest from the origin with an 81. The x and y coordinates of the particle at any time
acceleration of 6 m/s2 along the x-axis and 8 m/s2 are x = 8t – 2t2 and y = 40t respectively, where
along the y-axis. Its distance from the origin after x and y are in meters and t in seconds. The
4 seconds will be :- acceleration of the particle at t = 2s is :-
(1) 56 m (2) 64 m (3) 80 m (4) 128 m (1) 5 m/s2 (2) – 4 m/s2
r
79. The position vector of a particle R as a function (3) – 8 m/s2 (4) 0
of time is given by :- 82. A toy car with charge q moves on a frictionless
r
R = 2 sin(2pt)iˆ + 2cos(2pt)ˆj horizontal plane surface
r under the influencer of a
uniform electric field E . Due to the force qE , its
Where R is in meters, t is in seconds and î and ĵ
velocity increases from 0 to 12 m/s in one second
denote unit vectors along x and y-directions,
duration. At that instant the direction of the field is
respectively. Which one of the following statements
is wrong for the motion of particle ? reversed. The car continues to move for two more
(1) Path of the particle is a circle of radius 2 meter seconds under the influence of this field. The
r average velocity and the average speed of the toy
(2) Acceleration vectors is along -R car between 0 to 3 seconds are respectively :-
v2
(3) Magnitude of acceleration vector is where (1) 2 m/s, 4 m/s (2) 2 m/s, 6 m/s
R
v is the velocity of particle. (3) 1 m/s, 3.5 m/s (4) 1.5 m/s, 3 m/s
(4) Magnitude of the velocity of particle is
8 meter/second
83. If the velocity of a particle is v = At + Bt2, where
80. A particle moves so that its position vector is given
r A and B are constants, then the distance travelled
by r = cos wt xˆ + sin wt yˆ . Where w is a constant.
by it between 2s and 3s is :-
Which of the following is true ?
(1) Velocity and acceleration both are perpendicular 3
(1) A + 4B (2) 3A+7B
r 2
to r .
(2) Velocity and acceleration both are parallel to rr 5 19 A B
(3) A+ B (4) +
(3) Velocity is perpendicular to rr and acceleration 2 3 2 3
is directed towards the origin
(4) Velocity is perpendicular to rr and acceleration
is directed away from the origin

ANSWER KEY
Que. 1 2 3 4 5 6 7 8 9 10 11 12 13 14 15
Ans. 4 1 1 2 3 2 2 4 3 3 1 1 1 2 4
Que. 16 17 18 19 20 21 22 23 24 25 26 27 28 29 30
Ans. 4 2 4 1 4 2 2 2 3 1 2 3 4 4 2
Que. 31 32 33 34 35 36 37 38 39 40 41 42 43 44 45
Ans. 2 1 4 3 3 2 3 3 2 1 2 1 3 3 1
Que. 46 47 48 49 50 51 52 53 54 55 56 57 58 59 60
Ans. 3 1 3 1 4 3 1 2 2 2 3 4 2 3 3
Que. 61 62 63 64 65 66 67 68 69 70 71 72 73 74 75
Ans. 1 1 1 4 3 3 3 1 3 4 1 2 4 4 2
Que. 76 77 78 79 80 81 82 83
Ans. 1 2 3 4 3 2 2 3

E 49
Pre-Medical : Physics

LAWS OF MOTION AND FRICTION


w Force
A push or pull that one object exerts on another.
w Forces in nature
There are four fundamental forces in nature :
1. Gravitational force 2. Electromagnetic force 3. Strong nuclear force 4. Weak force
w Types of forces on macroscopic objects
(a) Field Forces or Range Forces :
These are the forces in which contact between two objects is not necessary.
Ex. (i) Gravitational force between two bodies.
(ii) Electrostatic force between two charges.
(b) Contact Forces :
Contact forces exist only as long as the objects are touching each other.
Ex. (i) Normal forces. (ii) Frictional force
(c) Attachment to Another Body :
Tension (T) in a string and spring force (F = kx) comes in this group.
w Newton's first law of motion (or Galileo's law of Inertia)
Every body continues in its state of rest or uniform motion in a straight line unless compelled by an external
force to change that state.
Inertia : Inertia is the property of the body due to which body opposes the change of it's state. Inertia of a body
is measured by mass of the body.

inertia µ mass
w Newton's second law
r dpr d r
r
dv r dm r r
F= = (mv) = m +v (Linear momentum p = mv )
dt dt dt dt
r r
r For constant mass system F = ma
r r
w Momentum : It is the product of the mass and velocity of a body i.e. momentum p = mv
SI Unit : kg m s–1 Dimensions : [M L T–1]
w Impulse : Impulse = product of force with time.

t
impulse = area under curve
t2
r
For a finite interval of time from t 1 to t2 then the impulse = ò Fdt
t1
r
If constant force acts for an interval Dt then : Impulse = F Dt
Impulse – Momentum theorem
Impulse of a force is equal to the change of momentum F Dt = Dp

w Newton's third law of motion : Whenever a particle A exerts a force on another particle B, B simultaneously
exerts a force on A with the same magnitude in the opposite direction.
Natural length
w Spring Force (According to Hooke's law) :
F
In equilibrium F=kx (k is spring constant) x
Note : Spring force is non impulsive in nature.
50 E
Pre-Medical : Physics
w Motion of bodies in contact
When two bodies of masses m1 and m2 are kept on the frictionless surface and a force F is applied on
one body, then the force with which one body presses the other at the point of contact is called force of
contact. These two bodies will move with same acceleration a.
(i) When the force F acts on the body with mass m1 as shown in fig.(i)
F = (m1 + m2)a
a

F
m1 m2

Fig.(1) : When the force F acts on mass m1

If the force exerted by m2 on m1 is f1 (force of contact) then for body m1, (F – f1) = m1a

a
a
F
m1 f1 f1
m2

Fig. 1(a) : F.B.D. representation of action and reaction forces.

For body m2 , f1=m2a Þ action of m1 on m2: f1 = m2F


m1 + m2
w Pulley system
A single fixed pulley changes the direction of force only and in general, assumed to be massless and frictionless.
SOME CASES OF PULLEY
Case – I
Let m1 > m2
T
now for mass m1, m1 g – T = m1a T
m2 a
for mass m2,T – m2 g = m2 a a m1

(m1 - m2 ) net pulling force


Acceleration = a = g=
(m1 + m2 ) total mass to be pulled

2m1m2 2 ´ Product of masses


Tension = T = g= g
(m1 + m2 ) Sum of two masses

4m1m2 g
Reaction at the suspension of pulley R = 2T =
(m1 + m2 ) a

m1
Case – II T

For mass m1 : T = m1 a
For mass m2 : m2g – T = m2 a
m2 a

m2 g m 1m 2
Acceleration a = and T = g Force on pulley = T 2
(m1 + m2 ) (m1 + m2 )

FRAME OF REFERENCE
• Inertial frames of reference : A reference frame which is either at rest or in uniform motion along the
straight line with respect to an inertial frame. A non–accelerating frame of reference is called an inertial
frame of reference.
All the fundamental laws of physics have been formulated in respect of inertial frame of reference.
• Non–inertial frame of reference : An accelerating frame of reference is called a non–inertial frame of
reference. Newton's laws of motion are not directly applicable in such frames, before application we must add
pseudo force.

E 51
Pre-Medical : Physics
w Pseudo force: The force on a body due to acceleration of non–inertial frame is called fictitious or apparent
r r r
or pseudo force and is given by F = - ma 0 , where a0 is acceleration of non–inertial frame with respect to
an inertial frame and m is mass of the particle or body. The direction of pseudo force must be opposite
to the direction of acceleration of the non–inertial frame.
• When we draw the free body diagram of a mass, with respect to an inertial frame of reference we apply
only the real forces (forces which are actually acting on the mass). But when the free body diagram is drawn
from a non–inertial frame of reference a pseudo force (in addition to all real forces) has to be applied to
r r
make the equation F = ma to be valid in this frame also.

w Man in a Lift
(a) If the lift moving with constant velocity v upwards or downwards. In this case there is no accelerated
motion hence no pseudo force experienced by observer inside the lift.
So apparent weight W´=Mg=Actual weight.
(b) If the lift is accelerated upward with constant acceleration a. Then forces acting on the man
w.r.t. observer inside the lift are
(i) Weight W=Mg downward
(ii) Fictitious force F0=Ma downward.
So apparent weight W´=W+F0=Mg+Ma=M(g+a)
(c) If the lift is accelerated downward with acceleration a<g.
Then w.r.t. observer inside the lift fictitious force F0=Ma acts upward while weight of man
W=Mg always acts downward.
So apparent weight W´=W+F0 = Mg – Ma = M(g–a)
Special Case :
If a=g then W´=0 (condition of weightlessness).
Thus, in a freely falling lift the man will experience weightlessness.
(d) If lift accelerates downward with acceleration a > g . Then as in Case (c). Apparent weight W´=M(g–a) is
negative, i.e., the man will be accelerated upward and will stay at the ceiling of the lift.

FRICTION
Friction is the force between two surfaces in contact which opposes relative motion. or the force of a medium
acting on a moving object. (i.e. air on aircraft.)
Frictional forces arise due to molecular interactions. In some cases friction acts as a supporting force and in
some cases it acts as opposing force.
w Cause of Friction: Friction is arises on account of strong atomic or molecular forces of attraction between the
two surfaces at the point of actual contact.
w Types of friction

Friction

Static friction Dynamic friction


(No relative motion (Kinetic friction)
between objects) (There is relative motion
between objects)

52 E
Pre-Medical : Physics
w Graph between applied force and force of friction
Friction force (f)
Limiting friction
f=fL

Dynamic friction

n
io
f block applied

ct
fri
\\\\\\\\\\\\\\\\\\\\\\\\\\\\\\\\\\\\\\\\\\\\\\\\\\
force

ic
at
St
45° Applied force F
F=fL

fL r r
• Static friction coefficient ms = , 0 £ fs £ msN , fs = - Fapplied
N
( fs )max = m sN = limiting friction

fk r
• Sliding friction coefficient mk = , f k = - ( m kN ) vˆ relative
N
w Angle of Friction (l)
N
Re f L a
of
su n d

l
lta N

fL mN
nt

Applied tan l = = s =µ s
fL N N
force

W
w Angle of repose : The maximum angle of an inclined plane for which a block remains stationary on the
plane.
N

fs

q Mgcosq
s in
Mg Mg
qR
tan qR= ms
• For smooth surface qR= 0

KEY POINTS
• Aeroplanes always fly at low altitudes because according to Newton's III law of motion as aeroplane displaces
air & at low altitude density of air is high.
• Rockets move by pushing the exhaust gases out so they can fly at low & high altitude.
• Pulling a lawn roller is easier than pushing it because pushing increases the apparent weight and hence friction.
• A moongphaliwala sells his moongphali using a weighing machine in an elevator. He gain more profit if the
elevator is accelerating up because the apparent weight of an object increases in an elevator while accelerating
upward.
• Pulling (figure I) is easier than pushing (figure II) on a rough horizontal surface because normal reaction is
less in pulling than in pushing.
F
F
m q q m
m m
Fig. I Fig. II
• While walking on ice, one should take small steps to avoid slipping. This is because smaller step ensure smaller
friction.
• A man in a closed cabin (lift) falling freely does not experience gravity as inertial and gravitational mass have
equivalence.

E 53
Pre-Medical : Physics
Illustration A force of 50 N acts in the direction as shown in figure. The block is
of mass 5kg, resting on a smooth horizontal surface. Find out the
acceleration of the block.

50 3
Solution Horizontal component of the force = 50 sin 60° =
2

component of force in the direction of acceleration


acceleration of the block, a =
mass

50 3 1
= ´ = 5 3 m/s 2
2 5

Illustration A ball of 0.20 kg hits a wall with a velocity of 25 m/s at an angle of 45°. If the mv cosq
ball rebounds at 90° to the direction of incidence, calculate the magnitude of
change in momentum of the ball. v
q
Solution Change in momentum = (– mv cos 45°) – (mv cos 45°) = – 2mv cos 45° q
-mv cosq

r 1 v
Dp = 2mvcos45° = 2 × 0.2 × 25 × = 5 2 N-s
2 q=45°

Illustration Figure shows an estimated force–time graph for a base ball struck by a bat. From this curve,

determine

(a) Impulse delivered to the ball

(b) Average force exerted on the ball.


Solution
(in N)
(a) Impulse = Area under F-t curve

1
= Area of DABC = × 18000 × (2.5 – 1)
2

= 1.35 × 104 kg–m/s

Impulse . ´ 104
135
(b) Average force = = = 9000 N
Time . - 1)
(25

Illustration A 600 kg rocket is set for a vertical firing. If the exhaust speed of gases is 1000 m/s, then
calculate the mass of gas ejected per second to supply the thrust needed to overcome the weight
of rocket.

dm
Solution Force required to overcome the weight of rocket F = mg and thrust needed = v rel
dt

dm dm mg 600 ´ 9.8
so vrel = mg Þ = =
dt dt v rel 1000 = 5.88 kg/s

54 E
Pre-Medical : Physics
Illustration Three blocks of masses m1 = 1 kg, m2 = 1.5 kg and
m3 = 2 kg are in contact with each other on a frictionless
F m2 m3
m1
surface as shown in fig. Find the (a) horizontal force F
needed to push the blocks as a single unit with an
acceleration of 4 m/s² (b) resultant force on each block
and (c) magnitude of contact forces between the blocks.
a a
Solution (a) F = (m1 + m2 + m3) a a
F N1 N1 N2 N2 m3
m2
= (1 + 1.5 + 2) × 4 m1

= 4.5 × 4 = 18 N F - N1 = m1a N1 - N2 = m 2a N2 = m 3a

(b) For m1

F – N1 = m1a = 1 × 4

Þ F – N1 = 4 N ......(i)

for m2,

N1 – N2 = m2 a = 1.5 × 4 = 6

Þ N1 – N2 = 6N ......(ii)

for m3,

N2 = m3a = 2 × 4

Þ N2 = 8N ......(iii)

(c) Contact force between m2 and m3 is N2 = 8 N

and contact force between m1 and m2 is N1 = N2 + 6 = 8 + 6 = 14N.

Illustration A block of mass M is pulled along a horizontal frictionless surface by a rope of mass m as shown
in fig. A horizontal force F is applied to one end of the rope. Find the (i) Acceleration of the
rope and the block (ii) Force that the rope exerts on the block. (iii) Tension in the rope at its
mid point.

M F
m

F
Solution (i) Acceleration a =
(m + M)
(ii) Force exerted by the rope on the block isa

a
M.F T T
M F
T = Ma = L
(m + M)
FBD of block FBD of rope

æm ö
(iii) T1 = ç + M÷ a = æç m + 2M ö÷ æ F ö
çm+M÷ a
è2 ø è 2 ø è ø
L/2 T1
(m + 2M)F M
Tension in rope at midpiont is T1 = m/2
2(m + M)
E 55
Pre-Medical : Physics
Illustration The system shown in fig. is in equilibrium. If the spring balance is calibrated in newtons, what
does it record in each case? (g = 10 m/s²)
Solution

T
T
one weight acts as support
and the other as weight
so tension T = 10 g
= 100 N
T =10 × 10× sin30°
= 10 × 10 ×
T = 2×10 × g = 2 × 10 × 10
= 200 N

Illustration A block of mass 25 kg is raised in two different ways by a


F
50 kg man as shown in fig. What is the action in the two cases ?
If the floor yields to a normal force of 700 N, which mode
should the man adopt to lift the block without the yielding of
the floor ? Mg
Solution Mass of the block, m = 25 kg ; F
Mg
mass of the man, M = 50 kg
Force applied to lift the block mg (a)
mg
(b)
F = mg = 25 × 9.8 = 245 N
Weight of the man,
Mg = 50 × 9.8 = 490 N
(a) When the block is raised by the man by applying a force F in the upward direction, reaction
being equal and opposite to F will act on the floor in addition to the weight of the man.
\ Action on the floor Mg + F = 490 + 245 = 735 N
(b) When the block is raised by the man applying force F over the rope (passing over the pulley) in
the downward direction, reaction being equal and opposite to F will act on the floor against the
weight of the man.
\ Action on the floor Mg – F = 490 – 245 = 245 N
since floor yields to a normal force of 700 N, mode (b) should be adopted by the man to lift the block.
Illustration In the figure blocks A, B and C have accelerations
a1, a2 and a3 respectively. F1 and F2 are external
A B C
forces of magnitudes 2mg and mg respectively.
Find the value of a1, a2 and a3.
Solution

2mg - mg 2m - m g m m m
a1 = =g ; a2 = g=
m 2m + m 3
2m m
F1=2mg
mg + mg - mg g F2=mg
a3 = = ; Clearly a1 > a3 > a2.
2m 2

56 E
Pre-Medical : Physics
Illustration A 12 kg monkey climbs a light rope as shown in fig. The rope passes
over a pulley and is attached to a 16 kg bunch of bananas. Mass and
friction in the pulley are negligible so that the effect of pulley is only to
reverse the direction of force of the rope. What maximum acceleration
can the monkey have without lifting the bananas? (Take g = 10 m/s2)
Solution For Monkey
T – 120 = 12 × a ....(i)
For Bananas
160 – T = N T
N T
Condition for just loosing the contact is N = 0
160 – T = 0 Þ T = 160 ....(ii)
from equation (i) & (ii)
160 – 120 = 12 × a Þ a = 3.33 m/s2 m1g = 120 N m2g = 160 N

Illustration In the system shown in figure all surfaces are smooth, string is massless and inextensible. (in
steady state) Find the
1 kg
T T
(a) acceleration of the system
A
(b) tension in the string and T
(c) extension in the spring if force constant of spring is T
2 B 2 kg
k = 50 N/m (Take g=10 m/s )
Solution (a) 3g – kx = 3a ..... (1)
2g + kx – T = 2a ..... (2)
C 3 kg
T=a ..... (3)
50 –2
\ Acceleration of the system is a = ms
6
a
(b) A T Free body diagram of 1 kg block gives T = ma = (1) æç 50 ö÷ N = 50 N
è 6ø 6

(c) Free body diagram of 3 kg block gives kx

50 C
30 – kx = ma but ma = 3 ´ = 25 N a
6

3g = 30 N
30 - 25 5
x= = = 0.1 m = 10 cm
k 50

Illustration What horizontal acceleration should be provided to the wedge so


that the block of mass m placed on the wedge falls freely?
Solution For free fall, normal reaction should be zero
N = 0
For equilibrium perpendicular to the wedge
0 + masinq = mgcosq

g
Þ a=
tan q

E 57
Pre-Medical : Physics
Illustration A student is able to lift a bag containing books of 20kg-wt by applying a force of 5kg-wt. Find the
mechanical advantage.
Solution W = 20kg-wt
F
F = 5kg-wt

20
M.A. = =4
5

Illustration Calculate the tensions T1, T2 and T3 in the massless strings 20kg
shown in figure (g = 10 ms–2)
30° 60°
Solution Considering the adjoining figure T1 T2
O
T3 = wt. of the 5 kg block (mg)
T3
T3 = 5 × 10 = 50 N
5 kg
Now applying Lami's theorem at point O.
30° 60°
T1 T2 T3
= = 90°
sin(90° + 60°) sin(90° + 30°) sin(180° - 60° - 30°) T1 T2
30° O 60°

T3
T1 T2 50
Þ = =
cos 60° cos 30° sin 90°
5kg
cos 60° cos 30°
T1 = 50 = 25 N and T2 = 50 = 25 3N
sin 90° sin 90° 50N

Illustration A 70 kg man standing on a weighing machine in a 50 kg lift pulls on


the rope, which supports the lift as shown in the figure. Find the force
with which the man should pull the rope to keep the lift stationary.
Also, find the weight of the man as shown by the weighing machine.
Solution Magnitude of tension everywhere in the string is same.
For equilibrium of the lift.

åF y =0 Þ 500 + N = 2T ...(i)

To analyse the equilibrium of the man let us assume him as a block

åF y =0 Þ N + T = 700 ...(ii)

From equations (i) & (ii), we have T = 400 N and N = 300 N


Here, T is the pull of mass and N is the reading of the weighing machine.
58 E
Pre-Medical : Physics
Illustration Length of a chain is L and coefficient of static friction is m. Calculate the maximum length of the
chain which can hang from the table without sliding. N
(L-y)
Solution f
Let y be the maximum length of the chain that can be
W' y
hang outside the table without sliding.
Length of chain on the table = (L – y)

M
Weight of the part of the chain on table W ' = (L - y) g
L

M
Weight of hanging part of the chain W = yg
L

For equilibrium :
limiting force of friction on (L–y) length = weight of hanging part of the chain of y length
mN = W Þ m W' = W

M M mL
r m ( L - y ) g = yg r mL - my = y Þ y = 1 + m
L L

Illustration A block of mass m rests on a rough horizontal surface as shown in figure (a) and (b). Coefficient of
friction between the block and surface is m. A force F = mg act at an angle q with the vertical side
of the block. Find the condition for which the block will move along the surface.

Solution For (a) : normal reaction N = mg – mg cos q, limiting frictional force = mN = m(mg – mg cos q)
Now, block can be pulled when : Horizontal component of force > limiting frictional force
i.e. mg sin q > m(mg – mg cos q)

q q
Þ 2sin cos ³ m(1 - cos q)
2 2

q q q q
Þ 2sin cos ³ 2m sin2 Þ cot ³ m
2 2 2 2

For (b) : Normal reaction N = mg + mg cos q = mg (1 + cos q)

Hence, block can be pushed along the horizontal surface when


horizontal component of force > limiting frictional force
i.e. mg sin q > m mg(1 + cos q)

q q q q
Þ 2sin cos ³ m ´ 2cos2 Þ tan ³ m
2 2 2 2

E 59
Pre-Medical : Physics
Illustration A body of mass m rests on a horizontal floor with which it has a coefficient of static friction m.
It is desired to make the body move by applying the minimum possible force F. Find the magnitude
of F and the direction in which it has to be applied.
Solution Let the force F be applied at an angle q with the horizontal as shown in figure.
For vertical equilibrium,
N + F sin q = mg i.e N = mg – F sin q ......(i)
for horizontal motion
F cos q > fL i.e. F cos q > mN [as fL = mN] ......(ii)
substituting expression for N from equation (i) in (ii),
m mg
F cos q > m(mg – F sin q) r F ³ ......(iii)
(cos q + m sin q)
For the force F to be minimum (cos q + m sin q) must be maximum,
F sinq F
N
d
(cos q + m sin q) = 0 q
dq
F cosq

or – sin q + m cos q = 0 i.e., tan q = m ..... (iv) fL

m 1
sin q =
\ and cos q = mg
1 + m2 1 + m2
substituting these in equation (iii)

m mg m mg
F³ i.e. F ³
1 m2 1 + m2
+
1+m 2
1+m 2

m mg
so that Fmin = with q = tan–1 (m)
2
1+ m

Note : As - A 2 + B2 £ A sin q + B cos q £ A 2 + B 2


µmg
So ( cos q + µ sin q ) max = 1 + µ 2 Therefore Fmin=
1 + µ2
Illustration A block of mass 1kg lies on a horizontal surface of a truck; the coefficient of static friction between
the block and the surface is 0.6, What is the force of friction on the block, if the acceleration of the
truck is 5 m/s2.
Solution Fictitious force (pseudo force) on the block opposite
to the acceleration of the block F = ma = 1 × 5 = 5N

While the limiting friction force


fL = msN = mSmg
= 0.6 × 1 × 9.8 = 5.88 newton
As applied force F is less than the limiting friction force, the block will remain at rest in the truck
and the force of friction will be equal to 5 N and in the direction of acceleration of the truck.

60 E
Pre-Medical : Physics
Illustration In the figure shown mA = 10 kg, mB = 15 kg. Find the maximum value of F, below which the blocks
move together.
m=0.25
A F
smooth
B B

F F
Solution Assuming that both blocks move together, their common acceleration is a C = =
10 + 15 25
FBD of block B (on which no externally applied force acts).
f
B
15F 3F
The required friction force f is equal to f = m B a C = =
25 5
Now, maximum static friction available is fL = mN1 = 0.25(100) = 25 N (here N1 = mAg =100N)
3F 125 125
\ f £ fL Þ £ 25 Þ F £ N Þ Fmax = N
5 3 3
Illustration For the figure shown mA = 10 kg , mB = 15 kg and F = 90 N. Find the accelerations of the blocks
and the frictional forces acting.
m1=0.4
A
m2=0.2
B F

Solution Step1 : Draw the FBD of the combined blocks system.

( f2 )L = m2N2 = 0.2 (25g ) = 50N (Q N2 = 25g )

A
B 90N
f2
Since 90 N > 50N, net unbalanced forces appear and hence movement begins.
Step 2 : Assuming that both the blocks move together, their combined acceleration is
90 - 50
aC = = 1.6 m/s 2
25
Draw the FBD of block A (on which externally applied force does not act).
The force required is f1 = mAaC = 10 × 1.6 = 16 N A
Now, (f1)L = m1N1 = 0.4 (10g) = 40 N. Clearly, f1 < (f1)L f1
\ The frictional force is strong enough to support the combined motion. 10g
\ Common acceleration is aC = 1.6 m/s2 and f1 = 16 N and f2 = 50 N
Illustration A is a 100 kg block and B is a 200 kg block. As shown in figure,
block A is attached to a string tied to a wall. The coefficient of friction
A
between A and B is 0.2 and the coefficient of friction between B and
B
floor is 0.3. Then calculate the minimum force required to move the
2
block B. (take g =10 m/s ).

Solution When B is made to move, by applying a force F, the frictional forces acting on it are f1 and
f2 with limiting values, f1 = (µ S)AmAg and f2 = (µ S)B (mA + mB)g

Then minimum value of F should be (such as to overcome these limiting values),

Fmin = f1 + f2 = 0.2 × 100 g + 0.3 × 300 g = 110 g = 1100 N


E 61
Pre-Medical : Physics
LAWS OF MOTION AND FRICTION EXERCISE
1. When a horse pulls a wagon, the force that causes 6. There are two forces on the 2.0 kg box in the overhead
the horse to move forward is the force view of figure but only one is shown. The figure also
(1) He exerts on the wagon shows the acceleration of the box. The second force
(2) The wagon exerts on him is nearly :-
(3) The ground exerts on him
(4) He exerts on the ground y

2. If the force of gravity suddenly disappears :-


(1) The mass of all bodies will become zero F1=20N
x
(2) The weight of all bodies will become zero
(3) Both mass and weight of all bodies will become
30°
zero
a = 12 m/s2
(4) Neither mass nor weight of all bodies will become
zero
3. The velocity acquired by a mass m in travelling a (1) -20jˆ N
certain distance d starting from rest under the action
of a constant force is directly proportional to :- (2) ( -20iˆ N + 20j)
ˆN

1 (3) ( -32iˆ - 21j)


ˆN
(1) m (2) m° (3) (4) m
m
(4) ( -21iˆ - 16ˆj) N
4. A water jet, whose cross sectional are is 'a' strikes a
wall making an angle ' q' with the normal and 7. A force represented as show in figure acts on a body
rebounds elastically. The velocity of water of density having a mass of 16 kg. The velocity of the body at t
'd' is v. Force exerted on wall is :- = 10 s, if the body starts from rest :-

F(N)
V
q 75
q
50
V

(1) 2 av2d cosq (2) av2d sinq t(s)


0 5 10
(3) 2 avd cosq (4) avd cosq
(1) 100 m/s (2) 50 m/s
5. Three forces act on a particle that moves
(3) 49 m/s (4) 39 m/s
r
with unchanging velocity v = (3iˆ - 4ˆj) m/s. 8. A player catches a ball of 200 g moving with a
r speed of 20 m/s. If the time taken to complete
ˆ and
Two of the forces are F1 = (3iˆ + 2jˆ - 4k)N
the catch is 0.5 s, the force exerted on the player's
r hand is :-
ˆ . The third force is :-
F2 = (-5iˆ + 8ˆj - 3k)N
(1) 8 N (2) 4 N (3) 2 N (4) 0
ˆ
(1) ( -2iˆ + 10jˆ - 7k)N 9. A block of metal weighing 2 kg is resting on a
frictionless plane. If struck by a jet releasing water at
ˆ
(2) (2iˆ - 10jˆ + 7k)N a rate of 1 kg/s and at a speed of 5 m/s. The initial
acceleration of the block will be :-
(3) (7iˆ - 2k + 10j)N
ˆ
(1) 2.5 m/s2 (2) 5.0 m/s2
(4) none of these (3) 10 m/s2 (4) None of the above

62 E
Pre-Medical : Physics
10. A disc of mass 1·0 kg is kept floating horizontally in 14. A string of negligible mass going over a clamped pulley
air by firing bullets of mass 0·05 kg each vertically of mass m supports a block of mass M as shown in
at it, at the rate of 10 per second. If the bullets the figure. The force on the pulley by the clamp is
rebound with the same speed, the speed with which given by :-
these are fired will be–

(1) 2 Mg (2) 2 mg

(1) 0·098 m/s (2) 0·98 m/s (3) ( )


(M + m)2 + m2 g (4) ( )
(M + m)2 + M2 g
(3) 9·8 m/s (4) 98·0 m/s
15. Three blocks of masses 4 kg, 6 kg and 8 kg are bang-
11. If force F = 500 – 100t, then impulse as a function
ing over a fixed pulley as shown. The tension in the
of time will be :- the string connecting 4 kg and 6 kg block is :-
(1) 500t – 50t2 (2) 50t – 10 (g = 10 m/s 2)
(3) 50 – t 2
(4) 100t 2 (1) 4 N
(2) 6 N
12. Find the tension in the string which connected the
blocks as shown in the following figure :- 320
(3) N 6kg
9
10N 8kg

60° 40 4kg
2kg 3kg (4) N
9
16. Figure shows four blocks that are being pulled along
(1) 2 N (2) 3 N (3) 5 N (4) 10 N
a smooth horizontal surface. The masses of the blocks
13. A block of mass M = 8 kg is connected to an empty and tension in one cord are given. The pulling force F
is :-
bucket of mass 1 kg by a massless cord running over
an ideal pulley . The coefficients of static and kientic F
60° 3N 2kg
friction between table top and block are 0.5 and 0.4 4kg 3kg 1kg
respectively. Sand is gradually added to the bucket
until the block just begin to slide. The mass of sand (1) 5 N (2) 10 N (3) 12.5 N (4) 20 N
added is : (g = 10 m/s2) 17. Two masses, M and m are connected together by a
pulley two strings and a stretched spring of force
constant k as shown. Assume that string, pulley and
spring all are massless and surface below m is smooth.
8kg The amount by which the spring is stretched:-
k
m

mMg 2mMg
(1) (2)
k(m + M) k(m + M)

(1) 5 kg (2) 6 kg (3) 7 kg (4) 10 kg Mg (m + M)g


(3) (4)
k 2k
E 63
Pre-Medical : Physics
18. A block of mass M is pulled along a horizontal smooth 22. Two masses m1 and m2 are joined by a spring as
surface by a rope of mass m. Force P is applied at shown. The system is dropped to the gorund from
one end of the rope. The force which the rope exert a certain height. The spring will be :-
on the block is :-
(1) Stretched when m2 > m1
P PM m1
(1) (2) (2) compressed when m2 < m1
(M - m) (M + m)
(3) neither compressed nor stretched only
Pm m when m1 = m2
(3) (4) P
(M + m) M (4) neither compressed nor stretched m2
regardless of the values of m1 and m2.
19. Two blocks, each having a mass M, rest on frictionless
23. The elevator shown in figure is descending, with
surface as shown in the figure. If the pulleys are light
and frictionless, and M on the incline is allowed to an acceleration of 2 ms–2. The mass of the block
move down, then the tension in the string will be :- A is 0.5 kg. The force exerted by the block A on
the block B is :

M
q M

(1) 2.3 Mg sin q (2) 1.3 Mg sin q


Mg (1) 2 N (2) 4 N (3) 6 N (4) 8 N
(3) sin q (4) 2 Mg sin q
2
24. A body kept on a smooth inclined plane of inclination
20. With what minimum acceleration can a firemman 1 in x will remain stationary relative to the inclined
slide down a rope whose breaking strength is 3/4th of plane if the plane is given a horizontal acceleration
his weight. equal to :-

g x2 - 1
(1) m / s2 (2) g m/s2 (1) (2) g
4 x 2 - 1g
x
3 gx g
(3) m / s2 (4) zero (3) (4)
4 2
x -1 2
x -1
21. A metal sphere is hung by a string fixed to a wall. 25. If a parrot starts flying upwards with an acceleration
The forces acting on the sphere are shown in fig. in an air tight cage, then the boy will feel the weight
Which of the following statements is correct ? of the cage:
(1) Unchanged (2) Reduced
(3) Increased (4) Nothing can be said
26. Two masses of 10 kg and 20 kg respectively are
connected by a massless spring as shown in the
figure. A force of 200 N acts on the 20 kg mass.
At the instant shown the 10 kg mass has an
acceleration 4 m/s 2 rightwards. What is the
acceleration of 20 kg mass ?
r r r
(a) R + T + W = 0 (b) T2 = R2 + W2

(c) T = R + W (d) R = W tan q


(1) a, b, c (2) b, c, d
(3) a, b, d (4) a, b, c, d (1) Zero (2) 10 m/s2
(3) 4 m/s2 (4) 8 m/s2

64 E
Pre-Medical : Physics
27. The pulley arrangements shown in the figure are 30. Adjoining figure shows two blocks A and B pushed
identical, the mass of the rope being negligible. In against the wall with a force F. The wall is smooth
case (a) mass m is lifted by attaching a mass of 2m but the surfaces in contact of A and B are rough.
to the other end of the rope. In case (b) the mass Which of the following is true for the system of blocks
m is lifted by pulling the other end of the rope with to be at rest against the wall ?
a constant downward force F = 2mg, where g is
the acceleration due to gravity. The acceleration
of mass m in case (a) is :-

(1) F should be be more than the weight of A and B


(2) F should be equal to the weight of A and B
(3) F should be less than the weight of A and B
(4) system cannot be in equilibrium
31. In the set-up shown, a 200 N block is supported
in equilibrium with the help of strings and a spring.
all knotted at ponit O. Extension in the spring is 4
(1) Zero cm. Force constant of the spring is closest to [g =
(2) More than that in case (b) 10 m/s2]
(3) Less than that in case (b)
(4) Equal to that in case (b)
37° 53°
28. A block is dragged on a smooth plane with the
help of a rope which moves with a velocity v as
shown in figure. The horizontal velocity of the block
is : O

200 N

(1) 30 N/m (2) 2500 N/m


(3) 3000 N/m (4) 4000 N/m
32. A lift of mass 100 kg starts moving from rests in the
upward direction. Fig shows the variation if speed
v v of the lift, T1, T2 and T3 stand for tension in the
(1) v (2) (3) v sinq (4)
sin q cos q rope form zero to two seconds, two to six seconds,
29. In the fig., the ends P and Q of an unstrechable six to seven seconds respectively then :-
string move downward with uniform speed v. Mass
M moves upwards with speed.

v v
t(s)

(1) T1 : T2 : T3 :: 1 : 1 : 1
(2) T1 : T2 : T3 :: 6 : 5 : 3
(1) v cosq (2) v/cosq (3) T1 : T2 : T3 :: 3 : 5 : 6
(3) 2v cosq (4) 2/v cosq (4) T1 : T2 : T3 :: 6 : 5 : 6

E 65
Pre-Medical : Physics
33. All surfaces are assumed to be frictionless. Calculate 36. Same spring is attached with 2 kg, 3 kg and
the horizontal force F that must be applied so that 1 kg blocks in three different cases as shown in
m1 and m2 do not move relative to m3 is :– figure. If x1, x2 and x3 be the respective extensions
in the spring in these three cases, then :-

m2g m2g
(1) (m1 + m2 + m3) (2) (m1 + m2)
m1 m1

2kg 2kg 3kg 2kg 1kg 2kg


m1 g m1 g
(3) (m2 + m3) m (4) (m1 + m3) m
2 2

34. Two masses of 1 kg and 2 kg are attached to the (1) x 1 = 0, x 3 > x 2


ends of a massless string passing over a pulley of (2) x 2 > x 1 > x 3
negligible weight. The pulley itself is attached to a
(3) x 3 > x 1 > x 2
light spring balance as shown in figure. The masses
(4) x 1 > x 2 > x 3
start moving; during this interval the reading of spring
balance will be:- 37. Two weights w1 and w2 are connected by a light
thread which passes over a light smooth pulley. If
(1) More than 3 kg.
the pulley is raised upwards with an acceleration
(2) Less than 3 kg. equal to g, then the tension in the thread will be :-
(3) Equal to 3 kg.
1kg
2w1w2 w1w 2
(4) None of the above 2kg (1) (2)
w1 + w2 w1 + w2
35. As shown in figure a monkey of 20 kg mass is
holding a light rope that passes over a frictionless
pulley. A bunch of bananas of the same mass 20
4w1w2 4w1w2
(3) (4)
kg is tied to the other end of the rope. In order to w1 + w2 w1 - w2
get access to the bananas the monkey starts
38. Three blocks of masses m, 3m and 5m are
climbing the rope. The distance between the
connected by massless strings and pulled by a force
monkey and the bananas :-
F on a frictionless surface as shown in the figure
below. The tension P in the first string is 16N.

F P Q
m 3m 5m

If the point of application of F is changed as given below,


the values of P' and Q' shall be :-

P' Q'
m 3m 5m F

(1) Decreases (1) 16N, 10N (2) 10N, 16N


(2) Increases
(3) 8 N, 2N (4) None of these
(3) remains unchanged
(4) Nothing can be stated

66 E
Pre-Medical : Physics
39. A 0.5 kg ball moving with a speed of 48 m/s strikes 42. A rigid ball of mass m strikes a rigid wall at 60° and
a hard wall at an angle of 30° with the wall. It is gets reflected without loss of speed as shown in the
reflected with the same speed and at the same angle. figure below. The value of impulse imparted by the
If the ball is in contact with the wall for wall on the ball will be :-
0.5 seconds, the average force acting on the wall m
V/2
is :-
30°

30°
V/2

mV mV
(1) 2mV (2) mV (3) (4)
2 3
(1) 24 N (2) 48 N (3) 72 N (4) 96 N
43. Two blocks A and B of masses m and 3m
40. A person used force (F), shown in figure to move a respectively are connected by a massless and
load with constant velocity on a given surface. inextensible string. The whole system is suspended
Identify the correct surface profile :- by a massless spring as shown in figure. The
magnitudes of acceleration of A and B immediately
F after the string is cut, are respectively :-
O
X
L

(1)
A m

B 3m
(2)
g
(1) 3g, g (2) g,g (3) 3g, 3g (4) g
3
(3)
44. A force F = kt is applied to a block A as shown in
figure, where t is time in second. The force is applied
at t = 0, when the system was at rest. Which of the
(4) following graphs correctly gives the frictional force on
block A as a function of time ?
41. The force 'F' acting on a particle of mass 'm' is F=kt
A
indicated by the force-time graph shown below. The
change in momentum of the particle over the time µs > µk
interval from zero to 6 s is :- f f

6
3 (1) (2)

0 t t
F (N)

2 4 6 8
–3 f f
t (s)
(1) 24 Ns (2) 12 Ns (3) (4)
(3) 6 Ns (4) 3 Ns t t

E 67
Pre-Medical : Physics
45. A 20 kg body is pushed with just enough force to 49. In the figure given, the system is in equilibrium.
start it moving across a floor and the same force What is the maximum value W can have if the
continues to act afterwards. The coefficient of static friction force on the 40 N block cannot exceed
and kinetic friction are 0.6 and 0.2 respectively. The 12.0 N ?
acceleration of the body is :- (g = 10 m/s 2)
(1) 1 m/s2 (2) 2 m/s2 30
0

(3) 3 m/s2 (4) 4 m/s2


40 N

46. A 2.5 kg block is initially at rest on a horizontal surface.


r W
A 6.0 N horizontal force and a vertical force P are
applied to the block as shown in figure. (1) 3.45 N (2) 6.92 N
The coefficient of friction for the block and surface is (3) 10.35 N (4) 12.32 N
0.4. The magnitude of friction force when P = 9N (g 50. A block of mass m is in contact with the cart C as
= 10 m/s2) shown in the figure. The coefficient of static friction
between the block and the cart is µ. The acceleration
®
P a of the cart that will prevent the block from falling
statisfies :-
6.0 N a

C m
(1) 6.0 N (2) 6.4 N (3) 9.0 N (4) zero

47. A block rests on a rough inclined plane making an


g mg
angle of 30° with the horizontal. The coefficient of (1) a < (2) a >
m m
static friction between the block and the plane is 0.8.
g g
If the frictional force on the block is 10 N, the mass (3) a > (4) a ³
mm m
of the block (in kg) is (take g = 10 m/s2) :-
51. Which one of the following statements is
(1) 2.0 (2) 4.0 (3) 1.6 (4) 2.5 incorrect ?
48. A block of mass m lying on a rough horizontal plane (1) Rolling friction is smaller than sliding friction
is acted upon by a horizontal force P and another (2) Limiting value of static friction is directly
force Q inclined at an angle q to the vertical. The proportional to normal reactions
block will remain in equilibrium if the coefficient (3) Frictional force opposes the relative motion
of friction between it and the surface is :- (4) Coefficient of sliding friction has dimensions of
force
52. A block of mass m is placed on a smooth inclined
Q q wedge ABC of inclination q as shown in the figure.
The wedge is given an acceleration 'a' towards the
P right. The relation between a and q for the block
to remain stationary on the wedge is :-
A
m
P + Q sin q P cos q + Q
(1) (2) a
mg + Q cos q mg - Q sin q q
C B
g g
P + Q cos q P sin q + Q (1) a = (2) a =
(3) (4) cosec q sin q
mg + Q sin q mg - Q cos q
g g
(3) a = (4) a =
tanq cot q
68 E
Pre-Medical : Physics
53. A block A of mass m1 rests on a horizontal table. 56. A block slides with constant velocity on a plane
inclined at an angle q. The same block is pushed
A light string connected to it passes over a frictionless
up the plane with an initial velocity v0. The distance
pulley at the edge of table and from its other end covered by the block before coming to rest is :-

another block B of mass m2 is suspended. The v 20 v 20


(1) (2)
coefficient of kinetic friction between the block and 2g sin q 4g sin q

the table is µ k. When the block A is sliding on the


v 20 sin2 q v 20 sin2 q
(3) (4)
table, the tension in the string is :- 2g 4g

(m2 – mk m1 )g m1m2 (1 + m k )g
(1) (2) 57. m=0.3
(m1 + m2 ) (m1 + m2 ) 4kg

Smooth 10kg F
m1m2 (1 – m k )g (m2 + mk m1 )g
(3) (m1 + m2 ) (4) (m + m )
1 2 Find maximum force for which both the block will
54. Three blocks, of masses m1 = 2.0, m2 = 4.0 and move with same acceleration.
m3 = 6.0 kg are connected by strings on a (1) 12N (2) 24N
frictionless inclined plane of 600, as shown in the (3) 36N (4) 42N
figure. A force F = 120 N is applied upwards along 58. Find the minimum value of m that should placed on
the incline to the uppermost block, causing an 10kg. So that system remain at rest.
upward movement of the blocks. The connecting
cords are light. The values of tensions T1 and T2 in
the cords are m
m=0.5 10kg
F
m3
T2
m2
T1
m1

0
60 10kg

(1) T1 = 20 N, T2 = 60 N
(2) T1 = 60 N, T2 = 60 N (1) 5kg (2) 10kg
(3) T1 = 30 N, T2 = 50 N (3) 15kg (4) 20kg
(4) T1 = 20 N, T2 = 1 00 N 59. Find acceleration of block A and B. If m1 = 0.8 and
55. A given object takes n times as much time to slide m2 = 0.4
down a 450 rough incline as it takes to slide down a
perfectly smooth 450 incline. The coefficient of A
m1 =0.8
kinetic friction between the object and the incline is 5kg F=50N
given by : m2 =0.4 10kg B
æ 1 ö æ 1 ö
(1) ç 1 - 2 ÷ (2) ç 2 ÷
è n ø è1 - n ø (1) 0 m/s2 and 0 m/s2
(2) 1 m/s2 and 1/2 m/s2
æ 1 ö æ 1 ö (3) 2 m/s2 and 0 m/s2
(3) ç1 - 2 ÷ (4) ç 2 ÷
è n ø è1 - n ø (4) 1/2m/s2 and 1/4 m/s2

E 69
Pre-Medical : Physics
60. Two blocks (A) 2 kg and (B) 5 kg rest one over the 61. Two blocks of masses M1 = 4 kg and M2 = 6 kg are
other on a smooth horizontal plane. The coefficient connected by a string of negligible mass passing
of static and dynamic friction between (A) and (B) is over a frictionless pulley as shown in the figure
the same and equal to 0.60. The maximum below. The coefficient of friction betwen the block
horizontal force F that can be applied to (B) in order M1 and the horizontal surface is 0.4. When the
that both (A) and (B) do not have any relative system is released, the masses M1 and M2 start
motion is : accelerating. What additional mass m should be
placed over M1 so that the masses (M1 + m) slide
with a uniform speed ?
2 kg
m
5 kg
M1

(1) 42 N (2) 42 kgf


M2
(3) 5.4 kgf (4) 1.2 N

(1) 12 kg (2) 11 kg (3) 10 kg (4) 9 kg

ANSWER KEY
Que. 1 2 3 4 5 6 7 8 9 10 11 12 13 14 15
Ans. 3 2 3 1 2 3 4 1 1 3 1 1 3 4 3
Que. 16 17 18 19 20 21 22 23 24 25 26 27 28 29 30
Ans. 4 1 2 3 1 3 4 2 4 3 4 3 2 2 4
Que. 31 32 33 34 35 36 37 38 39 40 41 42 43 44 45
Ans. 3 2 1 2 3 2 3 4 2 1 3 3 1 3 4
Que. 46 47 48 49 50 51 52 53 54 55 56 57 58 59 60
Ans. 1 1 1 2 4 4 4 2 1 1 2 4 2 3 1
Que. 61
Ans. 2

70 E
Pre-Medical : Physics
WORK, POWER AND ENERGY
WORK
Work is said to be done by a force when the force produces a displacement in the body on which it acts in any
direction except perpendicular to the direction of the force.
Work Done by Constant Force
Work is the product of the applied force and the displacement of the body in the direction of force.
r
W = F s cosq = F.sr
r r rr
l If F = Fxˆi + Fyˆj + Fz kˆ and s = xiˆ + yjˆ + zkˆ Then W = F.s = Fx x + Fy y + Fz z
Dimension : M1L2T–2 UNIT S I : joule C.G.S. : erg 1 joule = 107 erg
Work Done by a Variable Force
When magnitude and direction of the force varies with position or time, the work done by force for infinitesimal
r r
displacement ds is dW = F.ds
Br r B
The total work done for displacement from A to B is WAB = òA F.ds = òA (F cos q)ds B

In terms of rectangular components


r r ds q F
F = Fxˆi + Fyˆj + Fz kˆ , ds = dxiˆ + dyjˆ + dzkˆ
B x
ˆ
WAB = ò (Fxˆi + Fy ˆj + Fz k).(dxi ˆ = B F dx + yB F dy + zB F dz
ˆ + dyjˆ + dzk)
A
xA
ò x ò y ò z
yA zA A
(i) Work done by Stretching Force:

x 2

W= ò F cos q dx
x 1

l Work done to stretch spring by displacement x from normal length


x1 = 0 x2 = x and q = 0°
x

W = ò F dx
0

Q F = kx W = ò kx dx
0

1 2
W= kx
2
l Work done to stretch spring from displacement x1 to x2
Then x1 = x1 x2 = x 2 and q = 0°
x2
W= ò F dx
x1

x2
Q F = kx W= ò kx dx
x1

1
W= k ( x 22 - x 12 )
2

E 71
Pre-Medical : Physics
w Nature of work done : Although work done is a scalar quantity, yet its value may be positive, negative or
even zero

Negative work Zero work Positive work


F F
F
q
q S
S q
(q >90°) S
(q = 90°)
S (q < 90°)

F mg
S
f S

Work done by
Motion of particle mg
friction force
on circular path (uniform)
(q= 180°)
( q= 90°) Motion under gravity
(q= 0°)
S
N
f
B F
F=2.5 N A
mg fmax=10N
f=friction force
Work done by gravity Work done by friction
(q = 180°) mg = 100 N force on block A
(q = 0°)
As f = F, hence S = 0

Graphical Method to Calculate Work Done

If force displacement (F – x) curve is given then net area under F – x curve is equal to work done.
W = W1 + (–W2)

W 1(+ve)

W 2(–ve) x®

Facts:
l Work is defined for an interval or displacement, there is no term like instantaneous work similar to instantaneous
velocity.
l For a particular displacement work is independent of time, work will be same for same displacement whether the
time taken is small or large.
l When several forces act, work by a force for a particular displacement is independent of other forces.
l Displacement depends on reference frame so work done by a force is reference frame dependent so work done by a
force can be different in different reference frame.
l Work is done by the source or agent that applies the force.
l When q = 0º : A force does maximum positive work.
l When q = 180° A force does maximum negative work.

72 E
Pre-Medical : Physics
POWER
Power of a body is defined as the rate at which the body can do the work.

work W
Average Power Power = or P=
time t
W is the amount of the work done in total time t.
Instantaneous Power (P)
Instantaneous power is the power at any given instant. Suppose an agent does an infinitesimally amount of work
dW in an infinitesimally time dt then,
r r
dW F.ds r r r dsr rr
P= or P= (Q dW = F.ds) or P = F. or P = F.v
dt dt dt

æ joule ö
Dimensions : [M 1L2T –3] SI UNIT : watt = ç second ÷
è ø

Some other UNITS of power : A convenient rule of thumb


1 kilo watt (kW) = 1000 watt = 10 W 3 approximation is
1 mega watt = 1000,000 watt or 1MW = 103 kW
1hp ~~ 3 kW
1 horse power (h.p.) = 746 W = 550 ft. lb/s 4
FPS UNIT : (foot × pound)/second, 1W = 0.738 ft. lb/s
Efficiency
Machines are designed to convert energy into some of the useful work however, because of frictional effects, the
work performed by the machine is always less than the energy put into machine.
work done
The efficiency (h) of a machine h=
energy input

FACTS:
l When an agent delivers power at a uniform rate, the average power is equal to the instantaneous power.
l Power is the ratio of two scalars (Also' it is the scalar product of two vectors) so power is scalar quantity.
l When the time taken to complete a given amount of work is important we measure the power of agent doing work.
l The slope of work – time graph gives the instantaneous power

dW
slope = tanq = = P (instantaneous power)
dt R S
D C
l Area under power – time graph gives the work done.
Area under P–t graph = Area ABCD P

= å(PR × PQ) = å(P × dt) A dt B


P Q t
Area under power – time graph = ò Pdt = W

CONSERVATIVE AND NONCONSERVATIVE FORCE


Conservative Force
A force is said to be conservative if work done by the force on a particle moving between two points does not
depends on the path taken by the particle.

Nonconservative Force
A force is said to be non conservative, if work done by the force or against the force in moving a body from one
position to another, depends on the path.

E 73
Pre-Medical : Physics
Difference Between Conservative and Non-conservative Forces
Conservative force Non-conservative force
(i) Work done does not depend upon path (i) Work done depends upon path
(ii) Work done in a round trip is zero (ii) Work done in a round trip is not zero
(iii) When only a conservative force acts within (iii) Work done against a non-conservative
a system, the kinetic energy and potential energy force may be dissipated as heat energy.
can change. However their sum, the mechanical
energy of the system does not change.
(iv) Work done is completely recoverable. (iv) Work done is not completely recoverable.G

Example of Conservative Force and Non-conservative Force


Conservative force Non-conservative force
(i) Gravitational force. (i) Force of friction.
(ii) Electrostatic force. (ii) Viscous force.
(iii) Spring or restoring force. (iii) Force due to air resistance.
(iv) Intermolecular force. (iv) Magnetic force due to current carring element.
(v) Magnetic force produced by bar magnet.

In all 3 cases work done by gravity is same irrespective of their different paths. (W = –mgh)
Central Force
The forces acting along the line joining the particles are called as central force.
r
Fcentral = C.F(r). r̂ [C = constant, F(r) = function of position, r̂ = direction]
eg. Gravitational force, electrostatic force

FACTS:
l All central forces are conservative but all conservative forces are not central forces.
l The concept of potential energy exists only in the case of conservative force.
ENERGY
Energy is defined as the internal capacity of doing work.
When we say that a body has energy it mean that it can do work.
(i) Different forms of energy :
Mechanical energy, electrical energy, optical (light) energy, acoustical (sound)energy, molecular energy, atomic
and nuclear energy.
These forms of energy can change from one form to the other.
(ii) Mass energy relation
According to Einstein mass energy equivalence principle, mass and energy are inter convertible i.e. they can
be changed into each other.
Equivalent energy corresponding to mass m is E = mc 2
where, m : mass of the particle c : speed of light
l Energy is a scalar quantity
l Dimensions : [M1L2T–2]
l SI UNIT : joule
Other units 1 erg = 10–7 joule 1kWh = 36 × 105 joule
1 eV = 1.6 ×10 joule
–19
1 cal = 4.2 joule
In mechanics we only concerned with mechanical energy which is of two type.
(a) kinetic energy (b) Potential energy
74 E
Pre-Medical : Physics
Kinetic Energy
Kinetic energy is the internal capacity of doing work of the object by virtue of its motion.
or
K.E. of a body can be calculated by the amount of work done in stopping the moving body. or from the amount of
work done in giving the present velocity to the body from the state of rest.
If a particle of mass m is moving with velocity 'v' much less than the velocity of the light then the kinetic energy K.E.

1
is given by K.E. = mv2
2

Relation between K.E. (K) and linear momentum (p) :


2
1 1 p
p = mv and K= mv2 = (m2v2) =
2 2m 2m

p2
K= or p = 2mK
2m
Graph

p2 1
(i) K.E. = if p = constant. (ii) K.E. = mv2
2m 2
K.E. K.E.
1
K.E. µ K.E. µ v 2
m

1
yµ y µ x2
x m v

K.E. K.E.
p2
(iii) K.E. µ v2 (iv) K.E. =
2m
yµx K.E. µ p2
v
2
y µ x2 P

K.E.
(v) K.E. µ p2 (vi) 2 m K.E. = p2
yµx 2mK.E. = p

K.E. µ p
2
P
1 K.E.

K.E. µ 1
p

1
yµ 1
x
P

E 75
Pre-Medical : Physics
Questions based on % change:-
Case-I: change £ 5%

P2 DK 2DP
(i) K.E. = Þ K.E. µ P2 Þ =
2m K P

DP 1 DK
(ii) P= 2mK.E. Þ P µ K.E. Þ P µ ( K.E.)
1
2 Þ =
P 2 K
Case-II: More than 5% (³ 5%): Here E = Kinetic Energy

DE éæ P2 ö ù
2
DE é P22 - P12 ù = ê - 1ú ´ 100
EµP Þ E =ê
2
2 ú ´ 100 Þ E
ç ÷
êëè P1 ø úû
ë P1 û

DP é E 2 - E1 ù DP é E 2 ù
Pµ E Þ =ê ú ´ 100 Þ P = ê E - 1ú ´ 100
P êë E1 êë 1 úû
ûú
Work Energy Theorem
Work done by all forces (Net force) = change in K.E. (W = DK.E.)
Proof:
(i) For constant force u v
F m m
v2 = u2 + 2as
S
F
v2 – u2 = 2 s F.S = (v2 – u2)m
m

F.S = DK.E. W = DK.E.


(ii) For variable force

æ vdv ö
W= ò F dx = ò ma dx = ò m çè dx ÷ødx
v v
é v2 ù
W= ò mvdv
u
W= m ê ú
ë 2 ûu

m 2
W= é v - u2 ù W = DK.E.
2ë û
FACTS:
l If there is no change in the speed of a particle, there is no change in kinetic energy. So work done by the resultant
force is zero.
l If K.E. of the body decreases then work done is negative i.e. the force opposes the motion of the body.
l If K.E. of the body increases then work done is positive.
l In above discussion, we have assumed that the work done by the force is effective only in changing the kinetic energy
of the body. It should however be remembered that work done on a body may also be stored in the body in the
form of potential energy.
r r
l As mass m and v 2 or v. v are always positive so K.E. can never be negative.
l The kinetic energy depends on the frame of reference.
l Work done on the body is the measure of K.E. of the body.
1
l The expression K.E. = mv2 holds even when the force applied varies in magnitude or in direction or in both.
2

76 E
Pre-Medical : Physics
Potential Energy
The energy possesed by any body or a system is due to change in its position, shape or configuration is called as
potential energy.
or
Work done by external force against conservative force is equal to potential energy of that system.

dU d d
Potential energy gradient : = w ex = Fr cos q
dr dr dr
For conservative force q = 180º

dU dU
so =-F FC = –
dr dr

U2 r2
r r
ò dU = - ò F .dr
U1 r1
C U2 – U1 = –Wc DU = -Wc or DU = Wex

FACTS:
l It is a scalar quantity.
l It can be positive or negative or zero.
l It is a relative quantity & it depends on frame of reference but change in potential energy is independent of frame
of reference.
l Potential energy is always defined only for conservative force fields.
l Whenever work is done by the conservative forces potential energy decreses & whenever work is done against conservative
forces, potential energy increases.
l It is function of position doesn't depend on path.

TRANSLATIONAL EQUILIBRIUM
A system is said to be in translational equilibrium, if net force on that system is zero.
Types of equilibrium
Stable equilibrium Unstable equilibrium Neutral equilibrium

A
C

dF dF dF
(F = 0, < 0) (F = 0, > 0) (F = 0, = 0)
dx dx dx

dU d2 U dU d 2U dU d2 U
=0 & >0 =0& <0 =0& =0
dx dx2 dx dx2 dx dx2

Umin (point C) Umax (point B) Uconstant (point A)

E 77
Pre-Medical : Physics
LAW OF CONSERVATION OF ENERGY
Energy neither be created nor be destroyed, it only can be converted from one form to other form
Law of Conservation of Mechanical Energy
Under the effect of "conservative force" mechanical energy of the system always remains constant. This is called
law of conservation of mechanical energy.
M.E. = K.E. + U = constant

DK + DU = 0 DK = -DU
(i) For gravitational field
Let a ball of mass m is dropped from position (3) (as shown in figure)
At point 1: PE = 0

1
KE = mv 2 Q v = 2gh So, KE = mgh
2

At point 2:

1
PE = mgx KE = mv 2
2 P.E = 0
Q v = 2g(h - x) KE = mg(h – x)

At point 3: PE = mgh KE = 0
so during motion of ball at any position

PE = mgx KE = mg(h – x)

ME(1) = ME(2) = ME(3)

Spring Section

R.L.

R.L. [R.L. = Reference level]

R.L.

Step (1)- Reference level at the free end of natural length of spring.
Step (2)- If x is extension/compression in spring then spring force is in the opposite direction of
extension/compression and proportional to x.
ƒS µ – x
ƒS = –Kx

where K = spring force constant or stiffness constant.

1 2
Step(3)- Potential energy of spring. PE = Kx
2
78 E
Pre-Medical : Physics
Special cases:
(i) A particle of mass m is freely released from height h on a
spring, spring is compressed by x then

K
1 2 K
mg(h + x) = Kx
2

(ii) If body of mass m moving with speed v collides with a spring and spring is compressed by x.

1 1
mv 2 = Kx 2max v
2 2
m K
m
x max = v
k

(iii) If body is released from hight h and collide with spring .


m
1 2
mgh = Kx max h
2 K

2mgh
Þ = xmax
K
(iv) If block is attached with lower end of spring fixed at upper end in vertical position.
l Equilibrium condition: Fnet = 0

mg – Kx = 0
K
mg Kx
=x
K m
mg
l Maximum extension:
Decrease in G.P.E. of block = increase in P.E. of spring

1 2 2mg
mgxmax = Kx max = x max
2 K

Illustration A position dependent force F=7–2x+3x2 acts on a small body of mass 2 kg and displaces it from
x = 0 to x = 5 m. Calculate the work done in joules.

X2 5 5
é 2x2 3x3 ù
Solution W = ò Fdx = ò (7 - 2x + 3x )dx = ê7x -
2
+ ú = 135 J.
X1 0 ë 2 3 û0

Illustration Corresponding to the force-displacement diagram shown in adjoining diagram, calculate the work
done by the force in displacing the body y

from x=1 cm to x=5 cm. 20


10
Solution Work = Area between the curve F 0
(dyne) 1 2 3 4 5 6 7 8 x (cm)
and displacement axis -10
-20
= 10 + 20 – 20 + 10 = 20 ergs.
E 79
Pre-Medical : Physics
Illustration A 10 kg block placed on a rough horizontal floor is being pulled by a constant force of 50 N.
Coefficient of kinetic friction between the block and the floor is 0.4. Find the work done by each
individual force acting on the block over a displacement of 5 m.

Solution Forces acting on the block are (i) its weight (mg = 100 N),

mg = 100 N

F = 50 N
Dx = 5 m

f = 40 N

N = 100 N

(ii) normal reaction (N = 100 N) by the ground, (iii) force of kinetic friction (f = 40 N) and

(iv) the applied force (F = 50 N). All these force and the displacement of the block are shown
in the figure.
r r
All these forces are constant forces, therefore we use equation Wi® f = F × D r .

r r
Work done Wg by the gravity i.e. weight of the block Wg = 0 (Q mg ^ Dx )
r
Work done WN by the normal reaction WN = 0 (Q N ^ Dxr )
r r
Work done WF by the applied force (
WF = 250 J Q F P Dx )
r
Work done Wf by the force of kinetic friction Wf = – 200 J. (Q f ­¯ Dxr )
r
Illustration Calculate the work done by the force F = ( 3iˆ - 2jˆ + 4kˆ ) N in carrying a particle from point

(-2 m,1 m,3 m) to point (3 m,6 m,-2 m).


r r r
Solution The force F is a constant force, therefore we can use equation Wi® f = F × D r .

ur r
W = F . D r = (3iˆ - 2 ˆj + 4k).(5i
ˆ ˆ + 5ˆj - 5k)
ˆ = –15 J.

80 E
Pre-Medical : Physics
Illustration A 5 kg ball when falls through a height of 20 m acquires a speed of 10 m/s. Find the work done
by air resistance.
Position 1
Solution The ball starts falling from position 1, where its speed is zero;
hence, kinetic energy is also zero.
R
Free body diagram of
K1 = 0 J ...(i)
the ball at some
h intermediate position
During the downward motion of the ball, constant gravitational mg
force mg acts downwards and air resistance R of unknown
magnitude acts upwards as shown in the free body diagram. The
ball reaches position 20 m below the position-1 with a speed
Position 2
v = 10 m/s, so the kinetic energy of the ball at position 2 is
v
K 2 = 12 mv 2 = 250 J ...(ii)
Work done by gravity
Wg,1®2 = mgh = 1000 J ...(iii)

Denoting the work done by the air resistance as WR,1®2 and making use of eq. (i), (ii) and (iii) in
work-kinetic energy theorem, we have
W1®2 = K 2 - K1 Þ Wg,1®2 + WR,1®2 = K 2 - K1 Þ WR,1®2 = -750 J .

Illustration A box of mass m = 10 kg is projected up an inclined plane from its foot with a speed of
20 m/s as shown in the figure. The coefficient of friction m between the box and the plane is 0.5.
Find the distance travelled
by the box on the plane before it stops for the first time.
Solution The box starts from position 1 with speed v1 = 20 m/s 37°

and stops at position 2.


0 N=80N
Position 2 v 2 =

/s
20
m q
= g sin
v 1 m
Position 1 x

f=40N mg
Kinetic energy at position 1: K1 = 12 mv12 = 2000 J
Kinetic energy at position 2: K2 = 0
Work done by external forces as the box moves from position 1 to position 2 is,
W1®2 = Wg,1®2 + Wf,1®2 = -60x - 40x = -100x J
Applying work energy theorem for the motion of the box from position 1 to position 2, we have
W1®2 = K2 – K1 Þ – 100 x = 0 – 2000 Þ x = 20 m.
Illustration A particle of mass m moves with velocity v = a x where a is a constant. Find the total work done
by all the forces during a displacement from x = 0 to x = d.

1 1 2
Solution Work done by all forces = W = DKE = mv 22 – mv 1
2 2

1 1
Here v1 = a 0 = 0, v2 = a d , So W = ma2d – 0 = ma2d.
2 2

E 81
Pre-Medical : Physics
Illustration The potential energy for a conservative force system is given by U = ax2 – bx, where a and b
are constants. Find out the (a) expression for force, (b) equilibrium position and (c) potential energy
at equilibrium.

Solution (a) For conservative force F=–


dU
dx
= - 2ax - b = –2ax + bb g
b
(b) At equilibrium F = 0 Þ – 2ax + b = 0 Þ x =
2a

(c) U = a
FG b IJ 2
FG b IJ
b2 b2
-b
b2
H 2a K -
4a 2aH 2a K
=-
4a
=.

Illustration The potential energy of a particle of mass 1 kg free to move along the x-axis is given by
F x - xI
2

U(x) = GH 2 JK joules. If total mechanical energy of the particle is 2 J, then find its maximum
speed.
F x - xI
2
x2
Solution Potential energy U = GH 2 JK =
2
–x

dU 2x d 2U
For minimum U, = –1=0 and = 1 = positive
dx 2 dx 2
1
so at x = 1, U is minimum. Hence Umin = – J.
2
Total mechanical energy = Max KE + Min PE
1 FG - 1IJ 5 2 5
Þ Max KE =
2
mv2max = 2 –
H 2K =
2
Þ vmax = ´
1 2
= 5 ms–1.

1
Illustration A chain of mass m and length L is held on a frictionless table in such a way that th part is
n
hanging below the edge of table. Calculate the work done to pull the hanging part of the chain.
Solution Required work done = change in potential energy of chain
Now, let Potential energy (U) = 0 at table level L
n
so potential energies of chain initially and finally are respectively

æ Lö æ Mö L æ L ö MgL
Ui = –mg çè ÷ø = – çè ÷ø g çè ÷ø = - , Uf = 0
2n L n 2n 2n2

MgL
\ required work done = Uf – Ui =
2n2
Illustration For what minimum value of m1 will the block of mass m
just leave contact with the surface ?
k m1
Solution Let extension in the spring be x0 due to m1
1 2
then m1gx0 = kx Þ kx0 = 2m1g m
2 0
m m
but kx0 ³ mg so 2m1g ³ mg Þ m1 ³ ; therefore minimum value of m1 = .
2 2
Illustration A spring is initially compressed by x and then, it is further compressed by y. Find out the work
done during the latter compression. (spring constant is k.)

W1 = 1 2 kx ; W2 = 1 2 k(x + y)
2 2
Solution

= 1 2 k(x + y + 2xy) - 1 2 kx
2 2 2
W.D. = W2 – W1
1 k(y2 + 2xy) Þ 1 ky(y + 2x)
2 2

82 E
Pre-Medical : Physics
Illustration A body of mass m starting from rest from the origin moves along the x-axis with a constant power
(P). Calculate the :
(i) relation between velocity and time. (ii) relation between distance and time.
(iii) relation between velocity and distance.

t v
dv P P v2
Solution (i) P = Fv = mav = m
dt
v Þ ò
0
dt
m = ò vdv
0
Þ
m
t =
2

2P 1
Þ v= t Þ vµt 2 .....(1)
m

x t
dx 2P 1 2 2P 12
(ii)
dt
=
m
t Þ ò0
dx = ò
m 0
t dt

2P 2 3 2 3
Þ x= t Þ xµt 2 .......(2)
m 3

From (1) & (2), x µ ( t 1 2 )


3
(iii) Þ x µ v3 .......(3)

Illustration A truck of mass 10,000 kg moves up an inclined plane rising 1 in 50 with a speed of
36 km/h. Find the power of the engine (g = 10 m/s2).

1
Solution Force against which work is done F = mg sin q = 10,000 × 10 × = 2000 N
50

36 ´ 5
speed v = =10 m/s so P = 2000 × 10 = 20 kW.
18

Illustration An engine pumps water of density r, through a hose pipe. Water leaves the hose pipe with a
velocity v. Find the
(i) rate at which kinetic energy is imparted to water
(ii) power of the engine.

dE k d æ 1 2ö 1 2 dm 1 2 d
Solution (i) Rate of change of kinetic energy = = ç mv ÷ø = v = v (rAx)
dt dt è 2 2 dt 2 dt

1 dx 1
= rAv 2 = rAv3
2 dt 2

æ dm ö dm
(ii) Power = Fv = çè v ÷ v = v2 = v2(rAv) = rAv3
dt ø dt

E 83
Pre-Medical : Physics
WORK, POWER AND ENERGY EXERCISE
1. A body of mass m is displaced from point A(3, 1, 2) 4. A force F = Kx2 acts on a particle at an angle of
to point B(4, 2, 1) under the effect of a force 60° with the x–axis. the work done in displacing the
r
( )
F = ˆi + 2ˆj + 3kˆ N , calculate W.D. by the force. particle from x1 to x2 will be –

kx 2 k 2
(1) 57 J (2) 11 J (3) 0 (4) 22 J
(1)
2
(2)
2
e
x 2 - x12 j
2. Find work done by friction for displacement 'S' ?
k 3 k 3
F
(3)
6
e
x 2 - x13 j (4)
3
e
x 2 - x13 j
q
µK m S 5. A force acts on a 30 g particle in such a way that
the position of the particle as a function of time is
(1) µ K(mg + Fsinq).S (2) –µ K(mg + Fsinq).S
given by x = 3t – 4t2 + t3, where x is in metres and
(3) µ K(mg – Fsinq).S (4) –µ K(mg – Fsinq).S
t is in seconds. The work done during the first 4
3. Calculate the work done for following F-d curves
second is :-
(1) 5.28 J (2) 450 mJ (3) 490 mJ (4) 530 mJ
100
­
r
F(N)
6. A force F = (3x2+2x–7) N acts on a 2 kg body as
4 6
(A) 2 d(m) (P) 100 J a result of which the body gets displaced from x=0
®
to x=5m. The work done by the force will be–
–50
(1) 35 J (2) 70 J (3) 115 J (4) 270 J
15 7. A person of mass m is standing on one end of a plank
­ 10 of mass M and length L and floating in water. The
F(N)
person moves from one end to another and stops.
5
(B) (Q) 13.5 J Work done by normal force is –
1 2 3 4 5
d(m)
®
mMgL
(1) MgL (2) mgL (3) (4) 0
M+m
–15
8. A body of mass M tied to a string is lowered at a constant
acceleration of (g/4) through a vertical distance h. The
­ 15 work done by the string will be..............
F(N)
5
3 1 -3 -1
4 5 (1) Mgh (2) Mgh (3) Mgh(4) Mgh
1 2 3 d(m) 4 4 4 4
(C) ®
(R) 15 J
9. A body of mass 6 kg under a force which causes

–15
t2
displacement in it given ‘S = ’ metres where ‘t’ is
4
time. The work done by the force in 2 seconds is :-
­ 3
F(N) 2 (1) 12J (2) 9J (3) 6J (4) 3J
(D) 1
10. A stone of mass m is tied to a string of length l
1 2 3 4 5 6
d(m) at one end and by holding second end it is
® whirled into a horizontal circle, then work done
will be :–
(1) (A - P); (B - Q); (C - Q); (D - R)
F mv I 2

(2) G l J 2pl
(2) (A - P); (B - R); (C - R); (D - Q) (1) 0
H K
(3) (A - P); (B - P); (C - Q); (D - R)
F mv I 2

(4) G l J l
(4) (A - P); (B - P); (C - R); (D - Q) (3) (mg)·2pl
H K
84 E
Pre-Medical : Physics
11. As shown in the diagram a particle is to be carried 15. For the path PQR in a conservative force field the
from point A to C via paths (I), (II) and (III) in amounts work done in carrying a body from P to
gravitational field, then which of the following
Q and from Q to R are 8 Joule and 2 Joule
statements is correct :–
respectively. The work done in carrying the body
from P to R will be –

(1) 10 J (2) 6 J (3) 68 J (4) Zero


(1) Work done is same for all the paths
16. If K.E. increases by 4%. Then momentum will
(2) Work done is minimum for path (II)
increase by :-
(3) Work done is maximum for path (I) (1) 1.5% (2) 9%
(4) None of the above (3) 3% (4) 2%

12. A particle moves from a point ( -4iˆ + 6j)


ˆ to 1
17. The graph between E k and is
ˆ when a force of (2iˆ + 3j)
ˆ N is applied. p
(5ˆj + 3k)
(EK = kinetic energy and p = momentum) –
How much work has been done by the force ?

(1) 5 J (2) 2 J (3) 8 J (4) 11 J

13. The mass of a bucket full of water is 15 kg. It is being (1) (2)
pulled up from a 15m deep well. Due to a hole in
the bucket 6 kg water flows out of the bucket. The
work done in drawing the bucket out of the well will
be (g = 10m/s2)–

(1) 900 Joule (2) 1500 Joule (3) (4)


(3) 1800 Joule (4) 2100 Joule

14. A particle is moved from (0, 0) to (a, a) under


18. If the momentum of a body is increased n times,
r
a force F = (3iˆ + 4j)
ˆ from two paths. Path 1 is its kinetic energy increases.
OP and path 2 is OQP. Let W1 and W2 be the work (1) n times (2) 2n times
done by this force in these two paths. Then : (3) n times (4) n2 times
19. If K.E. body is increased by 100%. Then % change
y in 'P'.
P(a, a) (1) 50% (2) 41.4% (3) 10% (4) 20%
20. Velocity–time graph of a v (m/s)

45° particle of mass 2 kg 20


x
O Q moving in a straight line is
as shown in figure. Work
(1) W1 = W2 (2) W1 = 2W2 done by all the forces on t (s)
the particle is : 2
(3) W2 = 2W1 (4) W2 = 4W1
(1) 400 J (2) –400 J (3) –200 J (4) 200 J

E 85
Pre-Medical : Physics
21. The only force Fx acting on a 2.0 kg body as it (1) 1.96 mJ
moves along the x-axis varies as shown in the figure. (2) 3.92 mJ
The velocity of the body along positive x-axis at
x = 0 is 4 m/s. The kinetic energy of the body (3) 4.90 mJ
at x = 3.0 m is :- (4) 5.88 mJ
+4
(1) 4 J
Fx,(N) 1 2 3 4 5 26. A body is dropped from a height h. When loss in
(2) 8 J
0 its potential energy is U then its velocity is v. The
(3) 12 J x(m)
mass of the body is –
(4) 16 J –4

22. A uniform chain of length L and mass M is lying on U2 2v 2v 2U


(1) (2) (3) 2 (4)
2v U U v2
2
a smooth table and of its length is hanging down
3
over the edge of the table. If g is the acceleration 27. A 0.5 kg ball is thrown up with an initial speed

due to gravity, the work done to pull the hanging 14 m/s and reaches a maximum height of 8.0 m.
part on the table is :- How much energy is dissipated by air drag acting

MgL MgL 2MgL on the ball during the ascent ?


(1) MgL (2) (3) (4)
3 9 9
(1) 19.6 joules (2) 4.9 joules
23. A particle in a certain conservative force field has (3) 10 joules (4) 9.8 joules
20xy
a potential energy given by U = . The force 28. A ball of mass 4 kg and another of mass 8 kg are
z
dropped together from a 100 feet tall building.
exerted on it is
After a fall of 50 feet each towards earth, their
æ 20y ö ˆ æ 20x ö ˆ æ 20xy ö ˆ respective kinetic energies will be in the ratio of:-
(1) ç ÷i +ç ÷j+ç ÷k
è z ø è z ø è z2 ø (1) 1 : 4 (2) 1 : 2

(3) 1 : 2 (4) 2: 1
æ 20y ö ˆ æ 20x ö ˆ æ 20xy ö ˆ
(2) – ç ÷i -ç ÷j+ç ÷k
è z ø è z ø è z2 ø 29. A particle of mass m is moving in a horizontal ciricle
of radius R under a centripetal force equal to –
æ 20y ö ˆ æ 20x ö ˆ æ 20xy ö ˆ A
(3) – ç ÷i -ç ÷ j -ç ÷k (A = constant). The total energy of the particle
è z ø è z ø è z2 ø r2
is :-
æ 20y ö ˆ æ 20x ö ˆ æ 20xy ö ˆ
(4) ç ÷i +ç ÷j -ç ÷k (Potential energy at very large distance is zero)
è z ø è z ø è z2 ø

24. If the potential energy of two molecules is give by, A A A A


(1) (2) - (3) (4) -
R R 2R 2R
A B
U= - then at equilibrium position, its
r12 r6 30. Consider a drop of rain water having mass 1 g falling
potential energy is equal to : from a height of 1 km. It hits the ground with a speed
of 50 m/s. Take 'g' constant with a value
A2 B2 2B
(1) (2) - (3) (4) 3A 10 m/s2. The work done by the (i) gravitational force
4B 4A A
and the (ii) resistive force of air is :-
25. A 2 g ball of glass is released from the edge of (1) (i) 1.25 J (ii) – 8.25 J
a hemispherical cup whose radius is 20 cm. How
(2) (i) 100 J (ii) 8.75 J
much work is done on the ball by the gravitational
(3) (i) 10 J (ii) – 8.75 J
force during the ball's motion to the bottom of the
cup ? (4) (i) – 10 J (ii) – 8.25 J

86 E
Pre-Medical : Physics
31. A mass of 0.5 kg moving with a speed of 1.5 m/s 36. Calculate power generated by tension in the string
on a horizontal smooth surface, collides with a in first 4 seconds of motion :-
nearly weightless spring of force constant (1) 250 W
k=50N/m. The maximum compression of the
(2) 750 W a=
g
spring would be :- 2
(3) 1500 W
10
(4) 1000 W
37. A body of mass m starting from rest from origin
moves along x-axis with constant power (P).
(1) 0.12 m (2) 1.5 m Calculate relation between velocity and distance :-

(3) 0.5 m (4) 0.15 m (1) x µ v1/2 (2) x µ v 2


(3) x µ v (4) x µ v3
32. A vertical spring with force constant k is fixed on
38. A pump is used to deliver water at a certain rate
a table. A ball of mass m at a height h above the
from a given pipe. To obtain n times water from
free upper end of the spring falls vertically on the
the same pipe in the same time, by what factor, the
spring, so that the spring is compressed by a distance
d. The net work done in the process is: force of the motor should be increased?
(1) n times (2) n2 times
1 2
(1) mg(h + d) + kd 1
2 (3) n3 times (4) times
n
1 2 39. Water is falling on the blades of a turbine at a rate
(2) mg(h + d) - kd
2 of 100 kg/s from a certain spring. If the height
1 2 of the spring be 100 metres, the power transferred
(3) mg(h - d) - kd
2 to the turbine will be :-
1 2 (1) 100 kW (2) 10 kW
(4) mg(h - d) + kd
(3) 1 kW (4) 1000 kW
2
r
33. A block of mass M is attached to the lower end 40. A constant force F is acting on a body of mass
of a vertical spring. The spring is hung from a ceiling m with constant velocity nr as shown in the figure.
and has force constant value k. The mass is released The power P exerted is
from rest with the spring initially unstretched. the
F
maximum extension produced in the length of the
spring will be :- q v
(1) Mg/2k (2) Mg/k
(3) 2 Mg/k (4) 4 Mg/k
F cos q
(1) Fv cosq (2)
34. If a spring extends by x on loading then energy mg
stored by the spring is :- (T is tension in the spring,
Fmg cos q mg sin q
K = spring const.) (3) (4)
v F
T2 T2 2k 2T2 41. An engine pumps water continuously through a
(1) (2) (3) (4)
2x 2k T2 k hose. Water leaves the hose with a velocity v and
m is the mass per unit length of the water jet. What
35. Two men with weights in the ratio 2 : 3 run up a
is the rate at which kinetic energy is imparted to
staircase in times in the ratio 10 : 7. The ratio of
water :-
power of first to that of second is –
1 1
(1) m2v2 (2) mv3
2 2
7 15 10 7
(1) (2) (3) (4) 1
15 7 7 10 (3) mv3 (4) mv2
2

E 87
Pre-Medical : Physics
42. A particle of mass m is driven by a machine that 46. Work done in time t on a body of mass m which
delivers a constant power k watts. If the particle is accelerated from rest to a speed v in time t1 as
starts from rest the force on the particle at time a function of time t is given by :-
t is :-
1 v 2 v 2
-1 -1 (1) 2 m t t (2) m t t
(1) mk t 2 (2) 2mk t 2
1 1

2
1 -1 mk - 12 1 æ mv ö 2 1 v2 2
(3) mk t 2 (4) t (3) ç t t (4) 2 m t 2 t
2 2 2 è t1 ÷ø 1

43. A body of mass m accelerates uniformly from rest 47. A car of mass m starts from rest and accelerates
to v1 in time t1. The instantaneous power delivered so that the instantaneous power delivered to the
to the body as a function of time t is- car has a constant magnitude P0. The instantaneous
acceleration of this car is proportional to :-
mv1 t mv12 t mv1 t2 mv12 t
(1) t (2) (3) (4) (1) t1/2 (2) t/ m
1 t12 t1 t1
44. A body of mass 4 kg is moving up an inclined plane (3) t2P0 (4) t–1/2
rising 1 in 40 with velocity 40 m/sec if efficiency 48. The heart of a man pumps 10 litres of blood through
is 50% the calculate power required. th e arteri es per minut e at a press ure of
(1) 38.4 W (2) 55 W 75 mm of mercury. If the density of mercury be 13.6
×103 kg/m3 and g = 10m/s2 then the power of heart
(3) 78.4 W (4) 108 W
in watt is:
45. A 1.0 hp motor pumps out water from a well of (1) 1.50 (2) 1.70 (3) 2.35 (4) 3.0
depth 20 m and fills a water tank of volume 2238
liters at a height of 10 m from the ground. The 49. A body of mass 1 kg begins to move under the action
r
running time of the motor to fill the empty water of a time dependent force F = (2t ˆi + 3t 2ˆj)N , where
tank is (g = 10ms–2)
î and ĵ are unit vectors along x and y axis. What
(1) 5 minutes (2) 10 minutes power will be developed by the force at the
time t ?
(3) 15 minutes (4) 20 minutes
(1) (2t2 + 3t3)W (2) (2t2 + 4t4)W
(3) (2t3 + 3t4)W (4) (2t3 + 3t5)W

ANSWER KEY
Que. 1 2 3 4 5 6 7 8 9 10 11 12 13 14 15
Ans. 3 2 2 3 1 3 4 3 4 1 1 1 3 1 1
Que. 16 17 18 19 20 21 22 23 24 25 26 27 28 29 30
Ans. 4 3 4 2 2 3 4 2 2 2 4 4 2 4 3
Que. 31 32 33 34 35 36 37 38 39 40 41 42 43 44 45
Ans. 4 2 3 2 1 3 4 2 1 1 2 4 2 3 3
Que. 46 47 48 49
Ans. 4 4 2 4

88 E
Pre-Medical : Physics

CIRCULAR MOTION
When a particle moves in a plane such that its distance from a fixed point remains constant then its motion is called
as circular motion with respect to that fixed point. That fixed point is called centre and the distance is called radius
of circular path.
Kinematics of Circular Motion
Angular Displacement
Q
Angle traced by position vector Dq = angular displacement fixed
point Dq
Arc Arc (PQ)
Angle = Þ Dq = r
Radius r P

Frequency (n)
Number of revolutions described by particle per second is its frequency. Its unit is revolutions per second (r.p.s.) or
revolutions per minute (r.p.m.) r.p.m. = 60 r.p.m.
Time Period (T)
1
It is time taken by particle to complete one revolution. T =
n
Angular Velocity (w)
It is defined as the rate of change of angular position of moving particle .

Angle traced Dq dq
w= = Lim =
Time taken Dt ®0 Dt dt
Q
Relation between linear and Angular velocity
r
Arc Ds Dq Ds
Angle = or Dq = or Ds = rDq
Radius r
r
Ds rDq ds dq P
\ = if Dt ® 0 then =r v = wr
Dt Dt dt dt

® ® ® ®
v = w´ r (direction of v is according to right hand thumb rule)

Average Angular Velocity (wavg)

total angle of rotation q2 - q1 Dq 2p


wavg = = = = = 2pn
total time taken t2 - t1 Dt T
where q1 and q2 are angular position of the particle at instant t1 and t2.
Instantaneous Angular Velocity (Winst.)
r
Dq dq r dq
The angular velocity at a particular instant (w) = Lim = or w =
Dt ®0 Dt dt dt
Relative Angular Velocity

( v AB ) ^ Relative velocity of A w.r.t. B perpendicular to line AB


wAB = =
rAB seperation between A and B

v A sin q1 + v B sin q2
here (vAB)^ = vA sin q1 + vB sin q2 \ wAB =
r

E 89
TG: @Chalnaayaaar

Pre-Medical : Physics
ALLEN
Angular Acceleration (a)
Rate of change of angular velocity is called angular acceleration.
r
Dw dw r dw
Lim
a = Dt ®0 = or a =
Dt dt dt
r r r r
Its an axial vector quantity. It direction is along the axis of rotation according to if w­ Þ w || a , if w¯ Þ w a
Unit ® rad/s2 axis of rotation
r r
Relation between Angular and Linear Acceleration vr = w
w
´r
r r v
r r r r r dw r dr r
or a = a ´ r + w´ v ( = a and =v)
dt dt
r
r r r r r r r r r
or a = a T + a C ( a T = a ´ r is tangential acc. and a C = w´ v is centripetal acceleration.)

r r r
\ |a|= a2T + aC2 ( a T and a C are two component of net linear acceleration.)

Tangential Acceleration
r r
a T = ar ´ r , its direction is parallel to velocity..
r r r
r
v = w´ r and a T = ar ´ rr
axis of rotation
Centripetal acceleration w

r r r Þ ar = r r r r r) r
aC = w ´v C w ´ (w ´ r) (Q vr = w ´r O P

v2 r v2
Magnitude of centripetal acceleration, a C = wv = = w2 r ; aC = ( -ˆr)
r r
Equations of Motion in Circular Kinematics
If a particle moves along a circle with constant angular acceleration.
1 2
(i) w = w0 + at (ii) q = w0t + at
2

æ w0 + w ö
(iii) w2 = w02 + 2aq (iv) q= ç ÷t
è 2 ø
a
(v) qn = w0 + (2n – 1)
2
where, w0 = Initial angular velocity,
w = Instantaneous angular velocity a = Angular acceleration,
q = Angular displacement at time t, qn = Angular displacement in nth second
Uniform Circular Motion
When a particle moves in a circle at a constant speed then the motion is v2
said to be a uniform circular motion.
r v1
Speed is constant, so that a T = 0
r2
r r r r v r1
Acceleration of particle a = a C = w ´ v or a = wv ( but w = )
r O

v2
\ a= = w2 r = centripetal acceleration
r

due to centripetal acceleration, the velocity of the particle keeps on changing the direction i.e. the particle is
accelerated.
90 E
Pre-Medical : Physics
Non- Uniform Circular Motion
When a particle moving in a circle and if the speed of particle increases or decreses then the motion is non-uniform
circular motion.
r
In non- uniform circular motion |v| ¹ constant w ¹ constant
In non-uniform circular motion particle has two acceleration :

dv ds
(a) Tangential acceleration aT = = rate of change of speed; v = = speed; s=arc-length.
dt dt

v2
(b) Centripetal acceleration a c = = w2 r
r
r r r
Net acceleration of the particle a = ac + a T Þ a = a2C + a2T

-1 æa ö
\ q = tan ç T ÷
è aC ø

CIRCULAR MOTION IN VERTICAL PLANE


By conservation of mechanical energy between A and P.
1 1 O
0+ mu2 = mgh + mv2 or v = u 2 - 2gh
2 2
h
or v= u2 - 2gl (1 - cos q) [Q h = l(1 – cosq)]

Tension in string

mv 2
At point 'p' required centripetal force =
l
Net force toward centre

mv 2
T – mg cosq =
l

v2 = m 2
T = m [g cos q + ] [u - gl(2 - 3 cos q)] B vC
l l vB

O C
mv2A mu2
TA = + mg = + mg [q = 0°]
l l
A vA

mv 2B mu2
TB = - mg = - 5mg (q = 180°) [by law of conservation of energy v B2 = u2 – 2g (2l)]
l l
When particle is at point C of the circle –

mv 2C mu2
TC = = - 2mg (q = 90°) [by law of conservation of energy v C2 = u2 - 2g(l)]
l l
Thus we can conclude TA > TC > TB
TA - TB = 6mg
TA - TC = 3mg
TC – TB = 3mg

E 91
Pre-Medical : Physics
Cases

(i) if u > 5gl

In this case tension in the string will not be zero at any point, which implies that the particle will continue the
circular motion.

mu 2
(ii) u= 5gl ® just completes the loop (at point B tension becomes zero, TB = - 5mg )
l

critical velocity to complete the loop is u = vA = 5gl

vB = gl [ vB2 = vA2 – 4g(l) ]

vc = 3gl [ vC2 = vA2 – 2g(l)] and TA = 6mg, TB = 0, Tc = 3mg

(iii) 2gl < u < 5gl Þ not complete the loop and leave the circular path.
Tension becomes zero between points C and B but speed v ¹ 0 in this case. It moves in parabolic path beyond
that point where tension become zero upto the point at same height in other half cicular part.

(iv) u= 2gl Þ T= 0 and v = 0 at point C.


Particle will oscillate about point A.

(v) u< 2gl v = 0 in between A and C, but T ¹ 0 and particle will oscillate about 'A'.
Illustration A road makes a 90° bend with a radius of 190 m. A car enters the bend moving at 20 m/s. Finding
this too fast, the driver decelerates at 0.92 m/s². Determine the acceleration of the car when its
speed rounding the bend has dropped to 15 m/s.
Solution Since it is rounding a curve, the car has a radial acceleration associated with its changing direction,
in addition to the tangential deceleration that changes its speed. We are given that aT = –0.92
m/s², since the car is slowing down, the tangential acceleration is directed opposite the velocity.
ar a

v2 (15)2
The radial acceleration is ar = = = 1.2 m / s²
r 190
q
aT

Magnitude of net acceleration. a = a2r + a2T = [(1.2)² + (0.92)2]1/2 = 1.5 m/s²

æa ö æ 1.2 ö
and points at an angle q = tan-1 ç r ÷ = tan-1 ç = 53°
è aT ø è 0.92 ÷ø
relative to the tangent line to the circle.
Illustration A particle of mass 'm' tied with a string of length l is released from m
l
horizontal position as shown is fig. Find the velocity at the lowest
position.
1 2
Solution apply COME mgl = mv Þ v = 2gl
2
Illustration A 4 kg ball swing in a vertical circle at the end of a cord 1m long. Find the maximum speed at which it
can swing if the cord can sustain maximum tension of 104 N. (g = 10 m/s 2)

mv 2
Solution T= + mg \ v = 4 m/s
r

92 E
Pre-Medical : Physics
Illustration A ball is released from height 'h' as shown, which of the
condition hold good for the particle to complete the circular path.
Solution v= h
2gh ³ 5gR

5 R
Þ 2gh ³ 5gR r h ³ R
2
Illustration A circular overbridge having radius 20m, what is the maximum speed with which a car can cross the
bridge without leaving contact with the overbridge at the highest point ( g = 9.8 m/sec 2 )
N
mv 2
Solution For motion, mg – N =
r v

When reaction N becomes zero, contact is about to leave mg

mv2
\ mg = or v = rg = 14 m/s
r
Illustration A ring rotates about z-axis as shown in fig. in x-y plane. At a certain
instant the acceleration of a particle P (Shown in fig.) on the ring is
(6 $i - 8 $j ) m/sec2 . At that instant what is the angular acceleration and
angular velocity of the ring ?
r r r r r
Solution a = a T + a C here a C is along – $j and a T is along $i
r
given a = 6iˆ - 8ˆj r aT = 6 = a r
and aC = 8 = w2r
r 6 ˆ , r ˆ
now a = = 3 rad/s2 ( -k) w = 2 rad/s ( -k)
2
Illustration A particle of mass 'm' slide down form the vertex of hemisphere, without any initial velocity. At what
height from the horizontal will the particle leave the sphere.
m
mv 2 u=0 N
Solution At point P mg cosq – N = P
R
To leave the contact N = 0 so v2 = gR cosq
q
os

q v
gc

h
1
m

by law of conservation of energy 0 + mg R = mv2 + mgh q mg


2

h 2
v2 = 2g (R -h) = gR cosq (Q cos q = ) r h= R
R 3
Illustration A particle of mass 100 gm is suspended from the end of a weightless string of length 100cm and is allowed
to swing in a vertical plane. The speed of the mass is 200 cm/s, when the string makes an angle of 60°
with the vertical axis. Determine the tension in the string at 60°.

Solution Tension at point (B)

O vB=200cm/s
TB
mv 2B B
Fnet towards O = Fc Þ TB – mgcos60° = ,
60

mg
l cos
°

A 60°
60
°

vA mg
mv 2B 100 ( 200)
2
1
TB = + mgcos60° = + 100 × (1000) ×
l 100 2

= 40,000 + 50,000 = 90,000 dyne = 0.9 × 105 dyne = 0.9 N

E 93
Pre-Medical : Physics
Illustration In a vertical circular motion, tension at the highest point is equal to 2
mv
the weight of the particle, then find the speed and tension at the lowest m l
point. Mass of the particle is m = 10 kg and length of string is T
l
l = 10 m (g = 10 m/s2) mg

mv 2
Solution At the highest point T + mg = given T = mg
l A

mv 2
\ mg + mg = Þ v2 = 2lg Þ v = 2lg
l
by conservation of mechanical energy between top most & lowest point

1 1
mv2 + mg (2l) = mv12
2 2

1 1
r m (2lg) + 2mgl = mv12
2 2 T

6gl = v12 Þ v1 = 6g l
mv2
l mg
mv12 m
At lowest point T = mg + = mg + × 6gl
l l

T = 7mg = 700 N and v1 = 6g l = 10 6 m/s = 24.49 m/s

Illustration A particle is moving parallel to x–axis as shown in fig. such that the y
component of its position vector is constant at all instants and is equal
to 'b'. Find the angular velocity of the particle about the origin when its
radius vector makes an angle q with the x-axis.

b v sin q v 2
Solution \ wPO = = sin q
b b
sin q y

Illustration A particle is moving in clockwise direction in a circular path as shown II I


in figure. The instantaneous velocity of particle at a certain instant is x
r III IV
v = (3 $i + 3 $j ) m/s. Then in which quardant does the particle lie at that
instant? Explain your answer.
Solution II quardant. According to following figure x & y components of velocity are positive when the particle
is in II quardant. y

II I
x
III IV

94 E
Pre-Medical : Physics
Illustration A hemispherical bowl of radius R is rotating about its axis of symmetry w
which is kept vertical. A small ball kept in the bowl rotates with the bowl
without slipping on its surface. If the surface of the bowl is smooth and R
the angle made by the radius through the ball with the vertical is a . Find a Na
the angular speed at which the bowl is rotating. r A

mg
Solution N cosa = mg .....(1)
N sina = mrw2 .....(2)
r = Rsina .....(3)
From equations (2) & (3)
Nsina = mw2Rsina
N=mRw2 .....(4)
g
Þ (mRw2) cosa = mg Þ w =
R cos a
Illustration A particle of mass m tied to a string of length l and given a circular motion in the vertical plane.
If it performs the complete loop motion then prove that difference in tensions at the lowest and the
highest point is 6 mg.
Solution Let the speeds at the lowest and highest points be u and v respectively. H

mu 2
At the lowest point, tension = TL = mg + ...(i)
l
mv 2
At the highest point, tension = TH = - mg . ..(ii)
l
mu2 mv 2
By conservation of mechanical energy,
2
-
2
= mg 2l b g Þ u2 = v2 + 4gl

m v 2 + 4gl
Substituting this in eqn (i) TL = mg + ...(iii)
l
\ From eqn. (ii) & (iii) TL – TH = 6mg

Illustration A particle of mass m is connected to a light inextensible string of length


l such that it behaves as a simple pendulum. Now the string is pulled q2
q1
to point A making an angle q1 with the vertical and is released then obtain A
expressions for the :
(i) speed of the particle and [AIPMT (Mains) 2008]

(ii) the tension in the string when it makes an angle q2 with the vertical.
Solution (i) h = l(cosq2 – cosq1)
Applying conservation of mechanical energy between points A & B lcosq1 q2 l
1 lcosq2 q1
mv 2 = mgh Þ v= 2gh = 2gl(cos q2 - cos q1 ) A
2 T
h
mv 2 B
q2
(ii) At position B, T – mgcosq2 = where v = 2gl(cos q2 - cos q1 )
l mg mgcosq2

m
Þ T = mgcosq2 + [2gl(cosq2 – cosq1)] = mg(3cosq2 – 2cosq1).
l

E 95
Pre-Medical : Physics

CIRCULAR MOTION EXERCISE


1. If the equation for the displacement of a particle 6. A particle moving along a circular path. The angular
moving on a circular path is given by (q) = 2t3+0.5, velocity, linear velocity, angular acceleration and
where q is in radian and t in second, then the angular centripetal acceleration of the particle at any instant
velocity of the particle after 2 s from its start is :- respectively are w r , r , r , r . Which of the
v a ac
(1) 8 rad/s (2) 12 rad/s following relation is/are correct :–

(3) 24 rad/s (4) 36 rad/s ® ®


(a) w ^ v
2. The angular speed of second hand in a mechanical
watch is :- ® ®
(b) w ^ a

p
(1) rad / s (2) 2p rad/s ® ®
30 (c) v ^ a c

® ®
60 (d) w ^ a c
(3) p rad/s (4) rad / s
p
(1) a,b,d (2) b,c,d
r
3. What is the value of linear velocity, if w = 3iˆ - 4ˆj + kˆ (3) a,b,c (4) a,c,d

r 7. Two stones of masses m and 2 m are whirled in


and r = 5iˆ - 6ˆj + 6kˆ :-
horizontal circles, the heavier one in a radius r and
(1) 6iˆ + 2jˆ - 3kˆ the lighter one in radius 2r. The tangential speed
of lighter stone is n times that of the value of heavier
(2) -18iˆ - 13jˆ + 2kˆ stone when they experience same centripetal forces.
The value of n is :
(3) 4iˆ - 13jˆ + 6kˆ (1) 1 (2) 2 (3) 3 (4) 4

8. A particle is going in a spiral path as shown in figure,


(4) 6iˆ - 2jˆ + 8kˆ with constant speed.

4. A particle moves along a circle of radius R with


constant angular velocity w. Its displacement
magnitude in time t is :-

(1) wt (2) 2R sin wt

wt
(3) 2R cos wt (4) 2R sin
2

5. A particle of mass 'm' describes a circle of radius

4 (1) The velocity of particle is constant


(r). The centripetal acceleration of the particle is .
r2
(2) The acceleration of particle is constant
The momentum of the particle :–
(3) The magnitude of acceleration is constant

2m 2m 4m 4m (4) The magnitude of acceleration is increasing


(1) (2) (3) (4)
r r r r continuously
96 E
Pre-Medical : Physics
9. A point mass m is suspended from a light thread of 12. A motor cyclist moving with a velocity of 72 km/h on
length l, fixed at O, is whirled in a horizontal circle at a flat road takes a turn on the road at a point where
constant speed as shown. From your point of view, the radius of curvature of the road is 20 m. The
stationary with respect to the mass, the forces on the acceleration due to gravity is 10 m/sec 2. In order to
mass are avoid skidding, he must not bend with respect to the
vertical plane by an angle greater than :-
O
(1) q = tan–16 (2) q = tan–12
(3) q = tan 25.92
–1
(4) q = tan–14

13. A ball is suspended by thread of length l. What


minimum horizontal velocity has to be imparted to
l
the ball for it to reach the height of suspension ?

(1) (gl ) (2) (2gl ) (3) 2gl (4) gl


m
14. A tube of length L is filled completely with an
incompressible liquid of mass M and closed at both
T T the ends. The tube is then rotated in a horizontal
plane about one of its ends with a uniform angular
velocity w. The force exerted by the liquid at the other
F
end is :-
(1) (2)
MLw 2
(1) (2) MLw2
W W 2

T MLw 2 ML2w2
T (3) (4)
4 2

F F F 15. A car is moving in a circular horizontal track of radius


(3) (4) 10 m with a constant speed of 10 m/s. A plumb bob
is suspended from the roof of the car by a light rigid
W
W rod of length 1 m. The angle made by the rod with
track is :-
10. A car is travelling with linear velocity v on a circular
(1) zero (2) 30° (3) 45° (4) 60°
road of radius R. If its speed is decreasing at the rate
a m/s2, then the net acceleration will be :- 16. Figure shows a small mass connected to a string, which
v2 v2 is attached to a vertical post. If the mass is released
(1) +a (2) -a when the string is horizontal as shown, the magnitude
R R
of the total acceleration of the mass as a function of
2 2
æ v2 ö æ v2 ö the angle q is :-
çè R ÷ø + a çè R ÷ø - a
2 2
(3) (4)

11. One end of a string of length l is connected to a particle q


of mass m and the other to a small peg on a smooth
horizontal table. If the particle moves in a circle with
speed v, the net force on the particle (directed towards
the centre) is :-
(1) g sin q (2) g cos q
mv 2 mv 2
(1) T (2) T - (3) T + (4) zero (3) g 3 cos 2 q + 1 (4) g 3 sin2 q + 1
l l

E 97
Pre-Medical : Physics
17. A block follows the path as shown in the figure from 21. Keeping the banking angle of the road constant, the
height h. If radius of circular path is r, then relation maximum safe speed of passing vehicles is to be
that holds good to complete full circle is :- increased by 10%. The radius of curvature of the
road will have to be changed from 20 m to :-
(1) 16 m (2) 18 m
(3) 24.20 m (4) 30.5 m

h 22. Three identical particles are joined together by a


thread as shown in figure. All the three particles are
moving in a horizontal plane. If the velocity of the
r outermost particle is v0, then the ratio of tensions in
the three sections of the string is :-

5r 5r
(1) h < (2) h >
2 2 O A B C
l l l
5r 5r
(3) h = (4) h ³ (1) 3 : 5 : 7 (2) 3 : 4 : 5
2 2
18. Radius of the curved road on national highway is (3) 7 : 11 : 6 (4) 3 : 5 : 6
R. Width of the road is b. The outer edge of the 23. A mass m is attached to the end of a rod of length
road is raised by h with respect to inner edge so l. The mass goes around a verticle circular path
that a car with velocity v can pass safely over it. The with the other end hinged at the centre. What should
value of h is :– be the minimum velocity of mass at the bottom of
the circle so that the mass completes the circle ?
v2 b v v2R v2 b
(1) (2) (3) (4)
Rg Rgb bg R (1) 4gl (2) 3gl (3) 5gl (4) gl
19. A boy holds a pendulum in his hand while standing 24. A stone is tied to a string of length ‘ l’ and is whirled
at the edge of a circular platform of radius r rotating in a vertical circle with the other end of the string as
at an angular speed w. The pendulum will hang at the centre. At a certain instant of time, the stone is
an angle q with the vertical so that :– at its lowest position and has a speed ‘u’. The
w2 r 2 magnitude of the change in velocity as it reaches a
(1) tan q = 0 (2) tan q = position where the string is horizontal (g being
g
acceleration due to gravity) is :–
rw2 g
(3) tan q = (4) tan q =
g w2 r (1) u2 - gl (2) u – u2 - 2gl
20. A body tied to a string of length L is revolved in a
vertical circle with minimum velocity, when the body (3) 2gl (4) 2(u 2 - gl)
reaches the upper most point the string breaks and
25. A gramophone record is revolving with an angular
the body moves under the influence of the
velocity w. A coin is placed at a distance r from the
gravitational field of earth along a parabolic path.
centre of the record. The static coefficient of friction
The horizontal range AC of the body will be :–
is µ. The coin will revolve with the record if :-
(1) x = L
P v mg
(2) x = 2L (1) r ³ (2) r = mgw2
w2
m L
O
(3) x= 2 2L w2 mg
(3) r < (4) r £
x mg w2
A C
(4) x= 2L

98 E
Pre-Medical : Physics
26. A car of mass m is moving on a level circular track 29. In the given figure, a = 7.5 m/s2 represents the total
of radius R. If ms represents the static friction acceleration of a particle moving in the clockwise
between the road and tyres of the car, the maximum direction in a circle of radius R = 5 m at a given
speed of the car in circular motion is given by :- instant of time. The speed of the particle is :-

(1) mRg / m s (2) ms Rg

°
30
R
O a
(3) ms mRg (4) Rg / m s

27. A particle of mass 10 g moves along a circle of radius


6.4 cm with a constant tangential acceleration. What (1) 5.7 m/s (2) 6.2 m/s
is the magnitude of this acceleration if the kinetic
(3) 4.5 m/s (4) 5.0 m/s
energy of the particle becomes equal to 8 × 10–4
J by the end of the second revolution after the
beginning of the motion?
(1) 0.1 m/s2 (2) 0.15 m/s2
(3) 0.18 m/s2 (4) 0.2 m/s2
28. A car is negotiating a curved road of radius R. The
road is banked at an angle q. The coefficient of
friction between the tyres of the car and the road
is µ s. The minimum safe velocity on this road is :-

tan q + µ s tan q - µ s
(1) gR 2 (2) gR
1 - ms tan q 1 + m s tan q

g µ s + tan q g µ s + tan q
(3) R 1 -ms tan q (4) 2
1 +ms tan q
R

ANSWER KEY
Que. 1 2 3 4 5 6 7 8 9 10 11 12 13 14 15
Ans. 3 1 2 4 2 4 2 3 3 3 1 2 2 1 3
Que. 16 17 18 19 20 21 22 23 24 25 26 27 28 29
Ans. 4 4 1 3 2 3 4 1 4 4 2 1 2 1

E 99
Pre-Medical : Physics
COLLISION AND CENTRE OF MASS
w Centre of mass : For a system of particles centre of mass is that point at which its total mass is supposed to be
concentrated. y
w Centre of mass of system of discrete particles
Total mass of the body : M = m1 + m2 + ..... + mn then m1(x1, y1, z1)
r r r m2(x2, y2, z2)
r m r + m 2 r2 + m3 r3 + ... 1 r
R CM = 1 1 = Sm i ri m3(x3 , y3, z 3)
m1 + m2 + m 3 + ... M r1 r2 r3
mn(xn, yn, zn)
co-ordinates of centre of mass :
rn
1 1 1 x
x cm = Sm i x i , y cm = Sm i y i and z cm = Sm i z i (0,0,0)
M M M
w Centre of mass of continuous bodies z
y
r 1 r r 1 r r 1 r r 1 r
R CM = ò rdm Þ x cm =
M M ò x dm , y cm = ò y dm and z cm = ò z dm
M M
x, y, z are the co-ordinate of the COM of the dm mass.
dm
The centre of mass after removal of a part of a body
r
Original mass (M) – mass of the removed part (m)
= {original mass (M)} + { – mass of the removed part (m)} x
(0,0,0)
Mx - mx ¢ My - my ¢ Mz - mz ¢
The formula changes to : x CM = ; y CM = ; y CM = z
M-m M-m M-m
Where x', y' and z' represent the coordinates of the centre of mass of the removed part
Imporant points about centre of mass :
1. There may or may not be any mass present physically at the centre of mass.
2. Centre of mass may be inside or outside a body.
3. Centre of mass depends on the distribution of mass with in the body and is closer to massive portion.
4. For symmetrical bodies having homogeneous distribution of mass, centre of mass lie on the line of
symmetry and can be on centre of symmetry or geometrical centre.
CENTRE OF MASS OF SOME COMMON OBJECTS
Body Shape of body Position of centre of mass

Uniform Ring Centre of ring


CM

CM
Uniform Disc Centre of disc

Uniform Rod Centre of rod


CM

Solid sphere/ Centre of sphere


CM
hollow sphere

Triangular plane Point of intersection of the medians


lamina CM
of the triangle i.e. centroid

Plane lamina in Point of intersection of diagonals


the form of a
square or rectangle CM
or parallelogram

Hollow/solid CM
Middle point of the axis of cylinder
cylinder

100 E
Pre-Medical : Physics
Body Shape of body Position of centre
of mass
y

Half ring 2R
y cm =
CM p
ycm R
x

Segement of a ring Rsinq


CM y cm =
ycm q q
R
x

4R
CM y cm =
Half disc (plate) ycm 3p
x

Sector of a disc (plate) 2Rsinq


CM y cm =
ycm R
3q
q
x
y

CM
R
Hollow hemisphere ycm R y cm =
x 2
y

CM 3R
Solid hemisphere ycm R y cm =
x 8
y

h
Hollow cone h CM y cm =
ycm 3
x

h
Solid cone h CM y cm =
ycm 4
x

MOTION OF CENTRE OF MASS


r r r
r dR CM m1 v1 + m2 v 2 + ...
For a system of particles, velocity of centre of mass v CM = =
dt m1 + m2 + ....
r r
r d r m1a1 + m2a 2 + ...
Similarly acceleration CM dt ( CM )
a = v =
m1 + m 2 + ....

w Law of conservation of linear momentum


Linear momentum of a system of particles is equal to the product of mass of the system with velocity of its centre
of mass.
r
r d ( Mv CM )
From Newton's second law Fext. =
dt
r r r r r
If Fext. = 0 then Mv CM = constant or p final = pinitial
If no external force acts on a system the velocity of its centre of mass remains constant, i.e., velocity of centre
of mass is unaffected by internal forces. For example : Explosion of bomb at rest.
w Impulse – Momentum theorem r r
t 2

Impulse of a force is equal to the change of momentum òt Fdt = Dp


1

force time graph area gives change in momentum.

E 101
Pre-Medical : Physics
Collision of bodies
The event or the process, in which two bodies either coming in contact with each other or due to mutual
interaction at distance apart, affect each others motion (velocity, momentum, energy or direction of motion) is
defined as a collision.
In collision
• The particles come closer before collision and after collision they either stick together or move away from each
other.
• The particles need not come in contact with each other for a collision always. (But we genrally consiver those
collision which are in contact).
• The law of conservation of linear momentum is necessarily applicable in a collision, whereas the law of conser-
vation of mechanical energy is not.

T YPES O F CO LLISIO N

On the basis of direction On the basis of kinetic energy

one-dimensional Two dimensional


collision collision Elastic In-elastic Perfectly In-elastic
or or collision collision collision
Head on collision Oblique collision

The collision, in which The collision, in which A collision is said A collision is The collision, in which
the particles move along the particles move along to be elastic, if said to be particles gets sticked
the same straight line the same plane at together after the
the total kinetic inelastic, if the
collision, is called
before and after the different angles before energy before total kinetic perfectly inelastic
collision, is defined as and after collision, is and after energy does not collision. In this type of
one dimensional defined as oblique collision remains remains inelastic collision, loss of
collision. collision. the same constant energy is maximum.

w Coefficient of restitution (Newton's law)


r r
velocity of separation along line of impact v 2 - v1
e =- = r r
velocity of approach along line of impact u1 - u 2

Value of e is 1 for elastic collision, 0 for perfectly inelastic collision and 0 < e < 1 for inelastic collision.
w Head on collision
u1 u2 v1 v2
A B A B A B
m1 m2 m1 m2
Before collision Collision After collision
Head on inelastic collision of two particles
Let the coefficient of restitution for collision is e
(i) Momentum is conserved m1u1 + m2u2 = m1v1 + m2v2 ...(i)
(ii) Kinetic energy is not conserved.
v 2 - v1
(iii) According to Newton's law e = ...(ii)
u1 - u2

By solving eq. (i) and (ii) :

æ m - em2 ö æ (1 + e ) m2 ö m1u1 + m2u 2 - m2e ( u1 - u2 )


v1 = ç 1 ÷ u1 + ç ÷ u2 =
è m1 + m2 ø è m1 + m2 ø m1 + m2

æ m - em1 ö æ (1 + e ) m1 ö m1u1 + m2 u2 - m1e ( u2 - u1 )


v2 = ç 2 ÷ u2 + ç ÷ u1 =
è m1 + m2 ø è m1 + m2 ø m1 + m2

102 E
Pre-Medical : Physics
Elastic Collision (e=1)

• If the two bodies are of equal masses : m1 = m2 = m, v1 = u2 and v2 = u1


Thus, if two bodies of equal masses undergo elastic collision in one dimension, then after the collision, the bodies
will exchange their velocities.
• If the mass of a body is negligible as compared to other.
If m1>> m2 and u2 = 0 then v1 = u1, v2 = 2u1
When a heavy body A collides against a light body B at rest, the body A should keep on moving with same
velocity and the body B will move with velocity double that of A.

If m2 >> m1 and u2 = 0 then v2 = 0, v1 = – u1


When light body A collides against a heavy body B at rest, the body A should start moving with same velocity just
in opposite direction while the body B should practically remains at rest.
w Loss in kinetic energy in inelastic collision

m1m2 r r 2
DK = (1 - e2 ) u1 - u 2
2(m1 + m2 )

Oblique Collision
Conserving the momentum of system in directions along normal (x axis in our case)
m1u1cosa1 + m2u2cosa2 = m1v1cosb1 + m2v2cosb2

y
u2 v2
m2 m2
m1 m2
a2 b2
x
a1 b1

u1
v1
Before After
collision collision

Since no force is acting on m1 and m2 along the tangent (i.e. y–axis) the individual momentum of m1 and m2
remains conserved.
m1u1sina1 = m1v1sinb1 & m2u2sina2 = m2v2sinb2
By using Newton's experimental law along the line of impact

v 2 cos b2 - v1 cos b1
e=
u1 cos a1 - u2 cos a 2

KEY POINTS
r r
• Sum of mass moments about centre of mass is zero. i.e. åm r i i/ cm = 0

• A quick collision between two bodies is more violent then slow collision, even when initial and final
velocities are equal because the rate of change of momentum determines that the impulsive force small
or large.

• Heavy water is used as moderator in nuclear reactors as energy transfer is maximum if m1 ; m2.
• Impulse momentum theorem is equivalent to Newton's second law of motion.

• For a system, conservation of linear momentum is equivalent to Newton's third law of motion.

E 103
Pre-Medical : Physics
Illustration Three rods of the same mass are placed as shown in the figure. Calculate the coordinates of the
centre of mass of the system.

æa ö æ aö æa aö
Solution CM of rod OA is at ç ,0 ÷ , CM of rod OB is at ç 0, ÷ and CM of rod AB is at ç , ÷
è2 ø è 2ø è2 2ø
a a a a
m´ +m´0+m´ a m´0+ m´ + m´
For the system, xcm = 2 2 = Þ ycm = 2 2 = a
m+m+m 3 m+m+m 3
Illustration If the linear density of a rod of length L varies as l = A + Bx, determine the position of its centre
of mass. (where x is the distance from one of its ends)
Solution Let the X–axis be along the length of the rod with origin at one of its end as shown in figure.
As the rod is along x–axis, so, yCM = 0 and zCM = 0 i.e., centre of mass will be on the rod.
Now consider an element of rod of length dx at a distance x from the origin, mass of this element
dm = ldx = (A + Bx)dx so,
L L

ò xdm ò x(A + Bx)dx AL2 BL3


+
x CM = 0L = 0L = 2 3 = L(3A + 2BL)
BL2 3(2A + BL)
ò0 dm ò0 (A + Bx)dx AL + 2
dx

Note :
(i) If the rod is of uniform density then l = A = constant & B = 0 then xCM= L/2
(ii) If the density of rod varies linearly with x, then l = Bx and A = 0 then xCM = 2L/3
R
Illustration A disc of radius R is cut off from a uniform thin sheet of metal. A circular hole of radius is
2
now cut out from the disc, with the hole being tangent to the rim of the disc. Find the distance
of the centre of mass from the centre of the original disc.
Solution We treat the hole as a 'negative mass' object that is combined with the original uncut disc. (When
the two are overlapped together, the hole region then has zero mass). By symmetry, the CM lies
along the +y–axis in figure, so xCM = 0 . With the origin at the centre of the original circle whose
mass is assumed to be m.
Mass of original uncut circle m1 = m & Location of CM = (0,0)
m æ Rö
Mass of hole of negative mass : m2 = ; Location of CM = ç 0, ÷
4 è 2ø

æ möR y
m(0) + ç - ÷
m 1 y1 + m 2 y 2 è 4 ø2 = -R
Thus y CM = = R
m1 + m 2 æ mö 6 2
m + ç- ÷
è 4ø x
0
So the centre of mass is at the point æ 0, - R ö . R
çè ÷

Thus, the required distance is R/6.
104 E
Pre-Medical : Physics
Illustration Find coordinates of center of mass of a quarter sector of a uniform disk of radius r placed in
the first quadrant of a Cartesian coordinate system with centre at origin.

Solution. From the result obtained for sector of circular plate distance OC of the center of mass form the
center is
y
2r sin ( p / 4) 4 2r
OC = = yc C
3p / 4 3p

æ 4r 4r ö p/4
Coordinates of the center of mass (xc, yc) are çè , ÷ O xc x
3p 3p ø

Illustration A block of mass M is placed on the top of a bigger block of mass 10M as shown in figure. All
the surfaces are frictionless. The system is released from rest.

2.2 m

Find the distance moved by the bigger block at the instant when the smaller block reaches the
ground.
Solution If the bigger block moves toward right by a distance (x) then the smaller block will move toward
left by a distance (2.2 – x).
Now considering both the blocks together as a system, horizontal position of CM remains same.

As the sum of mass moments about centre of mass is zero i.e. åm x i i / cm = 0.

M(2.2 – x) = 10 Mx Þ x = 0.2 m.

Illustration A man of mass 80 kg stands on a plank of mass 40 kg. The plank is lying on a smooth horizontal
floor. Initially both are at rest. The man starts walking on the plank towards north and stops after
moving a distance of 6 m on the plank. Then
(A) the centre of mass of plank-man system remains stationary.
(B) the plank will slide to the north by a distance of 4 m
(C) the plank will slide to the south by a distance of 4 m
(D) the plank will slide to the south by a distance of 12 m
Solution Ans. (A,C)
Since net force is zero so centre of mass remains stationary
Let x be the displacement of the plank.
Since CM of the system remains stationary

6m

south x north

so 80 (6–x) = 40 x Þ 12 – 2x = x Þ x = 4 m.

E 105
Pre-Medical : Physics
Illustration Two bodies of masses m1 and m2 (<m1) are connected to the ends of a
massless cord and allowed to move as shown in figure. The pulley is massless
and frictionless. Calculate the acceleration of the centre of mass.
r r
Solution If a is the acceleration of m1,then – a is the acceleration of m2 then
r r
r m1a + m2 (– a) F
m1 – m2 r I
a cm =
m1 + m2
= GH
m +m
a
1 2
JK
F
m1 – m2 r I m1 – m2 F I 2
r
But
r
GH JK
r
a = m + m g so a cm= m + m
1 2 1 2
GH JK g.

Illustration In a gravity free room a man of mass m1 is standing at a height h above the floor. He throws a
ball of mass m2 vertically downward with a speed u. Find the distance of the man from the floor
when the ball reaches the ground.

h
Solution Time taken by ball to reach the ground t =
u

F m uI
By conservation of linear momentum, speed of man v = GH m JK
2

FG h IJ FG m u IJ = m
2 2
Therefore, the man will move upward by a distance = vt =
H uK H m K m
1 1
h

m2 m2 F I
Total distance of the man from the floor = h+ m h = 1 + m h.
1 1
GH JK
Illustration A simple pendulum of length 1m has a wooden bob of mass 1kg. It is struck by a bullet of mass
10–2 kg moving with a speed of 2 × 102 m/s. The bullet gets embedded within the bob. Obtain
the height to which the bob rises before swinging back.
Solution Applying principle of conservation of linear momentum

2
mu = (M + m) v Þ 10–2 × (2 × 102) = (1+.01) v Þ v = m/s
1.01

v
(M + m)
h
m
M
u

Initial KE of the block with bullet in it, is fully converted into PE as it


rises through a height h, given by

2
1 v2 æ 2 ö 1
(M + m)v 2 = (M + m)gh Þ v2 = 2gh Þ h = =ç ÷ ´ = 0.2 m.
2 2g è 1.01ø 2 ´ 9.8

106 E
Pre-Medical : Physics
Illustration A body falling on the ground from a height of 10 m, rebounds to a height 2.5 m calculate the:
(i) percentage loss in K.E. (ii) ratio of the velocities of the body just before and after the collision.
Solution Let v1 and v2 be the velocities of the body just before and just after the collision.

1 1 2 v2 h 10 v
KE1 = mv 12 = mgh1...(i) and KE2 = mv 2 = mgh2 ...(ii) Þ 12 = 1 = = 4 Þ 1 =2.
2 2 v 2 h2 2.5 v2

mg(h1 - h2 ) 10 - 2.5
Percentage loss in KE= ´ 100 = ´ 100 = 75%.
mgh1 10

Illustration A ball of mass m hits a floor with a speed v making an angle of incidence q = 45° with the normal
1
to the floor. If the coefficient of restitution is e = , find the speed of the reflected ball and the
2
angle of reflection. [AIPMT (Mains) 2005]

Solution

Since the floor exerts a force on the ball along the normal during the collision so horizontal component
of velocity remains same and only the vertical component changes.

v
Therefore, v'sinq' = vsinq =
2

1 1 v
and v'cosq' = evcosq = v× = .
2 2 2

v2 v2 3 2 3
Þ v'2 = + = v Þ v' = v
2 4 4 2

and tanq' = 2 Þ q' = tan 2 .


–1

Illustration A ball of mass 2 kg moving with a speed of 5 m/s collides directly with another ball of mass
3 kg moving in the same direction with a speed of 4 m/s. The coefficient of restitution is 2/3. Find
the velocities after collision.
Solution Denoting the first ball by A and the second ball by B, velocities immediately before and after the
impact are shown in the figure.

5 m/s 4 m/s v1 v2

A B A B
Before collision After collision

By COLM : 2(5) + 3(4) = 2v1 + 3v2 Þ 2v1 + 3v2 = 22 ...... (i)

v 2 – v1 2 v 2 – v1
By definition of e : e = u – u Þ = Þ 3v2 –3v1 =2 ...... (ii)
1 2 3 5–4

by solving equations (i) and (ii), we have v1 = 4 m/s and v2 = 4.67 m/s

E 107
Pre-Medical : Physics
COLLISION AND CENTRE OF MASS EXERCISE
1. The coordinate of the centre of mass of a system as 3a ˆ 3a ˆ
(1) aiˆ + ajˆ (2) i+ j
shown in figure :– 2 2
4a ˆ
Y (3) 2aiˆ + j (4) 2aiˆ + 2ajˆ
3
solid M
sphere 5. Find the position of centre of mass from the base of
(0,a)
a uniform solid cone of height 20 cm.
hollow M M disk
sphere 20
(1) 5 cm (2) cm
(0,0) (a,0)
3
(3) 12 cm (4) lie outside the cone

æa ö æa aö 6. If the linear density of a rod of length L varies as


(1) ç , 0 ÷ (2) ç , ÷
è3 ø è2 2ø l = kx2, determine the position of its centre of mass
æa aö æ aö (where x is the distance from one of its ends and k is
(3) ç , ÷ (4) ç 0, ÷ constant) :-
è 3 3ø è 3ø
2. The centre of mass of a system of three particles L L 2L 3L
(1) (2) (3) (4)
of masses 1g, 2g and 3g is taken as the origin of 2 3 3 4
a coordinate system. The position vector of a fourth 7. The variation of density of a cylindrical thick and
particle of mass 4g such that the centre of mass
of the four particle system lies at the point (1, 2, x2
long rod, is r = r0 , then position of its centre
ˆ , where a is a constant. The value
3,) is a (iˆ + 2jˆ + 3k) L2

of a is :- of mass from x = 0 end is :-

10 5 1 2 (1) 2L/3 (2) L/2 (3) L/3 (4) 3L/4


(1) (2) (3) (4)
3 2 2 5 8. Find the position of centre of mass from base for
3. A rigid body consists of a 5 kg mass connected to a a solid hemisphere of radius 24 cm,
10 kg mass by a massless rod. The 5 kg mass is (1) 4 cm (2) 9 cm (3) 8 cm (4) 12 cm
r
located at r1 = (5iˆ + 4j)
ˆ m and the 10 kg mass at
r 9. Two particles which are initially at rest, move towards
r2 = (4iˆ + 3j)
ˆ m. Find the coordinates of the centre
each other under the action of their internal
of mass. attraction. If their speeds are v and 2v at any instant,
æ 13 10 ö then the speed of centre of mass of the system will
(1) (11, 10) (2) ç , ÷
è 3 3 ø be :-
æ 11 10 ö (1) v (2) 2v (3) Zero (4) 1.5 v
(3) ç 15 , 15 ÷ (4) (3, 4)
è ø
10. Two bodies of masses 15 kg and 5 kg are connected
4. Find the position of centre of mass of given figure :- to the ends of a mass less cord and allowed to move
y as shown in figure. The pulley is massless and friction
less. Calculate the acceleration of the centre of mass:-
square plate
(1) 5 m/s2
(2) –5 m/s2
2a s
5
(3) m/s2
2
2a 2s 3s 5kg
5
(4) - m/s2 15 kg
2
x

108 E
Pre-Medical : Physics
11. If the system is released, then the acceleration of 15. A uniforme sheet is shown below if the shaded area
the centre of mass of the system :- is removed find new position of centre of mass.

g a
(1) (1) + a
4 6

g a
(2) (2) -
2 6
m
3m 2a
(3) g a O
(4) 2g (3) +
3
12. A cricket bat is cut at the location of its centre of
mass as shown. Then :- a
(4) - 2a
3
16. A circular disc of radius R has a uniform thickness.
A circular hole of diameter equal to radius of disc has
been cut out as shown. Distance of centre of mass of
(1) The two pieces will have the same mass remaining disc from O is :-
(2) The bottom piece will have larger mass R
(3) The handle piece will have larger mass (1)
14
(4) Mass of handle piece is double the mass of
bottom piece R
(2) R
12
13. Two spherical bodies of mass M and 5M and radii
R and 2R are released in free space with initial R
separation between their centres equal to 12R. If (3)
8
they attract each other due to gravitational force
only, then the distance covered by the smaller body R
(4)
before collision is :- 6
(1) 9.5R (2) 7.5R 17. A circular plate of uniform thickness has a diameter
(3) 4.5R (4) 3.5R 56 cm. A circular
14. A uniform solid sphere as shown below has a portion of diameter 42
spherical hole in it. Find the position of its centre of cm is removed from one
28 cm
mass. edge as shown in the 21 cm

figure. The centre of O2 O O1


y
mass of the remaining
b
portion from the centre
of plate will be :
(1) 5 cm (2) 7 cm (3) 9 cm (4) 11 cm
a
x
18. A 1 kg stationary bomb is exploded in three parts
having mass ratio 1 : 1 : 2. Parts having same mass
R
move in perpendicular directions with velocity 30
z m/s, then the velocity of bigger part will be :-

-a 3 b2 -a 3 b 10
(1) (2)
R 3 - a3 R 3 - a3 (1) 3 2 m/s (2) m/s
2
15
-a 3 b 3 - b3 a (3) 15 2 m/s (4) m/s
(3) (4) 2
R 3 - a3 R 3 - a3

E 109
Pre-Medical : Physics
19. A big ball of mass M, moving with velocity u strikes
æ m ö æ m+M ö
a small ball of mass m, which is at rest. Finally small (1) ç ÷v (2) ç ÷v
ball attains velocity u and big ball v. What is the èM–mø è M ø

value of v :-
æ M–m ö æ m ö
M-m m (3) ç ÷v (4) ç ÷v
(1) u (2) u è M ø è m+M ø
M M+m
2m M
(3) (4) v 25. A bullet of mass 10g moving horizontally with a
M+m M+m
velocity of 500 m/s strikes a wooden block of mass
20. A heavy nucleus at rest breaks into two fragments
which fly off with velocities 8 : 1. The ratio of radii 2 kg which is suspended by a light inextensible string
of length 5 m. As a result, the centre of gravity of
of the fragments is :-
the block is found to rise a vertical distance of
(1) 1 : 2 (2) 1 : 4
10 cm. The speed of the bullet after it emerges out
(3) 4 : 1 (4) 2 : 1
horizontally from the block will be :-
21. A person of mass m is standing on one end of a plank
(1) 220 m/s (2) 200 m/s
of mass M and length L and floating in water. The
person moves from one end to another and stops. (3) 160 m/s (4) 120 m/s
The displacement of the plank is –
26. A 10 gm lump of clay, moving with a velocity of
Lm 10 cm/s towards east, collides head–on with another
(1) (2) Lm(M + m)
(m + M) 20 gm lump of clay moving with 15cm/s towards
west. After collision, the two lumps stick together.
(M + m ) LM The velocity of the compound lump will be –
(3) (4)
Lm (m + M) (1) 5 cm/s towards east
(2) 5 cm/s towards west
22. Two man of mass 50 kg and 100 kg are standing on (3) 6.6 cm/s towards east
a 150 kg plank. Find the displacement of plank if (4) 6.6 cm/s towards west
both interchange their positions on 10 m long plank.
27. Two identical balls, one moves with 6 m/s and
50 kg 100 kg
second is at rest, collides elastically. After collision
velocity of second and first ball will be :
(1) 6m/s, 6m/s (2) 12m/s, 12m/s
Plank
(3) 6m/s, 0m/s (4) 0m/s, 6m/s
5 28. Two identical balls A and B having velocities of
(1) 5m, Right (2) m , Right
3 5 m/s and –3 m/s respectively collide elastically in
(3) 3m, Left (4) 0 one dimension. The velocities of B and A after the
23. A bullet of mass m is fired from a gun of mass M. collision respectively will be :-
The recoiling gun compresses a spring of force (1) –3 m/s and 5 m/s
constant k by a distance d. Then the velocity of the
(2) 3 m/s and 5 m/s
bullet is :-
(3) –5 m/s and 3 m/s
d
(1) kd M/m (2) km
M (4) 5 m/s and –3 m/s
d kM
(3) kM (4) d 29. A 1 Kg ball falls from a height of 25 cm and
m m
rebounds upto a height of 16 cm. The co–efficient
24. A bullet of mass m is fired into a large block of wood
of restitution is –
of mass M with velocity v. The final velocity of the
(1) 0·8 (2) 0·32 (3) 0·40 (4) 0·56
system is :-

110 E
Pre-Medical : Physics
30. A ball is dropped from a height of 10 m. If 60% of 36. If two masses m1 and m2 collide, the ratio of the
its energy is lost on collision with the earth then changes in their respective velocities is proportional
after collision the ball will rebound to a height of– to :-
(1) 10 m (2) 8 m
m1 m1
(3) 4 m (4) 6 m (1) m (2)
2 m2
31. Two balls of equal masses undergo a head-on collision
with speeds 6 m/s moving in opposite direction. If m2 m2
(3) m (4) m1
1
the coefficient of restitution is 0.5, find the speed of
each ball after impact in m/s. 37. In the diagrams given below the horizontal line
(1) 3 (2) 6 represents the path of a ball coming from left and
(3) 2 (4) 4 hitting another ball which is initially at rest. The other
32. Identify the wrong statement. two lines represents the paths of the two balls after
the collision. Which of the diagram shows a physically
(1) A body can have momentum without energy
impossible situation ?
(2) A body can have energy without momentum
(3) The momentum is conserved in an elastic
(1)
collision only.
(4) Kinetic energy is not conserved in an inelastic
collision
(2)
33. A sphere of mass m moving with a constant velocity
collides with another stationary sphere of same
(3)
mass. The ratio of velocities of two spheres after
collision will be, if the co-efficient of restitution is e-
(4)
1- e e -1
(1) (2) 38. A rubber ball is dropped from a height of 5m on
1+ e e +1
a plane, where the acceleration due to gravity is
1+ e e +1 not shown. On bouncing it rises to 1.8 m. The ball
(3) (4)
1- e e -1 loses its velocity on bouncing by a factor of :-
34. A ball is dropped from height h on the ground level. 16 2
(1) (2)
If the coefficient of restitution is e then the height 25 5
upto which the ball will go after nth jump will be –
3 9
h e 2n (3) (4)
5 25
(1) (2)
e2n h
39. Two masses m1 = 2kg and m2 = 5kg are moving
(3) he n
(4) he 2n on a frictionless surface with velocities 10m/s
35. A particle falls from a height ‘h’ upon a fixed and 3 m/s respectively. m2 is ahead of m 1. An
horizontal plane and rebounds. If ‘e’ is the coefficient ideal spring of spring constant k = 1120 N/m is
of restitution the total distance travelled before attached on the back side of m2. The maximum
rebounding has stopped is :- compression of the spring will be :-

F1+ e I2 F1- e I2

(1) h GH 1 - e JK
2 (2) h GH 1 + e JK
2

F
h 1 - e2 I F
h 1 + e2 I (1) 0·51 m (2) 0·062 m
(3) 2 GH
1 + e2
JK (4) 2 GH
1 - e2
JK (3) 0·25 m (4) 0·72 m

E 111
Pre-Medical : Physics
40. Two spheres A and B of masses m1 and m 2 41. A ball moving with velocity of 9m/s collides with
respectively collide. A is at rest initially and B is another similar stationary ball. After the collision
moving with velocity 2v along x-axis. After collision both the balls move in directions making an angle
B has a velocity v in a direction perpendicular to the of 30° with the initial direction. After the collision
original direction. The mass A moves after collision their speed will be –
in the direction.
5
(1) q = tan–1(1/2) to the x-axis (1) m/s (2) 3 3 m/s
3
(2) q = tan–1(–1/2) to the x-axis
(3) same as that of B
10
(4) opposite to that of B (3) m/s (4) 52 m/s
3

ANSWER KEY
Que. 1 2 3 4 5 6 7 8 9 10 11 12 13 14 15
Ans. 3 2 2 3 1 4 4 2 3 4 1 2 2 2 2
Que. 16 17 18 19 20 21 22 23 24 25 26 27 28 29 30
Ans. 4 3 3 1 1 1 2 3 4 1 4 3 4 1 3
Que. 31 32 33 34 35 36 37 38 39 40 41
Ans. 1 3 1 4 1 3 3 2 3 1,2 2

112 E
Pre-Medical : Physics
ROTATIONAL MOTION
RIGID BODY :
Rigid body is defined as a system of particles in which distance between each pair of particles remains
constant (with respect to time) that means the shape and size do not change, during the motion. Eg : Fan,
Pen, Table, stone and so on.

Type of Motion of rigid body

Pure Translational Pure Rotational Combined Translational and


Motion Motion Rotational Motion

ROTATIONAL MOTION
Moment of Inertia

The virtue by which a body revolving about an axis opposes the change in axis

rotational motion is known as moment of inertia. m


r
· The moment of inertia of a particles with respect to an axis of rotation
is equal to the product of mass of the particle and square of distance from
rotational axis. I = mr2
r = perpendicular distance from axis of rotation
axis discrete axis
· Moment of inertia of system of particle body continuous
r1 m1 dm body
I = m1r12 + m2 r22 + m3 r32 + ..... r2 r
m2

r3
· For Rigid Bodies :
2
I= r dm
m3
Moment of inertia of a rigid body about any axis of rotation.

ò
I = dm r 2

Radius of Gyration (K) K has no meaning without axis of rotation.


I = MK2 K is a scalar quantity

I
Radius of gyaration K = z
M
o y
Perpendicular axis Theorems : Iz = I x + I y
x
(Valid only for 2-dimensional body)
Parallel axis Theorem : I = ICM + Md2 ICM I=ICM+Md
2

(for all type of bodies) CM parallel


axis
ICM = moment of inertia about the axis d

Passing through the centre of mass

E 113
Pre-Medical : Physics
MOMENT OF INERTIA OF SOME REGULAR BODIES

Shape of Position of the axis Figure Moment of Radius of


the body of rotation Inertia (I) gyration (K)
(1) Circular (a) About an axis A
z
Ring perpendicular to the
plane and passes CM d MR2 R
through the centre
B

Mass = M
Iz
1 R
Radius = R (b) About the diametric y x' MR2
2 2

axis x y'

(c) About an axis

tangential to the rim I1 2MR2 2R


and perpendicular to R
CM
the plane of the ring
M

(d) About an axis

3 3
tangential to the rim MR2 R
O 2 2
R
and lying in the plane
of ring
(2) Circular Disc (a) About an axis

1 R
passing through the MR2
2 2
R

centre and perpendicular


M = Mass to the plane of disc
R = Radius

(b) About a diametric y MR2 R


x' 4 2
axis

x y'

(c) About an axis


CM 5 5
tangential to the rim O ICM MR2 R
R 4 2
and lying in the plane
of the disc

114 E
Pre-Medical : Physics

(d) About an axis Iz


3 3
tangential to the rim CM MR2 R
R 2 2
& perpendicular to the
plane of disc M
R

(3) Annular disc (a) About an axis


M 2 R12 + R22
passing through the R2 é R1 + R22 ùû

R2 R1
R1 2
centre and M
perpendicular to the
plane of disc
M = Mass
R1 = Internal (b) About a diameteric
R2 M 2 R12 + R22
Radius axis R1 éë R1 + R22 ùû
4 2
R2 = Outer M
Radius

(4) Solid Sphere (a) About its diametric I


2 2
axis which passes MR2 R
5 5
through its centre M R
of mass

M = Mass
R = Radius
(b) About a tangent to
I
7 7
the Sphere MR2 R
5 5
M R

(5) Hollow (a) About diametric axis I


M
2 2
Sphere passing through centre MR2 R
3 3
(Thin spherical of mass R
Shell)

table tennis ball

I
M 5 5
(b) About a tangent to MR2 R
3 3
the surface R
M = Mass
R = Radius
Thickness
table tennis ball
negligible

E 115
Pre-Medical : Physics
(6) Hollow (a) About its geometrical
Cylinder axis which is parallel to MR2 R

its length

M = Mass (b) About an axis which


MR2 ML2 R2 L2
R = Radius is perpendicular to its + +
2 12 2 12
L = Length length and passes
through its centre of
L
mass

(c) About an axis


MR2 ML2 R2 L2
perpendicular to its + +
2 3 2 3
length and passing
through one end of the
L
cylinder

I
(7) Solid (a) About its geometrical
R M MR2 R
Cylinder axis, which is along
2 2
M = Mass its length
R = Radius
L = Length

(b) About an axis


I
3 3
tangential to the M MR2 R
R 2 2
cylinderical surface and
parallel to its
geometrical axis

(c) About an axis


ML2 MR2 L2 R2
passing through the + +
12 4 12 4
centre of mass and
perpendicular to its
L
length

(8) Thin Rod (a) About an axis


ML2 L
passing through centre
12 12
of mass and
L
perpendicular to its
length

116 E
Pre-Medical : Physics
Thickness is
negligible (b) About an axis
ML2 L
w.r.t. length passing through one
3 3
Mass = M end and perpendicular
L
Length = L to length of the rod

(9) Rectangular (a) About an axis passing


Mb 2 b
Plate through centre of mass
12 2 3
b
and perpendicular to
a
side b in its plane

M = Mass (b) About an axis


Ma 2 a
a = Length passing through centre
b 12 2 3
b = Breadth of mass and
perpendicular to side a
a in its plane.

(c) About an axis


passing throught centre
(
M a2 + b 2 ) a2 + b 2
12 12
of mass and b
perpendicular to plane a

(10) Cube About an axis passes


Ma2 a
through centre of mass
6 6
and perpendicular to
face
a

Mass = M
Side a

TORQUE
® ® ®
Torque about point : t = r ´ F
Magnitude of torque = Force × perpendicular
distance of line of action of
force from the axis of rotation.
O
t = r F sinq
Direction of torque can be determined by using right hand thumb rule.

ANGULAR MOMENTUM (MOMENT OF LINEAR MOMENTUM)


Angular momentum of a body about a given axis is the product of
mv

q
p=

its linear momentum and perpendicular distance of line of action of r


q
linear momentum vector from the axis of rotation
r r r b=
L = r´p rsi
nq

E 117
Pre-Medical : Physics
Magnitude of Angular momentum = Linear momentum × Perpendicular distance of line of action of
momentum from the axis of rotation
L = mv × r sinq
Direction of angular momentum can be used by using right hand thumb rule.
r
r dL r
l According to Newtons Second Law’s for rotatry motion t = = Ia .
dt
l Angular Impulse = Change in angular momentum.
r
l If a large torque acts on a body for a small time then, angular impulse = tdt
Conservation of Angular Momentum
Angular momentum of a particle or a system remains constant if t ext = 0 about that point or axis of
rotation.
DL
If t = 0 then =0 r L = constant r Lf = Li or I1w1 = I2w2
Dt
Examples
l If a person skating on ice folds his arms then his M.I. decreases and 'w' increases.

l A diver jumping from a height folds his arms and legs (I decrease) in order to increase no. of rotation in air by
increasing 'w'.

l If a person moves towards the centre of rotating platform then 'I' decrease and 'w' increase.

ROTATIONAL KINETIC ENERGY


Kinetic Energy of Rotation
1 2
KER = Iw
2
1 2 L2 1
l other forms K= Iw = = Lw
2 2I 2
1
l If external torque acting on a body is equal to zero (t = 0), L=constant Kµ , Kµw
I
l Rotational Work Wr = tq (If torque is constant)
q2
Wr = ò tdq
q1
(If torque is variable)
l The work done by torque = Change in kinetic energy of rotation.
1 1 1
W = Iw22 - Iw12 = I(w22 - w12 )
2 2 2
dW dq w
l Instantaneous power Pin = = t = tw Average power Pav =
dt dt t
118 E
Pre-Medical : Physics
COMBINED TRANSLATIONAL AND ROTATIONAL MOTION OF A RIGID BODY
When a body perform translatory motion as well as rotatory motion then it is known as rolling.
In Pure Rolling Pure rolling
(i) If the velocity of point of contact with respect to the surface v =Rw CM CM

is zero then it is known as pure rolling. R


w CM
v
(ii) If a body is performing rolling then the velocity of any point of
CM

the body with respect to the surface is given by


r r r r
v = v CM + wCM ´ R Contact point

B VCM– w R=0

Only Translatory motion + Only Rotatory Motion = Rolling motion. For pure rolling above body
VA = 2VCM VE = 2 VCM
VF =
2 VCM VB = 0
Velocity at a point on rim of sphere

v net = v 2 + R 2 w2 + 2vRw cos q


For pure rolling v = Rw

q
v net = 2v cos
2
Rolling Kinetic Energy under pure rolling
1 2 1 2 1 2 1 æ v2 ö
Rolling Kinenetic Energy E = mv + Iw = mv + mK2 ç 2 ÷
2 2 2 2 èR ø
1 æ K2 ö
Rolling Kinenetic Energy E = mv 2 ç 1 + 2 ÷
2 è R ø
K2 K2
Etranslation : Erotation : ETotal = 1 : : 1 +
R2 R2

K2 E trans 1 K2
Body = 2 E trans 1 Erotation R2
Erotation K = =
R2 E total K2 E total K2
R2 1+ 2 1+ 2
R R
1 1
Ring 1 1
2 2
1 2 1
Disc 2
2 3 3
2 5 5 2
Solid sphere
5 2 7 7
2 3 3 2
Spherical shell
3 2 5 5
1 2 1
Solid cylinder 2
2 3 3
1 1
Hollow cylinder 1 1
2 2
E 119
Pre-Medical : Physics
Rolling Motion on an inclined plane
Velocity at bottom of inclined plane
Applying Conservation of energy
1 2 1 2
mgh = mv + Iw
2 2

2æ v ö
2
1 2 1
mgh = 2 mv + mK ç 2÷
2 èR ø

1 2æ K2 ö
mgh = 2 mv ç 1 + ÷ ...(1)
è R2 ø
h = s sinq ...(2)

2gh 2gs sin q


from (1) & (2) VRolling = =
K2
1+ 2 K2
R 1+
R2

g sin q
l Linear accleration on reaching the lowest point a =
1 + K 2 / R2

2s(1 + K 2 / R 2 )
l Time taken to reach the lowest point of the plane is t =
g sin q

K2
l Least, will reach first
R2

K2
Maximum, will reach last
R2

K2
equal, will reach together
R2
l When ring, disc, hollows sphere, solid sphere rolls on same inclined plane then
vS > vD > vH > vR aS > aD > aH > aR tS < tD < tH < tR
Illustration A wheel is rotating with angular velocity 2 rad/s. It is subjected to a uniform angular acceleration 2.0
rad/s2.
(a) What angular velocity does the wheel acquire after 10 s?
(b) How many revolutions will it make in this time interval?
Solution. The wheel is in uniform angular acceleration, Hence –

w = wo + at ® Substituting the values of wo, a and t, we have

w = 2 + 2 ´ 10 = 22 rad/s

q = qo + 12 ( wo + w) t ® Substituting qo = 0 for initial position, and wo from above equation, we have

q = 0 + 12 ( 2 + 22 )10 = 120rad.

In one revolution, the wheel rotates through 2p radians. Therefore the number of complete revolutions
n is

q 120
n= = » 19
2p 2p
120 E
Pre-Medical : Physics
Illustration A rigid lamina is rotating about an axis passing perpendiuclar to its plane through point O as shown
in the figure.

5m
3m w=10 rad/s

37°
O 4m A

The angular velocity of point B w.r.t. A is


Solution In a rigid body, angular velocity of any point w.r.t. any other point is constant and is equal to
the angular velocity of the rigid body.

Illustration Two masses m1 and m2 are placed at a separation r. Find out the moment of inertia of the system
about an axis passing through its centre of mass and perpendicular to the line joining the masses.

r
m2 r m1 r m1 m2
Solution. m1r1 = m2r2 and r1 + r2 = r Þ r1 = m + m , r2 = m + m CM
1 2 1 2
r1 r2

2 2
æ m2 r ö æ m1 r ö æ m1m2 ö 2
Moment of inertia I = m r 2
+mr 2
= m1 ç + m2 ç = ç r
1 1 2 2 ÷
è m1 + m2 ø ÷
è m1 + m2 ø è m1 + m2 ÷ø

m1 m2
Note : Here I = µr2 where µ (reduced mass) = .
m1 + m 2
Illustration Calculate the moment of inertia of an annular disc about an axis which lies in the plane of the
disc and tangential to the (i) inner circle and (ii) outer circle. Mass of the disc is M and its inner
radius is R1 and outer radius is R2.
M
Solution. (i) M.I. about an axis tangential to the inner circle is IAB = (R 2+R22) + MR12
4 1
R2 B
R1 D

A C

M
(ii) M.I. about an axis tangential to the outer circle is ICD= (R12+R22) + MR22
4
Illustration Calculate the moment of Inertia of a semicircular disc of mass M and radius R about an axis passing
through its centre and perpendicular to its plane.
Solution Let us assume a ring of radius ‘r’ & thickness ‘dr’

M 2Mrdr
dm = (prdr) = dr
pR 2 R2
r
2
R R R R
2Mr 2M 3 2M é r 4 ù MR2
I= ò ò
r2 dm = r2
0
R2
dr = 2
R 0 ò
r dr = 2 ê ú Þ I =
R ë 4 û0 2

E 121
Pre-Medical : Physics
Illustration The radius of gyration of a solid sphere of radius r about a certain axis is r. Calculate the distance
of that axis from the centre of the sphere.

Solution. From paralle axes theorem,

2 2
Q I = ICM + md2 \ mr2 = mr + md 2
5

Þ d = 3 r = 0.6 r..
5
Illustration Find the moment of inertia of the ring shown in figure about the axis AB.

Solution. From parallel axes theorem,

MR 2 3
IAB = ICM + MR2 = + MR2 = MR2.
2 2
Illustration The uniform solid block shown in figure has mass M and edge dimensions a, b, and c. Calculate
its rotational inertia about an axis passing through one corner and perpendicular to the large faces.

Solution. Use the parallel - axes theorem. The rotational inertia of a rectangular slab about

an axis through the centre and perpendicular to the large face is given by

Rotational
M 2
Icm =
12
(a + b2 ) axis
b
c

A parallel axis through a corner is at distance h = ( a / 2) 2 + ( b / 2) 2


a

M 2 M 2 M 2
from the centre, so I = Icm + Mh2 = (a + b2) + (a + b2) = (a + b2).
12 4 3

Illustration Three rods are arranged in the form of an equilateral triangle. Calculate
the M.I. about an axis passing through the geometrical centre and
perpendicular to the plane of the triangle (Assume that mass and length Iz
of each rod is M and L respectively). M,L M,L

2 x
3ML2 æ L ö ML2 ML2 ML2
Solution. I = 3ICM+3M x = 2
+ 3M ç ÷ = + = . M,L
12 è 2 3ø 4 4 2

Illustration Diameter of each spherical shell is R and mass M they are joined x
by a light and massless rod. Calculate the moment of inertia of the R R
2 R 2
system about xx' axis.

x'

2 æ R ö é2 æ R ö ù 1
2 2
13
Solution. Isystem = M ç ÷ + ê M ç ÷ + M(2R)2 ú = MR 2 + 4MR 2 = MR 2
3 è 2 ø êë 3 è 2 ø úû 3 3

122 E
Pre-Medical : Physics
Illustration Four holes of radius R are cut from a thin square plate of side 4R and mass M. Determine the
moment of inertia of the remaining portion about Z–axis.
Y
Solution. M = Mass of the square plate before the holes were cut.

é M ù 2 pM X
Mass of each hole m = ê 2ú
pR = .
ë16R û 16
\ Moment of inertia of the remaining portion,

M é mR2 ù é 8 10p ù
I = Isquare – 4Ihole= (16R 2 + 16R 2 ) - 4 ê + m( 2R)2 ú = 8 MR 2 - 10mR 2 = ê - 2
ú MR .
12 ë 2 û 3 ë 3 16 û

R
Illustration A thin uniform disc has a mass 9M and radius R. A disc of radius
3 2R
R 3
is cutoff as shown in figure. Find the moment of inertia of the remaining
O
disc about an axis passing through O and perpendicular to the plane
of disc.
Solution. As the mass is uniformly distributed on the disc,
2
9M 9Mp é R ù
so mass density (per unit area) = . Mass of removed portion = ´ =M
pR 2 pR 2 êë 3 úû
So the moment of inertia of the removed portion about the stated axis by theorem of parallel axes
is :
2 2
M éR ù é 2R ù MR 2
I1 = ê ú +Mê ú = .....(i)
2 ë3û ë 3 û 2

The moment of inertia of the original complete disc about the stated axis is I2 then
2
R
I2 = 9M .....(ii)
2
So the moment of inertia of the left over disc shown in fig. is I2 – I1.
i.e., I2 – I1 = 4MR2.

Illustration A uniform rod of 20 kg is hanging in a horizontal position with the


A B
help of two threads. It also supports a 40 kg mass as shown in the
l/4
figure. Find the tensions developed in each thread.
40 kg
Solution. Free body diagram of the rod is shown in the figure.
y
Translational equilibrium requires x T2
T1
SFy = 0 Þ T1 + T2 = 400 + 200 = 600 N (i) A D C B
l/4 l/2
Rotational equilibrium: Applying the condition about A, we get T2.
200 N
r r
St A = 0 Þ –400(l/4) – 200(l/2) + T2l = 0 400 N
T2 = 200 N
From equation (i)
T1 = 400 N.

E 123
Pre-Medical : Physics
Illustration Find the minimum value of F for the block to topple about an edge.
a
F
b M

Solution. When the block is about to topple the normal reaction N shifts to the edge through O.
N
FBD during toppling Taking torque about O
F
b
æ aö Mga a/2
F (b) = Mg çè ÷ø Þ Fmin = .
2 2b
Mg
Illustration A fixed pulley of radius 20 cm and moment of inertia 0.32 kg×m2 about its axle
has a massless cord wrapped around its rim. A 2 kg mass M is attached to the
end of the cord. The pulley can rotate about its axis without any friction. Find the
acceleration of the mass M. (Assume g = 10 m/s2)

Solution. For the motion of the block 2g – T=2a


For the motion of the pulley t = TR = Ia a

Ia Ia g R
Q a = aR \ T = 2 Þ 2g – 2 = 2a Þ a= I
R R 1+ T
2R2
10 10 10 T
Þa= = =2 ms–2.
0.32 1 + 4 5 a 2kg
1+
2 ´ 0.2 ´ 0.2
Illustration A belt moves over two pulleys A and B as shown in the figure. The pulleys are mounted on two
fixed horizontal axles. Radii of the pulleys A and B are 50 cm and 80 cm respectively. Pulley
A is driven at constant angular acceleration of 0.8 rad/s2 until pulley B acquires an angular velocity
of 10 rad/s. The belt does not slide on either of the pulleys.

C B
A

(a) Find the acceleration of a point C on the belt and angular acceleration of the pulley B.
(b) How long does it take for the pulley B to acquire an angular velocity of 10 rad/s ?
Solution. Since the belt does not slide on the pulleys, magnitudes of velocity and acceleration of any point
on the belt are same as that of any point on the periphery of either of the two pulleys.
r r r
aT = a ´ r aC = a A rA = a B rB

Substituting rA = 0.5 m, rB = 0.8 m and aA= 0.8 rad/s2,

aC a A rA
we have aC = 0.4 m/s2 and a B = = = 0.5 rad/s2
rB rB
wB - wBo
Q w = w o + at Þ t =
aB
Substituting w Bo = 0 , w B = 10 rad/s and a B = 0.5 rad/s2, we have t = 20 s
124 E
Pre-Medical : Physics
Illustration In the figure, the blocks have unequal masses m1 and m2 (m1 < m2). m2
has a downward acceleration a. The pulley P has a radius r, and some mass. P
The string does not slip on the pulley. Find the acceleration of block and
angular acceleration of pulley. (I = momentum of inertia of pulley)
T1 T2
m1
m1 a m2 a m2 a
Solution.

m1 g m2 g
T1 – m1g = m1a ....(1)
m2g – T2 = m2a ....(2)
Total torque = Ia
(T2 – T1)R = Ia ....(3)
a = aR ....(4)

(m2 – m1 ) a
By Eq. (1), (2) (3) and (4) Þ a = g Þ a=
I R
m1 + m2 +
r2
Illustration A spherical shell has a radius of 1.90 m. An applied torque of 960 N-m gives the shell an angular
acceleration of 6.20 rad/s2 about an axis passing through the centre of the shell. What are
(a) the rotational inertia of the shell about that axis and (b) the mass of the shell?
Solution. (a) Use t = Ia where t is the net torque acting on the shell, I is the rotational inertia of the shell,
and a as its angular acceleration.

t 960 N × m
This gives I = = = 155 kg-m2
a 6.20 rad/s2

æ 2ö
(b) The rotational inertia of the shell is given by I = çè ÷ø MR2
3

3I (
3 155kg × m2 )
\M= = = 64.4 kg.
2 (1.90 m )
2
2R 2

Illustration A solid cylinder of mass 'M' and radius 'R' is rotating along its axis with angular velocity w without
friction. A particle of mass 'm' moving with velocity v collides against the cylinder and sticks to its
rim. After the impact calculate angular velocity of cylinder. Before After
w w'
Solution. Initial angular momentum of cylinder = Iw
R R m
Initial angular momentum of particle = mvR
v
Before collision the total angular momentum L1 = Iw + mvR
m
After collision the total angular momentum L2 = (I + mR2)w'
L1 = L2 Þ (I+mR2)w' = Iw+mvR.

Iw + mvR
New angular velocity w' = .
I + mR 2
1 2 1
Note : Initial kinetic energy of the system = Iw + mv2.
2 2

1
Final kinetic energy of the system = (I+mR2)w'2
2

E 125
Pre-Medical : Physics
Illustration A light rod carries three equal masses A, B and C as shown in the figure.
What will be the velocity of B in the vertical position of the rod, if it is
released from horizontal position as shown in the figure ?

8gl 4gl 2gl 10gl


(A) (B) (C) (D)
7 7 7 7

Solution. Applying law of conservation of mechenical energy .


Loss in gravitational P.E. = Gain in rotational K.E. i.e.,

1æ æ lö ö
2 2
l æ 2l ö æ 2l ö
mg + mg çè 3 ÷ø + mg l = m
ç çè ÷ø + m ç ÷ + ml2 ÷ w 2
3 2è 3 è 3ø ø

36g 2l 36g 8gl


Þw= Þ v B = wl B = = .
14l 3 14l 7
Illustration A point mass is tied to one end of a cord whose other end passes through a vertical hollow tube,
caught in one hand. The point mass is being rotated in a horizontal circle of radius 2 m with a
speed of 4 m/s. The cord is then pulled down so that the radius of the circle reduces to 1m. Compute
the new linear and angular velocities of the point mass and also the ratio of kinetic energies in
the initial and final states.
Solution. The force on the point mass due to cord is radial and hence the torque about the centre of rotation
is zero. Therefore, the angular momentum must remain constant as the cord is shortened.
Let mass of the particle be m let it rotate initially in circle of radius r1 with linear velocity
v1 and angular velocity w1. Further let the corresponding quantities in the final state be radius
r2, linear velocity v2 and angular velocity w2.
Q Initial angular momentum = Final angular momentum

v1 v v1
\ I1w1 = I2w2 Þ mr12 = mr22 2 Þ r1v1 = r2v2
r1 r2 r1 w1

r2
r1 2 v2 8
\ v2 = v1 = ´ 4 = 8 m/s and w2 = = = 8 rad/s.
r2 1 r2 1
2
1 év ù
I w2 mr22 ´ ê 2 ú
Final K.E. 2 2 2 ë r2 û = v 2 = (8) = 4
2 2
= = .
Initial K.E. 1 é v1 ù
2
v 12 (4)2
I1w12 2
mr1 ´ ê ú
2
ë r1 û
Illustration A thin meter scale is kept vertical by placing its lower end hinged on floor. It is allowed to fall. Calculate
the velocity of its upper end when it hits the floor .

æ mgl ö æ 1 2 1 ml2 v 2 ö
Solution. Loss in PE ç ÷ = gain in rotational KE ç Iw = ´ 2 ÷ Þ v = 3gl .
è 2 ø è2 2 3 l ø

Illustration A solid sphere rolls without slipping on a rough surface and the centre of mass has a constant
speed v0. If the mass of the sphere is m and its radius is R, then find the angular momentum
of the sphere about the point of contact.
r r r r r r r
Solution Q L P = L cm + r ´ pcm = Icmw + R ´ mv cm ; here vcm = v
0 w
v0
Since sphere is in pure rolling motion hence w = v 0R

r 2
è5
v
( ) 7
( )
Þ L p = æç MR2 0 ö÷ - k̂ + Mv0R - k̂ = Mv0R - k̂
Rø 5
( )
126 E
Pre-Medical : Physics
Illustration A body of mass M and radius r, rolling with velocity v on a smooth horizontal floor, rolls up
a rough irregular inclined plane up to a vertical height (3v²/4g). Compute the moment of
inertia of the body and comment on its shape?

1 1
Solution The total kinetic energy of the body E = E + E = Mv² + Iw²
t r
2 2

1
Þ E= Mv² [1 + (I/Mr²)] [as v = rw]
2

When it rolls up on an irregular inclined plane of height h = (3v²/4g), its KE is fully converted

1 2 é I ù é 3v 2 ù
into PE, so by conservation of mechanical energy Mv ê1 + = Mg ê ú which on
2 ë Mr 2 úû ë 4g û
simplification gives I = (1/2) Mr². This result clearly indicates that the body is either a disc
or a cylinder.

E 127
Pre-Medical : Physics
ROTATIONAL MOTION EXERCISE
1. The moment of inertia of a solid cylinder about its 5. Four point masses (each of mass m) are arranged
own axis is the same as its moment of inertia about in the X-Y plane. The moment of inertia of this
an axis passing through its centre of gravity and array of masses about Y-axis is
perpendicular to its length. The relation between (1) ma2 Y
a (a,a)
its length L and radius R is
(2) 2ma 2
a (2a,0)
O a X
(1) L = 2 R (2) L = 3 R (3) 4ma 2
(0,0) a
(4) 6ma2 (a,–a)
(3) L = 3R (4) L = R
6. If the radius of gyration of a solid disc of mass 10 kg
2. Four similar point masses (each of mass m) are about an axis is 0.20 m, then the moment of inertia
placed on the circumference of a disc of mass M of the disc about that axis is
and radius R. The M.I. of the system about the (1) 0.4 kg m2 (2) 0.8 kg m2
normal axis through the centre O will be:-
(3) 1.6 kg m2 (4) 1 kg m2

7. Two rods each of mass m and length l are joined


O at the centre to form a cross. The moment of inertia
of this cross about an axis passing through the
common centre of the rods and perpendicular to
the plane formed by them, is :–
1
(1) MR2 + 4mR2 (2) MR2 + 4mR2 ml2 ml2 ml2 ml2
2 (1) (2) (3) (4)
12 6 3 2
8
(3) MR2 + mR2 (4) none of these 8. Three particles, each of mass m are situated at the
5
vertices of an equilateral triangle ABC of side l
3. The axis X and Z in the plane of a disc are mutually (as shown in the figure).
perpendicular and Y-axis is perpendicular to the The moment of inertia of the system about a line
plane of the disc. If the moment of inertia of the AX perpendicular to AB and in the plane of ABC,
body about X and Y axes is respectively 80 kg will be :-
m2 and 100 kg m2 then M.I. about Z-axis in will X m C
(1) 2 ml2
be :-
l l
(1) 180 kg m2 (2) 40 kg m2 5
(2) ml2
(3) 20 kg m2 (4) Zero 4
l m
3 m
4. The curve for the moment of inertia of a sphere (3) ml2 A B
2
of constant mass M versus its radius will be :-
3
(4) ml2
I
4
I
9. Four identical thin rods each of mass M and length
(1) (2)
l, form a square frame. Moment of inertia of this
R R frame about an axis through the centre of the square
and perpendicular to its plane is :-
I 1 4
(3) (4) I
(1) Ml2 (2) Ml2
3 3

2 13
R R (3) Ml2 (4) Ml2
3 3
128 E
Pre-Medical : Physics
10. Three identical spherical shells, each of mass m and 14. A particle performs uniform circular motion with
radius r are placed as shown in figure. Consider an an angular momentum L. If the frequency of
axis XX' which is touching to two shells and passing
particle's motion is doubled and its kinetic energy
through diameter of third shell. Moment of inertia
halved, the angular momentum becomes
of the system consisting of these three spherical
L L
shells about XX' axis is :- (1) 2L (2) 4L (3) (4)
2 4
(1) 3 mr2 X 15. Point masses m1 and m2 are placed at the opposite
16 ends of a rigid rod of length L, and negligible mass.
(2) mr 2 The rod is to be set rotating about an axis
5
perpendicular to it. The position of point P on this
(3) 4 mr2 rod through which the axis should pass so that the
X' work required to set the rod rotating with angular
11 2
(4) mr velocity w0 is minimum, is given by :-
5
11. A solid sphere and a hollow sphere of the same m2 L
(1) x = m + m w0
mass have the same M.I. about their respective 1 2

diameters. The ratio of their radii will be :- m1L


(2) x = m + m m1 P m2
1 2
(1) 1 : 2 (2) 3: 5 x (L–x)
m1
(3) x = m L
(3) 5: 3 (4) 5 : 4 2

m2
12. The moment of inertia of a ring of mass M and (4) x = m L
1
radius R about PQ axis will be :- r
16. If F is the force acting on a particle having position
r r
D P vector r and t be the torque of this force about the
M origin, then :-
R r r r r
O O' (1) r · t =0 and F · t ¹ 0

r r r r
(2) r · t ¹ 0 and F · t =0
D' Q
r r r r
(3) r · t >0 and F · t < 0

MR 2 r r r r
(1) MR2 (2) (4) r · t = 0 and F · t = 0
2
r
17. A force F = 2iˆ - 3kˆ acts o n a part icle at
3 r r
(3) MR2 (4) 2MR2 r = 0.5ˆj - 2kˆ . The torque t acting on the particle
2
relative to a point with co-ordinat es
13. In the rectangular lamina shown in the figure, (2.0 m, 0, –3.0 m) is
AB = BC/2. The moment of inertia of the lamina
ˆ -m
(1) ( -3.0iˆ - 4.5ˆj - k)N
is minimum along the axis passing through :-

(1) AB A E D ˆ -m
(2) (3iˆ + 6jˆ - k)N
(2) BC
F H
O ˆ -m
(3) ( -20iˆ + 4.0ˆj + k)N
(3) EG
B G C
(4) FH ˆ -m
(4) ( -1.5iˆ - 4.0jˆ - k)N

E 129
Pre-Medical : Physics
18. A uniform rod AB of length l and mass m is free to 22. A constant torque acting on a uniform circular wheel
rotate about A. The rod is released from rest in the changes its angular momentum from A0 to 4 A0
horizontal position. Given that the moment of inertia in 4 seconds. The magnitude of this torque is :–
ml2
of the rod about A is , the initial angular 3A 0
3 (1) (2) A0 (3) 4A0 (4) 12 A0
4
acceleration of the rod will be :–
23. If a ladder is not in balance against a smooth
l vertical wall, then it can be made in balance by :-
A B
(1) Decreasing the length of ladder
(2) Increasing the length of ladder
3g 2g l 3
(1) (2) (3) mg (4) gl (3) Increasing the angle of inclination
2l 3l 2 2
(4) Decreasing the angle of inclination
19. A solid cylinder of mass 50 kg and radius 0.5 m is
24. The figure shows a horizontal block of mass M
free to rotate about the horizontal axis. A massless
suspended by two wires A and B. The centre of
string is wound round the cylinder with one end
mass of the block is closer to B than A. (i) Is the
attached to it and other hanging freely. Tension in
magnitude of the torque due to wire A is greater,
the string required to produce an angular
less or equal to that due to B w.r.t. centre of mass
acceleration of 2 revolutions s–2 is :-
? (ii) Which wire A or B exerts more force on the
(1) 25 N (2) 50 N (3) 78.5 N (4) 157 N block ?

20. A ladder rests against a frictionless vertical wall, with


its upper end 6m above the ground and the lower A B
end 4m away from the wall. The weight of the ladder
is 500 N and its centre of gravity at (1/3)rd distance
from the lower end. Wall’s reaction will be, in N :- (1) (i) greater (ii) B (2) (i) equal (ii) B
(3) (i) less (ii) A (4) (i) greater (ii) A
(1) 111 N (2) 333 N (3) 222 N (4) 129 N
25. If a street light of mass M is suspended from the end
21. In the fig. (a) half of the meter scale is made of of uniform rod of length L in the different possible
wood while the other half of steel. The wooden patterns as shown in figure, then :-
part is pivoted at O. A force F is applied at the
end of steel part. In figure (b) the steel part is
pivoted at O' and the same force is applied at the
wooden end (In horizontal plane) :-

wood steel steel wood


(1) Pattern C is least sturdy
O P O' P' (2) Pattern B is least sturdy
(a) F (b) F (3) Pattern A is least sturdy
(4) All will have same sturdiness
(1) more angular acceleration will be produced 26. A wheel having moment of inertia 2 kg–m2 about
in (a) its vertical axis, rotates at the rate of 60 rpm about
(2) more angular acceleration will be produced the axis. The torque which can stop the wheel’s
in (b) rotation in one minute would be :–
(3) same angular acceleration will be produced p p
(1) N–m (2) N–m
in both conditions 12 15
(4) information is incomplete p 2p
(3) N–m (4) N–m
18 15

130 E
Pre-Medical : Physics
27. A rod of weight W is supported by two parallel knife 32. A small steel sphere of mass m is tied to a string
edges A and B and is in equilibrium in a horizontal of length r and is whirled in a horizontal circle with
position. The knives are at a distance d from each a uniform angular velocity 3w. The string is suddenly
other. The centre of mass of the rod is at pulled, so that radius of the circle is halved. The new
distance x from A. The normal reaction on A is :-
angular velocity will be
Wd W(d – x) (1) 3w (2) 6w (3) 8w (4) 12w
(1) (2)
x x
33. A circular platform is mounted on a frictionless
W(d – x) Wx vertical axle. Its radius R = 2m and its moment of
(3) (4) inertia about the axle is 200 kg m2. It is initially at
d d
rest. A 50 kg man stands on the edge of the
28. A uniform circular disc of radius 50 cm at rest is free
platform and begins to walk along the edge at the
to turn about an axis which is perpendicular to its
speed of 1 ms–1 relative to the ground.Time taken
plane and passes through its centre. It is subjected
by the man to complete one revolution is :-
to a torque which produces a constant angular
acceleration of 2.0 rad/s2. Its net acceleration in m/
p 3p
s2 at the end of 2.0 s is approximately : (1) 2p s (2) s (3) p s (4) s
2 2
(1) 8.0 (2) 7.0 (3) 6.0 (4) 3.0
29. A rope is wound around a hollow cylinder of mass
34. A particle of mass m is projected with a velocity
3 kg and radius 50 cm. What is the angular
v making an angle 45° with the horizontal. The
acceleration of the cylinder if the rope is pulled with
magnitude of the angular momentum of the
a force of 30 N ?
projectile about the point of projection when the
(1) 0.20 rad/s2 (2) 20 rad/s2 particle is at its maximum height h, is :
2 2
(3) 4 m/s (4) 20 m/s

30. A rotating table completes one rotation in 2 sec. mv3


and its moment of inertia is 100 kg-m2. A person (1) zero (2)
4 2g
of 50 kg. mass is standing at the centre of the
rotating table. If the person moves 2m. from the
centre, the angular velocity of the rotating table
will be: mv 3
(3) (4) m2 2gh3
2g
p p
(1) rad/sec (2) rad/sec
3 2
35. The graph between the angular momentum J and
2p angular velocity w for a body will be :-
(3) rad/sec (4) 2p rad/sec
3

31. A thin circular ring of mass M and radius ‘r’ is


J J
rotating about its axis with a constant angular velocity (1) (2)
w. Four objects each of mass m, are kept gently
w w
to the opposite ends of two perpendicular diameters
of the ring. The angular velocity of the ring will be:–

Mw Mw
(1) (2) J J
4m M + 4m (3) (4)
(M + 4m)w (M + 4m)w
(3) (4) w w
M M + 4m

E 131
Pre-Medical : Physics
36. A particle of mass m is rotating in a plane in a 41. A solid sphere is rotating freely about its symmetry
circular path of radius r. Its angular momentum is axis in free space. The radius of the sphere is
L. The centripetal force acting on the particle is increased keeping its mass same. Which of the
following physical quantities would remain constant
L2 L2 m L2 L2
(1) (2) (3) (4) for the sphere ?
mr r mr 2 mr 3
(1) Angular velocity
37. If the earth loses its atmosphere sudeenly, then the
duration of day will :- (2) Moment of inertia
(1) increase (3) Rotational kinetic energy
(2) decrease
(4) Angular momentum
(3) remain unchanged
(4) nothing can be definitely said
42. The rotational kinetic energy of a body is Krot and
38. An ant is sitting at the edge of a rotating disc. If
its moment of inertia is I. The angular momentum
the ant reaches the other end, after moving along
of body is
the diameter, the angular velocity of the disc will:-
(1) remain constant (1) IKrot (2) 2 IK rot
(2) first decreases and then increases
(3) first increases, then decrease (3) 2IK rot (4) 2IKrot
(4) Increase continuously
43. A thin rod of length L is suspended from one end
39. A round disc of moment of inertia I2 about its axis
and rotated with n rotations per second. The
perpendicular to its plane and passing through its
rotational kinetic energy of the rod will be :
centre is placed over another disc of moment of inertia
I1 rotating with an angular velocity w about the same 1
(1) 2mL2p2n2 (2) mL2p2n2
axis. The final angular velocity of the combination 2
of discs is :–
2 1
Iw (3) mL2p2n2 (4) mL2p2n2
(1) w (2) 1 3 6
I1 + I2
44. Two rotating bodies A and B of masses m and 2m
(I + I ) w I2 w
(3) 1 2 (4) with moments of inertia IA and IB (IB > IA) have equal
I1 I1 + I2
kinetic energy of rotation. If LA and LB be their
40. A small mass attached to a string rotates on a
frictionless table top as shown. If the tension in the angular momenta respectively, then :-
string is decreased by relasing the string causing
(1) LB > LA (2) LA > LB
the radius of the circular motion to increase by a
factor of 2, the kinetic energy of the mass will :- LB
(3) LA = (4) LA = 2LB
2

45. A solid sphere of mass m and radius R is rotating


r
about its diameter. A disc of the same mass and same
radius is also rotating about its geometrical axis with
an angular speed twice that of the sphere. The ratio

(1) Increase by a factor of 2 of their kinetic energies of rotation (Esphere / Edisc)


will be :-
(2) Remain constant
(1) 1 : 4 (2) 3 : 1
(3) Increase by a factor of 2
(3) 2 : 3 (4) 1 : 5
(4) Decrease by a factor of 4

132 E
Pre-Medical : Physics
46. Two discs of same moment of inertia rotating about 51. A body of mass m slides down an inclined plane
their regular axis passing through centre and and reaches the bottom with a velocity v. If the same
perpendicular to the plane of disc with angular mass were in the form of a ring which rolls down
velocities w1 and w2 . They are brought into contact this incline, the velocity of the ring at the bottom
face to face coinciding the axis of rotation. The would have been
expression for loss of energy during this process is:-
v æ 2ö
1 (1) v (2) v 2 (3) (4) çç 5 ÷÷ v
(1) I ( w1 - w2 )
2
(2) I ( w1 - w2 )
2 2 è ø
4
52. A ring is rolling without slipping. Its energy of
translation is E. Its total kinetic energy will be :-
I 1
(3) ( w1 - w2 )2 (4) I ( w1 + w2 )
2
(1) E (2) 2E (3) 3E (4) 4E
8 2
53. If rotational kinetic energy is 50% of total kinetic
47. Three objects, A : (a solid sphere), B : (a thin circular energy then the body will be :–
disk) and C = (a circular ring), each have the same (1) ring (2) cylinder
mass M and radius R. They all spin with the same (3) hollow sphere (4) solid sphere
angular speed w about their own symmetry axes. 54. A disc rolls down a plane of length L and inclined
at angle q, without slipping. Its velocity on reaching
The amounts of work (W) required to bring them to
the bottom will be :-
rest, would satisfy the relation :-
(1) WC > WB > WA (2) WA > WB > WC 4gL sin q 2gL sin q
(1) (2)
(3) WB > WA > WC (4) WA > WC > WB 3 3

48. A disc is rolling on an inclined plane without slipping 10gL sin q


then what fraction of its total energy will be in form (3) (4) 4gL sin q
7
of rotational kinetic energy :–
55. A ring takes time t1 and t2 for sliding down and
(1) 1 : 3 (2) 1 : 2 rolling down an inclined plane of length L
respectively for reaching the bottom. The ratio of
(3) 2 : 7 (4) 2 : 5
t1 and t2 is :-
49. A wheel is rolling along the ground with a speed of
5 ms–1. The magnitude of the linear velocity of the (1) 2 : 1 (2) 1 : 2

points at the extermities of the horizontal diameter (3) 1 : 2 (4) 2 : 1


of the wheel is equal to 56. A solid cylinder of mass M and radius R rolls without
slipping down an inclined plane of length L and height
(1) 5 10 ms -1 (2) 5 3 ms -1 h. What is the speed of its centre of mass when the
cylinder reaches its bottom :–
(3) 5 2 ms -1 (4) 5 ms–1 3
(1) 2 gh (2) gh
4
50. Calculate the ratio of the times taken by a uniform 4
(3) gh (4) 4 gh
solid sphere and a disc of the same mass and the 3
same diameter to roll down through the same 57. A sphere and a disc of same radii and mass are
distance from rest on a inclined plane. rolling on an inclined plane without slipping. as &
ad are acceleration and g is acceleration due to
(1) 15 : 14 (2) 15 : 14
gravity. Then which statement is correct ?

(3) 152 : 142 (4) 14 : 15 (1) as > ad > g (2) g > as > ad

(3) as > g > ad (4) ad > as > g

E 133
Pre-Medical : Physics
58. A disc of mass M and radius R rolls on a horizontal 59. A solid cylinder of mass 12 kg is rolling on a rough
surface and then rolls up an inclined plane as shown horizontal surface with velocity 4 ms–1. It collides with
in the figure. If the velocity of the disc is v, the height a horizontal spring of force constant 200 Nm–1. The
to which the disc will rise will be :- maximum compression produced in the spring will
be :-

(1) 0.7 m (2) 0.4 m

(3) 0.6 m (4) 1.2 m


h
v 60. A disk and a sphere of same radius but different
masses roll off on two inclined planes of the same
altitude and length. Which one of the two objects

3v 2 3v 2 gets to the bottom of the plane first ?


(1) (2) (1) Disk
2g 4g
(2) Sphere

v2 v2 (3) Both reach at the same time


(3) (4)
4g 2g (4) Depends on their masses

ANSWER KEY
Que. 1 2 3 4 5 6 7 8 9 10 11 12 13 14 15
Ans. 2 2 3 1 4 1 2 2 2 3 3 3 4 4 1
Que. 16 17 18 19 20 21 22 23 24 25 26 27 28 29 30
Ans. 4 4 1 4 1 2 1 3 2 1 2 3 1 2 1
Que. 31 32 33 34 35 36 37 38 39 40 41 42 43 44 45
Ans. 2 4 1 2 1 4 2 3 2 4 4 3 3 1 4
Que. 46 47 48 49 50 51 52 53 54 55 56 57 58 59 60
Ans. 1 1 1 3 4 3 2 1 1 2 3 2 2 4 2

134 E
Pre-Medical : Physics
GRAVITATION
w Newton's law of gravitation r
m1 m2
Gm1m 2
Force of attraction between two point masses F =
r2
Directed along the line joining of point masses.

• It is a conservative force field Þ mechanical energy is conserved.


• It is a central force field Þ angular momentum is conserved.

w Gravitational field due to spherical shell E

GM
r Outside the shell Eg= , where r > R r=R
r2 r
GM I=0
r On the surface Eg = , where r=R
R2 –GM
2
r Inside the shell Eg = 0, where r<R R
[Note : Direction always towards the centre of the sphere] E

w Gravitational field due to solid sphere


GM r=R
r Outside the sphere Eg = , where r > R r
r2
GM Iµr
r On the surface Eg = , where r=R
R2 –GM
2
GMr R
r Inside the sphere Eg = , where r< R
R3 g
w Acceleration due to gravity g = GM
2
(at surface)
R GM
r At height h above surface of earth, R2

GM
If h << R ; gh » gs æç 1 -
2h ö
gh = (R+h)2
è R ÷ø O r
R
(Taking direction towards
GM(R - d) æ dö
r At depth d below the surface of earth, gd= =gs çè 1 - ÷ø centre of earth as positive)
R3 R
r Effect of rotation on g
g' = g–w2Rcos2l where l is angle of latitude.

Condition of weightlessness on Earth's surface

If apparent weight of body is zero then angular speed of Earth can be calculated as mg' = mg – mRew2 cos2l

1 g
0 = mg – mRew2 cos2l Þ w =
cos l R e

g 1
But at equator l = 0° \ w= = rad/s = 0.00125 rad/s =1.25 × 10–3 rad/s.
R e 800
Note : If Earth were to rotate with 17 times of its present angular speed then bodies lying on equator
would fly off into the space. Time period of Earth's rotation in this case would be 1.4 h.
w Gravitational potential
GM
Due to a point mass at a distance V = –
r

E 135
Pre-Medical : Physics
w Gravitational potential due to spherical shell V

GM R
r Outside the shell V = – , r>R O r
r

GM GM
r Inside/on the surface the shell V = – , r£ R –
R R

w Potential due to solid sphere

V
GM
r Outside the sphere V = – , r > R
r R
O r

GM
r On the surface V = – , r = R
R GM

R

r Inside the sphere V = –


(
GM 3R 2 - r 2 ), r < R

3GM
2R
3
2R

GM
w Potential on the axis of a V = –
R2 + x2
thin ring at a distance x
from centre

2GM
w Escape velocity from a planet of ve =
R
mass M and radius R

w Orbital velocity of satellite v0 = GM GM


=
r (R + h)

GM v
r For nearby satellite v0 = = e
R 2

Here ve = escape velocity

w ve = 2v 0 , for near by setellite v0 = 8 km/s

2pr 2 pr 3 / 2
w Time period of satellite T= v =
GM

w Energies of a satellite

GMm E K.E.
r Potential energy U =- N
r
E
r
1 GMm R
r Kinetic energy K = mv 2 = G
E.

2 2r
T.

Y
GMm P.E.
r Mechanical energy E = U + K =-
2r

GMm
r Binding energy BE=–E =
2r

136 E
Pre-Medical : Physics
w Kepler's laws
r Ist Law : Law of orbit
Path of a planet is elliptical with the sun at a focus.

dA L
r IInd Law : Law of area : Areal velocity = constant =
dt 2m

3
r IIIrd Law : Law of periods T2 µ a3 or T2 µ æç rmax + rmin ö÷ µ ( mean radius )3
è 2 ø

For circular orbits T2 µ R3

KEY POINTS

• At the centre of earth, a body has centre of mass, but no centre of gravity.

• The centre of mass and centre of gravity of a body coincide if gravitation field is uniform.

• You do not experience gravitational force in daily life due to objects of same size as value of G is very small.

• Moon travellers tie heavy weight at their back before landing on Moon due to smaller value of g at Moon.

• Space rockets are usually launched in equatorial line from West to East because g is minimum at equator
and earth rotates from West to East about its axis.

• Angular momentum in gravitational field is conserved because gravitational force is a central force.

• Kepler's second law or constancy of areal velocity is a consequence of conservation of angular momentum.

Illustration Two stationary particles of masses M1 and M2 are 'd' distance apart. A third particle lying on the
line joining the particles, experiences no resultant gravitational force. What is the distance of this
particle from M1?
Solution Let m be the mass of the third particle

GM1m
Force on m towards M1 is F1 = M1 m M2
r2
r
d

GM2 m
Force on m towards M2 is F2 =
(d - r)2

Since net force on m is zero \ F1 = F2

GM1m GM2 m æ d - rö M
2
d M2 é M1 ù
Þ = Þ ç ÷ = 2 Þ -1 = Þ r = dê ú
r2 ( d - r)
2
è r ø M1 r M1 êë M1 + M2 úû

Illustration Three masses, each equal to M are placed at the three corners of a square of side a. Calculate
the force of attraction on unit mass placed at the fourth corner.

® ® GM
Solution Force on m = 1 due to masses at corners 1 and 3 are F1 and F3 with F1 = F3 = resultant
a2

® ®
GM
of F1 and F3 is Fr = 2 and its direction is along the diagonal i.e. toward corner 2
a2

E 137
Pre-Medical : Physics
M F1 m=1
1
F2
F3
a Fr

M a M
2 3

GM GM
Force on m due to mass M at 2 is F2 = = ; Fr and F2 act in the same direction.
( 2a) 2
2a2

Resultant of these two is the net force :

2GM GM GM é 1ù
Fnet = + 2 = 2 ê 2 + ú ; it is directed along the diagonal as shown in the figure.
a 2
2a a ë 2û

Illustration Three particles, each of mass m, are situated at the vertices of an equilateral triangle of side 'a'.
The only forces acting on the particles are their mutual gravitational forces. It is intended that
each particle moves along a circle while maintaining their original separation 'a'. Determine the
initial velocity that should be given to each particle and the time period of the circular motion.

Solution The resultant force on particle at A due to other two particles is

Gm
2 é Gm2 ù
...(i) êQ FAB = FAC = a2 ú
2 2
FA = FAB + FAC + 2FAB FAC cos 60° = 3
a2 ë û

a
Radius of the circle r =
3

If each particle is given a tangential velocity v, so that the resultant force acts as the centripetal
force,
A m
mv 2 mv 2
then = 3 ...(ii) F AB F AC
r a
FA
O
mv
2
Gm
2
3 Gm
From (i) and (ii) , 3 = Þ v= m m
a a
2
a a
B C

2pr 2pa a a3
Time period T = = = 2p .
v 3 Gm 3Gm

Illustration Two solid spheres of same size of a certain metal are placed in contact with each other.
M M
Prove that the gravitational force acting between them is directly
R
proportional to the fourth power of their radius.
Solution The weights of the spheres may be assumed to be concentrated R
at their centres.

é4 ù é4 ù
G ê pR 3r ú ´ ê pR 3rú
ë 3 û ë 3 û = 4 (Gp2r2 )R 4
So F= \ F µ R4
(2R)2 9

138 E
Pre-Medical : Physics
Illustration A body of mass m is placed on the surface of earth. Find the work required to lift this body by
a height

Re
(i) h = (ii) h = Re
1000

Re
Solution (i) h= , as h << Re , so
1000

æ GMe ö æ R e ö GMe m
we can apply Wext = mgh ; Wext = (m) ç 2 ÷ç ÷=
è R e ø è 1000 ø 1000R e
(ii) h = Re , in this case h is not very less than Re, so we cannot apply DU = mgh

éæ GMe ö æ GMe ö ù GMem


Wext = Uf - Ui= m(Vf - Vi) ; Wext = m êç - ÷ – ç- ÷ ú ; Wext = .
êëè R e + R e ø è R e ø úû 2R e

Illustration If velocity given to an object from the surface of the Earth is n times the escape velocity then
what will be its residual velocity at infinity ?

1 GMm 1
Solution Let the residual velocity be v, then from energy conservation m(nve)2 – = mv2 + 0
2 R 2

Þ v2 = n2ve2 –
2GM
= n2ve2 – ve2 = (n2 – 1) v2e Þ v = F I
n 2 - 1 ve.
R H K
Illustration A narrow tunnel is dug along the diameter of the earth, and a particle
R
of mass m0 is placed at distance from the centre. Find the escape
2
speed of the particle from that place.
Solution Suppose we project the particle with speed ve, so that it just reaches infinity (r ® ¥).
Applying energy conservation principle Me,R

Ki + Ui = Kf + Uf
R/2 at r®¥, v ®0
1 é GM ì æ R ö üù
2
m0 ve m0
m0 v 2e + m0 ê - e
í 3R 2
- ç ÷ ýú = 0
2 3
ë 2R î è 2 ø þû

11GM e
Þ ve = .
4R

Illustration A particle is projected vertically upwards from the surface of the earth (radius Re) with a speed equal
to one fourth of escape velocity. What is the maximum height attained by it ?

1 GMm GMm
Solution From conservation of mechanical energy, mv2 = R - R
2 e

1 1 2GM
Where R = maximum distance from centre of the earth Also v = ve =
4 4 Re

1 1 2GM GMm GMm 16 Re


Þ m´ ´ = - ÞR = R Þh = R–Re = .
2 16 Re Re R 15 e 15

E 139
Pre-Medical : Physics
ALLEN
Illustration Gravitational potential difference between a point on the surface of a planet and point 10 m above
is 4 J/kg. Considering the gravitational field to be uniform, how much work is done in moving
a mass of 2 kg from the surface to a point 5 m above the surface?

DV æ -4 ö 4
Solution Gravitational field g = - = -ç ÷ = J / kg - m
Dx è 10 ø 10
Work done in moving a mass of 2 kg from the surface to a point 5 m above the surface,

æ 4 J ö
W = mgh = (2 kg) ç (5 m) = 4 J
è 10 kg - m ÷ø

Illustration Two satellites S1 and S2 are revolving round a planet in coplanar and concentric circular orbits
of radii R1 and R2 in the same sense respectively. Their respective periods of revolution are 1
h and 8 h. The radius of the orbit of satellite S1 is equal to 104 km. Find the relative speed in
km/h when they are closest.

T2 T12 T22 1 64
Solution By Kepler's 3rd law, = constant \ = or (104 )3 = R 3 or R2 = 4 × 104 km
R3 R13 R 23 2

Distance travelled in one revolution, S1 = 2pR1 = 2p × 104 and S2 = 2pR2 = 2p × 4 × 104

S1 2p ´ 104 S 2p ´ 4 ´ 104
v1 = = = 2p ´ 104 km / h and v 2 = 2 = = p ´ 104 km / h
t1 1 t2 8
\ Relative velocity = v1 – v2 = 2p × 104 – p × 104 = p × 104 km/h

Illustration A planet is revolving round the sun in an elliptical orbit as shown in figure. Select correct alternative(s)
B
(A) Its total energy is negative at D. C

(B) Its angular momentum is constant


D ||||
A
||||
||||

||||
(C) Net torque on the planet about sun is zero O

(D) Linear momentum of the planet is conserved E


F
Solution (Ans. A, B, C)

For (A) : For a bound system, the total energy is always negative.

For (B) : For central force field, angular momentum is always conserved.

For (C) : For central force field, torque = 0.

For (D) : In presence of external force, linear momentum is not conserved.

140 E
Pre-Medical : Physics
GRAVITATION EXERCISE
1. Three identical point masses, each of mass 1 kg lie 6. Mars has a diameter of approximately 0.5 of
in the x–y plane at points (0,0) (0, 0.2m) and (0.2m, that of earth, and mass of 0.1 of that of earth.
0) respectively. The gravitational force on the mass The surface gravitational field strength on mars
as compared to that on earth is a factor of –
at the origin is :–
(1) 0.1 (2) 0.2
(1)1.67 x 10–11 (iˆ + ˆj) N
(3) 2.0 (4) 0.4
(2) 3.34 x 10–10 (iˆ + ˆj) N
(3) 1.67 x 10–9 (iˆ + ˆj) N 7. A stone dropped from a height 'h' reaches the
(4) 3.34 x 10–10 (iˆ – ˆj) N Earth's surface in 1 s. If the same stone is taken to
Moon and dropped freely from the same height then
2. Four particles of masses m, 2m, 3m and 4m are
it will reach the surface of the Moon in a time (The
kept in sequence at the corners of a square of side
a. The magnitude of gravitational force acting on 'g' of Moon is 1/6 times that of Earth) :–
a particle of mass m placed at the centre of the (1) 6 seconds (2) 9 seconds
square will be : (3) 3 seconds (4) 6 seconds

24m2 G 6m2 G 8. At which height from the earth's surface does the
(1) (2)
a2 a2 acceleration due to gravity decrease by 75% of its
value at earth's surface ?
4 2Gm 2
(1) 6400 Km (2) 3200 Km
(3) (4) Zero
a2 (3) 1600 Km (4) 12800 Km

9. Suppose the acceleration due to gravity at the


3. During the journey of space ship from earth to moon
earth's surface is 10m/s2 and at the surface of mars
and back, the maximum fuel is consumed :-
it is 4.0 m/s2. A 60kg passenger goes from the earth
(1) Against the gravitation of earth in return journey to the mars in a spaceship moving with a constant
velocity. Neglect all other objects in the sky. Which
(2) Against the gravitation of earth in onward journey
part of figure best represent the weight (Net
(3) Against the gravitation of moon while reaching gravitational force) of the passenger as a function
the moon of time :
(4) None of the above Weight (N)
600
4. The mass of the moon is 1% of mass of the earth.
The ratio of gravitational pull of earth on moon to 400
A
that of moon on earth will be : B
240
C
200
(1) 1 : 1 (2) 1 : 10 (3) 1 : 100 (4) 2 : 1

5. If the distance between the centres of earth and D Time

moon is D and mass of earth is 81 times that of


(1) A (2) B (3) C (4) D
moon. At what distance from the centre of earth
gravitational field will be zero : 10. Acceleration due to gravity at earth's surface
is 'g' ms–2. Find the effective value of acceleration
D 2D
(1) (2) due to gravity at a height of 32 km from sea
2 3
level : (Re = 6400 Km)
(1) 0.5 g ms–2 (2) 0.99 g ms–2
4D 9D
(3) (4) (3) 1.01 g ms –2
(4) 0.90 g ms–2
5 10
E 141
Pre-Medical : Physics
11. The dependence of acceleration due to gravity 'g' 16. When the radius of earth is reduced by 1% without
on the distance 'r' from the centre of the earth, changing the mass, then the acceleration due to
assumed to be a sphere of radius R of uniform gravity will
density, is as shown in figure below :- (1) increase by 2% (2) decrease by 1.5%
(3) increase by 1% (4) decrease by 1%
g g
17. Weight of a body of mass m decreases by 1% when
it is raised to height h above the earth's surface. If
(a) (b)
the body is taken to a depth h in a mine, then in
r r its weight will
R R
(1) decrease by 0.5% (2) decrease by 2%
g g (3) increase by 0.5% (4) increase by 1%
18. Read the following statements :
(c) (d) S1 : An object shall weigh more at pole than at
r r equator when weighed by using a physical balance.
R R
S2 : It shall weigh the same at pole and equator
The correct figure is :-
when weighed by using a physical balance.
(1) (a) (2) (b) (3) (c) (4) (d)
S3 : It shall weigh the same at pole and equator when
12. One can easily “weigh the earth” by calculating the
weighed by using a spring balance.
mass of earth using the formula (in usual notation)
S4 : It shall weigh more at the pole than at equator
G 2 g 2 g G 3 when weighed using a spring balance.
(1) RE (2) RE (3) RE (4) RE
g G G g
Which of the above statements is /are correct ?
13. The value of 'g' reduces to half of its value at surface
(1) S1 and S2 (2) S1 and S4
of earth at a height 'h', then :-
(3) S2 and S3 (4) S2 and S4.
(1) h = R (2) h = 2R
19. A spherical planet has a mass Mp and diameter Dp.
(3) h = ( 2 +1 R ) (4) h = ( 2 -1 R) A particle of mass m falling freely near the surface
14. If the earth stops rotating suddenly,the value of g
of this planet will experience an aceleration due to
at a place other than poles would :-
gravity, equal to :-
(1) Decrease
(2) Remain constant (1) GMp/DP2 (2) 4GMpm/Dp2

(3) Increase (3) 4GMp/Dp2 (4) GMpm/ Dp2

(4) Increase or decrease depending on the position 20. Starting from the centre of the earth having radius
of earth in the orbit round the sun R, the variation of g (acceleration due to gravity) is
shown by :-
15. Gravitation on moon is 1/6th of that on earth. When
a balloon filled with hydrogen is released on moon
then, this :- g g

æ gö
(1) Will rise with an acceleration less then ç ÷ (1) (2)
è6ø
O R r O R r
ægö
(2) Will rise with acceleration ç ÷
è6ø
æ 5g ö
(3) Will fall down with an acceleration less than ç ÷ g g
è 6 ø
ægö (3) (4)
(4) Will fall down with acceleration ç ÷
è6ø O r O r
R R

142 E
Pre-Medical : Physics
21. If the mass of the Earth were six times smaller and 25. At what height from the surface of earth the
the universal gravitational constant were six time gravitation potential and the value of g are
larger in magnitude, which of the following is not –5.4 × 107 J/kg2 and 6.0 m/s 2 respectively ?
correct ? Take the radius of earth as 6400 km :
(1) Raindrops will fall faster (1) 1600 km (2) 2600 km
(3) 1400 km (4) 2000 km
(2) Walking on the ground would become more
26. If Me is the mass of earth and Mm is the mass of
difficult
moon (Me = 81 Mm). The potential energy of an
(3) Time period of a simple pendulum on the Earth object of mass m situated at a distance R from the
would decrease centre of earth and r from the centre of moon, will
(4) 'g' on the Earth will not change be :-
22. The intensity of gravitational field at a point situated æ R ö 1 æ 81 1 ö
at a distance 8000 km from the centre of Earth is (1) -GmM m ç + r÷ 2 (2) -GmM e ç + ÷
è 81 øR è r Rø
6.0 N/kg. The gravitational potential at that point
in N-m/kg will be :– æ 81 1 ö æ 81 1 ö
(3) -GmM m ç + ÷ (4) GmM m ç - ÷
(1) 6 (2) 4.8 × 107 è R rø è R rø
(3) 8 × 105 (4) 4.8 × 102 27. A body of mass m is situated at a distance 4Re above
23. A particle of mass M is situated at the centre of a the Earth's surface, where Re is the radius of Earth.
spherical shell of same mass and radius a. The What minimum energy should be given to the body
a so that it may escape ?
gravitational potential at a point situated at (1) mgRe (2) 2mgRe
2
distance from the centre, will be :- mgR e mgR e
(3) (4)
5 16
4GM 3GM 2GM GM
(1) - (2) - (3) - (4) - 28. The ratio of radii of two satellites is p and the ratio
a a a a
of their acceleration due to gravity is q. The ratio
24. Which of the following curve expresses the variation
of their escape velocities will be :
of gravitational potential with distance for a hollow
sphere of radius R :
1 1
æqö 2
æpö 2
(1) ç ÷ (2) ç ÷ (3) pq (4) pq
èpø èqø
R
29. A black hole is an object whose gravitational field
V
(1) is so strong that even light cannot escape from it.
r To wha t approximate radius wou ld e arth
(mass = 5.98 × 1024 kg) have to be compressed to
R r be a black hole ?
(1) 10–9 m (2) 10–6 m (3) 10–2 m (4) 100 m
(2) 30. A particle falls on earth :
V
(i) from infinity, (ii) from a height 10 times the radius
of earth. The ratio of the velocities gained on
R r reaching at the earth's surface is :

(1) 11 : 10 (2) 10 : 11
(3) V
(3) 10 : 11 (4) 11 : 10
31. Escape velocity of a body from earth is 11.2 km/s.
Escape velocity, when thrown at an angle of 45°
R
from horizontal will be :-
r (1) 11.2 km/s (2) 22.4 km/s
(4) V
(3) 11.2/ 2 km/s (4) 11.2 2 km/s

E 143
Pre-Medical : Physics
32. The escape velocity from the earth is 11.2 km/s the 38. The relay satellite transmits the television
mass of another planet is 100 times of mass of earth programme continuously from one part of the world
and its radius is 4 times the radius of earth. The to another because its :
(1) Period is greater than the period of rotation of
escape velocity for the planet is :-
the earth about its axis
(1) 56.0 km/s (2) 280 km/s (2) Period is less than the period of rotation of the
(3) 112 km/s (4) 11.2 km/s earth about its axis
(3) Period is equal to the period of rotation of the
33. A projectile is fired verticaly upward from the sur-
earth about its axis
face of earth with a velocity KVe, where Ve is the (4) Mass is less than the mass of earth
escape velocity and K < 1. Neglecting air resistance, 39. Two identical satellites are at the heights R and 7R
the maximum height to which it will rise measured from the earth's surface. Then which of the following
from the centre of the earth is : (Where are R = radius statement is incorrect :–
of earth) :- (R = Radius of the earth)
(1) Ratio of total energy of both is 5
R R
(1) (2) (2) Ratio of kinetic energy of both is 4
1 - K2 K2
(3) Ratio of potential energy of both 4
(4) Ratio of total energy of both is 4
1 - K2 K2
(3) (4) 40. Two satellites of earth, S1 and S2, are moving in
R R
the same orbit. The mass of S1 is four times the mass
of S2. Which one of the following statements is true ?
34. The ratio of escape velocity at earth (ve) to the escape
velocity at a planet (vp) whose radius and mean (1) The kinetic energies of the two satellites are
density are thrice as that of earth is :- equal
(2) The time period of S1 is four times that of S2
(1) 1 : 3 (2) 1 : 3 3 (3) The potential energies of earth and satellite in
(3) 1 : 9 (4) 1 : the two cases are equal
3
(4) S1 and S2 are moving with the same speed
35. If the gravitational force were to vary inversely as 41. The radii of circular orbits of two satellites A and
mth power of the distance, then the time period of B of the earth, are 4R and R, respectively. If the
a planet in circular orbit of radius r around the Sun speed of satellite A is 3v, then the speed of satellite
B will be :-
will be proportional to :-
(1) 3v/2 (2) 3v/4 (3) 6v (4) 12v
(1) r–3m/2 (2) r3m/2 42. A remote - sensing satellite of earth revolves in a
(3) r m+1/2
(4) r (m+1)/2 circular orbit at a height of 0.25 ×106 m above the
surface of earth. If earth's radius is 6.38 × 106 m
36. Two satellites A and B, having ratio of masses
and g=9.8 ms–2, then the orbital speed of the
3 : 1 are in circular orbits of radius r and 4r.
satellite is :
Calculate the ratio of total mechanical energies of
(1) 6.67 km s–1 (2) 7.76 km s–1
A to B.
(3) 8.56 km s–1 (4) 9.13 km s–1
(1) 4 : 1 (2) 12 : 1 (3) 1 : 12 (4) 6 : 1 43. Two solid spherical planets of equal radii R having
37. A satellite of earth of mass 'm' is taken from orbital masses 4M and 9M their centre are separated by
radius 2R to 3R, then minimum work done is :- a distance 6R. A projectile of mass m is sent from
the planet of mass 4 M towards the heavier planet.
GMm GMm What is the distance r of the point from the centre
(1) (2)
6R 12R of lighter planet where the gravitational force on
the projectile is zero ?
GMm GMm
(3) (4) (1) 1.4 R (2) 1.8 R (3) 1.5 R (4) 2.4 R
24R 3R
144 E
Pre-Medical : Physics
44. An earth's satellite is moving in a circular orbit with 49. The orbital velocity of an artificial satellite in a
a uniform speed v. If the gravitational force of the circular orbit just above the earth’s surface is v0. The
earth suddenly disappears, the satellite will :- orbital velocity of satellite orbiting at an altitude of
half of the radius is :-
(1) vanish into outer space
(2) continue to move with velocity v in original orbit 2
3
(1) v0 (2) v0
(3) fall down with increasing velocity 2 3
(4) fly off tangentially from the orbit with velocity v
2 3
45. One projectile after deviating from its path starts (3) v0 (4) v0
3 2
moving round the earth in a cirular path of radius
equal to nine times the radius of earth R. Its time 50. A satellite of mass m is orbiting the earth
period will be :-
(of radius R) at a height h from its surface. The total
R R energy of the satellite in terms of g0, the value of
(1) 2p (2) 27 ´ 2p acceleration due to gravity at the earth's surface, is:-
g g

R R 2mg0 R2 2mg 0 R 2
(3) p (4) 0.8 ´ 3p (1) (2) -
g g R+h R +h

46. Potential energy and kinetic energy of a two particle mg0 R 2 mg0 R2
system under imaginary force field are shown by (3) (4) -
2(R + h) 2(R + h)
curves KE and PE. respectively in figure. This system
is bound at : 51. A planet is revolving round the sun. Its distance from
(1) only point A the sun at Apogee is rA and that at Perigee is rp.
The mass of planet and sun is m and M respectively,
Energy

(2) only point D KE Distance


® vA and vP is the velocity of planet at Apogee and
(3) only point A B C Perigee respectively and T is the time period of
D
A, B, and C revolution of planet round the sun.
PE
(4) All points A,
p2 p2
( rA + rP ) ( rA + rP )
2 2 2 3
B, C and D (a) T = (b) T =
2Gm 2GM
47. If the length of the day is T, the height of that TV (c) vArA = vPrP (d) vA < vP , rA > rP
satellite above the earth's surface which always
appears stationary from earth, will be : (1) a, b, c (2) a, b, d

1 1
(3) b, c, d (4) all
é 4p2 GM ù
3
é 4p2 GM ù 2
(1) h = ê ú (2) h = ê ú -R
2 2 52. The figure shows elliptical orbit of a planet m about
ë T û ë T û
the sun S. The shaded area SCD is twice the shaded
1 1

é GMT 2 ù 3 é GMT2 ù
3
area SAB. If t1 is the time for the planet to move
(3) h = ê 2 ú
-R (4) h = ê 2 ú
+R
ë 4p û ë 4p û from C to D and t2 is the time to move from A to
48. A communication satellite of earth which takes 24 B then :-
hrs. to complete one circular orbit eventually has
v
to be replaced by another satellite of double mass. (1) t1 = t2 m
If the new satellites also has an orbital time period B C
of 24 hrs, then what is the ratio of the radius of the (2) t1 < t2
S
new orbit to the original orbit ?
A D
(3) t1 = 4t2
(1) 1 : 1 (2) 2 : 1
(4) t1 = 2t2
(3) 2 :1 (4) 1 : 2

E 145
Pre-Medical : Physics
53. A planet moving along an elliptical orbit is closest
to the sun at a distance r 1 and farthest away at a(aphelion)
a distance of r2. If v1 and v2 are the linear velocities
ra
v1
at these points respectively, then the ratio v
2
rp earth
is :-
(1) (r1/r2)2 (2) r2/r1 (3) (r2/r1)2 (4) r1/r2 P(perihelion)
54. A satellite S is moving in an elliptical orbit around r r r r
(i) L a = L P (ii) L a = –L P
the earth. The mass of the satellite is very small
compared to the mass of the earth. Then, r r r r
(iii) ra ´ L a = rP ´ L P
(1) the acceleration of S is always directed towards
Which of the above relations is/are true ?
the centre of the earth.
(1) (i) only (2) (ii) only
(2) the angular momentum of S about the centre of
(3) (iii) only (4) (i) and (iii)
the earth changes in direction, but its magnitude
57. Two astronauts are floating in gravitational free
remains constant.
space after having lost contact with their spaceship.
(3) the total mechanical energy of S varies
The two will :-
periodically with time.
(1) Move towards each other.
(4) the linear momentum of S remains constant in
(2) Move away from each other.
magnitude.
(3) Will become stationary
55. The maximum and minimum distances of a comet
(4) Keep floating at the same distance between them.
from the sun are 8 × 1012 m and 1.6 × 1012 m
58. The speed of a planet in an elliptical orbit about the
respecting. If its velocity when it is nearest to the
Sun, at positions A, B and C are VA, VB and VC
sun is 60 m/sec then what will be its velocity in
m/s when it is farthest ? respectively. AC is the major axis and SB is
(1) 12 (2) 60 (3) 112 (4) 6 perpendicular to AC at the position of the Sun S
56. Consider a satellite orbiting the earth as shown in as shown in the figure. Then
B
the figure below. Let La and Lp represent the
angular momentum of the satellite about the earth
A C
when at aphelion and perihelion respectively. S
Consider the following relations.
(1) VA < VB < VC (2) VA > VB > VC
(3) VB < VA < VC (4) VB > VA > VC

ANSWER KEY
Que. 1 2 3 4 5 6 7 8 9 10 11 12 13 14 15
Ans. 3 3 2 1 4 4 1 1 3 2 4 2 4 3 4
Que. 16 17 18 19 20 21 22 23 24 25 26 27 28 29 30
Ans. 1 1 4 3 4 4 2 2 3 2 3 3 4 3 1
Que. 31 32 33 34 35 36 37 38 39 40 41 42 43 44 45
Ans. 1 1 1 2 4 2 2 3 1 4 3 2 1 4 2
Que. 46 47 48 49 50 51 52 53 54 55 56 57 58
Ans. 3 3 1 3 4 3 4 2 1 1 1 4 2

146 E
Pre-Medical : Physics
PROPERTIES OF MATTER AND FLUID MECHANICS
ELASTICITY

Internal restoring force F


STRESS = Area of cross - section = A .
Re s

There are three types of stress :-

w Longitudinal Stress

F F ll
wa
(a) Tensile Stress : l Dl
F
tensile stress
tensile stress

F
Dl
l Compressive
Stress
(b) Compressive Stress :

• Volume Stress / Hydraulic


A F
B
• Tangential Stress or Shear Stress L

Change in dim ension of the body


w Strain = F
Original dim ension of the body D C
l
change in length of the body DL
w Longitudinal strain = initial length of the body = L F
A
l A' B B'
L
change in volume of the body DV f
w Volume / Hydraulic strain= original volume of the body = V F
D C

l l displacement of upper face with respect to lower face


w Shear strain : tan f = for small angle f= =
L L distance between two faces

Breaking Stress

The stress required to cause the actual fracture of a material is called


the breaking stress or ultimate strength.

F
Breaking stress = ; F = force required to break the body.
A
Dependence of breaking stress : (i) Nature of material (ii) Temperature (iii) Impurities.

Independence of breaking stress : (i) Cross sectional area or thickness (ii) Applied force.

Maximum load (force) which can applied on the wire depends on

(i) Cross sectional area or thickness (ii) Nature of material (iii) Temperature (iv) Impurities.

E 147
Pre-Medical : Physics
w Relation between angle of twist (q) & angle of shear (f)
fixed B
AA' = r q and Arc AA'=lf

rq
So rq = lf Þ f = f
l q'
l r
where q = angle of twist,
q
f = angle of shear O
tw is A A'
te d
w Stress – Strain Graph

Breaking B
strength

Elastic
limit Elastic E Y C
region Plastic Region
lim rtion
po
it

Stress
Pro

0 Strain

w Hooke's Law within elastic limit Stress µ strain

Longitudinal stress Fl
w Young's modulus of elasticity Y = =
Longitudinal strain A Dl
w If L is the length of wire, r is radius and l is the increase in length of the wire by suspending a weight Mg

at its one end then Young's modulus of elasticity of the material of wire Y = ( Mg / pr ) = MgL
2

2
(l / L) pr l
MgL r g L2
w Increment in length due to own weight Dl = =
2AY 2Y

w Bulk modulus of elasticity K = Volume stress = F / A = P


Volume strain æ -DV ö æ -DV ö
èç V ø÷ èç V ø÷

1 1
w Compressibility C = =
Bulk modulus K

shearing stress ( Ftan gential ) / A


w Modulus of rigidity h = =
shearing strain f

æ DD ö
lateral strain b ç where b = - D ÷
w Poisson's ratio ( s ) = = ç ÷
Longitudinal strain a çç DL ÷
a= ÷
è L ø
–1 £ s £ 0.5 (theoritical limit), s » 0.2 to 0.4 (practical limit)
w Work done in stretching wire
W = ½ × Y × (strain)2 × volume ; W = ½ (stress) (strain) (volume).
1 F Dl 1
W= ´ ´ × A × l = F × Dl
2 A l 2

(Stress)2
Energy density/energy stored per unit volume =
2Y
148 E
Pre-Medical : Physics
Factor Affecting Elasticity
l Effect of Temperature
T ­ Þ Y¯ Due to weekness of intermolecular force.
When temperature is increased, the elastic properties in general decreases i.e. elastic constants decrease.
Plasticity increases with temperature.
For a special kind of steel, elastic constants do not vary appreciably with temperature. This steel is called
INVAR steel.
l Interatomic Force Constant :
k or ka = Y . r0
Y = Young's modulus ; r0 = interatomic distance under normal circumstances
l Relation between Y, K, h and s : To be remembered

9 3 1
Y = 3K (1–2s), Y = 2h (1+s), = + .
Y h K

HYDROSTATICS
mass
w Density =
volume

weight
w Specific weight = = rg
volume

density of given any body


w Relative density =
density of pure water at 4°C

w Density of a Mixture of substance in the proportion of mass

M + M + M ....
the density of the mixture is r = M 1 M 2 M 3
1 2
+ 3
+ + ....
r1 r2 r3

w Density of a mixture of substance in the proportion of volume


r1 V1 + r2 V2 + r3 V3
the density of the mixture is r =
V1 + V2 + V3 + ....

2r1r2
If liquids with same masses are mixed i.e. m1 = m2 = m then rmix. = (Harmonic mean of individual
r1 + r2

densities)
r1 + r2
If liquids with same volumes are mixed i.e. V1 = V2 = V then rmix. = (Arithmetic mean of individual
2
densities)

w Pressure = Normal force = Thrust


Area Area
w Variation of pressure with depth
Pressure is same at two points in the same horizontal level P1 = P2
The difference of pressure between two points separated by a depth h
(P2–P1) = hrg

E 149
Pre-Medical : Physics
w Pascal's Law
• A liquid exerts equal pressures in all directions.
• If the pressure in an enclosed fluid is changed at a particular point, the change is transmitted to every point of
the fluid and to the walls of the container without being diminished in magnitude. [for ideal fluids]
w Types of Pressure : Pressure is of three types
(i) Atmospheric pressure (P0)
(ii) Gauge pressure (Pgauge)
(iii) Absolute pressure (Pabs.)
w Atmospheric pressure : Force exerted by air column on unit cross–section area at sea level called atmospheric
pressure (Po)

F
Po = = 101.3 kN/m2 \ Po = 1.013 × 105 N/m2
A
Barometer is used to measure atmospheric pressure.
Which was discovered by Torricelli.
Atmospheric pressure varies from place to place and at a particular place from time to time.
• Gauge Pressure :
Excess Pressure ( P– Patm) measured with the help of pressure measuring instrument called Gauge pressure.
Pgauge = hrg or Pgauge µ h

Gauge pressure is always measured with help of "manometer"

• Pressure due to liquid on a vertical wall is different at different depths, so average fluid pressure on side wall a
hrg
of container filled upto height h = mean pressure =
2
• Absolute Pressure :

Sum of atmospheric and Gauge pressure is called absolute pressure.

Pabs = Patm + PgaugeÞ Pabs = Po + hrg

The pressure which we measure in our automobile tyres is gauge pressure.

w Buoyant force = Weight of displaced fluid = Vsg

w Apparent weight = Weight – Upthrust

Density of body
w Relative density of body =
Density of water at 4°C

Principle of Floatation

When a body of density (r) and volume (V) is completely immersed in a liquid of density (s), the forces acting
on the body are :

(i) Weight of the body W = Mg = Vrg directed vertically downwards through the Centre of gravity of the body.

(ii) Buoyant force or Upthrust Th = Vsg directed vertically upwards through Centre of buoyancy.

150 E
Pre-Medical : Physics
The following three cases are possible :

Case I Density of the body is greater than that of liquid (r > s)

In this case W > Th

So the body will sink to the bottom of the liquid.

WApp = W – Th = Vrg – Vsg = Vrg (1 – s/r) = W (1 – s/r).

Case II Density of the body is equal to the density of liquid (r = s)

In this case W = Th

So the body will float fully submerged in the liquid. It will be in neutral equilibrium.

WApp = W – Th = 0

Case III Density of the body is lesser than that of liquid (r < s)

In this case W < Th

So the body will float partially submerged in the liquid. In this case the body will move up and will
rise partially the volume of liquid displaced by the body (Vin) will be less than the volume of body
(V). This ensures that Th equally to W

\ WApp = W – Th = 0 Þ w = Th Þ Vrg = Vin s g

The above three cases constitute the laws of floatation which states that a body will float in a liquid if weight
of the liquid displaced by the immersed part of the body is at least equal to the weight of the body.

HYDRODYNAMICS
w Steady and Unsteady Flow : Steady flow is defined as that type of flow in which the fluid characteristics like
velocity, pressure and density at a point do not change with time.
w Streamline Flow : In steady flow all the particles passing through a given point follow the same path and hence
a unique line of flow. This line or path is called a streamline.
w Laminar and Turbulent Flow : Laminar flow is the flow in which the fluid particles move along well–defined
streamlines which are straight and parallel.
w Equation of continuity A1v1 = A2v2 Based on conservation of mass

1
Bernoulli's theorem : P + rv + rgh = constant
2
w
2

Based on energy conservation


C P2
A2 v2
B
P1 v1 h2
A1
h1

w For horizontal flow in venturimeter


A
B
1 2 1
P1 + rv 1 = P2 + rv 22
2 2

E 151
Pre-Medical : Physics
P v2
or + + h = constant (Energy per unit weight)
rg 2g
P v2
In the above equation is called the pressure head, is called the velocity head and h is called the
rg 2g

gravitational/potential head.
A P0

w Velocity of efflux v = 2gh h


H v
w Horizontal range R = 2 h(H - h) B
H–h
Time taken to empty the container

A 2é
t= H 1 - H 2 ùû
a gë

A – Area of container
a – Area of orifice
H1 – Initial level from Orifice t = 0
H2 _ Final level from Orifice t
l Magnus Effect (Observed in a Spinning Ball) l Aerofoil

high speed, reduced pressure


force on ball speed of air
flow increases
\pressure reduces

spin aerofoil

speed of air lift low speed, increased


flow decreases \pressure increased pressure
motion of a spin ball principle of an aerofoil

l Sprayer or Atomizer l Blowing-off of Tin Roof Tops in Wind Storm

spray p
wind v large so
air flowing out with
Rubber p<p0
high velocity p0
bulb

VISCOSITY

dv
w Newton's law of viscosity F = -hA
dy

N´s
• SI UNITS : or deca poise or 1 pa-s or 1 poiseuille
m2
Shear stress
• CGS UNITS : dyne–s/cm2 or poise (1 decapoise = 10 poise) h=
Shear strain rate
152 E
Pre-Medical : Physics
w Dependency of viscosity of fluids
On Temperature of Fluid
(a) Since cohesive forces decrease with increase in temperature as increase in K.E.. Therefore with the rise in
temperature, the viscosity of liquids decreases.
(b) The viscosity of gases is the result of diffusion of gas molecules from one moving layer to other moving layer.
Now with increase in temperature, the rate of diffusion increases. So, the viscosity also increases. Thus, the
viscosity of gases increases with the rise of temperature.

On Pressure of Fluid
(a) The viscosity of liquids increases with the increase of pressure.
(b) The viscosity of gases is practically independent of pressure.
On Nature of Fluid

4
dV ppr
w Poiseuille's formula Q= =
dt 8 hL

r r
w Viscous force F = -6phrv (Stokes law)

vT = 2 r (r - s )g Þ vT µ r2
2
w Terminal velocity r - density of body, s - density of medium
9 h

Inertial force
w Reynolds number Re =
Viscous force

Re < 1000 >2000 between 1000 to 2000


Type of flow streamline often turbulent unsteady

SURFACE TENSION
Surface tension is basically a property of liquid. The liquid surface behaves like a stretched elastic membrane
which has a natural tendency to contract and tends to have a minimum possible area. This property of liquid
is called surface tension.
Intermolecular forces
(a) Cohesive force
The force acting between the molecules of one type of molecules of same substance is called cohesive force.
(b) Adhesive force
The force acting between different types of molecules or molecules of different substance is called adhesive
force.
r Intermolecular forces are different from the gravitational forces not obey the inverse–square law
r The distance upto which these forces effective, is called molecular range. This distance is nearly 10–9 m.
Within this limit this increases very rapidly as the distance decreases.
r Molecular range depends on the nature of the substance
Properties of surface tension
• Surface tension is a scalar quantity.
• Force due to surface tension is acts tangential to liquid surface.
• Surface tension is always produced due to cohesive force.
• More is the cohesive force, more is the surface tension.
• When surface area of liquid is increased, molecules from the interior of the liquid rise to the surface. For this,
work is done against the downward cohesive force.
E 153
Pre-Medical : Physics
Dependency of Surface Tension
• On Cohesive Force : Those factors which increase the cohesive force between molecules increase the surface
tension and those which decrease the cohesive force between molecules decrease the surface tension.
• On Impurities : If the impurity is completely soluble then on mixing it in the liquid, its surface tension increases.
e.g., on dissolving ionic salts in small quantities in a liquid, its surface tension increases. If the impurity is partially
soluble in a liquid then its surface tension decreases because adhesive force between insoluble impurity molecules
and liquid molecules decreases cohesive force effectively, e.g.
(a) On mixing detergent in water its surface tension decreases.
(b) Surface tension of water is more than (alcohol + water) mixture.
• On Temperature
On increasing temperature surface tension decreases. At critical temperature and boiling point it becomes
zero.
Note : Surface tension of water is maximum at 4°C
• On Contamination
The dust particles or lubricating materials on the liquid surface decreases its surface tension.
Definition of surface tension
The force acting per unit length of an imaginary line drawn on the free liquid surface at right angles to the line
and in the plane of liquid surface, is defined as surface tension.
r For floating needle 2Tl sinq = mg
Ts inq
Ts inq

T T
q q
Tcosq Tcosq
Mg

w Required excess force for lift

r Wire Fex = 2Tl r Hollow disc Fex = 2pT (r1 + r2)

r For ring Fex = 4prT r Circular disc Fex=2prT

r Square frame Fex = 8aT r Square plate F ex = 4aT

w Work = surface energy = TDA

r Liquid drop W = 4pr2T r Soap bubble W = 8pr2T

w Splitting of bigger drop into smaller droples R = n1/3 r

(SE)Big = T(4pR2) (SE)small = T(4pR2) . n1/3

æ1 1 ö
W = DSE = 4pR 3 T ç - ÷ = 4pR2T (n1/3 – 1)
èr Rø
w Excess pressure Pex = Pin – Pout
2T 4T
r In liquid drop Pex = r In soap bubble Pex =
R R

154 E
Pre-Medical : Physics
ANGLE OF CONTACT (qC)
The angle enclosed between the tangent plane at the liquid surface and the tangent plane at the solid surface
at the point of contact inside the liquid is defined as the angle of contact.

The angle of contact depends the nature of the solid and liquid in contact.

w Angle of contact q < 90° Þ concave shape, Liquid rise up

Angle of contact q > 90° Þ convex shape, Liquid falls

Angle of contact q = 90° Þ plane shape, Liquid neither rise nor falls

w Effect of Temperature on angle of contact

é 1ù
On increasing temperature surface tension decreases, thus cosqc increases êQ cos q c µ ú and qc decrease. So
ë Tû
on increasing temperature, qc decreases.

w Effect of Impurities on angle of contact

(a) Solute impurities increase surface tension, so cosqc decreases and angle of contact qc increases.

(b) Partially solute impurities decrease surface tension, so angle of contact qc decreases.

w Effect of Water Proofing Agent

Angle of contact increases due to water proofing agent. It gets converted acute to obtuse angle.
w Capillary rise (i) Pressure Balance Method
2T
rgh =
R
2T
h=
Rrg
2T cos q
h=
rrg
(ii) Force Balance Method
(2pr)Tcosq = mg

Enlarged view
P
C
P h
PC
R

A B qC
r
qC
PC

R= Radius of the r
meniscus R = cos q

1
• Zurin's law h µ
r
r1 r2
w When two soap bubbles are in contact then radius of curvature of the common surface r= (r1 > r2)
r1 - r2

w When two soap bubbles are combining to form a new bubble then radius of new bubble r= r12 + r22

2AT
w Force required to separate two plates F=
d

E 155
Pre-Medical : Physics
Illustration The stress versus strain graphs for two materials A and B are shown below.
Explain the following
(a) Which material has greater Young's modulus ?
(b) Which material is more ductile ?
(c) Which material is more brittle ?
(d) Which of the two is more stronger material ?
Solution (a) Material A has greater value of Young's modulus, because slope of A is greater than that of B.
(b) Material A is more ductile because there is a large plastic deformation range between the
elastic limit and the breaking point.
(c) Material B is more brittle because the plastic region between the elastic limit and breaking
point is small.

(d) Strength of a material is determined by the stress required to cause fracture. Material A is
stronger than material B.

Illustration Two wires of diameter 0.25 cm, one made of steel and the other made

of brass are loaded as shown in Fig. The unloaded length of steel wire is

1.5 m and that of brass wire is 1.0 m. Young's modulus of steel is 2.0 × 1011 Pa and

that of brass is 0.91 × 1011 Pa. Calculate the elongations of the steel and brass

wires. (1 Pa = 1 Nm–2)

Mgl S (4 + 6) ´ 9.8 ´ 1.5


Solution Q The elongation in steel wire DlS = pr 2 Y = =1.50 ×10–4 m
( )
2
-2 11
S 3.14 ´ 0.125 ´ 10 ´ 2 ´ 10

Mgl B 6 ´ 9.8 ´ 1.0


The elongation in brass wire DlB = = =1.32 × 10–4 m
( )
pr YB 3.14 ´ 0.125 ´ 10-2 2 ´ 0.91 ´ 1011
2

Illustration Calculate the force required to increase the length of a steel wire of cross-sectional area
10–6 m2 by 0.5%. given : Y(for steel) = 2 × 1011 N-m–2.
l l
Solution ´ 100 = 0.5% r = 5 ´ 10–3
L L
l
so F = Y A =2 ´ 1011 ´ 10-6 ´ 5 ´10–3 = 103 N.
L
Illustration The graph shows the extension of a wire of length 1m suspended from a roof at one end and with
a load W connected to the other end. If the cross sectional area of the wire is 1 mm2, then the Young's
modulus of the material of the wire is
W(N)

100
80
60
40
20

1 2 3 4 5 Dl (mm)

F/A Wl W YA l 1 æ 40 - 20 ö -2
= slope Þ Y = (slope) = -6 ÷ = 2 ´ 10 Nm .
10
Solution Y= = Þ = ç (2 - 1) ´ 10-3
Dl / l ADl Dl l A 10 è ø

156 E
Pre-Medical : Physics
Illustration A sphere contracts in volume by 0.01% when taken to the bottom of sea 1 km deep. Find the
bulk's modulus of the material of the sphere. Given : density of sea water is 1 gcm –3, g = 980 cms–2.

DV 0.01
Solution = , h = 1 km = 105 cm, r = 1 gcm–3 ; DP = 105 × 1× 980 dyne-cm–2, K = ?
V 100

DP DP ´ V 105 ´ 980 ´ 100


K= = = dyne-cm–2 = 9.8 × 1011 dyne-cm–2.
DV V DV 0.01

Illustration Young modulus of elasticity of steel is 2 x 1011 N/m2. If interatomic distance for steel is 3.2 A°,
then find the interatomic force constant.

Solution k = Y× r0 = 2 × 1011 × 3.2 × 10–10 = 64 Nm–1.

Illustration Two immiscible liquids of densities 2.5 g/cm3 and 0.8 g/cm3 are taken in the ratio of their masses
as 2:3 respectively. Find the average density of the liquid combination.

æ 2M 3M ö
Solution Let masses be 2M & 3M then V = V1 + V2 = ç + ÷ cm
3

è 2.5 0.8 ø

Total mass = 2M+3M = 5M

5M 5M 5 10
Therefore, the average density r av = = = = g/cm3 = 1.09 g/cm3
V 2M 3M 2 3 9.1
+ +
2.5 0.8 2.5 0.8

Illustration Calculate the depth of a well if the pressure at its bottom is 15 times that at a depth of 3 metres.
Atmospheric pressure is 10 m column of water.

Solution Let the depth of the well be h then according to the question,

Patm + hrw g = 15 (Patm + 3rw g)

hrw g = 14 Patm+ 45 rw g = 14 (10 × rw g) + 45 rw g

h = 185 m.

Illustration A vertical U–tube of uniform cross–section contains


mercury in both arms. A glycerine (relative density =
1.3) column of length 10 cm is introduced into one of C
h
the arms. Oil of density 800 kg m–3 is poured into the
other arm until the upper surface of the oil and glycerine
0.1m
are at the same horizontal level. Find the length of the
(0.1-h) m
oil column. Density of mercury is 13.6 × 103 kgm–3.

Solution Pressure at A and B must be same


A B
Pressure at A = P0 + 0.1 × (1.3 × 1000) × g

Pressure at B = P0 + h × 800 × g + (0.1 – h) × 13.6 × 1000 g

Þ 0.1 × 1300 = 800 h + (0.1 – h) × 13600

Þ h = 0.096 m = 9.6 cm
E 157
Pre-Medical : Physics
Illustration An iceberg is floating partially immersed in sea-water. The density of sea-water is 1.03 g/cm3 and
that of ice is 0.92 g/cm3. What is the fraction of the total volume of the iceberg above the level of
sea-water ?
Solution In case of floatation weight = upthrust i.e.
mg = Vinsg or Vrg = Vinsg

r
or Vin = V
s

é rù
so Vout = V – Vin = V ê1 - ú
ë s û
\ The required fraction is,

Vout é rù é 0.92 ù 0.11


or fout = = ê1 - ú = ê1 - = = 0.106
V ë sû ë 1.03 úû 1.03

Illustration A syringe containing water is held horizontally with its


nozzle at a height h above the ground as shown in fig.
The cross-sectional areas of the piston and the nozzle
v
are A and a respectively. The piston is pushed with a
constant speed v. Find the horizontal range R of the
stream of water on the ground
.
Solution let v' be the horizontal speed of water when it emerges from the nozzle then from equation of
continuity

Av
Av = av' Þ v' =
a
1 2
Let t be the time taken by the stream of water to strike the ground then h = gt
2

2h 2h Av 2h
Þ t = Þ horizontal distance R = v' = .
g g a g

Illustration Water is flowing through two horizontal pipes of different diameters which are connected together.
In the first pipe the speed of water is 4 m/s. and the pressure is 2 × 104 N/m2. Calculate the
speed and pressure of water in the second pipe. The diameters of the pipes are 3 cm and 6
cm respectively ?

Solution If A is the area of cross–section of a pipe at a point and v is the velocity of flow of water at
that point, then by the principle of continuity Av = constant Þ A1v1 = A2v2

Þ p r12 v1 = p r22 v2

FG r IJ 2
F 15. ´ 10 I
-2
2

Þ v2 =
1
Hr K
2
v1 = GH 3 ´ 10 JK
-2 × 4 = 1 m/s.

1 1 1
From Bernoulli's theorem : P1 + rv 2 = P2 + rv 2 Þ P2 = P1 + r(v12 – v22)
2 1 2 2 2
1
\ P2 = 2 × 104 + × (103) × (16 – 1) = 2 × 104 + 7.5 × 103 = 2.75 × 104 N/m2
2
158 E
Pre-Medical : Physics
Illustration The diagram (fig.) shows venturimeter through which water is flowing.
5.1mm
The speed of water at X is 2 cm/s. Find the speed of water at Y
(taking g = 1000 cm/s2).
X Y
Solution By using Bernoulli's principle –
1 1 1
P1 + 1 rv12 = P2 + rv22 Þ P1 – P2 = r(v22 – v12) Þ rgh = r(v22 – v12)
2 2 2 2
1 2
putting the values in equation 1000 × 0.51 = (v – 22) Þ v2 = 32 cm/s
2 2

Illustration A cylindrical tank 1m in radius rests on a platform 5 m high.


Initially,the tank is filled with water to a height of 5 m. A small
plug whose area is 10–4 m2 is removed from an orifice located
on the side of the tank at the bottom. Calculate the :
(i) initial speed with which water flows out from the orifice
(ii) initial speed with which the water strikes the ground.
Solution (i) Applying Bernoulli's theorem between the water surface and the orifice,

1 1
P0 + r(0)2 + rgh = P0 + rv2 + rg(0)
2 2

1
rgh = rv2 ; v = 2gh = 2 ´ 10 ´ 5 =10 m/s.
2
(ii) Let v' be the initial velocity with which the water strikes the ground
Then, applying Bernoulli's theorem between the top of the tank and the ground level,
we get

v' = b
2g H + h g = 2 ´ 10 ´ 10 = 10 2 = 14.1 m/s.

Illustration The velocity of water in a river is 18 km/h at the surface. If the river is 5 m deep and the flow
is streamlined, find the shearing stress between the horizontal layers of water assuming uniform
veloicty gradient. Viscosity of water is 10–3 poiseuille.
Solution As velocity at the bottom of the river will be zero,
dv 18 ´ 103
Velocity gradient = = 1s –1
dy 60 ´ 60 ´ 5
F dv
Shear stress = =h = 10–3 × 1 = 1 × 10–3 N/m2.
A dy
Illustration A spherical ball of radius 1 × 10–4 m and density 104 kg/m3 falls freely under gravity through a
distance h before entering a tank of water. If the velocity of the ball does not change, after entering
the water find h. Viscosity of water is 9.8 × 10–6 N-s/m2.

Solution After falling a height h velocity of the ball will become v = 2gh. After entering into the water

as this velocity does not change, this velocity is equal to the terminal velocity,

2 2 ér - s ù
2gh = r g
9 êë h úû

2
é2 (104 –103 ) ´ 9.8 ù 20 ´ 20
2gh = ê ´ (10 –4 )2 ´ –6 ú Þ h = 2 ´ 9.8 = 20.41m
ë9 9.8 ´ 10 û

E 159
Pre-Medical : Physics
Illustration Calculate the work done against surface tension in blowing a soap bubble from a radius 10 cm to
20 cm if the surface tension of soap solution is 25 × 10–3 N/m. Then compare it with a liquid drop
for same radii.

Solution (i) For soap bubble : Extension in area = 2 × (4pr22 – 4pr12) = 8p [(0.2)2– (0.1)2] = 0.24p m2

Work done W1 = surface tension × extension in area = 25 × 10–3 × 0.24 p = 6p × 10–3 J.

(ii) For Liquid Drop : in case of liquid drop there is only one free surface, so extension in area will be
half that of soap bubble

W1
\ W2 = = 3p × 10–3 J
2

Illustration If W is the amount of work done in forming a soap bubble of volume V, then calculate the amount
of work done in forming a bubble of volume 2V from the same solution.

4 3
Solution. Volume of bubble V = pr Þ V µ r3
3

3
2V æ r2 ö r
= ç ÷ Þ 2 = 21 / 3
V r
è 1ø r1

Work done in forming the bubble W = 8pr2T Þ W µ r2

2
W2 æ r2 ö
= ( 21/ 3 ) = 22 / 3
2
=
W1 çè r1 ÷ø

W2 = 22/3 W
Illustration A water drop of radius 1mm is split into 106 identical drops. Surface tension of water is
72 dynes/cm. Find the energy spent in this process.

4 4 R
Solution As volume of water remains constant, so pR 3 = n pr3 Þ r = 1 3
3 3 n

Increase in surface area DA = n (4pr2) – 4pR2 = 4p(n1/3 – 1) R2 = 4p(100 – 1)10–6


\ Energy spent = TDA = 4p × 99 × 10–6 × 72 × 10–3 = 89.5 × 10–6 J
Illustration Prove that If two bubbles of radii r1 and r2 (r1 < r2) come in contact with each other then the radius
r1r2
of curvature of the common surface r= r - r .
2 1

Solution Q r1 < r2 \ P1 > P2 Small portion of bubbles is in contact and in equilibrium

4T 4T 4T 4T r1r2
Þ P1– P2 = Þ r – r = Þr = r -r
r 1 2 r 2 1

P1 P2
r1 r2

160 E
Pre-Medical : Physics
Illustration A hollow sphere which has a small hole in its bottom is immersed in water to a depth of
30 cm before any water enters in it. If the surface tension of water is 75 dynes/cm then find
the radius of the hole in metres (taking g=10 m/s2)

2T 2 ´ 75 ´ 10-3
Solution Radius of the hole r = = = 5× 10–5 m.
hdg 30 ´ 10 -2 ´ 103 ´ 10

Illustration A U - tube is supported with its limbs vertical and is partly filled with water. If the internal diameters
of the limbs are 1 × 10–2 m and 1 × 10–4 m respectively. What will be the difference in heights
of water in the two limbs? (Surface tension of water is 0.07 N/m.)

Solution Let h1 and h2 be the heights of water columns in the limbs of radis r1 and r2.

2T cos q 2 ´ 0.07 ´ cos 0°


Then h1 = = = 2.8 × 10–3 m = 0.028 × 10–1 m
r1dg 0.5 ´ 10-4 ´ 1000 ´ 9.8

2T cos q 2 ´ 0.07 ´ cos 0°


similarly h2 = = = 2.8 × 10–1 m
r2 dg 0.5 ´ 10-2 ´ 1000 ´ 9.8

Therefore difference in heights = h2 – h1 = (2.8 – 0.028) × 10–1 m = 2.772 × 10–1 m


= 0.277 m.

E 161
Pre-Medical : Physics
PROPERTIES OF MATTER AND FLUID MECHANICS EXERCISE
1. The breaking stress of steel is 15.8 ´ 108 N/m2 6. One end of uniform wire of length L and of weight
and density is 7.9 ´ 103 kg/m3. What should be W is attached rigidly to a point in the roof and a
the maximum length of a steel wire so that it may weight W1 is suspended from its lower end. If s is
not break under its own weight ? the area of cross-section of the wire, the stress in
(1) 2 km (2) 20 km (3) 1 km (4) 10 km the wire at a height (L/4) from its lower end is
2. A mass of 4 kg is suspended from a steel wire of é Wù
length 5 meter to form a pendulum arrangement. W1 ê W1 + 4 ú
(1) (2) ë û
If the mass is moved to one side and released from s s
the horizontal position of wire then find the maximum é 3W ù
extension in the length of the wire. ê W1 + 4 ú W1 + W
(3) ë û (4)
s 4

Fixed end 7. The dimensions of two wires A and B are the same.
But their materials are different. Their load-
l=5m extension graphs are shown. If YA and YB are the
values of Young's modulus of elasticity of A and B
respectively then
(Given Ysteel = 2 × 1011 N/m2 and area of cross– A
section of wire = 2 mm2, g = 10 m/s2)

load
B
(1) 0.2 mm (2) 0.8 mm
(3) 1.5 mm (4) 0.9 mm
extension
3. Young modulus of elasticity of brass is 1011 N/m2.
The increase in its energy on pressing a rod of length (1) YA > YB (2) YA < YB
0.2 m and cross–sectional area 4 cm2 made of brass (3) YA = YB (4) YB = 2YA
with a force of 40 N along its length, will be ............ 8. Two wires of the same material and length but
(1) 4 µJ (2) 3 µJ (3) 2 µJ (4) 1 µJ diameters in the ratio 1 : 2 are stretched by the
4. A fixed volume of iron is drawn into a wire of length same force. The potential energy per unit volume
l . The extension produced in this wire by a constant for the two wires when stretched will be in the ratio.
force F is proportional to - (1) 16 : 1 (2) 4 : 1 (3) 2 : 1 (4) 1 : 1

1 1 9. The load versus elongation graph for four wires of


(1) 2 (2) (3) l 2 (4) l the same material and same length is shown in the
l l
figure. The thinnest wire is represented by the line.
5. In determination of young's modulus of elasticity of
D
wire, a force is applied and extension is recorded. C
load

B
Initial length of wire is '1m'. The curve between
A
extension and stress is depicted then Young's
O elongation
modulus of wire will be :-
8mm
(1) OA (2) OB (3) OC (4) OD
10. Copper of fixed volume 'V; is drawn into wire of
Extension length 'l'. When this wire is subjected to a constant
1mm force 'F', the extension produced in the wire is 'Dl'.
Which of the following graphs is a straight line ?
1000 8000
Stress 1
(KN/m )
2 (1) Dl versus (2) Dl versus l2
l
(1) 2 × 109 N/m2 (2) 1 × 109 N/m2 1
(3) Dl versus 2 (4) Dl versus l
(3) 2 × 10 10
N/m 2
(4) 1 × 1010 N/m2 l
162 E
Pre-Medical : Physics
11. Two wires are made of the same material and have 18. A U-tube contains two liquids in static equilibrium:
the same volume. The first wire has cross-sectional Water of density rw (=1000 kg/m3) is in the right
area A and the second wire has cross-sectional area arm, oil of unknown density r is the left arm as
2A. If the length of the first wire is increased by Dl shown in figure. Measurement give l = 135 mm and
on applying a force 2F, how much force is needed d = 12.5 mm. The density of oil is :-

to stretch the second wire by the same amount ?


(1) 8F (2) 6F (3) 4F (4) 2F d
12. An increase in pressure required to decrease the
Oil
200 litres volume of a liquid by 0.004% in container Water l

is : (Bulk modulus of the liquid = 2100 MPa) Interface

(1) 188 kPa 2) 8.4 kPa


(3) 18.8 kPa (4) 84 kPa (1) 1092 kg/m3 (2) 961 kg/m3
13. A ball falling in a lake of depth 400 m shows 0.01% (3) 915 kg/m3 (4) 843 kg/m3
decrease in its volume at the bottom. What is the 19. A 800 g solid cube having an edge of length
bulk modulus of the material of the ball : 10 cm floats in water. What volume of the cube is
(1) 19.6 × 108 N/m2 (2) 39.2 × 109 N/m2 outside water ?
(3) 19.6 × 1010 N/m2 (4) 19.6 × 10–8 N/m2 (1) 200 cm3 (2) 300 cm3
14. The bulk modulus of a spherical object is 'B'. If it is (3) 500 cm3 (4) 800 cm3
20. A sphere is floating in water its 2/3rd part is outside
subjected to uniform pressure 'p', the fractional the water and when sphere is floating in unknown
decrease in diameter is :-
3
liquid, its th part is outside the liquid then density
4
B 3p p p
(1) (2) (3) (4) of liquid is
3p B 3B B (1) 4/9 gm/c.c. (2) 9/4 gm/c.c.
15. For a given material, the Young's modulus is (3) 4/3 gm/c.c. (4) 3/8 gm/c.c.
2.4 times that of rigidity modulus. Its Poisson's 21. The spring balance A read 2 kg. with a block m
ratio is : suspended from it. A balance B reads 5 kg. when a
beaker with liquid is put on the pan of the balance.
(1) 2.4 (2) 1.2 (3) 0.4 (4) 0.2
The two balances are now so arranged that the
16. Two vessels A and B have the same base area and
hanging mass is inside the liquid in the beaker as
contain water to the same height, but the mass of
shown in fig. In this situation :–
water in A is four times that in B. The ratio of the
liquid thrust at the base of A to that at the base of B
is :-
A

A B
m
H B

l l
(1) The balance A will read more than 2 kg.
(1) 4 : 1 (2) 2 : 1
(2) The balance B will read more than 5 kg.
(3) 1 : 1 (4) 16 : 1 (3) The balance A will read less than 2 kg. and B
17. Hydraulic press is based upon will read more than 5 kg.
(1) Archimede's principle (2) Bernoulli's theorem (4) The balance A and B will read 2 kg. and 5 kg.
(3) Pascal's law (4) Reynold's number respectively.

E 163
Pre-Medical : Physics
22. A jar is filled with two non-mixing liqudis 1 and 2 26. A solid uniform ball having volume V and density r
having densities r1 and r2 , respectively. floats at the interface of two immiscible liquids as
shown in figure.
A solid ball, made of a material of density r3, is
dropped in the jar. It comes to equilibrium in the

position shown in the figure. Which of the following


is true for r1, r2 & r3 The densities of the upper and the lower liquids are
r1 and r2 respectively, such that r1 < r < r2 . What
(1) r3 < r1 < r2
Liquid 1
fraction of the volume of the ball will be in the lower
(2) r1 > r3 > r2 liquid :–
(3) r1 < r2 < r3 r - r2 r1
Liquid 2
(1) r - r (2) r - r
(4) r1 < r3 < r2 1 2 1 2

r1 - r r1 - r2
23. A wooden block, with a coin placed on its top, floats (3) r - r (4)
1 2 r2
in water as shown in figure. The distance l and
h are shown there. After sometime the coin falls 27. Two syringes of different cross section (without
into the water. Then :– needes) filled with water are connected with a tightly
fitted rubber tube filled with water. Diameter of
coin
smaller and larges piston are 1.0 cm and 3.0 cm
respectively. Find the force exerted on the larger
piston when a force of 10 N is applied to the smaller
l
h piston.
(1) 10 N (2) 30 N (3) 90 N (4) 60 N

(1) l decreases and h increases 28. The cylindrical tube of a spray pump has a radius
(2) l increases and h decreases R, one end of which has n fine holes, each of radius
(3) both l and h increase r. If the speed of flow of the liquid in the tube is v,
(4) both l and h decrease the speed of ejection of the liquid through the hole
is :-
24. An object of weight W and density r is submerged 1
væRö væRö 2

in a fluid of density r1 . Its appearent weight will be (1) ç ÷ (2) ç ÷


nè r ø nè r ø
3 2
( r - r1 ) væRö 2
v æ Rö
(1) W (r - r1 ) (2) (3) ç ÷ (4) ç ÷
W nè r ø nè r ø

æ r ö 29. The cylindrical tube of a spray pump has radius


(3) W ç1 - 1 ÷ (4) W( r1 - r)
è rø R, one end of which has n fine holes, each of
radius r. If the speed of the liquid in the tube is V,
25. A wooden block is taken to the bottom of a lake of
the speed of the ejection of the liquid through
water and then released. it rise up with a
the holes is :-
(1) Constant acceleration
V2R VR 2
(2) Decreasing acceleration (1) (2)
nr n2r 2
(3) Constant velocity
VR 2 VR 2
(3) (4)
(4) Decreasing velocity nr 2 n3 r 2

164 E
Pre-Medical : Physics
30. Fire is caught at height of 125 m from the 34. The flow speeds of air on the lower and upper
fire brigade. To extinguish the fire, water is
surfaces of the wing of an aeroplane are v and 5v
coming out from the pipe of cross section
respectively. The density of air is r and surface area
12.8 cm with rate 3800 litre/min. Find out
of wing is A. The dynamic lift on the wing is :
minimum velocity of water exiting from fire
brigade tank (g = 10m/s 2) (1) rv2A (2) 2 rv2A
(1) 5 m/s (2) 10 m/s (3) 25 m/s (4) 50 m/s (3) (1/2) rv2A (4) 2rv2A
31. Water from a tap emerges vertically downwards 35. A tank is filled upto a height h with a liquid and is
with an in it ial speed o f 5. 0 m/s. The placed on a platform of height h fromt he ground
cross–sectional area of tap is 10–4 m2. Assume that To get maximum range xm a small hole is punched
the pressure is constant throughout the stream of at a distance of y from the free surface of the liquid.
water and that the flow is steady, the cross–sectional Then :-
area of stream 3.75 m below the tap is :–
(1) 5.0 × 10–4 m2
(2) 1.0 × 10–4 m2
y
h
(3) 5.0 × 10–5 m2
(4) 2.0 × 10–5 m2
32. A wind with speed 40 m/s blows parallel to the roof
of a house. The area of the roof is 250 m 2. h
Assuming that the pressure inside the house is
xm
atmospheric pressure, the force exerted by the wind
on the roof and the direction of the force will be:
(rair = 1.2 kg/m3)
(1) xm = 3h (2) xm = 1.5h
(1) 4.8 × 105 N, upwards
(3) y = h (4) y = 0.75 h
(2) 2.4 × 105 N, upwards
36. A tank of height 5 m is full of water. There is a hole
(3) 2.4 × 105 N, downwards
of cross sectional area 1 cm2 in its bottom. The initial
(4) 4.8 × 105 N, downwards
volume of water that will come out from this hole
33. Water flows through a frictionless duct with a
per second is
cross-section varying as shown in figure. Pressure P
(1) 10–3 m3/s (2) 10–4 m3/s
at points along the axis is represented by
(3) 10 m3/s (4) 10–2 m3/s.
37. Scent sprayer is based on
(1) Charle's law
(2) Archimede's principle
(3) Boyle's law
(4) Bernoulli's theorem
P P
38. A large open tank has two holes in the wall. One
(1) (2) is a square hole of side L at a depth y from the
x x top and the other is a circular hole of radius R
at a depth 4y from the top. When the tank is
completely filled with water, the quantities of water
P P
flowing out per second from the holes are both
(3) (4) same. Then, R is equal to :–
L L
(1) (2) 2pL (3) L (4)
2p 2p

E 165
Pre-Medical : Physics
ALLEN
39. A metal block of film area 0.10 m2 is connected to 44. If the terminal speed of a sphere of gold
a 0.010 kg mass via a string that passes over an (density = 19.5 kg/m3) is 0.2 m/s in a viscous liquid
ideal pulley (considered massless & frictionless). A (density = 1.5 kg/m3), find the terminal speed of
a sphere of silver (density=10.5 kg/m3) of the same
liquid with a film thickness of 0.30 mm is placed
size in the same liquid.
between the block and the table. When released
(1) 0.4 m/s (2) 0.133 m/s
the block moves to the right with a constant speed
(3) 0.1 m/s (4) 0.2 m/s
of 0.085 m/s. Find the coefficient of viscosity of
the liquid. (g = 9.8 m/s2) 45. If a ball of steel (density r = 7.8 g cm–3) attains a
terminal velocity of 10 cm s –1 when falling
Film
in a tank of water (coefficient of viscosity
hwater = 8.5 × 10–4 Pa.s) then its terminal velocity
0.01 kg
in glycerine (r = 1.2 g cm–3, h = 13.2 Pa.s) would
be nearly :-
(1) 6 × 10–2 Pa.s (2) 3.45 × 10–5 Pa.s
(1) 1.6 × 10–5 cm s–1 (2) 3.98 × 10–4 cm s–1
(3) 3.45 × 10–3 Pa.s (4) 3.45 × 10–6 Pa.s
(3) 6.25 × 10–4 cm s–1 (4) 1.5 × 10–5 cm s–1
40. A square plate of 1m side moves parallel to a
46. A non-viscous fluid of constant density of 1000 kg/m3
second plate with velocity 4 m/s. A thin layer of
flows in a stream line motion along a tube of variable
water exists between plates. If the viscous force is
cross-section.
2 N and the coefficient of viscosity is 0.01 poise
then find the distance between the plates in mm.
P
(1) 2 mm (2) 4 mm (3) 6 mm (4) 8 mm
41. An air bubble of radius 1 mm is allowed to rise 5m Q
through a long cylindrical column of a viscous liquid
of radius 5 cm and travels at a steady rate of 3m
2.1 cm per second. If the density of the liquid is
1.47 g/cc, find its viscosity. Assume g=980 cm/s2 The area of cross-section at two P and Q at lengths
and neglect the density of air.
5 m and 3 m are 80 cm2 and 40 cm2 respectively.
(1) 2 poise (2) 3 poise
If velocity of fluid at P is 3 m/s then find velocity
(3) 4 poise (4) 1.52 poise
42. A small ball is left in a viscous liquid from very much of fluid at Q.
height. Correct graph of its velocity with time after (1) 3 m/s (2) 4 m/s
it enters in liquid is :
(3) 5 m/s (4) 6 m/s
B
A 47. A thin liquid film formed between a U-shaped wire
an d a light slider suppor ts a weight of
velocity

C 1.5 × 10–2 N (see figure). The length of the slider is


D 60 cm and its weight negligible. The surface tension
time of the liquid film is :-
(1) A (2) B (3) C (4) D
43. A small drop of water falls from rest through a large Film
height h in air. The final velocity is
(1) almost independent of h
w
(2) proportional to h
(1) 0.025 Nm–1 (2) 0.0125 Nm–1
(3) proportional to h
(3) 0.1 Nm–1 (4) 0.05 Nm–1
(4) inversely proportional to h

166 E
Pre-Medical : Physics
48. The potential energy U of two atoms of a diatomic 53. Consider a soap film on a rectangular frame of wire
molecule as a function of distance r between the of area 5 × 5 cm2 . If the area of the soap film is
atoms is shown in the given figures. increased to 6 × 5 cm2, the work done in the
process will be (The surface tension of the soap film
(+)
is 3 × 10–2 N/m)
U
r (1) 12 × 10–6 J (2) 24 × 10–6 J
r1
r2 (3) 30 × 10–6 J (4) 96 × 10–6 J
(–) r3
54. A liquid drop of diameter D breaks into 27 tiny
Read the following statements carefully. drops. The resultant change in energy is –
A. The equilibrium separation distance between
the atoms is equal to r2. (1) 2p TD2 (2) 4p TD2
B. At r = r1, the force between the atoms is
(3) p TD2 (4) None of these
repulsive.
C. For r > r3, the force between the atoms is 55. If the surface tension of a liquid is T and its surface
attractive. area is increased by A, then the surface energy of
Which of the above statements is true ? that surface will be increased by –
(1) A only (2) B only
(1) AT (2) A/T (3) A2T (4) A2T2
(3) C only (4) B and C
49. Spiders and insects move and run about on the 56. The excess pressure inside a soap bubble A is twice
surface of water without sinking because : that in another soap bubble B. The ratio of volumes
(1) Elastic membrane is formed on water due to of A and B is
propery of surface tension
(1) 1 : 2 (2) 1 : 4 (3) 1 : 8 (4) 1 : 16
(2) Spiders and insects are ligther
(3) Spiders and insects swim on water 57. A certain number of sphereical drops of a liquid of
(4) Spiders and insects experience up-thrust radius 'r' coalesce to form a single drop of radius 'R'
50. Adding detergents to water helps in removing dirty and volume 'V'. If 'T' is the surface tension of the
greasy stains. This is because liquid, then :
(a) It increases the oil-water surface tension
(b) It decreases the oil-water surface tension æ1 1 ö
(1) energy = 4VT ç - ÷ is released
(c) It increases the viscosity of the solution èr Rø

(d) Dirt is held suspended surrounded by detergent


molecules æ1 1 ö
(2) energy = 3VT ç + ÷ is absorbed
èr Rø
(1) (b) and (d) (2) (a) only
(3) (c) and (d) (4) (d) only
æ1 1 ö
51. Find the work done in increasing the volume of a (3) energy = 3VT ç - ÷ is released
èr Rø
soap bubble by 700% if its radius is R and surface
tension is T. (4) Energy is neither released nor absorbed
(1) 24pR2T (2) 12pR2T
58. Consider a soap film on a rectangular frame of wire
(3) 6pR2T (4) pR2T
of area 4× 4 cm2 . If the area of the soap film is
52. Two small drops of mercury, each of radius R,
increased to 4 × 5cm2, the work done in the
coalesce to form a single large drop. The ratio of
process will be (The surface tension of the soap film
the total surface energies before and after the
is 3 × 10–2 N/m)
change is :–
(1) 1 : 21/3 (2) 21/3 : 1 (1) 12 × 10–6 J (2) 24 × 10–6 J
(3) 2 : 1 (4) 1 : 2
(3) 60 × 10–6 J (4) 96 × 10–6 J

E 167
Pre-Medical : Physics
59. A rectangular film of liquid is extended from 64. Th ree liquids of densities r 1 , r 2 and r 3
(7 cm × 2 cm) to (5 cm × 4 cm). If the work done (with r1 > r2 > r3), having the same value of surface
is 1.5 × 10–4 J, the value of the surface tension of
tension T, rise to the same height in three identical
the liquid is :-
capillaries. The angles of contact q1, q2 and q3 obey:-
(1) 0.2 Nm–1 (2) 8.0 Nm–1
p
(3) 0.250 Nm–1 (4) 0.125 Nm–1 (1) < q1 < q2 < q3 < p
2
60. If the difference between pressure inside and outside
p
of a soap bubble is 4 mm of water and its radius is (2) p > q1 > q2 > q3 >
2
16 mm. What is the surface tension in dynes
per cm. p
(3) > q1 > q2 > q3 ³ 0
2
(1) 117.6 (2) 256
(3) 378 (4) 160 p
(4) 0 £ q1 < q2 < q3 <
61. Two soap bubbles of radii r1 and r2 equal to 4cm 2
and 5 cm are touching each other over a common 65. On dipping one end of a capillary in a liquid and
surface S1S2 (shown in figure). Its radius will be :– inclining the capillary at angles 37° and 60° with
the vertical, the lengths of liquid columns in it are
S1
4cm 5cm
found to be l1 and l2 respectively. Find the ratio of
l1 and l2 ?
S2
(1) 4/5 (2) 8/5 (3) 5/8 (4) 1/2
(1) 4 cm. (2) 20 cm. 66. When a capillary tube is dipped inside water, water
(3) 5 cm. (4) 4.5 cm. rises inside the capillary tube up to 0.030 m. If the
62. A glass tube of uniform internal radius (r) has a valve surface tension of water is 75 × 10–3 N/m calculate
separating the two identical ends. Initially, the valve the radius of the capillary tube ?
is in a tightly closed position. (1) 0.5 mm (2) 2 mm
(3) 1 mm (4) 4 mm
67. If a capillary of radius r is dipped in water, the height
of water that rises in it is h and its mass is M. If the
radius of the capillary is doubled the mass of water
that rises in the capillary will be
2 1
M
End 1 has a hemispherical soap bubble of radius r. (1) 4M (2) 2M (3) M (4)
2
End 2 has sub–hemispherical soap bubble as shown
in figure. Just after opening the valve, 68. A liquid flows through two capillary tubes connected
(1) Air from end 1 flows towards end 2. No change in series. Their lengths are l and 2l and radii r and
in the volume of the soap bubbles. 2r respectively, then the pressure difference across
the first and second tubes are in the ratio......
(2) Air from end 1 flows towards end 2. Volume of
the soap bubble at end 1 decreases. (1) 4 : 1 (2) 8 : 1
(3) No change occurs (3) 16 : 1 (4) 64 : 1
(4) Air from end 2 flows towards end 1. Volume of 69. In a capillary tube expertiment, a vertical 50 cm
the soap bubble at end 1 increases. long capillary tube is dipped in water. The water
63. Shape of meniscus for a liquid of zero angle of rises up to a height of 20 cm due to capillary action.
contact is - If this experiment is conducted in a freely falling
(1) plane (2) parabolic elevator, the length of the water column becomes :
(3) hemi-spherical (4) cylindrical (1) 50 cm (2) 20 cm (3) 30 cm (4) Zero

168 E
Pre-Medical : Physics
70. On dipping a capillary of radius 'r' in water, water 71. A capillary tube of radius r can support a liquid of
rises upto a height H and potential energy of water weight 6.28 × 10–4 N. If the surface tension of the
is u1. If a capillary of radius 2r is dipped in water, liquid is 5 × 10–2 N/m. The radius of capillary
u1 must be :-
then the potential energy is u2. The ratio
u 2 is
(1) 2 × 10–3 m (2) 2 × 10–4 m
(1) 2 : 1 (2) 1 : 2 (3) 4 : 1 (4) 1 : 1
(3) 1.5 × 10–3 (4) 12.5 × 10–4 m

ANSWER KEY
Que. 1 2 3 4 5 6 7 8 9 10 11 12 13 14 15
Ans. 2 3 1 3 2 2 1 1 1 2 1 4 2 3 4
Que. 16 17 18 19 20 21 22 23 24 25 26 27 28 29 30
Ans. 3 3 3 1 3 3 4 4 3 1 3 3 4 3 1
Que. 31 32 33 34 35 36 37 38 39 40 41 42 43 44 45
Ans. 3 2 1 4 3 1 4 1 3 1 4 3 1 3 3
Que. 46 47 48 49 50 51 52 53 54 55 56 57 58 59 60
Ans. 4 2 4 1 1 1 2 3 1 1 3 3 2 4 4
Que. 61 62 63 64 65 66 67 68 69 70 71
Ans. 2 2 3 4 3 1 2 2 1 4 1

E 169
Pre-Medical : Physics
THERMAL PHYSICS
TEMPERATURE, THERMAL EXPANSION
l Zeroth law of thermodynamics
If objects A and B are separately in thermal equilibrium with a third object C (say thermometer), then objects
A and B are in thermal equilibrium with each other. Zeroth law of thermodynamics introduce the concept
of temperature.
l Comparison between Different Temperature Scales

K - 273 C F - 32 X – LFP
= = =
100 100 180 UFP – LFP

F THERMAL EXPANSION
When matter is heated without any change in its state, it usually expands.
l Solids can expand in one dimension (Linear expansion), two dimensions (Superficial expansion) and three
dimensions (Volumetric expansion) while liquids and gases usually suffers change in volume only.
l Linear expansion : l = l0 (1 + aDq)
l Superficial (areal) expansion : A = A0 (1 + bDq)
l Volumetric expansion : V = V0 (1 + gDq) a: b : g= 1: 2: 3

HEAT
Heat required to increase the temperature of a substance is Q = mcdT Here c = Specific heat
Heat required for change of phase is Q = mL Here L = Latent heat
l Value of specific heats of gas can vary from zero (0) to infinity depending on the condition to be heated.
l Generally two types of specific heats are defined for a gas –
(a) Specific heat at constant volume (Cv) (b) Specific heat at constant pressure (CP)
Thermal capacity (Heat capacity)
The quantity of heat required to raise the temperature of the whole of that substance through 1°C is called
thermal capacity. The thermal capacity of substance is = mc
F Effect of change in pressure on M.P. and B.P. for ice/water

If P ­ ¾¾¾¾¾ ® M.P. ¯ & B.P. ­


then result
l l If P ¯ ¾¾¾¾
then result
¾
® M.P. ­ & B.P. ¯
Law of Mixtures :

Principle of calorimetry represents the law of conservation of heat energy.

MODES OF HEAT TRANSFER


l Conduction
The process in which the material takes an active part by molecular action and energy is passed from one
particle to another is called conduction. It is predominant in solids.
l Convection
The transfer of energy by actual motion of particle of medium from one place to another is called convection.
It is predominant in fluids (liquids and gases).
l Radiation
Quickest way of transmission of heat is known as radiation. In this mode of energy transmission, heat is transferred
from one place to another without effecting the inter–venning medium.

170 E
Pre-Medical : Physics
F Thermal conduction
T1 T T – dT T2
IH IH Q DT
Rate of heat transfer (Heat current) IH = = -KA
t Dx
Dx
Q KA(T1 - T2 )
= K= Thermal conductivity ® it is the measure of the ability of material to conduct heat.
t L

æ dT ö
Here K is a constant depending on nature of the material of quantity ç ÷ is called temperature gradiant.
è dx ø
The (–) sign shows heat flows from high to low temperature (DT is a –ve quantity).
Application of Thermal Conduction
• Cooking utensils are made of aluminium and brass whereas their handles are made of wood.
• Ice is covered in gunny bags to prevent melting of ice.
• Two thin blankets are warmer than a single blanket of double the thickness.
Thermal Resistance to conduction

IH T1 T 2 IH DT T1 - T2 L
R= = ; R=
R IH IH KA
In series combination In parallel combination
L1 + L 2 SL i K 1 A 1 + K 2 A 2 SK i A i
Keq = = K eq = =
L1 L 2 L A1 + A 2 SA i
+ S i
K1 K 2 Ki
For identical rods For identical rods
2K1K 2 K1 + K 2
KS = KP =
K1 + K 2 (Harmonic mean) 2
(Arithmatic mean)

F Thermal Radiation
The process of the transfer of heat from one place to another place without heating the intervening medium
is called radiation.
FUNDAMENTAL DEFINATION
Qa
• Absorptive power or absorptive coefficient 'a' : a = (unitless)
Q
• Emissive power (e) : e = Q/At (watt/m2)
¥

• Spectral Emmisive power (el ) : Emissive power or total emissive power e = ò el dl SI UNIT : W/m2 Å
0

Q GB e emitted radiation by gray body


• Emissivity (er) : er = = GB =
Q IBB E IBB emitted radiation by ideal black body
GB = gray or general body, IBB = Ideal black body
(i) No unit (ii) For ideal black body er = 1 (iii) range 0 < er < 1
• Prevost's theory of heat energy exchange
According to Prevost at every possible temperature ( except zero kelvin temperature) there is a continuous
heat energy exchange between a body and its surrounding and this exchange carry on for infinite time.
• KIRCHHOFF'S LAW :
At constant temperature

el é el ù é el ù
= E l = constant ê ú = ê ú = constant hence el µ al
al ë a l û1 ë a l û2
Good absorbers are good emitters and bad absorbers are bad emitters
E 171
Pre-Medical : Physics
F Applications of Kirchoff Law
• Fraunhoffer's lines
Fraunhoffer lines are dark lines in the spectrum of the Sun. When white light emitted from the central core
of the Sun (Photosphere) passes through its atmosphere (chromosphere) radiations of those wavelengths will
be absorbed by the gases present, resulting in dark lines in the spectrum of Sun.
vapour chromosphere
state
K
Ca SUN photo
Na sphere
107K

6000K

At the time of total solar eclipse direct light rays emitted from photosphere cannot reach on the Earth and
only rays from chromosphere are able to reach on the Earth surface. At that time we observe bright fraunhoffer
lines.
• In deserts days are hot and nights are cold
• STEFAN'S LAW
Amount of radiation emitted E µ T4
E = s T4 (This law is true for only ideal black body)
SI Unit : E = watt/m 2
s = Stefen's constant = 5.67 x10–8 watt /m2 K4 (universal constant)
Dimensions of s : M1 L0 T–3 q–4
Total radiation energy emitted out by surface of area A in time t :
Ideal black body QIBB = s A T4 t and for any other body QGB = ersA T4 t
Including the temp of sorrounding (T0 = temp of surrounding)
Q
RH = = sAe r (q4 - q40 ) ; RH = Rate of heat loss
t
dq sAe r 4
RF = = (q - q40 ) ; RF = Rate of fall in temp.
dt mcJ
dq
µ (q – q0) [(when (q – q0) > 35°C]
dt
Eg. Two identical hollow and solid spheres are cooled in the same sorrounding the solid sphere will cool late.
• NEWTON'S LAW OF COOLING q
For Numerical Problems, Newton's Law of cooling

é q1 - q2 ù é q1 + q2 ù
ê t ú = + K ê 2 - q0 ú q0
ë û ë û
t
Spectral Energy distribution curve of Black Body radiations
E T T >T >T 1
lm µ
3
lm3
(i) 3 2 1

T
J sec-m2

l <l <l m3 m2 m1

different
wavelength
(ii) E lm µ T5
T2 and emitted ¥
E lm2
Spectral radiation

radition
(iii) Area = òE dl = E = s T 4
intens ity

l
0

T1
E lm1

lm3 lm2 lm1 l

b
Wein's Displacement Law lm = where b = 2.89 × 10–3 m–k
T
172 E
Pre-Medical : Physics
• Solar constant 'S'
The Sun emits radiant energy continuously in space of which an in significant part reaches the Earth. The
solar radiant energy received per unit area per unit time by a black surface held at right angles to the Sun's
rays and placed at the mean distance of the Earth (in the absence of atmosphere) is called solar constant.
The value of solar constant is 1340 watt/m2 or 1.937 cal/cm2–min.
2
æ Rö
S = sT 4 ç ÷
è dø

KINETIC THEORY OF GASES


Intermolecular force Solid > liquid > real gas > ideal gas (zero)
Potential energy Solid < liquid < real gas < ideal gas (zero)
At a given temperature for solid, liquid and gas:
(i) Internal kinetic energy : Same for all
(ii) Internal potential Energy : Maximum for ideal gas (PE = 0) and Minimum for solids (PE = –ve)
(iii) Internal Energy : Maximum for Ideal gas and Minimum for solid
Ideal Gas Concept
• Volume of gas molecules is negligible as compared to volume of container so volume of gas = volume of
container (Except 0 K)
• No intermoleculer force act between gas molecules.
Equation of state for Ideal gas
M éRù P RT kT
PV = µRT Þ PV = RT = ê ú N T = NkT Þ = =
r Mw m
Mw ë N0 û M=constant
Gas Laws T=constant

1 P
• Boyle's Law V µ if M and T = Constant Þ P1 V1 = P2 V2
P
• Charle's Law V

V1 T1 µR
V µ T if M and P = Constant Þ = Slope of V–T curve =
V2 T2 P V
• Gay–Lussac's Law
T(K)

P1 T1 µR
P µ T if M and V = constant Þ = Slope of P–T curve =
P2 T2 V
P

• Avogadro's Law N1 = N2 if P,V and T are same. T(K)


• Dalton's Partial Pressure Mixture Law : P = P1 + P2 + .... If V, T ® Same
• Maxwell's law of distribution of velocities
Each velocity increases with increase in temp. and no. of molecules corresponding to most probable speed decreases.
Area under the curve gives the total number of molecules which is constant at each temperature. i.e. for
each curve.
Nmax (T1)
T 1=500K
number of molecules (N)

vmp=most probable speed


v max =maximum speed of molecule
Nmax (T2)
T2=1000K
Nmax (T3)
T3=2000K
T3 > T2 > T1

v mp (T ) v mp (T )
1
v mp (T )
2 3

velocity of molecule v
Degree of freedom (f)
• The number of independent ways in which a molecule or an atom can exhibit motion or have energy is called
it's degrees of freedom.
• The degrees of freedom are of three types :

E 173
Pre-Medical : Physics
(a) Translational Degree of freedom : Maximum three degree of freedom are there corresponding
to translational motion.
(b) Rotational Degree of freedom : The number of degrees of freedom in this case depends on the
structure of the molecule.
(c) Vibrational Degree of freedom : It is exhibited at high temperatures.
• For monoatomic gas : f = 3 (T = 3)
For diatomic gas : f = 5 (T = 3, R = 2)
For triatomic/polyatomic gas (Linear) : f = 5 (T = 3, R = 2)
For triatomic/polyatomic gas (non-linear) : f = 6 (T = 3, R = 3)
Maxwell's law of equipartition of energy
The total kinetic energy of a gas molecules is equally distributed among its all degree of freedom and the
1
energy associated with each degree of freedom at absolute temperature T is kT
2
Different K.E. of gas (Internal Energy)
1 3 3 3 M 3 N
• Translatory kinetic energy (ET) ET = Mv 2rms = PV = µRT = RT = RT
2 2 2 2 Mw 2 N0

Total energy E 1 éMù 2 1 2


• Energy per unit volume or energy density (EV) EV = = ; E V = ê ú v rms = r v rms
Volume V 2ëVû 2

3 3RT 3kT
Emolar = RT v rms = =
2 Mw m

3 RT 2RT 2kT
Egram = v mp = =
2 Mw Mw m

3 RT 3 8RT 8kT
Emolecule = 2 N = 2 kT v av = =
0 Mw pm
f
for 'f' degree of freedom E = RT
2
Mean freepath ® Average distance travelled between two successive collisions
n = no. of molecules per unit volume

1 V kT
l= 2
= 2
=
2pd n 2pd N 2pd 2P

THERMODYNAMIC
W Vf
• Work done by thermodynamic system W=ò dW = ò PdV
0 Vi

F First law of thermodynamics dU = dQ – dW or DU = Q – W Here DU = µCVDT & Q = µCDT


Heat supplied to the system and work done by the system are path dependent. Change in internal energy
DU = Uf – Ui does not depends on path it depends only on initial and final positions of the system.
F DIFFERENT PROCESSES sign convesion
• Isometric or Isochoric Process (V = constant or P µ T) Q = +ve heat is absorbed by the gas
W = 0 and Q = DU = µ Cv DT Q = –ve heat is rejected by the gas
• Isobaric Process (P = constant or V µ T) W = +ve (expansion)
Q = µCP DT ; W = µRDT = P(Vf – Vi); DU = µCVDT W = –ve (compression)
Isothermal Process (T = constant or PV = constant)
Work Done

V2 µRT é V2 ù éP ù
\ Work done : W = ò
V1 V
dV = µRT log e ê ú = 2.303µRT log ê 1 ú
ëê V1 ûú
10 P
ë 2û
[Q P1V1 = P2V2]

174 E
Pre-Medical : Physics
P
• Two isotherms for a given gas at two different temperatures
T2 > T1
T1 and T2 are shown in figure
The curves drawn for the same gas at different temperatures are
T2
mutually parallel and do not cut each other.
T1
• If changes are executed in a vessel of infinite conductivity
then they will be isothermal. V

• Adiabatic Process (Q = 0 Þ W = –DU)


Eqn of state PVg = const. ; TVg-1 = const. ; P1–gTg = const.

dP P dP slope of adiabatic changes


=g = g Þ =g P
dV adia V dV iso slope of isothermal changes AD
IT
µR AD IT
Work done W = (T1 - T2 )
( g - 1) V
Examples of adiabatic process
• If a gas is suddenly expanded by moving the piston outwards, there will be a decrease in the temperature
of the gas.
• Propagation of sound waves in a gas.
Relation between degrees of freedom and specific heat of gas

dU f R éf ù gR CP 2
CV = = R= . C P = C V + R = ê + 1ú R = and g = C = 1 + f
dT 2 g -1 ë2 û g -1 V

R R
General expression for C (CP or CV) in the process PVx = constant C = +
g -1 1 - x
R
For isobaric process P = constant so x = 0 \ C = CP = + R = CV + R
g -1
For isothermal process, PV = constant so x = 1 \ C= ¥
g
For adiabatic process PV = constant so x = g \ C = 0

Atomicity of gas f Cv CP g
3 5 5
Monoatomic 3 R R = 1.67
2 2 3
Diatomic, Triatomic and Triatomic linear 5 7 7
5 R R = 1.4
(at normal temperature) 2 2 5
6 8 4
Poly atomic Triangular Non-linear 6 R = 3R R = 4R = 1.33
2 2 3
F Why CP is greater than CV ?
In comparision to constant volume, at constant pressure additional heat is required to do the work
So, CP is more than CV
l When a gas expands its volume increases, then final pressure is less for adiabatic expansion. But, when a
gas compresses its volume decreases, then the final pressure is more in case of adiabatic compression.
adiabatic W IB > W IT > W AD WIB > W IT > W AD
P P P
iso P final P IB > P IT > P AD P final P AD > P IT > P IB
th
er DP DPAD > DPIT > DPIB DP DPAD > D PIT > DPIB
m
al IB AD
isobaric
IT
isobaric IT
iso the AD IB
r m al
adiabatic
V V 2V V V/2 V V

E 175
Pre-Medical : Physics
F HEAT ENGINE Source
T1
work done
[% efficiency = ×100 ] Q1
Qinput Working
W
substance
Heat engine is a device which converts heat into work. In a cycle of Q2
heat engine the working substance extracts heat Q1 from source, T2
does some work W and rejects remaining heat Q2 to the sink. Sink

work done (W) T - T2 Q1 - Q2


Efficiency of heat engine h = Þ 1 =
heat taken from source (Q1 ) T1 Q1
F REFRIGERATOR Hot reservoir
It is reverse of heat engine. It extracts heat (Q2) from a cold reservoir, Q1
same external work W is done on it and rejects heat (Q1) to hot reservoir. Working
W
substance
Q2 Q2 1 Q2
Heat extracted from cold reservoir = = =
b= W Q1 - Q2 Q1 Cold reservoir
Work done on refrigerator -1
Q2

Q1 T1 T2
\ b = Q2 =
= 1 1
For Carnot reversible refrigerator Q2 T2 = Þ b =
W é Q1 ù éT ù T1 - T2
ê - 1 ú ê 1 - 1ú
ë Q2 û ë T2 û
1
Relation between h & b ® b = -1
h
Illustration A thermometer with an arbitrary scale has the ice point at –20° and the steam point at 180°.
When the thermometer reads 5°, a centigrade thermometer will read
(1) 7.5°C (2) 12.5°C (3) 16.5°C (4) –9.37°C

C –0 t - ( -20) C 5 + 20
Solution : = (Here t = 5°) Þ = Þ C = 12.5°C
100 – 0 180 - ( -20) 100 200
Illustration The temperature of an iron piece is raised from 30°C to 90°C. What is the change in its temperature
on the Fahrenheit scale and on the Kelvin scale?
Solution DC=90°–30° = 60°C
9 9
Temperature difference on Fahrenheit Scale DF = DC = (60°C ) = 108°F
5 5
Temperature difference on Kelvin Scale DK = DC = 60K
Illustration A small ring having small gap is shown in figure on heating what will happen
to the size of gap.
Solution : Gap will also increase due to thermal expansion.
Illustration A rectangular plate has a circular cavity as shown in the figure. If we increase its temperature
then which dimension will increase in following figure.

Solution : Distance between any two point on an object increases with increase in temperature.
So, all dimension a, b, c and d will increase
176 E
Pre-Medical : Physics
Illustration There are two spheres of same radius and material at same temperature but one being solid
while the other hollow. Which sphere will expand more if they are heated to the same temperature,
Solution As thermal expansion of isotropic solids is similar to true photographic enlargement,
expansion of a cavity is same as if it had been a solid body of the same material
i.e. DV = Vg Dq

V V

As here V, g and Dq are same for both solid and hollow spheres treated (cavity) ; so the expansion
of both will be equal.
Illustration What is the percentage change in length of 1m iron rod if its temperature changes by 100ºC.
a for iron is 2 × 10–5/ºC.
Solution : percentage change in length due to temperature change

Dl
× 100 = aDq × 100 = 2 × 10–5 × 100 × 100 = 0.2%
l

Illustration 5 kg of steam at 100°C is mixed with 10 kg of ice at 0°C. Choose incorrect alternative
(Given swater = 1 cal/g°C, LF = 80 cal/g, LV = 540 cal/g)
(A) Equilibrium temperature of mixture is 160°C
(B) Equilibrium temperature of mixture is 100°C

1
(C) At equilibrium, mixture contains 13 kg of water
3
2
(D) At equilibrium, mixture contains 1 kg of steam
3
Solution Ans. (A)
Required heat Available heat
10 kg ice (0°C) 5 kg steam (100°C)

800 kcal 2700 Kcal

10 g water (0°C) 5 g water (100°C)

1000 kcal

10 g water (100°C)
So available heat is more than required heat therefore final temperature will be 100°C.

800 + 1000 10 10 40 1
Mass of heat condensed = = kg. Total mass of water = 10 + = = 13 kg
540 3 3 3 3

10 5 2
Total mass of steam = 5 - = = 1 kg
3 3 3

E 177
Pre-Medical : Physics
Illustration The temperature of equal masses of three different liquids A, B, and C are 10°C 15°C and
20°C respectively. The temperature when A and B are mixed is 13°C and when B and C are
mixed, it is 16°C. What will be the temperature when A and C are mixed?

Solution : when A and B are mixed

mS1 × (13 – 10) = m × S2 × (15 – 13)

Þ 3S1 = 2S2 .....(1)

when B and C are mixed S2 × 1 = S3 × 4 ....(2)

when C and A are mixed S1(q – 10) = S3 × (20 – q) ....(3)

140
by using equation (1), (2) and (3) we get q = °C
11

Illustration Two vessels of different materials are identical in size and wall–thickness. They are filled with
equal quantities of ice at 0°C. If the ice melts completely, in 10 and 25 minutes respectively
then compare the coefficients of thermal conductivity of the materials of the vessels.

Solution Let K1 and K2 be the coefficients of thermal conductivity of the materials, and t1 and t2 be the
time in which ice melts in the two vessels. Since both the vessels are identical, so A and L in
both the cases is same.

Now, Q =
K1A (q1 - q2 )t1
=
b
K 2 A q1 - q 2 t 2 g K1 t 2 25 min
ÞK =t = =
5
L L 2 1 10 min 2
Illustration Two plates of equal areas are placed in contact with each other. Their thickness are 2.0 cm
and 5.0 cm respectively. The temperature of the external surface of the first plate is –20°C
and that of the external surface of the second plate is 20°C. What will be the temperature of
the contact surface if the plate (i) are of the same material, (ii) have thermal conductivities in
the ratio 2 : 5.
2cm 5cm
Solution Rate of flow of heat in the plates is
plate 1

plate 2
Q K1A (q1 - q) K 2 A (q - q 2 )
= L1 = L2 ...(i)
t
-20 C
0
q 0
20 C
(i) Here q1 = –20°C, q2 = 20°C,

L1 = 2 cm = 0.02 m, L2 = 5 cm = 0.05 m and K1 = K2 = K

\ equation (i) becomes


b
KA -20 - q g =
b
KA q - 20 g
0.02 0.05
\ 5(–20–q) = 2(q – 20) Þ –100 – 5q = 2q – 40 Þ 7q = –60 Þ q = –8.6°C

K1 2 2
(ii) K 2 = 5 or K1 = 5 K2

\ from equation (i)


b
2 5 K 2 A -20 - q g = K Abq - 20g
2
Þ –20 – q = q – 20
0.02 0.05
\ q = 0°C

178 E
Pre-Medical : Physics
Illustration Three identical rods of length 1m each, having cross-section area of 1cm2 each and made of
Aluminium, copper and steel respectively are maintained at temperatures of 12°C, 4°C and 50°C
respectively at their separate ends. Find the temperature of their common junction.
[ KCu=400 W/m-K , KAl = 200 W/m-K, Ksteel = 50 W/m-K ]
50ºC

el
ste
12ºC
Aluminium

co
pp
er
4ºC

L 1 104
Solution RAl = = =
KA 200 ´ 10-4 200

104 104
Similarly Rsteel = and Rcopper =
50 400

Let temperature of common junction = T


50ºC
then from Kirchoff;s current laws, iAl + isteel + iCu = 0
iS
T - 12 T - 50 T - 4
Þ + + =0 iAl Rs
R Al R steel R Cu 12ºC
RAl T
Þ (T – 12) 200 + (T – 50) 50 + (T – 4) 400 = 0 RCu
iCu
Þ 4(T – 12) + (T – 50) + 8 (T – 4) = 0
Þ 13T = 48 + 50 + 32 = 130 Þ T = 10°C 4ºC
Illustration Water in a closed tube is heated with one arm vertically placed above the lamp. In what direction
water will begin the circulate along the tube ?

Solution On heating the liquid at A will become lighter and will rise up. This will push A B
the liquid in the tube upwards and so the liquid in the tube will move clockwise
i.e. from B to A.

Illustration Draw a graph between log E and log T y


4
Solution E= sT (taking log) log E
4
logE = log (sT ) x
log T
log s
logE = 4logT +logs

This is equilant to y = mx – C (s < 1 so its log is a negative quantity)

Illustration If temperature of ideal black body is increased by 50%, what will be percentage increase in
quantity of radiations emitted from its surface.
4 4
é 15 ù 4 é 3 ù 4 81 4
Solution E µ T 4
and \ E' µ (1.5) T µ ê ú T µ ê ú T µ
4 4 T
ë 10 û ë2 û 16

é 81 4 4ù
E '- E ê 16 T - T ú
´ 100% = ê ú ´ 100% = 406 % » 400 %
E ê T4 ú
ëê ûú
E 179
Pre-Medical : Physics
Illustration If lm for the moon is 14.5 micron, then find its temperature. (Assume balck body)
b 2.89 ´ 10-3
Solution Wien's displacement law lmT = b \ T = l = = 199.3K
m 14.5 ´ 10-6
Illustration Calculate the temperature at which a perfect black body radiates at the rate of 5.67 W cm–2.
Stefan's constant is 5.67 × 10–8 J s–1 m–2 K–4.
Solution Given E = 5.67 W cm–2 = 5.67 × 10+4 W m–2 , s = 5.67 × 10–8 J s–1 m–2 K–4
1 1

MNL Es OPQ
E 4 é 5.67 ´ 10+4 ù 4
Using, E = sT4; T4 = or T = = ê -8 ú = (1012)1/4 = 1000 K
s êë 5.67 ´ 10 úû

Illustration Two bodies A and B have thermal emissiviities of 0.01 and 0.81 respectively. The outer surface
areas of the two bodies are same, the two bodies emit total radiant power at the same rate.
The wavelength lB corresponding to maximum spectral radiancy of B is shifted from the wavelength
corresponding to maximum spectral radiancy in the radiation of A by 1.0 mm. If the temperature
of A is 5802K, Calculate :-
(a) The temperature of B (b) Wavelength lB
Solution (a) As both bodies A and B having same radiant power
\ PA = PB Þ eAsAATA4 = eBsABTB4 Þ (0.01)sATA4 = (0.81)sATB4
1/ 4
æ 0.01 ö TA 5802
TB = ç ÷ TA = = = 1934 K
è 0.81 ø 3 3
(b) According to wein's displacement law
æ 5802 ö
lATA = lBTB Þ lB = ç ÷ l = 3lA
è 1934 ø A

lB 2l B
As lB – lA = 1 µm Þ lB – = 1 µm Þ = 1 µm Þ lB = 1.5 µm.
3 3

Illustration Assuming Newton's law of cooling to be valid. The temperature of body changes from
60°C to 40°C in 7 minutes. Temperature of surroundings being 10°C, Find its temperature
after next 7 minutes.

q2 – q1 æ q1 + q2 ö
Solution According to Newton's law of cooling =K ç – q0 ÷
t è 2 ø
Since the temperature decreases from 60°C to 40°C in 7 minutes

60 – 40 FG 60 + 40 – 10IJ 20 1
7
= K
H 2 K Þ
7
= K (50 – 10) Þ K =
14
40 – q' 1 F 40 + q' I
7
= G
14 H 2
– 10J
K
1
Þ 40 – q' = (40 + q' – 20) Þ 160 – 4q' = 20 + q' Þ 5q' = 140 Þ q' = 28°C
4
Illustration Define (i) Steady state and (ii) Temperature gradient in conduction of heat through a conducting
rod.
Solution (i) When one end of a rod is heated, the temperature of various points of the rod changes continuously
but after some time a state is reached, when the temperature of each cross–section becomes
steady which is called steady state. In this state the heat received by any section will be totally
transfered to the next section so no heat is absorbed by any cross section.
(ii) Temperature gradient is defined as the rate of change of temperature with distance in the
direction of flow of heat.

180 E
Pre-Medical : Physics
Illustration By increasing temperature of gas by 5° C its pressure increases by 0.5% from its initial value
at constant volume then what is initial temperature of gas ?

DT DP 5 ´ 100
Solution Q At constant volume T µ P \ ´ 100 = ´ 100 = 0.5 Þ T = = 1000K
T P 0.5

Illustration A closed container of volume 0.02 m3 contains a mixture of neon and argon gases at a temperature
of 27°C and pressure of 1 × 105 N/m2. The total mass of the mixture is 28 g. If the gram
molecular weights of neon and argon are 20 and 40 respectively, find the mass of the individual
gases in the container, assuming them to be ideal. Given : R = 8.314 J/mol-K.

Solution Let m gram be the mass of neon. Then, the mass of argon is (28 – m)g.

m 28 - m 28 + m
Total number of moles of the mixture, µ= + = ...(i)
20 40 40

PV 1 ´ 105 ´ 0.02
Now, m= = =
0.8 ...(ii)
RT 8.314 ´ 300

28 + m
By (i) and (ii), = 0.8 Þ 28 + m = 32 Þ m = 4 gram
40
or mass of argon = (28 – 4)g = 24 g
Illustration At the top of a mountain a thermometer reads 7°C and barometer reads 70 cm of Hg. At the
bottom of the mountain they read 27°C and 76 cm of Hg respectively. Compare the density
of the air at the top with that at the bottom.

P R é M M ù
Solution By gas equation PV = M RT Þ = êQ m =
Mw
and
V
= rú
Mw rT M w ë û

éPù éPù
Now as MW and R are same for top and bottom ê rT ú = ê rT ú
ë û T ë ûB

rT PT TB 70 300 75
So r = P ´ T = ´ = = 0.9868
B B T 76 280 76

Illustration A vessel of volume 8.0 × 10–3 m3 contains an ideal gas at 300 K and 200 k Pa. The gas
is allowed to leak till the pressure falls to 125 kPa. Calculate the amount of the gas leaked
assuming that the temperature remains constant.
Solution As the gas leaks out, the volume and the temperature of the remaining gas do not change.
PV
The number of moles of the gas in the vessel in given by µ = .
RT

P1 V
The number of moles in the vessel before the leakage is m1 = and that after the leakage
RT
P2 V
is µ2 = .
RT

(P1 - P2 )V (200 - 125) ´ 103 ´ 8.0 ´ 10 -3


The amount leaked is µ 1 – µ 2 = = = 0.24 mole
RT 8.3 ´ 300

E 181
Pre-Medical : Physics
Illustration 1500 ml of a gas at a room temperature of 23°C is inhaled by a person whose body temperature
is 37°C, if the pressure and mass stay constant, what will be the volume of the gas in the lungs
of the person ?
Solution T1 = 273 + 23 = 296 K; T2 = 273 + 37 = 310 K. Pressure and amount of the gas are kept
constant,

V1 V2 T
So = \ V2 = V1 ´ 2 = 1500 ´ 310 = 1570.95 ml
T1 T2 T1 296

Illustration The velocities of ten particles in ms–1 are 0, 2, 3, 4, 4, 4, 5, 5, 6, 9. Calculate


(i) average speed and (ii) rms speed (iii) most probable speed.
0+2+3+ 4 + 4 + 4 +5 +5 +6+9
Solution (i) average speed, vav=
10
42
= = 4·2 ms–1
10
12
é (0)2 + (2)2 + (3)2 + (4)2 + (4)2 + (4)2 + (5)2 + (5)2 + (6) 2 + (9) 2 ù
(ii) rms speed, vrms = ê ú
ë 10 û
1/ 2
é 228 ù –1
= ê ú = 4.77 ms
ë 10 û
(iii) most probable speed vmp = 4 m/s
Illustration At what temperature root mean square velocity of hydrogen becomes double of its value at S.T.P.,
pressure remaining constant ?
Solution Let v1 be the r.m.s. velocity at S.T.P. and v2 be the r.m.s. velocity at unknown temperature T2.
v12 T
\ 2
= 1
v2 T2
2
év ù
or T2 = T1 ê 2 ú = 273 × (2)2 = 273 × 4 = 1092 K = (1092 – 273) = 819°C
ë v1 û
Illustration The pressure of one mole monoatomic gas increases linearly from 4 × 105 Nm–2 to 8 × 10+5
Nm–2 when its volume increases from 0.2m3 to 0.5 m3. Calculate.
(i) Work done by the gas, (ii) Increase in the internal energy,
(iii) Amount of heat supplied, (iv) Molar heat capacity of the gas R = 8.31 J mol–1 K–1
Solution P1 = 4 × 105 Nm–2 P2 = 8 × 10+5 Nm–2, V1 = 0.2 m3, V2 = 0.5 m3
(i) Work done by the gas = Area under P–V graph (Area ABCDEA)
P
1 1 8 B
= (AE +BD) × AC = (4 × 105 + 8 × 105) × (0.5–0.2)
2 2
(10 5 × N/m 2)

1
= × 12 × 105 × 0.3 = 1.8 × 105 J 4 C
2 A

E D
(ii) Increase in internal energy 0 0.2 3 0.5 V
(m )
CV CV
DU = CV (T2 – T1) = R(T2–T1) = (P2V2 – P1V1)
R R
3
For monoatomic gas CV = R
2
3 3
\ DU = [(8 × 105 × 0.5) – (4 × 105 × 0.2)] = [4 × 105 – 0.8 × 105 ] = 4.8 × 105 J
2 2
(iii) Q = DU + W = 4.8 × 105 + 1.8 × 105 = 6.6 × 105 J
Q QR = QR 6.6 ´ 105 ´ 8.31
(iv) C = =
n(P2 V2 - P1 V1 )
= = 17.14 J/mole K
nDT nRDT 1 ´ 3.2 ´ 105

182 E
Pre-Medical : Physics
Illustration As shown in figure when a system is taken from state a to state b, along the path
P
c b

a d

a ® c ® b, 60 J of heat flow into the system, and 30 J of work is done :

(i) How much heat flows into the system along the path a ® d ® b if the work is 10 J.

(ii) When the system is returned from b to a along the curved path, the work done by the
system is –20 J. Does the system absorb or liberate heat, and how much?

(iii) If, Ua = 0 and Ud = 22 J, find the heat absorbed in the process a ® d and d ® b.

Solution For the path acb DU = Q – W = 60 – 30 = 30 J or Ub – Ua = 30 J

(i) Along the path adb Q = DU + W = 30 + 10 = 40 J

(ii) Along the curved path ba Q = (Ua – Ub) + W = (–30) + (–20) = –50 J, heat liberates from system

(iii) Qad = Ud – Ua + Wad

but Wad = Wadb – Wdb = 10 – 0 = 10 Hence Qad = 22 – 0 + 10 = 32 J

and Qdb = Ub – Ud + Wdb = 30 – 22 + 0 = 8 J

Illustration Figure shows a process ABCA performed on an ideal gas. Find the net heat given to the system
during the process.

V
V2 C

A B
V1

T1 T2 T

Solution Since the process is cyclic, hence the change in internal energy is zero.

The heat given to the system is then equal to the work done by it.

The work done in part AB is W1 = 0 (the volume remains constant). The part BC represents
V2
an isothermal process so that the work done by the gas during this part is W2 = nRT2 ln
V1
nRT
During the part CA : V µ T So, V/T is constant and hence, P = is constant
V
The work done by the gas during the part CA is

W3 = P(V1 – V2) = nRT1 – nRT2 = – nR(T2 – T1).

The net work done by the gas in the process ABCA is

é V2 ù
W = W1 + W2 + W3 = nR ê T2 ln - (T2 - T1 )ú
ë V1 û
The same amount of heat is given to the gas.
E 183
Pre-Medical : Physics
Illustration Two samples of a gas initially at same temperature and pressure are compressed from a volume
V
V to . One sample is compressed isothermally and the other adiabatically. In which sample
2
is the pressure greater?
Solution Let initial volume, V1 = V and pressure, P1 = P , final volume,
V
V2 = and final pressure, P2 = ?
2

P1 V1 PV
For isothermal compression P2V2 = P1V1 or P2 = = = 2P
V2 V
2

g g
é V1 ù é V ù g
For adiabatic compression P2' = P1 ê ú Þ P2 ' = P ê ú =2 P
V
ë 2û ë V / 2 û
g g
Þ g
P ´ = 2 P > 1 \ 2 > 2 and P ' > P
2 2 2
Pressure during adiabatic compression is greater than the pressure during isothermal compression.
Illustration A sample of oxygen with volume of 500 cc at a pressure of 2 atm is compressed to a volume
of 400 cc. What pressure is needed to do this if the temperature is kept constant ?

V1 é 500 ù
Solution Temperature is constant, so P1 V1 = P2V2 \ P2 = P1 = 2ê ú = 2.5 atm
V2 ë 400 û
Illustration During an experiment it is found that an ideal gas follows and additional law VP2 = costant.
Initial temperature and volume of gas is T and V then what will be final temperature of the
gas it is expanded 2V volume ?

Solutions According to question VP2 = constant. Using gase equation PV = mRT

æ K ö mR
ç ÷ V = mRT , i.e., V= T
è Vø K
V1 æT ö V T
\ =ç 1÷ i.e., = or T' = ( 2 )T
V2 è T2 ø 2V T '
Illustration Plot P – V , V – T graph corresponding to the P–T graph for an ideal gas shown in figure.
Explain your answers.

A
V
Solution C
B

(P - V curve) T
1 (V - T curve)
For process AB T = constant so P µ For process BC P = constant so V µ T
V
1
For process CD T = constant so V µ For process DA P = constant so V µ T
P

184 E
Pre-Medical : Physics
Illustration A carnot engine working between 400 K and 800 K has a work output of 1200 J per cycle.
What is the amount of heat energy supplied to the engine from source per cycle?

T2 W 400 1200 1200


Solution W = 1200J, T1 = 800K, T2 = 400K \ h = 1 – = Þ 1 - 800 = Q Þ 0.5 = Q
T1 Q1 1 1

1200
Heat energy supplied by source Q1 = = 2400 joule per cycle
0.5
Illustration The temperatures T1 and T2 of the two heat reservoirs in an ideal carnot engine are 1500°C
and 500°C respectively. Which of the following : increasing T1 by 100°C or decreasing T2 by
100°C would result in a greater improvement in the efficiency of the engine?
Solution T1 = 1500°C = 1500 + 273 = 1773 K and T2 = 500°C = 500 + 273 = 773 K.

T2 773
The efficiency of a carnot's engine h = 1 - = 1- = 0.56
T1 1773

When the temperature of the source is increased by 100°C, keeping T2 unchanged, the new
temperature of the source is T´1 = 1500 + 100 = 1600°C = 1873 K.

T2 773
The efficiency becomes h´= 1 - = 1- = 0.59
T1 ' 1873

On the other hand, if the temperature of the sink is decreased by 100°C, keeping T1 unchanged,
the new temperature of the sink is T´2 = 500 – 100 = 400°C = 673 K. The efficiency now
becomes

T´2 673
h´´= 1 - =1- = 0.62
T1 1773

Since h´´ is greater than h´, decreasing the temperature of the sink by 100°C results in a greater
efficiency than increasing the temperature of the source by 100°C.
Illustration A heat engine operates between a cold reservoir at temperature T2 = 300 K and a hot reservoir
at temperature T1. It takes 200 J of heat from the hot reservoir and delivers 120 J of heat
to the cold reservoir in a cycle. What could be the minimum temperature of hot reservoir?

W 80
Solution Work done by the engine in a cycle is W = 200 – 120 = 80 J. h = = = 0.4
Q 200

T2 300 300 300


From carnot's Theorem 0.4 £ 1 - = 1- Þ £ 0.6 Þ T1 ³ Þ T1 ³ 500
T1 T1 T1 0.6

Illustration A refrigerator takes heat from water at 0°C and transfer it to room at 27°C. If 100 kg of water
is converted in ice at 0°C then calculate the work done. (Latent heat of ice is 3.4 × 105 J/kg)

T2 273 273
Solution Coefficient of performance (COP) = = =
T1 - T2 300 - 273 27

Q2 mL 100 ´ 3.4 ´ 105 100 ´ 3.4 ´ 105 ´ 27


W= = = = = 3.36 × 106 J
COP COP 273 / 27 273

E 185
Pre-Medical : Physics
THERMAL PHYSICS EXERCISE
1. Using which of the following instrument, the 5. A new scale of temperature (which is linear) called
temperature of the sun can be determined ? the W scale, the freezing and boiling points of water
(1) Platinum thermometer are 39°W and 239°W respectively. What will be the
temperature on the new scale, corresponding to a
(2) Gas thermometer
temperature of 39°C on the Celsius scale ?
(3) Pyrometer
(1) 200° W (2) 139° W (3) 78° W (4) 117°W
(4) Vapour pressure thermometer 6. At STP a rod is hung from a frame as shown in
figure, leaving a small gap between the rod and
2. Which of the curves in figure represents the relation
floor. The frame and rod system is heated uniformly
between Celsius and Fahrenheit temperature?
upto 350 K. Then
(1) Curve a (2) Curve b
e
m
(3) Curve c (4) Curve d rf a
Rod

C
c
b
(1) The rod will never touch the floor in any case.
F
(2) If arod > aframe, then rod may touch the floor.
d a (3) If arod < aframe, then rod may touch the floor.
(4) None of the above

7. The figure below shows four isotropic solids having


3. The graph AB shown in figure is a plot of positive coefficient of thermal expansion. A student
temperature of a body in degree Celsius and predicts that on heating the solid following things
degree Fahrenheit. Then can happen. Mark true (T) or False (F) for comments
made by the student.
100°C B

A
Centigrade

a
B

32°F 212°F Fahrenheit (i) The angle a in figure (1) will not change.
A

(ii) The length of line in figure (2) will decrease.


(1) slope of line AB is 9/5
(iii) The radius of inner hole will decrease.
(2) slope of line AB is 5/9
(3) slope of line AB is 1/9 (iv) The distance AB will increase.
(4) slope of line AB is 3/9
(1) T F F T (2) F T T F (3) T T T T (4) F F T F
4. Two thermometers X and Y have ice points marked
8. Suppose there is a hole in a copper plate. On
at 15° and 20° and steam points marked as 75°
and 100° respectively. When thermometer X heating the plate, diameter of hole, would :
measures the temperature of a bath as 60° on it, (1) always increase
what would thermometer Y read when it is used to (2) always decrease
measure the temperature of the same bath ? (3) always remain the same
(1) 60° (2) 75° (3) 80° (4) 90° (4) none of these

186 E
Pre-Medical : Physics
9. The table gives the initial length l 0, change in
L 5L
temperature DT and change in length Dl of four rods. (1)
5g
(2)
g
Which rod has greatest coefficient of linear expansion

Rod l0(m) D T(°C) D l(°C) gL mL


(3) (4)
5m 5g
A1 1 100 1

A2 1 100 2 16. A bullet of mass 10 g moving with a speed of


A3 1.5 50 3 20 m/s hits an ice block of mass 990 g kept on a
A4 2.5 20 4 frictionless floor and gets stuck in it. How much ice
will melt if 50% of the lost KE goes to ice ? (initial
(1) A1 (2) A2 (3) A3 (4) A4
temperature of the ice block and bullet = 0°C)
10. Two rods one of aluminium of length l1 having
coefficient of linear expansion aa, and other steel (1) 0.001 g (2) 0.002 g
of length l2 having coefficient of linear expansion (3) 0.003 g (4) 0.004 g
as are joined end to end. The expansion in both the 17. A piece of ice falls from a height h so that it melts
rods is same on variation of temperature. Then the
completely. Only half of the heat produced is
l1
value of l + l is absorbed by the ice and all energy of ice gets
1 2
converted into heat during its fall. The value
as as
(1) a + a (2) a - a of h is :
a s a s
[Latent heat of ice is 3.4 × 105 J/kg and
aa + as
(3) as (4) None of these g = 10 N/kg]
11. An iron bar (Young’s modulus = 1011 N/m2 , (1) 34 km (2) 136 km
a = 10–6 /°C) 1 m long and 10–3 m2 in area is (3) 68 km (4) 544 km
heated from 0°C to 100°C without being allowed to
18. The amount of heat required to convert 1 gm of ice at
bend or expand. Find the compressive force
0°C into steam at 100°C, is
developed inside the bar.
(1) 10,000 N (2) 1000 N (1) 716 cal. (2) 500 cal.
(3) 5000 N (4) 105 N (3) 180 cal. (4) 100 cal.
12. A rod of length 2m rests on smooth horizontal floor. 19. The graph shown in the figure represent change in
If the rod is heated from 0°C to 20°C. Find the the temperature of 5 kg of a substance as it abosrbs
longitudinal strain developed? heat at a constant rate of 42 kJ min–1. The latent
(a = 5 × 10–5/°C) heat of vapourazation of the substance is :
(1) 10–3 (2) 2 × 10–3 (3) Zero (4) None
13. 4
An ideal gas is expanding such that PT = constant.
The coefficient of volume expansion of the gas is :

1 3 5 4
(1) (2) (3) (4)
Temperature

T T T T
14. A bullet moving with velocity v collides against wall.
consequently half of its kinetic energy is converted
into heat. If the whole heat is acquired by the bullet,
the rise in temperature will be:–
(1) v2/4S (2) 4v2 / 2S (3) v2 / 2S (4) v2 / S
15. A block of ice with mass m falls into a lake. After
(1) 630 kJ kg–1
impact, a mass of ice m/5 melts. Both the block of
ice and the lake have a temperature of 0°C. If L (2) 126 kJ kg–1
represents the heat of fusion, the minimum distance (3) 84 kJ kg–1
the ice fell before striking the surface is (4) 12.6 kJ kg–1
E 187
Pre-Medical : Physics
20. Figure shows the temperature variation when heat 24. 1 kg of ice at – 10°C is mixed with 4.4 kg of water
is added continuously to a specimen of ice (10 g) at at 30°C. The final temperature of mixture is :
–40 °C at constant rate. (1) 2.3°C (2) 4.4°C (3) 5.3°C (4) 8.7°C
(Specific heat of ice = 0.53 cal/g °C and Lice = 80 25. 2 kg ice at – 20°C is mixed with 5 kg water at
cal/g, Lwater= 540 cal/g) 20°C. Then final amount of water in the mixture
would be;
Temp. (°C) Given specific heat of ice = 0.5 cal/g°C,
100 Specific heat of water = 1 cal/g°C,
Latent heat of fusion for ice = 80 cal/g.
(1) 6 kg (2) 5 kg (3) 4 kg (4) 2 kg
0 26. Equal volume of H2, O2 and He gases are at same
Q(cal) temperature and pressure. Which of these will have
-40
large number of molecules :-
Q1 Q2 Q3 Q4
(1) H2
Column–I Column–II (2) O2
(A) Value of Q1 (in cal) (P) 800 (3) He
(B) Value of Q2 (in cal) (Q) 1000 (4) All the gase will have same number of
molecules
(C) Value of Q3 (in cal) (R) 5400
27. The equation of state for 22g of CO2 at a pressure
(D) Value of Q4 (in cal) (S) 212
P and temperature T, when occupying a volume V,
(T) 900
will be :– (where R is the gas constant.)
(1) A®S; B®P; C®Q; D®T (1) PV = 5 RT (2) PV = (5/2) RT
(2) A®P; B®S; C®Q; D®R (3) PV = (5/16) RT (4) PV = (1/2)RT
(3) A®P; B®S; C®R; D®Q 28. Two vessels separately contain two ideal gases
(4) A®S; B®P; C®Q; D®R A and B at the same temperature, the pressure of
21. A 2100 W continuous flow geyser (instant geyser) A being twice that of B. Under such conditions, the
has water inlet temperature = 10°C while the water density of A is found to be 2.5 times the density of
flows out at the rate of 20 g/sec. The outlet B. The ratio of molecular weight of A and B
temperature of water must be about is:
(1) 20°C (2) 30°C
1 3
(3) 35°C (4) 40°C (1) (2)
2 4
22. Two identical bodies are made of a material for
which the heat capacity increases with temperature. 5 4
One of these is at 100 °C, while the other one is (3) (4)
4 3
at 0°C. If the two bodies are brought into contact,
29. A given sample of an ideal gas occupies a volume
then, assuming no heat loss, the final common
V at a pressure P and absolute temperature T. The
temperature is :-
mass of each molecule of the gas is m. Which of the
(1) less than 50 °C but greater than 0 °C following gives the density of the gas ?
(2) 0 °C (1) P/(kTV) (2) mkT
(3) 50 °C (3) P/(kT) (4) Pm/(kT)
(4) more than 50 °C 30. A balloon contains 500 m of helium at 270C and
3

23. Steam at 100°C is added slowly to 1400 gm of 1 atmosphere pressure. The volume of the helium
water at 16°C until the temperature of water is raised at –3 0C temperature and 0.5 atmosphere
to 80°C. The mass of steam required to do this is pressure will be-
(LV = 540 cal/gm) : (1) 500 m3 (2) 700 m3
(1) 160 gm (2) 125 mg (3) 250 gm (4) 320 gm
(3) 900 m3 (4) 1000 m3

188 E
Pre-Medical : Physics
31. On increasing the temperature of a gas filled in a 37. An ideal gas mixture filled inside a balloon expands
closed container by 10C its pressure increases by according to the relation PV2/3 = constant. The
0.4%, then initial temperature of the gas is- temperature inside the balloon is
(1) increasing (2) decreasing
(1) 250C (2) 2500C (3) 250 K (4) 25000C
(3) constant (4) can’t be said
32. 28 gm of N2 gas is contained in a flask at a pressure 38. An ideal gas follows a process PT = constant. The
10 atm. and at a temperature of 57°C. It is found correct graph between pressure & volume is :-
that due to leakage in the flask, the pressure is
reduced to half and the temperature reduced to
27°C. The quantity of N2 gas that leaked out is –
(1) (2)
(1) 11/20 gm (2) 80/11 gm
(3) 5/63 gm (4) 63/5 gm
33. A vessel has 6g of oxygen at pressure P and
temperature 400 K. A small hole is made in it so (3) (4)
that oxygen leaks out. How much oxygen leaks out
if the final pressure is P/2 and temperature is
300 K ? 39. A cyclic process ABCA is shown in P–T diagram.
When presented on P–V, it would
(1) 3g (2) 2g (3) 4g (4) 5g
34. Two closed containers of equal volume filled with air
at pressure P 0 and temperature T 0. Both are
connected by a narrow tube. If one of the container
is maintained at temperature T 0 and other at
temperature T, then new pressure in the conainers
will be :-
(1) (2)
2P0 T P0 T
(1) T + T (2) T + T
0 0

P0 T T + T0
(3) 2(T + T ) (4) P0 (3) (4)
0

35. Two identical glass bulbs are interconnected by a 40. For P-V diagram of a thermodynamic cycle as shown
thin glass tube at 0ºC. A gas is filled at N.T.P. in in figure, process BC and DA are isothermal. Which
these bulb is placed in ice and another bulb is placed of the corresponding graphs is correct ?
in hot bath, then the pressure of the gas becomes P
1.5 times. The temperature of hot bath will be :– A B

C
D
V
Ice Hot water P A B P A B

(1) 100°C (2) 182°C (3) 256°C (4) 546°C (1) C (2) C
D D
36. An ideal gas expands according to the law (0,0)
(0,0) T T
P2V = constant. The internal energy of the gas :-
V D C V C B
(1) Increases continuously
(2) Decreases continuously B A
(3) (4)
(3) Remains constant A D
(4) First increases and then decreases (0,0) T (0,0) T
E 189
Pre-Medical : Physics
41. Figure shows the isotherms of a fixed mass of an 47. Two containers of same volume are filled with atomic
ideal gas at three temperatures TA, TB and TC then Hydrogen and Helium respectively at 1 and 2 atm
pressure. If the temperature of both specimen are
1 A same then average speed V for hydrogen atoms
V will be -
B
(1) VH = 2 VHe

C
(2) VH = VHe

O P (3) VH = 2 VHe

(1) TA > TB > TC


VHe
(2) TA < TB < TC (4) VH =
2
(3) TB < TA < TC 48. The root mean square (rms) speed of oxygen
(4) TA = TB = TC molecules O2 at a certain temperature T (absolute)
42. Consider a gas with density r and c as the root is v. If the temperature is doubled and oxygen gas
mean square velocity of its molecules contained in dissociates into atomic oxygen. The rms speed :
a volume. If the system moves as whole with velocity (1) becomes v/ 2 (2) remains v
v, then the pressure exerted by the gas is
(3) becomes 2v (4) becomes 2v
1 1
(1) r ( c)2 (2) r( c + v)2
3 3 49. The molecules o f a given mass o f a gas
have r.m.s. velocity of 200 m/s at 127°C and
1 1
(3) r( – v)2 (4) r(c–2 – v)2 1.0 × 105 N/m2 pressure. When the temperature
3 c 3
and pressure of the gas are respectively, 227°C and
43. Three particles have speeds of 2u , 10u and 11u.
0.05 ×105 N/m2, the r.m.s. velocity of its molecules
Which of the following statements is correct?
in m/s is :
(1) The r.m.s. speed exceeds the mean speed by
about u. (1) 100 2 (2) 100 5

(2) The mean speed exceeds the r.m.s. speed by 200 500 2
(3) (4)
about u. 3 3

(3) The r.m.s. speed equals the mean speed. 50. A gas is equilibrium at T kelvin. If mass of one
(4) The r.m.s. speed exceeds the mean speed by molecule is m and its component of velocity in
more than 2u.
y direction is vy. Then mean of its vy2 is
44. For the molecules of an Ideal gas, Which of the
following velocity average can not be zero 3kT 2kT
(1) (2)
m m
(1) < v > (2) < v4 >
(3) < v3 > (4) < v5 > kT
(3) (4) zero
45. The speeds of 5 molecules of a gas (in arbitary m
units) are as follows 2,3,4,5,6 The root mean square 51. According to Maxwell's law of distribution of
speed for these moecules is - velocites of molecules, the most probable velocity
(1) 2.91 (2) 3.52 (3) 4.00 (4) 4.24 is :-
46. If the root mean square speed of hydrogen (1) greater than the mean velocity
molecules is equal to root mean square speed of
(2) equal to the mean velocity
oxygen molecules at 470C, the temperature of
hydrogen is- (3) equal to the root mean square velocity
(1) 20 K (2) 47 K (3) 50 K (4) 80 K (4) less than the root mean square velocity

190 E
Pre-Medical : Physics
52. The reason for the absence of atmosphere on moon 58. The mean free path of molecules of a gas, (radius 'r')
is that the : is inversely proportional to :-
(1) Value of vrms of the molecules of gas is more
(1) r3 (2) r2 (3) r (4) r
than the value of escape velocity
(2) Value of vrms of gas is less than escape velocity 59. The ratio of number of collisions per second at the
(3) Value of vrms is negiligible walls of containers by He and O 2 gas molecule kept
(4) None of the above at same volume and temperature, is (assume normal
53. A gas mixture consists of 2 moles of oxygen and 4 incidence on walls)
moles of argon at temperature T. Neglecting all (1) 2 : 1 (2) 1 : 2
vibrational modes, the total internal energy of the
(3) 2 2 : 1 (4) 1 : 2 2
system is
(1) 4 RT (2) 15 RT (3) 9 RT (4) 11 RT 60. The specific heat of an ideal gas depends on
54. N molecules of an ideal gas at temperature T1 and temperature is -
presseure P1 are contained in a closed box. If the
molecules in the box gets doubled, Keeping total 1
(1)
kinetic energy as same then if new pressure is P2 T
and temperature is T2, Then :
(2) T
(1) P2 = P, T2 = T1
(2) P2 = P1, T2 = T1 / 2 (3) T
(3) P2 = 2 P1, T2 = T1
(4) P2 = 2P1, T2 = T1 / 2 (4) Does not depends on temperature
55. Relation between pressure (P) and energy density 61. The equation of state of a gas is given by
(E) of an ideal gas is -
(1) P = 2/3 E (2) P = 3/2 E æ aT2 ö C
çP + ÷ V = (RT + b), where a, b, c and R are
è V ø
(3) P = 3/5 E (4) P = E
56. Three monoatomic perfect gases at absolute constants. This isotherms can be represented by
temperature T1, T2 and T3 are mixed. If number of P = AVm – BVn, where A and B depend only on
molecules of the gases are n1, n2 and n3 respectively temperature and
then temperature of mixture will be (assume no loss
(1) m = – c and n = – 1
of energy)
(2) m = c and n = 1
T1 + T2 + T3 2 2 2
n T1 + n T2 + n T3
1 2 3 (3) m = – c and n = 1
(1) (2)
3 3 (4) m = c and n = – 1

n1 T1 + n2 T2 + n 3 T3 T1 + T2 + T3 62. For hydrogen gas cP – cV = a and for a oxygen gas


(3) n1 + n2 + n3 (4) n + n + n cP – cV = b then the relation between a and b is
1 2 3

(where cP & cV are gram specific heats)


57. Which of the following statement is true according
(1) a = 16 b (2) b = 16 a
to kinetic theory of gases?
(3) a = b (4) None of these
(1) The collision between two molecules is inelastic
and the time between two collisions is less than R
63. For a gas = 0.4. This gas is made up of
the time taken during the collision. CV
(2) There is a force of attraction between the
molecules which are :
molecules
(1) Monoatomic
(3) All the molecules of a gas move with same
(2) Mixture of diatomic and polyatomic molecules
velocity
(3) Diatomic
(4) The average of the distances travelled between
(4) Polyatomic
two successive collisions is mean free path.

E 191
Pre-Medical : Physics
64. If 2 gm moles of a diatomic gas and 1 gm mole of a 71. In an isometric change :
mono-atomic gas are mixed then the value of g (= (1) dQ = dU (2) dW = dU
Cp/Cv) for mixture will be :–
(3) dQ + dW = dU (4) None of these
13 19 7 5
(1) (2) (3) (4) 72. Calculate the work done for B ® A, :-
19 13 5 3
V(L)
65. For a certain process, pressure of diatomic gas varies
A
according to the relation P = aV2, where a is 4
constant. What is the molar heat capacity of the gas
for this process ?
2 B
C
17R 6R 13R 16R
(1) (2) (3) (4) 2
6 17 6 7 0 1 5 P(N/m )
66. Molar specific heat at constant volume, for a non- (1) 6 × 10–3 J (2) 12 × 10–3 J
linear triatomic gas is (vibration mode neglected) (3) 3 × 10–3 J (4) 4 × 10–3 J
(1) 3R (2) 4R (3) 2R (4) R 73. In a cyclic process shown on the P – V diagram the
67. Relation between the ratio of specific heats (g) of magnitude of the work done is :
gas and degree of freedom 'f'' will be
P

1 1 1 P2
(1) g = f + 2 (2) = +
g f 2
P1
(3) f = 2 / (g-1) (4) f = 2( g-1) V
O V1 V2
68. One mole of an ideal monatomic gas undergoes a
process described by the equation PV5 = constant. 2 2
æ P2 - P1 ö æ V2 - V1 ö
The heat capacity of the gas during this process is (1) p ç ÷ (2) p ç ÷
è 2 ø è 2 ø
3 5 3 5
(1) R (2) R (3) R (4) R p
2 4 4 2 (3) (P – P1) (V2 – V1) (4) p (P2V2 – P1V1)
4 2
69. Consider the process on a system shown in figure.
74. A thermodynamic system undergoes cyclic process
During the process, the work done by the system.
ABCDA as shown in fig. The work done by the
system in the cycle is :-
Pressure

1 2 P
C B
3P 0

2P 0

P0 D
Volume A

(1) Continuously increases V0 2V 0 V


(2) Continuously decreases
(3) First increases then decreases P0 V0
(1) P0V0 (2) 2P0V0 (3) (4) Zero
(4) First decreases then increases 2
70. One mole of an ideal gas at temperature T1 expends 75. If the ratio of specific heat of a gas at constant
P pressure to that at constant volume is g, the change
according to the law 2 = a (constant). The work
V in internal energy of gas, when the volume changes
done by the gas till temperature of gas becomes T2 from V to 2V at constant pressure P, is:-
is :
PV
1 1 (1) (2) PV
(1) R(T2 – T1) (2) R(T2 – T1) ( g - 1)
2 3
1 1 R gPV
(3) R(T2 – T1) (4) R(T2 – T1) (3) (4)
4 5 ( g - 1) ( g - 1)

192 E
Pre-Medical : Physics
76. A system is taken along the paths A and B as shown. 81. A thermodynamical process is shown in figure with
If the amounts of heat given in these processes are PA = 3 ×104 Pa; VA = 2 ×10–3 m3; PB = 8 ×104 Pa,
DQA and DQB and change in internal energy are VC = 5 ×10–3 m3. In the processes AB and BC, 200
DUA and DUB respectively then :- J and 600 J of heat is added to the system
respectively. The change in internal energy of the
(1) DQA = DQB; DUA < DUB A system in process AC would be :-
(2) DQA ³ DQB; DUA = DUB P i f
(3) DQA < DQB; DUA > DUB B P
B C
(4) DQA > DQB; DUA = DUB V
77. The amount of heat energy required to raise the
temperature of 1 g of Helium at NTP, from T1 K
to T2 K at constant volume is :- A

5 æ T2 ö 5 V
(1) Na kB ç T ÷ (2) N k (T – T1)
4 è 1ø 8 a B 2
(1) 560 J (2) 800 J (3) 600 J (4) 640 J
5 5
(3) N k (T – T1) (4) N k (T – T1) 82. When a system is taken from state ‘a’ to state
2 a B 2 4 a B 2
‘b’ along the path ‘acb’, it is found that a quan-
78. 1 kg of a gas does 20 kJ of work and receives
tity of heat Q = 200 J is absorbed by the system
16 kJ of heat when it is expanded between two
and a work W = 80J is done by it. Along the
states. A second kind of expansion can be found
path ‘adb’, Q = 144J. The work done along the
between the same initial and final state which
path ‘adb’ is
requires a heat input of 9 kJ. The work done by the
gas in the second expansion is :
p
(1) 32 kJ (2) 5 kJ (3) –4 kJ (4) 13 kJ c b
79. A P-T graph is shown for a cyclic process. Select
correct statement regarding this
P
C a d
V

B (1) 6J (2) 12 J (3) 18 J (4) 24 J


D
A 83. A gas for which g = 4/3 is heated at constant
O T
pressure. The percentage of total heat given that
(1) During process CD, work done by gas is negative
will be used for external work is :
(2) During process AB, work done by gas is positive
(3) During process BC internal energy of system
(1) 40% (2) 25% (3) 60% (4) 20%
increases
(4) During process BC internal energy of system 84. For monoatomic gas the relation between pressure
decreases of a gas and temperature T is P2 a TC where C is.
80. In the diagram shown Q ia f = 80 cal and (For adiabatic process)
Wiaf = 50 cal. If W = –30 cal for the curved path
fi, value of Q for path fi, will be :- 5 5 3 10
(1) (2) (3) (4)
3 2 5 2
P
a f 85. An ideal monatomic gas at 300 K expands
adiabatically to 8 times its volume. What is the final
i temperature ?
(0, 0) V
(1) 75 K (2) 300 K
(1) 60 cal (2) 30 cal (3) –30 cal (4) –60 cal (3) 560 K (4) 340 K
E 193
Pre-Medical : Physics
91. One mole of a monatomic ideal gas is taken through
æ 4ö
86. The volume of a poly-atomic gas ç g = ÷ a cycle ABCDA as shown in the P-V diagram.
è 3ø
Column-II gives the characteristics involved in the
1
compressed adiabatically to of the original cycle. Match them with each of the processes given
27th
volume. If the original pressure of the gas is P0 the in Column-I.

new pressure will be


P
(1) 8 P0 (2) 81 P0 B
3P A
(3) 16 P0 (4) 2 P0
87. A diatomic gas undergoes adiabatic compression
and its volume reduces to half of initial volume then 1P
C D
its final pressure would be if gas initial pressure P
(1) 21.4P (2) P/2 0 1V 3V 9V V
(3) 2P (4) 3.07 P
Column-I Column-II
88. The molar heat capacity in a process of a diatomic (A) Process A ® B (p) Internal energy decreases
Q (B) Process B ® C (q) Internal energy increases
gas, if it does a work of when a heat of Q is
4 (C) Process C ® D (r) Heat is lost
supplied to it, is :-
(D) Process D ® A (s) Heat is gained
2 5 (t) Work is done on the gas
(1) R (2) R
5 2
(1) A ® p,r,t ; B ® p,r ; C ® q,s ; D ® r,t

10 6 (2) A ® r,t ; B ® q,r ; C ® q,s ; D ® r,s


(3) R (4) R
3 7 (3) A ® p,r,t; B ® p,q,r ; C ® s ; D ® r,t

89. Which of the following graphs correctly represents (4) A ® p,r,t ; B ® p,r ; C®q; D ® r,s
the variation of b = –(dV/dP)/V with P for an ideal
92. One mole of an ideal gas goes from an initial state
gas at constant temperature?
A to final state B via two processes. It firstly
undergoes isothermal expansion from volume V to
3V and then its volume is reduced from 3V to V at
(1) (2) constant pressure. The correct P-V diagram
representing the two processes is :-

A A
(3) (4)
P B P B

(1) (2)
90. An ideal gas undergoes the process 1 ® 2 as shown V 3V V 3V
in the figure, the heat supplied and work done in V V
the process is DQ and DW respectively. The ratio
Q : W is
B A
(1) g : g – 1
P A P
(2) g
(3) g – 1 (3) (4) B
V 3V V 3V
(4) g – 1g V V

194 E
Pre-Medical : Physics
93. Thermodynamic processes are indicated in the 96. A gas speciman in one vessel is expended
following diagram : isothermally to double its volume and a simillar
specimen in the second vessel is expanded
P adiabatically the same extent, then :
(1) In the second vessel, both pressure and work
i IV f
I III
done are more
II f (2) In the second vessel, pressure in more, but the
f 900K
f 700K work done isless.
500K
V (3) In the first vessel, both pressure & work done
are more.
Match the following
(4) In the first vessel, pressure is more, but work
Column-1 Column-2 done is less
97. A gas is compressed isothermally to half its initial
P. Process I a. Adiabatic
volume. The same gas is compressed separately
Q. Process II b. Isobaric through an adiabatic process until its volume is again
reduced to half. Then :-
R. Process III c. Isochoric
(1) Compressing the gas isothermally will require
S. Process IV d. Isothermal more work to be done.
(2) Compressing the gas through adiabatic process
(1) P ® c, Q ® a, R ® d, S ® b will require more work to be done.
(2) P ® c, Q ® d, R ® b, S ® a (3) Co mpre ssin g th e ga s i soth erma lly or
adiabatically will require the same amount of
(3) P ® d, Q ® b, R ® a, S ® c
work.
(4) P ® a, Q ® c, R ® d, S ® b (4) Which of the case (whether compression through
isothermal or through adiabatic process) requires
94. P-V plots for two gases during adiabatic processes
more work will depend upon the atomicity of
are shown in the figure. Plots 1 and 2 should
the gas.
correspond respectively to 98. A closed container is fully insulated from outside.
One half of it is filled with an ideal gas X separated
by a plate P from the other half Y which contains a
vacuum as shown in figure. When P is removed, X
moves into Y. Which of the following statements is
correct?

(1) He and O2 (2) O2 and He X Y


(3) He and Ar (4) O2 and N2 gas vacuum
p
95. In which of the figure no heat exchange between
(1) No work is done by X
the gas and the surroundings will take place, if the
(2) X decreases in temperature
gas is taken along curve:
(3) X increases in internal energy
(curves are isothermal and adiabatic) (4) X doubles in pressure
99. According to the second law of thermodynamics :
(1) heat energy cannot be completely converted
to work
Pressure

A C
(2) work cannot be completely converted to heat
B D
energy
Volume (3) for all cyclic processes we have dQ/T < 0
(1) A (2) B (3) C (4) D (4) the reason all heat engine efficiencies are less
than 100% is friction, which is unavoidable

E 195
Pre-Medical : Physics
100. A Carnot engine takes 3 × 106 cal of heat from 107. A refrigerator transfer 360 joule of energy in one
reservoir at 627°C and gives it to a sink at 27°C. second from temperature –3°C to 27°C. Calculate
Then work done by the engine is the average power consumed, assuming no energy
(1) 4.2 × 106 J (2) 8.4 × 106 J losses in the process.
(3) 16.8 × 106 J (4) zero (1) 18 W (2) 54 W
101. A Carnot engine working between 300 K and (3) 40 W (4) 120 W
600 K has a work output of 800 J per cycle. The 108. A reversible refrigerator operates between a low
amount of heat energy supplied to engine from the temperature reservoir at TC and a high temperature
source in each cycle is :- reservoir at TH. Its coefficient of performance is
(1) 800 J (2) 1600 J (3) 3200 J (4) 6400 J given by :
102. The efficiency of an ideal heat engine working (1) (TH – TC)/TC (2) TC/(TH – TC)
between the freezing point and boiling point of
(3) (TH – TC)/TH (4) TH/(TH – TC)
water, is :-
(1) 26.8% (2) 20% (3) 6.25% (4) 12.5% 109. A refrigerator transfer 180 joule of energy in one
103. A Carnot engine whose sink is at 300 K has an second from temperature –0°C to 30°C. Calculate
efficiency of 40%. By how much amount should the the average power consumed, assuming no energy
temperature of source be increased so as to increase losses in the process. (approx)
its efficiency by 50% of original efficiency? (1) 18 W (2) 54 W (3) 20 W (4) 120 W
(1) 150 K (2) 250 K (3) 300 K (4) 450 K
110. The coefficient of performance of a refrigerator is
104. A Carnot engine takes 3000 kcal of heat from a 5. If the temperature inside freezer is –20°C, the
reservoir at 627°C and gives a part of it to a sink temperature of the surroundings to which it rejects
at 27°C. The work done by the engine is :- heat is :
(1) 4.2 × 106 J (1) 21°C (2) 31°C (3) 41°C (4) 11°C

(2) 8.4 × 106 J 111. A refrigerator works between 4°C and 30°C. It is
required to remove 300 calories of heat every
(3) 16.8 × 106 J
second in order to keep the temperature of the
(4) Zero refrigerated space constant. The power required is:
105. The efficiency of a Carnot's engine at a particular (Take 1 cal = 4.2 Joules)
1 (1) 1.182 W (2) 11.82 W
source and sink temperature is . When the sink
2
(3) 118.25 W (4) 1182 W
temperature is reduced by 100°C, the engine
112. The temperature inside a refrigerator is t2°C and the
2
efficiency becomes . Find the source temperature. room temperature is t1°C. The amount of heat
3
delivered to the room for each joule of electrical
(1) 300 K (2) 600 K (3) 900 K (4) 1200 K energy consumed ideally will be :-

106. The efficiency of carnot engine is 50% and


t2 + 273 t1 + t2
temperature of sink is 500K. If temperature of (1) t1 - t 2 (2) t + 273
1
source is kept constant and its efficiency raised to
60%, then the required temperature of the sink
t1 t1 + 273
will be :- (3) t - t (4)
1 2 t1 - t 2
(1) 100 K (2) 600 K (3) 400 K (4) 500 K

196 E
Pre-Medical : Physics
119. The figure shows the face and interface
1
113. A carnot engine having an efficiency of as heat temperature of a composite slab containing of four
20
layers of two materials having identical thickness.
engine, is used as a refrigerator. If the work done on Under steady state condition, find the value of
the system is 20 J, the amount of energy absorbed temperature q.
from the reservoir at lower temperature is :-

(1) 380 J (2) 399 J (3) 400 J (4) 10 J


114. The P-V diagram represents the thermodynamic
cycle of an engine, operating with an ideal
monoatomic gas. The amount of heat, extracted
from the source in a single cycle is :
(1) 5°C (2) 10°C (3) –15°C (4) 15°C
P
120. Three rods of same dimensions have thermal
2p0 B C conductivities 3K, 2K and K. They are arranged as
shown, with their ends at 100°C, 50°C and 0°C. The
p0 temperature of their junction is :-
A D
50°C
V 2K
v0 2v0 3K
100°C
13 K
(1) p0v0 (2) æç ö÷ p0 v 0 0°C
è 2 ø
200 100
æ 11 ö (1) 75°C (2) °C (3) 40°C (4) °C
(3) ç ÷ p0 v 0 (4) 4p0v0 3 3
è 2 ø
121. The coefficient of thermal conductivity of copper is
115. In the above question efficiency of cycle ABCDA is nine times that of steel. In the composite cylindrical
nearly : bar shown in the figure what will be the temperature
(1) 12.5% (2) 15.2% (3) 9.1% (4) 10.5% at the junction of copper and steel ?
116 On a cold morning, a metal surface will feel colder
to touch than a wooden surface because (1) 80°C 100°C 20°C

(1) Metal has high specific heat (2) 67°C copper iron

(2) Metal has high thermal conductivity (3) 33°C 18cm 6cm

(3) Metal has low specific heat (4) 25°C


(4) Metal has low thermal conductivity 122. A Cylinder of radius R made of material of thermal
117. The ratio of cofficient of thermal conductivity of two conductivity K1 is surrounded by a cylindrical shell
different materials is 4:9. If the thermal resistance of inner radius R and outer radius 2 R made of a
of rods of same thickness of these material is same, meterial of thermal conductivity K2. The two ends
then what is ratio of length of these rods - of combined system are maintained at two different
(1) 3:5 (2) 4:9 (3) 25:9 (4) 9:25 temp there is no loss of heat across cylindrical surface
118. Which of the following cylindrical rods will conduct and system is in steady state calculate effective
most heat, when their ends are maintained at the thermal conductivity of system.
same steady temperature
K1 + 3K 2
(1) Length 1 m; radius 1 cm (1) (2) K1 + K
4
(2) Length 2 m; radius 1 cm
(3) Length 2 m; radius 2 cm K1 + 8K 2 8K1 + K 2
(3) (4)
(4) Length 1 m; radius 2 cm 9 9

E 197
Pre-Medical : Physics
123. Two conductors having thickness d 1 and d 2, 126. Two identical square rods of metal are welded end
thermal conductivity k 1 and k2 are placed one to end as shown in figure (1), 20 calories of heat
above the another. Find the equivalent thermal flows through it in 4 minutes. If the rods are welded
conductance :- as shown in figure (2), the same amount of heat will
flow through the rods in -
T1
d1
d2 0°C 0°C 100°C
100°C
T2 l l
l
(a) (b)
(d1 + d2 )(k1d 2 + k 2 d1 )
(1) 2(k1 + k2 )
(1) 1 minute
(d1 - d2 )(k1 d2 + k2 d1 ) (2) 2 minutes
(2) 2(k1 + k 2 )
(3) 4 minutes
k1k2 (d1 + d2 ) (4) 16 minutes
(3) d k + d k
1 2 2 1 127. In natural convection, a heated portion of a liquid
(4) None of these moves because :
(1) Its molecular motion becomes aligned
124. The three rods shown in figure have identical
dimensions. Heat flows from the hot end at a rate (2) Of moleuclar collisions within it
of 40 W in the arrangement (a). Find the rates of (3) Its density is less than that of the surrounding
heat flow when th e rods are jo ined as in fluid
arrangement (b). (Assume KAl = 200 W/m °C and
(4) Of currents of the surrounding fluid
KCu = 400 W/m °C)
128. It is hotter at the same distance over the top of a
0°C Al Cu Al 100°C fire than it is in the side of it, mainly because
(a) (1) Air conducts heat upwards
Al (2) Heat is radiated upwards
0°C Cu 100°C (3) Convection takes more heat upwards
Al
(4) Convection, conduction and radiation all
(b)
contribute significantly transferring heat upward
(1) 75 W (2) 200 W (3) 400 W (4) 4 W 129. The power radiated by a black body is P and it
125. Two rods A and B of different materials are welded radiates maximum energy at wavelength l0. If the
together as shown in figure. Their thermal temperature of the black body is now changed so
conductivities are K 1 and K 2 . The thermal that it radiates maximum energy at wavelength
conductivity of the composite rod will be :-
3
l 0 , the power radiated by it becomes nP. The
5
T1 T2 value of n is :-

3 4
(1) (2)
d 5 3

3(K1 + K 2 )
(1) (2) K1 + K2 625 81
2 (3) (4)
81 625
K1 + K 2
(3) 2 (K1 + K2) (4)
2
198 E
Pre-Medical : Physics
130. Two stars appear to be red and blue, what is true 135. Three objects coloured black, gray and white can
about them - withstand hostile conditions upto 2800°C. These
(1) The red star is nearer objects are thrown into a furnace where each of
(2) The blue star is nearer them attains a temperature of 2000°C. Which
object will gloss brightest :-
(3) The temperature of red star is more
(1) The white object
(4) The temperature of blue star is more
(2) The black object
131. The um – T curve for a perfect black body is – (3) All glow with equal brightness
(um ® frequency corresponding to maximum (4) Gray object
emission of radiation)
136. A piece of iron is heated in a flame. It first becomes
dull red then becomes reddish yellow and finally turns
to white hot. The correct explanation for the above
D
B observation is possible by using :-
C
um (1) Newton's Law of cooling
(2) Stefan's Law
A
(3) Wein's displacement Law
T
(4) Kirchoff's Law
137. The rectangular surface of area 8cm × 4 cm of a
(1) A (2) B (3) C (4) D black body at a temperature of 127°C emits energy
132. The spectral emissive power El for a body at at the rate of E per second. If the length and breadth
temperature T1 is plotted against the wavelength of the surface are each reduced to half of the initial
and area under the curve is found to be A. At a value and the temperature is raised to 527°C, the
different temperature T2 the area is found to be rate of emission of energy will become.
625A. Then l1/l2 =
81 9
(1) 4E (2) E (3) E (4) 16 E
16 16
138. A black body, at a temperature of 227°C, radiates
heat at a rate of 7 cal cm–2 s–1. At a temperature
of 727°C, the rate of heat radiated in the same
units will be :–
(1) 80 (2) 60 (3) 50 (4) 112
(1) 5 (2) 1/5 (3) 1 5 (4) 5 139. A spherical black body with a radius of 10 cm
radiates 810 watt power at 900 K. If the radius were
133. The temperature of a furnace is 23240C and the
halved and the temperature doubled, the power
intensity is maximum in its radiation spectrum nearly
radiated in watt would be :-
at 12000 A0. If the intensity in the spectrum of a
star is maximum nearly at 4800 A0, then the sur- (1) 450 (2) 1000
face temperature of star is (3) 3240 (4) 225
(1) 8400 C
0
(2) 7200 C 0 140. Cooling rate of a sphere of 600 K at external
(3) 6219.5 C 0
(4) 59000C environment (200 K) is R. When the temperature
of sphere is reduced to 400 K then cooling rate of
134. Two stars A and B of surface area Sa and Sb & the sphere becomes :
temperature T a an d T b glow red an d blue
respectively. Choose the correct option. 3 16
(1) R (2) R
16 3
(1) Ta > Tb (2) Ta < Tb

(3) TaSa = TbSb (4) TaSb = TbSa 9


(3) R (4) None
27
E 199
Pre-Medical : Physics
141. A solid cube and sphere are made of same 145. A liquid in a beaker has temperature q at time t
substance and both have same surface area. If the and q0 is temperature of surroundings, then
temperature of both bodies 1200 C then : according to Newton's law of cooling, correct graph
between loge(q – q0) and t is:
(1) Both will loss of Heat by same rate

loge (q – q0)
loge (q – q0)
(2) Rate of loss of Heat of cube will be more than
that of the sphere
(3) Rate of loss of Heat of the sphere will be more (1) (2)
than that of the cube t t
(4) Rate of loss of Heat will be more for that whose
mass is more

loge (q – q0)
loge (q – q0)
142. Two spheres of radii in the ratio 1 : 2 and densities
in the ratio 2 : 1 and of same specific heat, are (3) (4)
heated to same temperature and left in the same 0
0 t
t
surrounding. Their rate of falling temperature will
be in the ratio : 146. A body cools down from 80°C to 60°C in 10 minutes
when the temperature of surroundings is 30°C. The
(1) 2 :1 (2) 1 : 1
temperature of the body after next 10 minutes will
(3) 1 : 2 (4) 1 : 4 be :-
(1) 30°C (2) 48°C (3) 50°C (4) 52°C
143. Newton's law of cooling is used in laboratory for the
determination of the 147. A body cool from 90°C to 70°C in 5 minutes if
(1) Specific heat of the gases temperature of surrounding is 20°C find the time
(2) The latent heat of gases taken by body to cool from 60°C to 30°C. Assuming
(3) Specific heat of liquids Newton's law of cooling is valid.
(4) Latent heat of liquids (1) 10 min (2) 12 min (3) 18 min (4) 5 min

144. If a piece of metal is heated to temperature q and 148. A body cools from a temperature 3T to 2T in
then allowed to cool in a room which is at 15 min. The room temperature is T. Assume that
temperature q 0 , the graph between the Newton's law of cooling is applicable. The
temperature T of the metal and time t will be closest
temperature of the body at the end of next
to
15 min will be :-

4 7 3
(1) T (2) T (3) T (4) T
3 4 2

(1) (2) 149. The total radiant energy per unit area, normal to
the direction of incidence, received at a distance R
from the centre of a star of radius r, whose outer
surface radiates as a black body at a temperature
T Kelvin is given by :-

4psr2 T 4 sr 2 T 4
(3) (4) (1) (2)
R2 R2

sr2 T 4 sr 4 T 4
(3) (4)
4 pr 2 r4
(Where s is Stefan's Constant)
200 E
Pre-Medical : Physics
150. If el and al be the emissive power and absorption 151. If E is the total energy emitted by a body at a
power respectively of a body and El be the emissive temperature T K and Emax is the maximum energy
power of an ideal black body, then from Kirchhoff's emitted by it at the same temperature, then -
laws (1) E µ T4; Emax µ T5 (2) E µ T4; Emax µ T–5
(1) al = El / el (2) al / el = El (3) E µ T–4; Emax µ T4 (4) E µ T5; Emax µ T4
(3) el / al = El (4) el = El / al

ANSWER KEY
Que. 1 2 3 4 5 6 7 8 9 10 11 12 13 14 15
Ans. 3 1 2 3 4 2 1 1 4 1 1 3 3 1 1
Que. 16 17 18 19 20 21 22 23 24 25 26 27 28 29 30
Ans. 3 3 1 3 4 3 4 1 4 1 4 4 3 4 3
Que. 31 32 33 34 35 36 37 38 39 40 41 42 43 44 45
Ans. 3 4 2 1 4 1 1 3 3 2 2 1 1 2 4
Que. 46 47 48 49 50 51 52 53 54 55 56 57 58 59 60
Ans. 1 3 4 2 3 4 1 4 2 1 3 4 2 3 4
Que. 61 62 63 64 65 66 67 68 69 70 71 72 73 74 75
Ans. 1 1 3 2 1 1 3 2 1 2 1 1 3 4 1
Que. 76 77 78 79 80 81 82 83 84 85 86 87 88 89 90
Ans. 4 2 4 3 4 1 4 2 4 1 2 1 3 1 1
Que. 91 92 93 94 95 96 97 98 99 100 101 102 103 104 105
Ans. 1 2 1 2 2 3 2 1 1 2 2 1 2 2 2
Que. 106 107 108 109 110 111 112 113 114 115 116 117 118 119 120
Ans. 3 3 2 3 2 3 4 1 2 2 2 2 4 1 2
Que. 121 122 123 124 125 126 127 128 129 130 131 132 133 134 135
Ans. 1 1 3 3 4 1 3 3 3 4 2 1 3 2 2
Que. 136 137 138 139 140 141 142 143 144 145 146 147 148 149 150
Ans. 3 1 4 3 1 1 2 3 2 3 2 3 4 2 3
Que. 151
Ans. 1
E 201
Pre-Medical : Physics

OSCILLATIONS
(SHM, DAMPED AND FORCED OSCILLATIONS & RESONANCE)
1. PERIODIC MOTION AND ITS CHARACTERISTICS AND TYPES OF SHM
1.1 Periodic Motion
(i) Any motion which repeats itself after regular interval of time is called periodic motion or harmonic
motion.
(ii) The constant interval of time after which the motion is repeated is called time period.
Examples : (i) Motion of planets around the sun.
(ii) Motion of the pendulum of wall clock.
1.2 Oscillatory Motion
(i) The motion of a body is said to be oscillatory or vibratory motion if it moves back and forth (to and fro)
about a fixed point after regular interval of time.
(ii) The fixed point about which the body oscillates is called mean position or equilibrium position.
Examples : (i) Vibration of the wire of 'Sitar'.
(ii) Oscillation of the mass suspended from spring.
1.3 Simple Harmonic Motion
l In SHM particle does to and fro motion about a fixed point called mean position.
l The force acting on the particle which tends to bring the particle towards its mean position, is known as
restoring force.
l This force is always directed towards the mean position and proportional to displacement from mean
postion.
l SHM motion can be represented by
x = A sinwt,
x = A coswt,
x = A sinwt ± B coswt,
x = A sin2wt,
x = A cos2(wt + f)
Amplitude
The maximum displacement of particle from its mean position is define as amplitude.
Time period (T), Frequency (n) and Angular frequency (w)
l The time after which the particle keeps on repeating its motion is known as time period.
1
l It is given by T = 2p , T = where w is angular frequency and n is frequency.
w n
Phase (SHM as a uniform circular motion) Y constant
w angular
(a) Projection of particle's position on Y-axis. velocity
N t
y = A sin(wt + f)
y A
The quantity (wt + f) represents the phase angle at that instant. t=0
wt f
(b) The phase angle at time t = 0 is known as initial phase or X' O x X
epoch.
(c) The difference of total phase angles of two particles executing
S.H.M. with respect to the mean position is known as phase
Y'
difference.
(d) If the phase angles of two particles executing S.H.M. are (wt + f1) and (wt + f2) respectively, then the
phase difference between two particles is given by
Df = (wt + f2 ) – (wt + f1) or Df = f2 – f1

202 E
Pre-Medical : Physics
1.4 Types of Simple harmonic motion (S.H.M.)
(i) S.H.M. are of two types
(a) Linear S.H.M.
When a particle moves to and fro about a point
(called equilibrium position) along a straight line then m
its motion is called linear simple harmonic motion.
Example : Motion of a mass connected to spring.
(b) Angular S.H.M.
When a system oscillates angularly with respect to a axis then
its motion is called angular simple harmonic motion.
Example :- Motion of a bob of simple pendulum.

(ii) Comparison between linear and angular S.H.M.

Linear S.H.M. Angular S.H.M.

F µ–x t µ–q
F= –kx t = – Cq
Where k is the restoring force constant Where C is the restoring torque constant
Where x is disp. from mean position Where q is angular disp. from mean position.

k C
a =- x a =- q
m I
whee m is the mass of body.

d2 x k d2q C
+ x=0 + q=0
dt 2 m dt2 I

It is known as differential equation of It is known as differential equation of


linear S.H.M. angular S.H.M.
x = A sinwt considering initial phase = 0 q = q0 sin wt considering initial phase zero.
a = – w2x a=–wq 2

where w is the angular frequency

k C
w2 = w2 =
m I

k 2p C 2p
w= = = 2pn w= = = 2pn
m T I T

where T is time period and n is frequency

m I
T = 2p T = 2p
k C

1 k 1 C
n= n=
2p m 2p I

This concept is valid for all types of linear This concept is valid for all types of angular
S.H.M. S.H.M.

E 203
Pre-Medical : Physics
2. SIMPLE HARMONIC MOTION (SHM) AND ITS EQUATION; VELOCITY,
ACCELERATION
2.1 Displacement, Velocity and Acceleration in S.H.M.
Displacement in S.H.M.
(i) The displacement of a particle executing linear S.H.M. at any instant is defined as the position of the
particle from the mean position at that instant.
(ii) It can be given by relation x = A sinwt or x = A coswt or x = A sin(wt + f)
Ex. What will be the equation of displacement in the following different conditions ?
(i) –A +A (ii) –A +A

(iii) –A +A (iv) –A +A

p
Sol. (i) x = A sinwt (ii) x = A sin(wt + ) Þ x = A coswt
2
3p
(iii) x = A sin(wt + p) Þ x = –A sinwt (iv) x = A sin(wt + ) Þ x = –A coswt
2
2.2 Velocity in S.H.M. v a vmax v a
(i) Velocity in S.H.M. is given by
amax amax
dx d
v= = (A sin wt)
dt dt
x=-A v a M.P. v a x=+A
r v = Aw coswt r 2
v = ±w (A - x ) 2
vmax = Aw

v2 x2 x2 v2 v
OR 2 2
=1- 2 r 2
+ 2 2 =1
w A A A A w
wA
This is equation of ellipse. So curve between displacement and velocity
of particle executing S.H.M. is ellipse. x
(ii) The graph between velocity and displacement is shown in figure.
If particle oscillates with unit angular frequency (w = 1) then curve A
between v and x will be circle.
2.3 Acceleration in S.H.M.
a
dv d w2A
(i) Acceleration in S.H.M. is given by a = = (Aw cos wt)
dt dt
a = –w A sin wt
2
r a=–wx 2
+A
r amax = –w2A –A x
(ii) The graph between acceleration and displacement is a straight line
as shown in figure. –w2A
Important points :
• In linear S.H.M., the length of S.H.M. path = 2A
• In S.H.M., the total work done and displacement in one complete oscillation is zero but total travelled length is 4A.
p
• Velocity is always ahead of displacement by phase angle radian
2
• Acceleration is ahead of displacement by phase angle p radian i.e., opposite to displacement.
p
• Acceleration leads the velocity by phase angle radian.
2
204 E
Pre-Medical : Physics
3. ENERGY IN SHM – POTENTIAL & KINETIC ENERGIES
3.1 Potential Energy (U or P.E.)
(i) In terms of displacement
dU 1 2
The potential energy is related to conservative force by the relation F = -
dx
Þ ò dU = -ò Fdx ÞU=
2
kx + U0
1 2
Where the potenital energy at equilibrium position = U0 If U0 = 0 then U = kx
2
(ii) In terms of time
KE
1 1 m w2A2
Since x = A sin(wt + f), U = kA2 sin2(wt +f) Umax=
2 2
PE
3.2 Kinetic Energy (K) 1 mw2A2
KEmax=
–A 2
+A
(i) In terms of displacement
If mass of the particle which is executing S.H.M. is m and Its velocity is v then kinetic energy at any instant.

1 1 1
K= mv2 = mw2 (A2 – x2) = k(A2 – x2)
2 2 2

(ii) In terms of time where ET represents total energy


K,U

Q v = Aw cos(wt + f) Kmaxor Umax or ET

1 K.E.
\K= mw2A2 cos2 (wt + f)
2
P.E.
3.3 Total energy (E)
t
Total energy in S.H.M. is given by ; E = potential energy + kinetic energy = U + K
1 2 1 1
w.r.t. position E = kx + k (A2 – x2) Þ E= kA2 = constant
2 2 2
3.4 Average energy in S.H.M.

1 1 1 1
(a) <KE>t = mw2 A 2 = kA 2 , <KE >x = mw2 A 2 = kA2
4 4 3 3

1 1 1 1 2
(b) <PE>t = mw 2 A 2 + U0 = kA 2 + U0 , <PE>x = mw 2 A 2 + U0 = kA + U0
4 4 6 6

• The frequency of oscillation of potential energy and kinetic energy is twice as that of displacement or
velocity or acceleration of a particle executing S.H.M.
• Frequency of total energy is zero because it remains constant.
4. OSCILLATIONS OF A SPRING BLOCK SYSTEM
4.1 Spring Block System
(i) When a small mass is suspended from a spring then this arrangement is known as spring block system.
For small linear displacement the motion of spring block system is simple harmonic.
(ii) For a spring block system
k
2p m
Time period T = = 2p ,
w k
1 k m
Frequency n =
2p m

E 205
Pre-Medical : Physics
(iii) Time period of a spring block system is independent of acceleration due to gravity. This is why a clock based on
oscillation of spring block system show proper time everywhere on a hill or moon or in a satellite or different
places of earth, where gravity is varying.

(iv) If a spring block system oscillates in a vertical plane is made to oscillate on a horizontal k
surface or on an inclined plane then time period will remain unchanged.
mm1

m
(v) If two masses m1 and m2 are connected by a spring and made to oscillate then time period T = 2p
k
k
m1m2 m1 m2
Here, m = m + m = reduced mass of a system.
1 2

(vi) If the stretch in a vertically loaded spring is y 0 then for equilibrium of mass m.
m y0
ky0 = mg i.e., = k
k g ky0

y0 m
m y0
So, time period T = 2p = 2p m mg
k g

But remember time period of spring pendulum is independent of acceleration due to gravity (y0 will change with
changing value of g).
4.2 Various Spring Arrangements
• Series combination of springs

In series combination same restoring force exerts in all springs but extension will be different.
If equivalent force constant is ks then F = – ksx
1 1 1 k1 k 2 k1 k2
Where = + Þ ks = m
k s k1 k 2 k1 + k 2

m
Time period Ts = 2p
ks

• Parallel Combination of springs

k1
m
k1 k2
k1 k2 m

m k2

In parallel combination, displacement on each spring is same but restoring force is different.
If equivalent force constant is kP, then F = – kPx, so kP = k1 + k2

m m
Time period TP = 2p = 2p
kP k1 + k 2

206 E
Pre-Medical : Physics
5. SIMPLE PENDULUM
If a heavy point mass is suspended by a weightless, q

inextensible and perfectly flexible string from a rigid Effective l T


length
support, then this arrangement is called a simple pendulum

l y

mg
T=2p s in
q

co s
g CM mg

q
Important points : mg

1. The time period of pendulum is independent from mass of the bob but it depends on size of bob (position
of centre of mass). So in pendulum when a solid iron bob is replaced by light aluminium bob of same
radius then time period remains unchanged.

æ 1 ö
2. If simple pendulum is shifted to poles, equator or hilly areas, then its time period may be different ç T µ ÷
ç g ÷ø
è
3. If a clock based on oscillation of simple pendulum is shifted from earth to moon then it becomes slow
g 1
because its time period increases and becomes 6 times compare to earth. gM = 6 Þ TM = 6TE
E

4. Periodic time of simple pendulum in reference (system) frames.


v
l
T = 2p
geff
geff = g
(a) If reference system is lift
(i) If velocity of lift v = constant

acceleration a = 0 and geff. = g \


l
T = 2p a
g
(ii) If lift is moving upwards with acceleration a
geff = g+a
geff. = g + a
l
T = 2p Þ T decreases
g+a
a
(iii) If lift is moving downwards with acceleration a
geff. = g – a
l geff = g–a
\ T = 2p Þ T increases
g-a
(iv) If lift falls downwards freely
geff. = g – g = 0 Þ T=¥ simple pendulum will not oscillate
(b) A simple pendulum is mounted on a moving truck
l
(i) If truck is moving with constant velocity, time period remains same T = 2p
g
(ii) If truck accelerates forward with acceleration 'a'.

l l
So effective acceleration, geff. = g2 + a 2 and T' =2p a
g eff.
g2 + a 2 g
l a
Time period T' = 2p Þ T' decreases
g2 + a 2
E 207
Pre-Medical : Physics
6. If a simple pendulum of density s is made to oscillate in a liquid of density r then its time period will
increase as compare to that of air and is given by

æ rö
gnet = g ç 1 - ÷ Fb
è s ø
v
l s r
T = 2p
é rù Fg
ê1 - s ú g
ë û

7.(a) If the bob of simple pendulum has positive charge q and pendulum
is placed in uniform electric field which is in downward direction. +q
E m
l
T = 2p
qE mg + qE
g+
m
(b) If the bob of simple pendulum has positive charge q and is made is
oscillate in uniform electric field acting in upward direction.
E +q
m
l
T = 2p mg – qE
qE
g-
m

l
8. T = 2p is valid when length of simple pendulum (l) is negligible as compare to radius of
g
earth (l << R) but if l is comparable to radius of earth

1
then time period T = 2p
é1 1 ù
êl + R úg
ë û
The time period of oscillation of simple pendulum of infinite length

R 1
T = 2p ~ 84.6 minute » 1 hour It is maximum time period.
g 2
9. Second's pendulum
If the time period of a simple pendulum is 2 second then it is called second's pendulum. Second's pendulum
take one second to go from one extreme position to other extreme position.

l
For second's pendulum, time period T = 2 = 2p
g
At the surface of earth g = 9.8 m/s2 » p2 m/s2,
So length of second pendulum at the surface of earth l » 1 m and at surface of moon l » 1/6 m
10. If simple pendulum is shifted to the centre of earth, freely falling lift or in artificial satellite then it will not
oscillate and its time period is infinite (Q geff. = 0).
11. Variation in Time Period with Temperature

l 0 (1 + aDt)
T = 2p where Dt is charge in temp.
8

DT 1
or = µ DT where DT increase in time period
T0 2

208 E
Pre-Medical : Physics
If time period of clock based on simple pendulum increases then clock will be slow and if time period
decreases then clock will be fast.
6. DIFFERENT TYPES OF OSCILLATIONS
Different types of oscillations
Free, Damped, Forced oscillations and Resonance
(a) Free oscillation
(i) The oscillations of a particle with fundamental frequency under the influence of restoring force are
defined as free oscillations.
(ii) The amplitude, frequency and energy of oscillations remain constant.

xm

O
–xm

(b) Damped oscillations


(i) In these oscillations the amplitude of
A – gt
oscillations decreases exponentially x = x me
due to damping forces like frictional
force, viscous force etc.
(ii) If initial amplitude is A0 then amplitude t
O
after time t will be A = A 0e- gt where

g = Damping coefficient
For example A0 is initial amplitude
t0 A0 t0 A
A0 ¾¾® ¾¾® 20 and so on.
n n
(c) Forced oscillations
(i) The oscillations in which a body oscillates under the influence of an external periodic force (driver)
are known as forced oscillations.
(ii) The driven body does not oscillate with its natural frequency rather it oscillates with the frequency
of the driver.
(iii) The amplitude of forced vibration is determined by the difference between the frequency of the
applied force and the natural frequency. If the difference between frequencies is small then the
amplitude will be large.
(d) Resonance
(i) When the frequency of external force (driver) w is equal to the natural frequency w0 of the oscillator
(driven), then this state of the driver and the driven is known as the state of resonance.
(ii) In the state of resonance, there occurs maximum transfer of energy from the driver to the driven.
(iii) When w = w0, amplitude of oscillations is maximum. This state is resonance. Energy of the oscillations
is also maximum in this state. Amplitude vs w graph is as shown.
Amplitude

w0 w
E 209
Pre-Medical : Physics
7. EXAMPLES OF SIMPLE HARMONIC MOTION
PHYSICAL SYSTEM THAT MOVES WITH SHM
(1) Torsional oscillator (Angular SHM)
I hpr 4
T = 2p , C=
C 2l
wire h = modulas of elasticity of the wire
C disk r = radius of the wire
l = length of the wire
I
I = MI of the disc

(2) Oscillation liquid in a V-shape tube : l


T = 2p g(cos q + cos q )
1 2

l = Total length of liquid column in tubes


l q1 & q2 = angles of tubes with horizontal
q2 q1 (for U-shaped tube q1 = q2 = 90°)
g = gravitational acceleration

m Ld h
T = 2p Arg , T = 2p rg , T = 2p g
(3) Floating block : h L
m = mass of block, A = Area of block
r = density of liquid, L = length of cylinder
d = density of cylinder, h = length of cylinder inside the
liquid at mean position

(4) Oscillation of piston in a gas chamber piston:


Vm
Ball T = 2p
A 2K
x v = volume of cylinder, m = mass of piston
piston
A = area of cylinder ball
DP
Gas x K = bulk modulus =
D
- V
gas V
For (1) Isothernal process K=P, (2) adiabatic process K = gP
cylinder

(5) Longitudinal oscillation of an elastic wire lm


T = 2p
or rubber string : AY
l = length of string
wire of m = mass of ball
ll
rubber string A = Area of cross section
Y = young's modulus
m
Dl

(6) Tunnel across earth : Re


T = 2p g , Re = 6400 km, T = 84.6 minute

Time taken to go from one end of the tunnel to other


end is T/2 i.e. 42.3 minutes

210 E
Pre-Medical : Physics
Illustration An object performs S.H.M. of amplitude 5 cm and time period 4 s. If timing is started when the
object is at the centre of the oscillation i.e., x = 0 then calculate –

(i) Frequency of oscillation

(ii) The displacement at 0.5 sec.

(iii) The maximum acceleration of the object.

(iv) The velocity at a displacement of 3 cm.

1 1
Solution (i) Frequency f = = = 0.25 Hz
T 4

(ii) The displacement equation of object x = Asinwt

p 5
so at t = 0.5 s x = 5sin(2p × 0.25 × 0.5) = 5 sin = cm
4 2

(iii) Maximum acceleration amax = w2A = (0.5 p )2 × 5 = 12.3 cm/s2

(iv) Velocity at x = 3 cm is v = ±w A2 - x2 = ±0.5p 52 - 32 = ±6.28 cm/s

Illustration Amplitude of a harmonic oscillator is A, when velocity of particle is half of maximum velocity, then
determine position of particle.

v max Aw
Solution v = w A 2 - x2 but v= =
2 2

Aw
= w A 2 - x2 r A 2 = 4[A 2 - x 2 ]
2

4A 2 - A 2 3A
r x2 = r x =±
4 2

Illustration A particle performing SHM is found at its equilibrium position at t = 1 sec and it is found to have
a speed of 0.25 m/s at t = 2 sec. If the period of oscillation is 8 sec. Calculate the amplitude of
oscillations.

Solution x = Asin(wt + f)

at t = 1 sec. particle at mean position

æ 2p ö p
0 = A sin ç ´1 + f÷ Þ f=-
è 8 ø 4

at t = 2 sec. velocity of particle is 0.25 m/s

æp pö
0.25 = Aw cos ç ´ 2 - ÷
è4 4ø

Aw 2
0.25 = Þ A=
2 p

E 211
Pre-Medical : Physics
é pù
Illustration If two S.H.M. are represented by equation s y1 = 10 sin ê3pt + ú and
ë 4û
y 2 =5 ésin(3pt) + 3 cos(3pt) ù then find the ratio of their amplitudes and phase difference in between
ë û
them.
é1 3 ù
Solution As y 2 = 5 éësin(3pt) + 3 cos(3pt) ùû = 10 ê sin(3pt) + cos(3pt) ú
ë2 2 û
é p p ù æ pö A1 10
Þ 10 êcos sin 3pt + sin cos 3 pt ú = 10 sin ç 3 pt + ÷ Þ =
ë 3 3 û è 3ø A2 10
p p p
Þ A1 : A2 = 1 : 1 and Phase difference = - = rad.
3 4 12
Illustration The velocity of a particle in S.H.M. at position x1 and x2 are v1 and v2 respectively. Determine
value of time period and amplitude.
Solution v = w A 2 - x2 r v2 = w2 (A2 – x2)
At position x1 velocity v12 = w2 (A2 – x12 ) ... (i)
At position x2 velocity v 22 = w2 (A2 – x22 ) ... (ii)

v12 - v 22
Subtracting (ii) from (i) v12 - v 22 = w2 (x22 - x12 ) r w=
x 22 - x12

2p x 22 - x12
Time period T= r T = 2p
w v 12 - v 22

v12 A 2 - x12
Dividing (i) by (ii) = r v 12 A2 - v12 x22 = v 22 A2 - v 22 x12
v 22 A 2 - x 22

v12 x22 - v 22 x12


So A (v - v ) = v x - v x r
2 2 2 2 2 2 2 A=
1 2 1 2 2 1 v12 - v 22
Illustration In case of simple harmonic motion –
(a) What fraction of total energy is kinetic and what fraction is potential when displacement is
one half of the amplitude.
(b) At what displacement the kinetic and potential energies are equal.
Solution In S.H.M.
1 1 2 1
KE = k(A2 – x2) PE = kx and TE = kA2
2 2 2

KE A 2 - x 2 PE x 2
(a) fKE = = fPE = =
TE A2 TE A 2
A A2 - A 2 4 3 A2 4 1
at x = fKE = = and fPE = =
2 A2 4 A2 4
1 1 A
(b) KE = PE r k (A2 – x2) = kx2 r 2x2 = A2 r x= ±
2 2 2
Illustration A particle starts oscillating simple harmonically from its equilibrium position with time period T.
T
Determine ratio of K.E. and P.E. of the particle at time t = .
12

T 2p T p A
Solution at t = x = A sin ´ = A sin =
12 T 12 6 2

1 3 1 2 1 2 1 1 2 K.E. 3
so K.E. = k (A2 – x2) = ´ kA and P.E. = kx = ´ kA \ =
2 4 2 2 4 2 P.E. 1
212 E
Pre-Medical : Physics
Illustration The potential energy of a particle oscillating on x-axis is U = 20 + (x – 2)2. Here U is in joules
and x in meters. Total mechanical energy of the particle is 36 J.
(a) State whether the motion of the particle is simple harmonic or not ?
(b) Find the mean position.
(c) Find the maximum kinetic energy of the particle.
dU
Solution (a) F=- = –2(x – 2) By assuming x – 2 = X, we have F = –2X
dx
Since, F µ - X The motion of the particle is simple harmonic
(b) The mean position of the particle is X = 0 Þ x – 2 = 0, which gives x = 2m
(c) Maximum kinetic energy of the particle is, Kmax = E – Umin = 36 – 20 = 16 J
Note : Umin is 20 J at mean position or at x = 2m.
Illustration A body of mass m attached to a spring which is oscillating with time period 4 seconds. If the mass
of the body is increased by 4 kg, its time period increases by 2 sec. Determine value of initial mass m.

m m m+4
Solution In Ist case : T = 2p Þ 4 = 2p ...(i) and in IInd case: 6 = 2p ...(ii)
k k k

4 m 16 m
Divide (i) by (ii) = Þ = Þ m = 3.2 kg
6 m+4 36 m + 4
Illustration One body is suspended from a spring of length l, spring constant k and has time period T. Now if
spring is divided in two equal parts which are joined in parallel and the same body is suspended from
this arrangement then determine new time period.
Solution Spring constant in parallel combination k' = 2k + 2k = 4k

m m m 1 T T
\ T' = 2p = 2p = 2p ´ = =
k' 4k k 4 4 2
Illustration A block of mass m is attached from a spring of spring constant k and dropped from its natural length.
Find the amplitude of S.H.M.
Solution Let amplitude of S.H.M. be A then by work energy theorem W = DKE

1 2 2mg
mgx0 – kx0 = 0 Þ x 0 =
2 k
x=0, v=0 m
mg
So amplitude A = x=x 0, v=0 m
k
Illustration Periodic time of oscillation T1 is obtained when a mass is suspended from a spring. If another spring
is used with same mass then periodic time of oscillation is T 2. Now if this mass is suspended from series
combination of above springs then calculate the time period.
m 4 p2 m m 4p2 m 4p2m
Solution T1 = 2p Þ k1 = and T2 = 2p Þ k2 =
k1 2
T1 k2 T22 so K eq. = 2
Teq.

k1 k2 k1

m m
k2

m
2
1 1 1 Teq. T2 T2
In series combination = + Þ 2
= 21 + 22 Þ Teq. = T12 + T22
K eq. K1 K 2 4p m 4p m 4p m

E 213
Pre-Medical : Physics
Illustration Infinite springs with force constants k, 2k, 4k, 8k, ..... respectively are connected in series. Calculate
the effective force constant of the spring.
1 1 1 1 1
Solution = + + + + .............¥
k eff k 2k 4k 8k
a
(For infinite G.P. S¥ = where a = First term, r = common ratio)
1-r
é ù
1 1é 1 1 1 ù 1ê 1 ú 2
= ê1 + + + + ...........ú = ê ú = so keff = k/2
k eff k ë 2 4 8 û k ê1 - ú 1 k
ë 2û
Illustration A simple pendulum is suspended from the ceiling of a lift. When the lift is at rest, its time period is
T. With what acceleration should lift be accelerated upwards in order to reduce its time period to
T
?
2

l T l
Solution In stationary lift T = 2p ...(i) In accelerated lift = T ' = 2p ...(ii)
g 2 g+a

g+a
Þ 2= Þ g + a = 4g Þ a = 3g
g
13. If length of a simple pendulum is increased by 4%. Then determine percentage change in time
period.

l DT 1 Dl
Solution T = 2p Þ T µ l1/ 2 Þ =
g T 2 l

DT 1 Dl
Percentage change in time period ´ 100% = ´ 100 [Q Dg = 0]
T 2 l
Dl DT 1
According to question ´ 100 = 4% \ ´ 100% = × 4% = 2%
l T 2
Illustration A bob of simple pendulum is suspended by a metalic wire. If a is the coefficient of linear expansion
and dq is the change in temperature then prove that percentage change in time period is 50adq.
Solution With change in temperature dq, the effective length of wire becomes l' = l (1 + a dq)

l' l T' l'


T ' = 2p and T = 2p Hence = = (1 + a dq) 1/2 =
g g T l

1
1+ adq
2

é T '- T ù éT' ù
\ Percentage increase in time period = ê
T ú ´ 100 = ê - 1ú ´ 100
ë û ë T û

é adq ù
= ê1 + - 1ú ´ 100 = 50 a dq
ë 2 û
Illustration The amplitude of a damped oscillator becomes half in one minute. The amplitude after
1
3 minutes will be times of the original. Determine the value of x.
x
Solution Amplitude of damped oscillation is A = A0e–gt [from x = xme–g t]
A0 A0
at t = 1 min A= so = A 0e - g Þ eg = 2
2 2
A0 A0
After 3 minutes A= so = A 0 e - g´ 3 Þ x = e3g = (eg)3 = 23 = 8
x x
214 E
Pre-Medical : Physics
OSCILLATIONS EXERCISE
1. A particle of mass m is executing S.H.M. If amplitude 6. A particle executes SHM of type x = asinwt. It takes
is a and frequency n, the value of its force constant a a
time t1 from x = 0 to x = and t2 from x = to
will be : 2 2
x = a. The ratio of t1 : t2 will be :
(1) mn2 (2) 4mn2a2 (3) ma2 (4) 4p2mn2
(1) 1 : 1 (2) 1 : 2 (3) 1 : 3 (4) 2 : 1
2. The equation of motion of a particle executing
S.H.M. where letters have usual meaning is : 7. The period of a particle is 8s. At t = 0 it is at the
mean position. The ratio of distance covered by the
d2 x k d2 x particle in first second and second will be-
(1) =– x (2) = +w2x
dt2 m dt2
2 -1 1
(1) (2)
2 2
d2 x d2 x
(3) 2 = –w2x2 (4) 2 = –kmx
dt dt 1
(3) (4) éë 2 - 1ùû
2 -1
3. The equation of motion of a particle executing 8. A particle is executing SHM with time period T.
simple harmonic motion is a+16p2x = 0. In this Starting from mean position, time taken by it to
equation, a is the linear acceleration in m/s2 of the 5
complete oscillations, is :-
particle at a displacement x in metre. Find the time 8
period. T T 5T 7T
(1) (2) (3) (4)
(1) 0.50 (2) 0.15 12 6 12 12
(3) 0.155 (4) 0.25 9. Two bodies performing S.H.M. have same amplitude
and frequency. Their phases at a certain instant
4. Out of the following functions representing motion are as shown in the figure. The phase difference
of a particle which represents SHM : between them is

(A) y = sinwt – cos wt (B) y = sin3wt 0.5 A


(-x) + (+x)
0
æ 3p ö A A
(C) y = 3 cos ç - 5wt ÷ (D) y = 1 + wt + w2t2
è 4 ø

(-x) + (+x)
(1) Only (A) 0.5 A 0

(2) Only (D) does not represent SHM


11 p 3
(1) p (2) p (3) (4) p
(3) Only (A) and (C) 6 3 5
(4) Only (A) and (B) 10. The velocity-time diagram of a harmonic oscillator
is shown in the adjoining figure. The frequency of
5. The phase of a particle in SHM at time t is p/6. oscillation is :
The following inference is drawn from this:
(1) The particle is at x = a/2 and moving in
V(m/sec)
+ X-direction +4
+2
(2) The particle is at x = a/2 and moving in 0.02 0.04
0
– X-direction 0.01 0.03 t (in sec.)
–2
(3) The particle is at x = –a/2 and moving in –4
+ X-direction
(4) The particle is at x = –a/2 and moving in (1) 25 Hz (2) 50 Hz
– X-direction (3) 12.25 Hz (4) 33.3 Hz

E 215
Pre-Medical : Physics
11. The plot of velocity (v) versus displacement (x) of 16. Two identical pendulums oscillate with a constant
a particle executing simple harmonic motion is
p
shown in figure. The time period of oscillation of phase difference and same amplitude. If the
4
particle is :-
maximum velocity of one is v, the maximum velocity
v(m/s) of the other will be
0.4
v
(1) v (2) 2v (3) 2v (4)
x(cm)
2
–10 0 10
17. The acceleration of a particle in SHM at 5 cm from
–0.4 its mean position is 20 cm/sec2. The value of angu-
lar velocity in radian/second will be :
(1) 2 (2) 4 (3) 10 (4) 14
p 18. If the displacement, velocity and acceleration of a
(1) s (2) p s (3) 2p s (4) 3p s
2
particle in SHM are 1 cm, 1cm/sec, 1cm/sec 2
12. A particle is executing S.H.M. of frequency respectively its time period will be (in seconds) :
300 Hz and with amplitude 0.1 cm. Its maximum (1) p (2) 0.5p (3) 2p (4) 1.5p
velocity will be :
19. The variation of acceleration (a) and displacement
(1) 60p cm/s (2) 0.6p cm/s (x) of the particle executing SHM is indicated by
(3) 0.50p cm/s (4) 0.05p cm/s the following curve :
13. Average velocity of a particle performing SHM in a a
one time period is :-

Aw (1) x (2) x
(1) Zero (2)
2

Aw 2Aw a a
(3) (4)
2p p

14. A particle performing S.H.M. is found at its (3) x (4) x


equilibrium at t = 1 s and it is found to have a speed
of 0.25 m/s at t = 2 s. If the period of oscillation
20. A body oscillates with SHM according to the
is 6s. Calculate amplitude of oscillation
equation x = 5.0 cos(2pt + p). At time t = 1.5 s,
3 3 it s di splacement, spee d an d acceleration
(1) m (2) m respectively is :
2p 4p
(1) 0, –10p, + 20p2 (2) 5, 0, –20p2
6 3 (3) 2.5, +20p, 0 (4) –5.0, +5p, –10p2
(3) m (4) m
p 8p 21. Two simple Harmonic Motions of angular frequency
15. Two simple harmonic motions are represented by 500 and 5000 rads–1 have the same displacement
amplitude. The ratio of their maximum accelerations
æ pö is:-
the equations y 1 = 0.1 sin çè 100pt + ÷ø and
3 (1) 1 : 103 (2) 1 : 104 (3) 1 : 10 (4) 1: 102
y2 = 0.1 cos100pt. The phase difference of the 22. A particle is executing a simple harmonic motion.
velocity of particle 1, with respect to the velocity Its maximum acceleration is a and maximum velocity
of particle 2 is- is b. Then its time period of vibration will be :-

–p p –p p a
(1) (2) (3) (4) 2pb b2 b2
6 3 3 6 (1) (2) (3) (4)
a a2 b a

216 E
Pre-Medical : Physics
23. A particle executes linear simple harmonic motion 28. The total energy of a harmonic oscillator of mass
with an amplitude of 5 cm. When the particle is at 2kg is 9 joules. If its potential energy at mean position
3 cm from the mean position, the magnitude of its is 5 joules, its K.E. at the mean position will be :
velocity is equal to that of its acceleration. Then its
(1) 9J (2) 14J (3) 4J (4) 11J
time period in seconds is :-
29. The particle executing simple harmonic motion has
2 3p 2p 3
(1) (2) (3) (4) a kinetic energy Ko cos2 wt. The maximum values
3p 2 3 2p of the potential energy and the total energy are
respectively :-
24. A body executes S.H.M. with an amplitude A. At (1) Ko and Ko (2) 0 and 2Ko
what displacement from the mean position, is the
Ko
potential energy of the body one-fourth of its total (3) and Ko (4) Ko and 2Ko
2
energy?
30. If <E> and <V> denotes the average kinetic and
A
(1) average potential energies respectively of mass
4
describing a simple harmonic motion over one
A
(2) period then the correct relation is:
2
(1) <E> = <V> (2) <E> = 2<V>
3A
(3) (3) <E> = –2<V> (4) <E> = – <V>
4
(4) Some other fraction of A 31. The potential energy of a simple harmonic oscillator
at mean position is 3 joules. If its mean K.E. is
25. A particle of mass 4 kg moves simple harmonically
4 joules, its total energy will be :
such that its PE (U) varies with position x, as shown.
The period of oscillations is :- (1) 7J (2) 8J (3) 10J (4) 11J

32. Simple pendulum of large length is made equal to the


U(x)joule radius of the earth. Its period of oscillation will be :
(1) 84.6 min. (2) 59.8 min.
1.0 (3) 42.3 min. (4) 21.15 min.

33. A lift is ascending with acceleration g/3. What will


O 0.2
be the time period of a simple pendulum suspended
x(m)® from its ceiling if its time period in stationary lift is
T?
2p p 2 T 3T 3T T
(1) s (2) s (1) (2) (3) (4)
25 5 2 2 4 4

4p 2p 2 34. A simple pendulum performs simple harmonic


(3) s (4) s
5 5 motion about x = 0 with an amplitude a and time
period T. The speed of the pend ulum at
26. The force acting on a 4gm mass in the energy region x = a/2 will be :-
U = 8x2 at x = –2cm is :
pa 3 pa 3 pa 3p 2 a
(1) 8 dyne (2) 4 dyne (1) (2) (3) (4)
T 2T T T
(3) 16 dyne (4) 32 dyne
35. The period of oscillation of simple pendulum of
27. A particle describes SHM in a straight line about O.
length L suspended from the roof of the vehicle
O P
which moves without friction, down on an inclined
If the time period of the motion is T then its kinetic plane of inclination a, is given by :-
energy at P be half of its peak value at O, if the time
L L
taken by the particle to travel from O to P is (1) 2p (2) 2p
g cos a g sin a
1 1 1 1
(1) T (2) T (3) T (4) T
2 4 2 2 8 L L
(3) 2p (4) 2p
g g tana
E 217
Pre-Medical : Physics
36. The time period of oscillations of a simple pendulum 42. As shown in the figure, two light springs of force
is 1 minute. If its length is increased by 44%, then constant K1 and K2 oscillate a block of mass M. Its
its new time period of oscillation will be :- effective force constant will be :
(1) 96 s (2) 58 s (3) 82 s (4) 72 s
37. Two pendulums of length 1.21 m and 1.0 m start
vibrating. At some instant, the two are in the mean
K1 K2
position in same phase. After how many vibrations
M
of the longer pendulum, the two will be in phase?
(1) 10 (2) 11 (3) 20 (4) 21
38. A pendulum is hung from the roof of a sufficiently
(1) K1K2 (2) K1 + K2
high building and is moving freely to and fro like a
simple harmonic oscillator. The acceleration of the 1 1 K 1K 2
(3) + (4)
bob of the pendulum is 16 m/s2 at a distance of 4 K1 K 2 K1 + K 2
m from the mean position. The time period of
43. Two springs of force constant k and 2k are
oscillation is :-
connected to a mass as shown below. The frequency
(1) 2p s (2) p s (3) 2 s (4) 1 s of oscillation of the mass is :
39. Some springs are combined in series and parallel
arrangement as shown in the figure and a mass M
is suspended from them. The ratio of their
frequencies will be :

K K K
1 k 1 2k
(1) (2)
2p m 2p m
K
K
1 3k 1 m
M (3) (4)
M 2p m 2p k

44. In an artificial satellite, the use of a pendulum watch


(1) 1 : 1 (2) 2 : 1 (3)
3 : 2 (4) 4 : 1 is discarded, because :
40. Two particles A and B of equal masses are (1) The satellite is in a constant state of motion
suspended from two massless springs of spring
(2) The effective value of g becomes zero in the
constants k1 and k2, respectively. If the maximum
velocities during oscillations are equal, the ratio of artificial satellite
amplitudes of A and B is- (3) The periodic time of the pendulum watch is re-
duced
(1) k1 / k 2 (2) k1/k2
(4) None of these
(3) k 2 / k1 (4) k2/k1
45. A body of mass m is attached to the lower end of
41. A block of mass m is suspended separately by two a spring whose upper end is fixed. The spring has
different springs have time period t1 and t2. If same
negligible mass. When the mass m is slightly pulled
mass is connected to parallel combination of both
down and released, it oscillates with a time period
springs, then its time period is given by :-
of 3s. When the mass m is increased by 2 kg, the
t1 t 2 t1 t2 time period of oscillations becomes 5 s. The value
(1) t + t (2) of m in kg is :-
1 2 t12 + t 22

8 9 9 16
t1 t 2 (1) (2) (3) (4)
9 8 16 9
(3) t1 + t 2 (4) t1 + t2

218 E
Pre-Medical : Physics
46. The amplitude of a SHM reduces to 1/3 in first 20 49. A block is resting on a piston which executes simple
second then in first 40 second its amplitude becomes: harmonic motion with a period 2.0 s. The maximum
velocity of the piston, at an amplitude just sufficient
1 1 1 1 for the block to separate from the piston is :- (g =
(1) (2) (3) (4)
3 9 27 3 10 m/s2)
(1) 1.57 ms–1 (2) 3.14 ms–1
47. Amplitude of vibrations remains constant in case of (3) 1 ms –1 (4) 6.42 ms–1
(i) free vibrations
50. A simple pendulum has time period T 1. The point
(ii) damped vibrations
of suspension is now moved upward according to
(iii) maintained vibrations the relation y = Kt2, (K = 1 m/s2) where y is the
(iv) forced vibrations vertical displacement. The time period now
(1) i, iii, iv (2) ii, iii
T12
(3) i, ii, iii (4) ii, iv becomes T2. The ratio of is : (g = 10 m/s2)
T22
48. In the following four : 6 5 4
(i) Time period of revolution of a satellite just (1) (2) (3) 1 (4)
5 6 5
above the earth’s surface (Tst)
(ii) Time period of oscillation of ball inside the
tunnel bored along the diameter of the earth
(Tma)
(iii) Time period of simple pendulum having a
length equal to the earth’s radius in a uniform
field of 9.8 newton/kg (Tsp)
(iv) Time period of an infinite simple pendulum in
the earth’s gravitational field (Tis)
Which of the following is true
(1) Tst > Tma (2) Tma > Tst
(3) Tsp > Tis (4) Tst = Tma = Tsp = Tis

ANSWER KEY
Que. 1 2 3 4 5 6 7 8 9 10 11 12 13 14 15
Ans. 4 1 1 3 1 2 3 4 3 1 1 1 1 1 1
Que. 16 17 18 19 20 21 22 23 24 25 26 27 28 29 30
Ans. 1 1 3 1 2 4 1 2 2 4 4 4 3 1 1
Que. 31 32 33 34 35 36 37 38 39 40 41 42 43 44 45
Ans. 4 2 2 1 1 4 1 2 3 3 2 4 3 2 2
Que. 46 47 48 49 50
Ans. 2 1 4 2 1

E 219
Pre-Medical : Physics
WAVE MOTION & DOPPLER'S EFFECT

WAVES & ITS CHARACTERISTIC


• A wave is a disturbances that propagate in space, transports energy and momentum from one point to another
without the transport of matter.
• In wave motion, the disturbance travels through the medium due to repeated periodic oscillations of the
particles of the medium about their mean positions.
• Each particle receives disturbance a little later than its preceding particle i.e., there is a regular phase
difference between one particle and the next.
• The velocity with which a wave travels is different from the velocity of the particles with which they oscillate
about their mean positions.
CLASSIFICATION OF WAVES

Oscillation

(EMW)
or Non-Mechanical Waves

• In the propagation of mechanical waves elasticity and density of the medium plays an important role therefore
mechanical waves are also known as elastic waves.
• A mechanical wave will be transverse or longitudinal depending on the nature of medium and mode of
excitation.
• In strings, mechanical waves are always transverse when string is under a tension.
• In the bulk of gases and liquids mechanical waves are always longitudinal e.g. sound waves in air or water. This
is because fluids cannot sustain shear.
• In solids, mechanical waves (may be sound) can be either transverse or longitudinal depending on the mode of
excitation. The speed of the two waves in the same solid are different. (Longitudinal waves travels faster than
transverse waves).
TRANSVERSE WAVE MOTION
• Mechanical transverse waves are produced Vw (Wave velocity)
in such type of medium which have shearing
property, so they are known as shear wave
or S-wave. Particle
· Particles of the medium vibrate at right angles
to the direction of wave motion
· Particle velocity is always perpendicular to wave velocity
· Can be polarized e.g. String waves, Waves on surface of solid or liquid
• A crest is a portion of the medium, which is raised temporarily above the normal position of rest of particles
of the medium , when a transverse wave passes.
• A trough is a portion of the medium, which is depressed temporarily below the normal position of rest of
particles of the medium , when a transverse wave passes.

220 E
Pre-Medical : Physics
LONGITUDINAL WAVES MOTION
In this type of waves, oscillatory motion of the medium Wave
particles produces regions of compression (high pressure)

and rarefaction (low pressure) which propagated in space


with time (see figure).

· Particles of the medium vibrate in the direction of wave motion

· Particle velocity is parallel or antiparallel to wave velocity


· Can not be polarized e.g. Sound waves, Waves in gases
PLANE PROGRESSIVE WAVES
• Equation of progressive wave in positive x direction -

æ x ö é 2p 2p ù
y(x, t) = A sin (wt – kx) = A sin w ç t - V ÷ = A sin ê t - x
è Wø ëT l úû

• Equation of progressive wave in negative x direction -

æ x ö é 2p 2p ù
y(x, t) = A sin (wt + kx) = A sin w çè t + V ÷ø = A sin ê t + x
W ëT l úû

Here y(x, t) = Displacement of medium particle A = amplitude


wt = time dependent phase, kx = position dependent phase f = Initial phase
k = propagation constant T = time period f = frequency
l = wavelength VW = wave speed 1/l = wave number

¶2 y 1 ¶2 y
• Differential equation : ¶x2 = V 2 ¶t2
w

w l
• Wave velocity VW = = = fl
k T

¶y æ ¶y ö
• Particle velocity Vp = = Aw cos ( wt - kx ) Vp = - VW ´ slope = - VW ç ÷
¶t è ¶x ø

¶2 y
• Particle acceleration : ap = = -w2 A sin ( wt - kx ) = -w2 y
¶t2
y
1 • For particle 1 : vp ¯ and ap ¯
2
x • For particle 2 : vp ­ and ap ¯
4
3
• For particle 3 : vp ­ and ap ­

• For particle 4 : vp ¯ and ap ­

• Relation between phase difference, path difference & time difference

p 2p
0 Df Dx Dt
T/2 T = =
2p l T
l

E 221
Pre-Medical : Physics
• Speed of transverse wave on string :

T Tl T
v= = = , where m = mass/length, T = tension in the string, r = density, r = radius of wire,
m M rpr 2

M = mass of string. Expression derived for the velocity of wave always given velocity w.r.t. medium not
w.r.t. ground.
1 2 2
• Energy density : U = [Average total energy / volume] = rw A
2

æ1 ö
• Power : P = ç rw2 A 2 VW (S)÷ [where S = Area of cross-section]
è 2 ø

Power 1
• Intensity : I = = rw2 A 2 VW
Area of cross-section 2
WAVEFRONT
• An imaginary surface on which waves incident perpendicular & in same phase.

Wave front Plane Spherical Cylindrical

Source Source at infinite distance Point source Linear source


Sun, Torch (Bulb, small siren) (tubelight)
Area of A = l ×b A = 4pr2 A = 2prl
wavefront A = const. A µ r2 A µr
1 1 1
Iµ µ a2 I = constant Iµ µ a2 Iµ µ a2
Area r2 r
1 1
Amplitude a = constant aµ aµ
r r
variation

INTERFERENCE
When two coherent waves of same frequency propagate in same direction and superimpose on one another
then the intensity of resultant wave becomes maximum at some points and at some points it becomes minimum.
This phenomena of intensity variation w.r.t. position is known as interference.
Mathematical Analysis : At a time t, at point x two waves of equal frequency
y1 = A1sin(wt + f1), y2 = A2sin(wt + f2)
Resultant wave : A1

A
A = A12 + A 22 + 2A 1A2 cos( Df)

as I µ A 2
Df
A2
I = I1 + I 2 + 2 I1I2 cos( Df)

where Df = Phase difference = k(x 2–x1) = kDx


Dx = path difference
A = Resultant amplitiude
222 E
Pre-Medical : Physics
• For constructive interference [Maximum intensity]
Df = 2np or path difference = nl where n = 0, 1, 2, 3, ....

( )
2
I max = I1 + I 2 ; A max = A1 + A 2; I max µ (A 1 + A 2 )2

• For destructive interference [Minimum Intensity]


l
Df = (2n+1)p or path difference = (2n+1) where n = 0, 1, 2, 3, ....
2

( );A
2
I min = I1 - I2 min = A1 : A 2; I min µ (A1 - A 2 ) 2

Imax + Imin
• Average intensity of interference pattern Iavg = = I1 + I2
2

æ Imax – Imin ö
• Degree of Interference pattern (f) = ç I ÷ ´ 100%
è max + I min ø
REFLECTION AND REFRACTION (TRANSMISSION) OF WAVES
Rarer Medium : A medium in which speed of wave is greater.
Incident v1 v2
Denser Medium : A medium in which speed of wave is smaller. wave

yi=Aisin(wt-k1x)
• The frequency of the wave remain unchanged. Ai transmitted
æv -v ö i wave
• Amplitude of reflected wave A r = ç 2 1 ÷ A i Medium-1
t
Medium-2
è v1 + v2 ø Ði = Ðr r
Ar
At
• Amplitude of transmitted wave A t = æç 2v 2 ö÷ A i
è v1 + v 2 ø
Reflected
• Wave is moving from rarer to denser medium. wave
v1
v2
Incident
wave Reflection from denser medium
v2 gives phase change of p
Þ v2 < v1
v1
Reflected wave Transmitted wave

• Wave is moving from denser to rarer medium

v1 v2
Incident wave
Reflection from rarer medium gives no
v1 v2 phase change. The transmitted wave is
Þ v2 > v1
always in phase with incident wave.
Reflected wave Transmitted wave

• Reflection from fixed end :

v
Fixed

Fixed

Þ
Phase change of p
Incident v

• Reflection from free end :


frictionless

v v
ring

Þ No phase change

E 223
Pre-Medical : Physics
STANDING WAVES
• When two waves of same frequency and amplitude travel in opposite direction at same speed, their superposition
gives rise to a new type of wave, called stationary waves or standing waves.

B¢ D¢
A, B, C, D & E are nodes.
A
C¢ E A¢, B¢, C ¢ & D¢ are antinodes.
A' B C D

• Formation of standing wave is possible only in bounded medium.


• Let two waves are y1 = A sin ( wt - kx ) ; y 2 = A sin ( wt + kx )
r r r
By principle of superposition y = y1 + y 2 = 2A cos kx sin wt ¬ Equation of stationary wave

• Its amplitude is not constant but varies periodically with position.


l 3l 5l
• Nodes ® amplitude is minimum : cos kx = 0 Þ x = , , ,......
4 4 4

• Antinodes ® amplitude is maximum : cos kx = 1 Þ x = 0, l , l, 3l ,......


2 2
l
· Distance between consecutive nodes = distance between consecutive antinodes = .
2
l
· Distance between adjacent node and antinodes = .
4
• The nodes divide the medium into segments (loops). All the particles in a segment vibrate in same phase
but in opposite phase with the particles in the adjacent segment.
• As nodes are permanently at rest, so no energy can be transmitted across them, i.e. energy of one region
(segment) is confined in that region.
TRANSVERSE STATIONARY WAVES IN STRETCHED STRING
• Fixed at both ends [fixed end ®Node & free end®Antinode]
Fundamental or v
f=
first harmonic 2l
l
l=
2
All Harmonics are present
second harmonic 2v
f= (n+1)V
first overtone 2l n th overtone =
l=l 2l
third harmonic 3v (n)V
f= n th harmonic =
second overtone 2l 2l
3l
l=
2
fourth harmonic 4v
f=
third overtone 2l
l=2l
• Fixed at one end
v
Fundamental f=
4l
l
l=
4
third harmonic 3v
f= Only Odd Harmonics are present
first overtone 4l
3l (2n+1)V
l= n th overtone =
4
4l
fifth harmonic 5v (n)V
f= n th harmonic =
second overtone 4l 4l
5l
l=
4
seventh harmonic 7v
f=
third overtone 4l
7l
l=
224 4 E
Pre-Medical : Physics
Sonometer : In this case, transverse stationary waves are formed.

T = Mg
(tension in wire)
l
l
Mg

n T
• The wire vibrates in n loops, then f =
2l m

1 1
• fµ T (Law of tension) fµ (Law of length) fµ (Law of mass)
l m
• The point where string is plucked it is antinode
• The point where string is touched it is node.
• If arm of tuning fork is filed, then its frequency increases.
• If arm of tuning fork is loaded with wax, then its frequency decreases.
SOUND WAVES
• Displacement and pressure wave
A sound wave can be described either in terms of the longitudinal displacement suffered by the particles of
the medium (called displacement wave) or in terms of the excess pressure generated due to compression
and rarefaction (called pressure wave).
Displacement wave y = Asin(wt–kx)
Pressure wave p = p0cos(wt–kx)
where p0 = ABk = rAvw
A = displacement of amplitude, B = Bulk modulus, k = propagation constant,
p0 = Amplitude of pressure wave, p = Excess pressure in sound wave
• As sound-sensors (e.g., ear or mike) detect pressure changes, description of sound as pressure wave is preferred
over displacement wave.
• The pressure wave is 90° out of phase w.r.t. displacement wave, i.e. displacement will be maximum when
pressure is minimum and vice-versa.
INTENSITY OF SOUND

p20
• Intensity in terms of pressure amplitude I =
2rv

æ Iö
• Sound level in dB = 10 log10 ç ÷
è I0 ø
• Where I0 = threshold intensity of human ear = 10–12 W/m2
CHARACTERISTICS OF SOUND
• Loudness ® Sensation received by the ear due to intensity of sound.
• Pitch ® Sensation received by the ear due to frequency of sound.
• Quality (or Timbre)® Sensation received by the ear due to waveform of sound.
SPEED OF SOUND IN DIFFERENT MEDIA

Y
(1) For solid medium : Vsolid = , where Y = Young's modulus and r = Density of medium
r

B
(2) For liquid : VLiquid = , where B = Bulk modulus or Modulus of elasticity
r

E 225
Pre-Medical : Physics
(3) For gaseous medium : Propagation of sound in a gas is an adiabatic process.

B gP gRT
v gas = = = where B = gP = Adiabatic elasticity of gas
r r Mw

MW = Molecular weight or molar mass


e.g. soft iron vsolid = 5150 m/sec > For water vLiquid = 1450m/s > For air vgas = 330 m/s
(4) Factors effecting the speed of sound
Where
1
é t ù 2 v t = velocity of wave at t°c
• Effect of temperature v µ T Þ vt = v0 ê1 +
273 ú v 0 = velocity of wave at 0°c
ë û
• Effect of pressure : No effect (if temperature is constant)

1
• Effect of moisture r moist air < r dry air , v µ , [velocity in moist air is greater than velocity in dry air].
r

• Effect of speed of medium – If the direction of sound wave (v) and air medium is same than (v + w) and in case
of opposite directions (v – w).
• Effect of frequency – There is no effect of frequency on the speed of sound, known as non dispersiveness.

g
• Relation between speed of sound and R.M.S. value of gas particle v Sound = v rms
3
VIBRATIONS OF ORGAN PIPES
Closed end ® displacement node, pressure antinode
Open end® displacement antinode, pressure node
• Closed end organ pipe

• Only odd harmonics are present

l v 3l 3v 5l 5v v
l= Þf= l= Þf= l= Þf= • Frequency of mth overtone = (2m + 1)
4 4l 4 4l 4 4l 4l

v
Fundamental Ist overtone 2nd overtone • Frequency of mth harmonic = ( m )
4l
Mode 3rd harmonic 5th harmonic
• Open end organ pipe

• All harmonics are present

l v 2v 3l 3v v
l= Þf= l= lÞf= l= Þf= • Frequency of mth overtone = ( m + 1)
2 2l 2l 2 2l 2l

mv
Fundamental Ist overtone 2nd overtone • Frequency of mth harmonic =
2l
Mode 2nd harmonic 3rd harmonic
• End correction :
Due to finite momentum of air molecules in organ pipes reflection takes place not exactly at open end but
some what above it, so antinode is not formed exactly at free end but slightly above it.

In closed organ pipe f0 = v where e = 0.6 R (R=radius of organ pipe)


4 (l + e )
v
In open organ pipe f0 =
2 ( l + 2e )
226 E
Pre-Medical : Physics
INTERFERENCE IN TIME : BEATS
When two sound waves of same amplitude and different frequencies superimpose, then intensity at any point in
space varies periodically with time. This effect is called beats.
If the equation of the two interfering sound waves emitted by s1 and s2 at point O are,
ì æC+Dö æ C - D öü s1
p1 = p0 sin (2pf1 t) ísin C + sin D = 2 sin ç ÷ cos ç ÷ý
î è 2 ø è 2 øþ
p2 = p0 sin (2pf2 t)
O
By principle of superposition
s2
p = p1 + p2 = 2p0 cos {p(f1 – f2)t} sin {p(f1 + f2) t}

y1

(A) Wave of amplitude A and frequency 18

y2

(B) Wave of amplitude A and frequency 21

(C) Resultant wave amplitude 2A and frequency 19.5

æ f1 + f2 ö
• The resultant sound at point O has frequency çè
2 ÷ø
æ f1 - f2 ö
• Pressure amplitude at point O varies with time with a frequency of çè .
2 ÷ø
• Sound intensity will vary with a frequency f1 – f2. This frequency is called beat frequency (fB)
• The time interval between two successive intensity maxima (or minima) is called beat time period (TB)
1
fB = f1 ~ f2 TB =
f1 ~ f2
• The Beat frequency should be less than 10 Hz, for it to be audible.
RESONANCE TUBE

a a
b b
B A
A

l1 l/4

c N
S N l2 3l/4

N
P

Wavelength l=2(l2-l1) l2-3l1


End correction e=
2
E 227
Pre-Medical : Physics
QUINCKE'S TUBE
• Quincke tube is practical method for finding the speed of sound in gaseous medium

Galton's
whishtle
Moveable part
Fixed part Scale

Flame (detector)

• As we slide movable part of tube by l unit, path difference will become 2l so Dx = 2l


l
• If vibration in flame become maximum to minimum or minimum to maximum Dx Þ = 2l
2
• If vibration in flame become minimum to minimum or maximum to maximum then Dx Þ l = 2l
DOPPLER'S EFFECT IN SOUND
• The apparent change in frequency or pitch due to relative motion of source and observer along the line of sight
is called Doppler Effect.

æv±v ±v ö
Observed frequency f ' = ç v ± v m v ÷ f v0 = velocity of observer, vs = velocity of source, vm = velocity of medium
m 0

è m s ø

f' = Apparent frequency & f = frequence of source


• If source and observer approach each other, observed frequency increases.
• If source and observer move away from each other, observed frequency decreases.
SPECIAL CASES
Case-I If medium moves in a direction opposite to the direction of propagation of sound, then

æ v - vm ± vO ö
f' =ç ÷f
è v - vm ± vS ø
Case-II Source in motion towards the observer. Both medium and observer are at rest.

æ v ö
f' = ç ÷f ; Clearly f' > f
è v - vS ø

Case-III Source in motion away from the observer. Both medium and observer are at rest.

æ v ö
f' = ç ÷f ; Clearly f' < f
è v + vS ø
Case-IV Observer in motion towards the source. Both medium and source are at rest.

æ v + vO ö
f' = ç ÷f ; Clearly f' > f
è v ø
Case-V Observer in motion away from the source. Both medium and source are at rest.

æ v - vO ö
f' = ç ÷f ; Clearly f' < f
è v ø

Case-VI Both source and observer are moving away from each other. Medium at rest.

æ v - vO ö
f' = ç ÷f ; Clearly f' < f
è v + vS ø

228 E
Pre-Medical : Physics
Case-VII When source moves towards stationary target.
f'D = direct apparent frequency, f' = apparent frequency at reflector, f'R = reflected apparent frequency

æ v ö
l n 'D = ç v - v ÷ n
è S ø

æ v ö
n' = ç v - v ÷n
è S ø

n 'R = n' Þ Beats = Dn = n 'R – n 'D = 0

æ v ö
l n 'D = çv+v ÷n
è S ø

æ v ö
n' = çv-v ÷n
è S ø

æ v v ö 2v S vn æ 2v ö
Beats = Dn = ç v - v - v + v ÷ n = v2 - v2 @ ç S ÷n
è S S ø S è v ø
DOPPLER'S EFFECT IN LIGHT :
Doppler effect holds also for EM waves. As speed of light is independent of relative motion between source and
observer, the formula are different from that of sound. Here when either source or observer (detector) or both
are in motion, only two cases are possible (approach or recession)

Case I : In case of approach Observer Light Source

O v S

æ 1 + vc ö æ vö ü
Frequency n¢ = ç v ÷
n » ç1 + ÷ n ï
è 1 - c ø
è c ø ï æ vö
ý Blue Shift Dl = l ' - l = - çè ÷ø l
æ 1 - vc ö æ vö ï c
Wavelength l ¢ = ç ÷ l » çè 1 - ÷l ï
è 1 + vc ø cø þ

Case II : In case of recession Observer Light Source

O v S

æ 1 - vc ö æ vö ü
Frequency n¢ = ç v ÷
n » ç1 - ÷n ï
è 1+ c ø è cø ï æ vö
ý Red Shift Dl = l' - l = + çè c ÷ø l
æ 1 + vc ö æ vö ï
Wavelength l ¢ = ç v ÷
l » ç1 + ÷l ï
è 1- c ø è cø þ
Illustration A progressive wave of frequency 500 Hz is travelling with a velocity of 360 m/s. How far apart
are two points 60o out of phase.
v 360
Solution We know that for a wave v = f l So l = = = 0.72 m
f 500
Phase difference Df = 60o = (p/180) x 60 = (p/3) rad,
l 072
. p
so path difference Dl= (Df)= x =0.12 m
2p 2p 3
E 229
Pre-Medical : Physics
Illustration A man generates a symmetrical pulse in a string by moving his hand up and down. At t = 0
the point in his hand moves downward. The pulse travels with speed of 3 m/s on the string &
his hands passes 6 times in each second from the mean position. Then the point on the string
at a distance 3m will reach its upper extreme first time at time t =

6 1 æ1ö
Solution Frequency of wave = = 3 Þ T = s ; l = VT = ( 3) ç ÷ = 1m
2 3 è3ø

3 3T
Total time taken = + = 1.25 sec
3 4

ép pù
Illustration The equation of a wave is, y(x, t) = 0.05 sin ê (10x - 40t) - ú m
ë 2 4 û

Find : (a) The wavelength, the frequency and the wave velocity
(b) The particle velocity and acceleration at x=0.5 m and t=0.05 s.

Solution (a) The equation may be rewritten as, y(x, t) = 0.05 sin æç 5 px - 20pt - p ö÷ m
è 4ø

Comparing this with equation of plane progressive harmonic wave,

2p
y(x, t) = A sin(kx - wt + f) we have, wave number k = = 5 p rad / m \l = 0.4m
l

The angular frequency is, w = 2pf = 20p rad /s \ f = 10Hz

w
The wave velocity is, V =f l = = 4ms-1 in + x direction
k

¶y æ 5p pö
(b) The particle velocity and acceleration are, vp = = -(20p )(0.05) cos ç - p - ÷ = 2.22m/s
¶t è 2 4ø

¶2 y æ 5p pö
ap = 2
= - (20p )2 (0.05) sin ç - p - ÷ =140 m/s2
¶t è 2 4ø

Illustration Calculate the ratio of intensity of wavetrain A to wavetrain B.

IA a 2A n2A
Q I µ a2n2 \ I = 2 2 =
FG 2IJ 2
FG 1IJ 2

Solution
B a B nB H 1K ×
H 2K =1

230 E
Pre-Medical : Physics
Illustration Determine the change in volume of 6 liters of alcohol if the pressure is decreased from 200 cm
of Hg to 75 cm. [velocity of sound in alcohol is 1280 m/s, density of alcohol = 0.81 gm/cc,
density of Hg = 13.6 gm/cc and g = 9.81 m/s 2]

Solution For propagation of sound in liquid v = (B / r) i.e., B = v2r

DP DP V( -DP)
But by definition B = – V So –V = v2r, i.e. DV =
DV DV rv 2
Here DP = H2rg – H1rg = (75 – 200) ´ 13.6 ´ 981 = –1.667 ´ 106 dynes/cm2

So DV =
(6 ´ 10 )(1.667 ´ 10 )
3 6

= 0.75 cc
0.81 ´ (1.280 ´ 10 ) 5 2

Illustration (a) Speed of sound in air is 332 m/s at NTP. What will be the speed of sound in hydrogen at
NTP if the density of hydrogen at NTP is (1/16) that of air.
(b) Calculate the ratio of the speed of sound in neon to that in water vapour at any temperature.
[Molecular weight of neon = 2.02 ´ 10–2 kg/mol and for water vapours = 1.8 ´ 10–2 kg/mol]

E gP g RT
Solution The velocity of sound in air is given by v= r = r = Mw

vH PH rair rair
(a) In terms of density and pressure v = ´ = [as Pair = PH]
air rH Pair rH

rair 16
Þ vH = vair ´ = 332 ´ = 1328 m/s
rH 1

v Ne g Ne M W
(b) In terms of temperature and molecular weight ´
vW = M Ne g W
[as TNe = TW]

Now as neon is mono atomic (g = 5/3) while water vapours poly atomic (g = 4/3) so

v Ne b5 / 3g ´ 18. ´ 10 -2
5 18
´
.
vW = b4 / 3g ´ 2.02 ´ 10 -2 =
4 2.02
= 1.055

Illustration In interference phenomena if the degree of interference pattern in interference is 60% then find
the ratio of intensity & amplitudes of interferring wave form.

I max - I min 60 3 Imax 5 + 3 4


Solution. = = By C & D = =
Imax + Imin 100 5 Imin 5 - 3 1

a1 + a 2 2 a1 2 + 1 3 I1 9
Thus = & = = thus = Ans.
a1 - a 2 1 a2 2 - 1 1 I2 1
Illustration T.F. having n = 300 Hz produces 5 beats/sec. with another T.F. If impurity is added on the arm
of known tuning fork number of beats decreases then find frequency of unknown T.F. ?

Solution. 295 300 305


wax is added
If it would be 305 Hz, beats would have increased but with 295 Hz beats decreases so answer is
295 Hz.

E 231
Pre-Medical : Physics
Illustration A T. F. having n = 158 Hz, produce 3 beats/sec. with another T. F. As we file the arm of
unknown, beats become 7 then find frequency of unknown.
Solution. before filling 158 + 3 so 155 or 161
after filling b=7
155 158 161 165 (after filling)
165 (after filling)
filling filling
Both T.F. give 7 beat/sec. after filling. So answer is both.
Illustration 41 tuning forks are arranged in a series in such a way that each T.F. produce 3 beats with its
neighbouring T.F. If the frequency of last is 3 times of first then find the frequency of 1st 11 th 16th
21st & last T.F.
Solution. n1 = n (let) So n41 = 3n (according to Que.)
n2 = n + b
n3 = n + 2b So n41 = n + 40 × 3
n4 = n + 3b 3n = n + 120
n41= n + 40 b n = 60 Hz
n11= n + 10 b= 90 Hz, n16 = n + 15 b = 105 Hz
n21 = n + 20b = 120 Hz
Illustration Two loudspeakers as shown in fig. below separated by a distance 3 m, are in phase. Assume that
the amplitudes of the sound from the speakers is approximately same at the position of a listener,
Who is at a distance 4.0 m in front of one of the speakers. For what frequencies does the listener
hear minimum signal ? Given that the speed of sound in air is 330 ms–1.

3m

Listener

4.0m

( 3) + (4)
2 2
Solution The distance of the listener from the second speaker = = 25 = 5 m

path difference = (5 – 4.0) m = 1 m

For fully destructive interference 1 m = (2m + l)l/2

Hence l = 2/(2m + 1) m

The corresponding frequencies are given by

n = [330 × (2m + 1)]/2 s–1, for m = 0, 1, 2, 3, 4, ....................

= 165 (2m + 1) s–1, for m = 0, 1, 2, 3, 4, ...........

Therefore the frequencies for which the listener would hear a minimum intensity 165 Hz.
495 Hz, 825 Hz, .............

232 E
Pre-Medical : Physics
Illustration Three tuning forks of frequencies 200, 203 and 207 Hz are sounded together. Find out the beat
frequency.

1 2 FG 3IJ
Solution
3 3 H 3K
200 203 207
1 2 3 FG 4IJ 3 4
Divide 1 second
into 3, 4 or 7
4 4 4 H 4K equal divisions
7

1 2 3 4 5 6 FG 7IJ
7 7 7 7 7 7 H 7K
Eliminate common time instants. Total Maxima in one second 3 + 3 + 6 = 12
Þ 12 beats per second
Illustration Two tuning forks A and B produce 8 beats/s when sounded together. A gas column 37.5 cm long
in a pipe closed at one end resonate to its fundamental mode with fork A whereas a column of length
38.5 cm of the same gas in a similar pipe is required for a similar resonance with fork B. Calculate
the frequency of these two tuning forks. [ AIPMT
2006]

Solution For tuning fork 'A'


l1
= 37.5 so n1 =
v
=
v U|
4 l1 4 ´ 37.5 |V
For tuning fork 'B'
l2
= 38.5 \ n2 =
v
=
v ||
4 l2 4 ´ 38.5 W
v v
\ n1 – n2 = 8 Þ – = 8 \ v = (8 × 4 × 37.5 × 38.5)
4 ´ 37.5 4 ´ 38.5

8 ´ 4 ´ 37.5 ´ 38.5
n1 = = 308 Hz and n2 = 308 – 8 = 300 Hz
4 ´ 37.5
Illustration A transverse wave, travelling along the positive x-axis, given by y=Asin(kx–wt) is superposed with
another wave travelling along the negative x-axis given by y=–Asin(kx+wt). The point x=0 is
Solution At x =0, y1 = Asin (–wt) and y2 = –Asinwt; y1 + y 2 = -2A sin wt (antinode)

Illustration If an OOP (open organ pipe) of fundamental frequency 1400 Hz dipped 30% in water then calculate
produced frequency.

v v
Solution = 1400 Hz Þ = 2800 Hz
2L L

v v v 2800
n' = = = = = 1000 Hz
4L ' 4[0 × 7L] 2× 8L 2×8
Illustration A string with a mass density of 4 × 10–3 kg/m is under tension of 360 N and is fixed at both ends.
One of its resonance frequencies is 375 Hz. The next higher resonance frequency is 450 Hz. Find
the mass of the string. [AIPMT 2007]

p T p +1 T
Solution n1 = 375 = and n2 = 450 = where p is number of loops
2l m 2l m

E 233
Pre-Medical : Physics
450 p + 1
Þ = Þ p = 5
375 p
p T 5 360
so l= = = 2m
2 ´ n1 m 2 ´ 375 4 ´ 10-3
Þ Mass of wire = (m) (l) = (4 × 10–3) (2) = 8 × 10–3 kg

OR

1 T
Difference between two consecutive resonating frequency n2 – n1 =
2l m

1 360 1 360 1 6 ´ 102


Þ 450 – 375 = Þ l= = ´ =2m
2l 4 ´ 10 -3 2 ´ 75 4 ´ 10 -3 150 2
Þ Mass of wire = (m) (l) = (4 × 10–3) (2) = 8 × 10–3 kg
Illustration For given C.O.P. (closed organ pipe) if 9th O.T. (over tone) has frequency 1900 Hz. then fundamental
frequency of same length O.O.P. is ?
Solution 19n = 1900 ; n = 100
OOP = 2n = 200 Hz
For same length OOP have double freq. than COP.
Illustration Two C.O.P. having length 20 cm & 20.5 cm produce 5 beat/sec determine the freg of both
C.O.P.
n 20
Solution = ; n = 200 For 20.5 cm
n+5 20.5
n + 5 = 205 For 20 cm
Illustration Two tuning forks A and B lying on opposite sides of observer ‘O’ and of natural frequency 85 Hz
move with velocity 10 m/s relative to stationary observer O. Fork A moves away from the observer
while the fork B moves towards him. A wind with a speed 10 m/s is blowing in the direction of
motion of fork A. Find the beat frequency measured by the observer in Hz. [Take speed of sound
in air as 340 m/s]
é v sound - v medium ù 33
Solution fobserver for source 'A' = f0 ê ú= f0 ;
ë v sound - v medium + v source û 34
é v sound + v medium ù 35
fobserver for source 'B' = f0 ê ú= f0
ë v sound + v medium - v source û 34
æ 35 - 33 ö
\ Beat frequency = f1 - f2 = çè ÷f =5
34 ø 0

Illustration

If engine of train produce horn at B point then find apparent frequency observed by observer
at A point.

234 E
Pre-Medical : Physics

æ v ö
Solution n' = ç n ; Direction in AB velocity
è v - v s cos q ÷ø

é 5ù 5 3
= vs cosq = ê120 ´ ×cos q= 120 ´ ´ = 20 m/sec.
ë 18 úû 18 5

æ 340 ö
Þ n' = ç ×640 = 680 Hz.
è 340 - 20 ÷ø

Illustration A SONAR system fixed in a submarine operates at a frequency 40.0 kHz. An enemy submarine moves
towards the SONAR with a speed of 360 km h–1. What is the frequency of sound reflected by the
submarine ? Take the speed of sound in water to be 1450 ms–1.
Solution As the sound is observed by enemy subsmarine.
Here observer (enemy submarine) is moving towards the source (SONAR)

æ v + v0 ö æ 1450 + 100 ö 1550


\ Apparent frequency n' = ç ÷n = ç ÷ ´ 40 ´ 103 = ´ 40 ´ 10 3 Hz
è v - v s ø è 1450 ø 1450
After the sound is reflected, enemy submarine acts. as a source of frequency n'. This source moves
with a speed of 100 ms–1 towards the observer (SONAR)
\ Apparent frequency of sound reflected by the enemy submarine

æ v - v0 ö æ 1450 - 0 ö æ 1550 3ö
n" = ç
v - v
÷n' = ç
1450 - 100 ÷ ´ ç 1450 ´ 40 ´ 10 ÷ = 45.93 kHz
è s ø è ø è ø
Illustration Two trains travelling in opposite directions at 126 km/hr each, cross each other while one
of them is whistling. If the frequency of the note is 2.22 kHz find the apparent frequency as
heard by an observer in the other train :
(a) Before the trains cross each other, (b) After the trains have crossed each other.
(vsound = 335 m/sec)
5
Solution Here v1 = 126 × = 35 m/s
18
(i) In this situation v1 v1

Observed freq
æ v + v1 ö
n' = ç ÷´n =
FG 335 + 35IJ ´ 2220 = 2738 Hz
è v - v1 ø H 335 - 35K
(ii) In this situation v1 v1

Observed freq
æ v - v1 ö
n' = ç ÷´n =
FG 335 - 35IJ ´ 2220 = 1800 Hz
è v + v1 ø H 335 + 35K
Illustration A star which is emitting radiation at a wavelength of 5000 Å, is approaching the earth with a velocity
3
of 1·5 × 10 m/s. Calculate the change in wavelength of the radiation as received by the earth.

v . ´ 103
15
Solution Dl = l= ´ 5000 = 0.025Å
c 3 ´ 108

E 235
Pre-Medical : Physics
WAVE MOTION & DOPPLER'S EFFECT EXERCISE
1. Water waves are of the nature : 9. The graph between wave number ( n ) and angular
(1) Transverse frequency (w) is :
(2) Longitudinal
(3) Sometimes longitudinal and some times

frequency (w)

frequency (w)
transverse and longitudinal both

Angular

Angular
(4) Neither transverse nor longitudinal
(1) (2)
2. Sound wave are not polarized because :
(1) Their speed is less Wave no. (–
n) Wave no. (–
n)

(2) A medium is needed for their propagation


(3) These are longitudinal

frequency (w)

frequency (w)
(4) Their speed depands on temperature

Angular

Angular
3. Transverse waves can propagate (3) (4)
(1) only in solids
Wave no. (–
n) Wave no. (–
n)
(2) both in solids and gases
(3) neither in solids nor in gases 10. The figure shows an instantaneous profile of a rope
(4) only in gases carrying a progressive wave moving from left to
4. Transverse elastic waves can be propagate in right, then
(1) Both solid & gas
(2) In solid but not gas y
(3) Neither solid nor gas
(4) None
5. Th e eq uati on o f progressive wave is A x
B
ì æ t xö p ü
Y = 4 sin í p ç - ÷ + ý where x and y are
î è 5 9 ø 6 þ
(a) the phase at A is greater than the phase at B
in cm. Which of the following statement is true ?
(b) the phase at B is greater than the phase at A
(1) l = 18 cm
(2) amplitude=0.04 cm (c) A is moving upwards
(3) velocity v =50 cm/s (d) B is moving upwards
(4) frequency f = 20 Hz (1) a & c (2) a & d (3) b & c (4) b & d
6. The equation y = 4 + 2 sin (6t – 3x) represents a 11. An earthquake generates both transverse (S) and
wave motion with longitudinal (P) sound waves in the earth. The speed
(1) amplitude 6 units of S waves is about 4.5 km/s and that of P waves
(2) amplitude 4 units is about 8.0 km/s. A seismograph records P and
(3) wave speed 2 units S waves from an earthquake. The first P wave
(4) wave speed 1/2 units arrives 4.0 min before the first S wave. The
7. Due to propagation of longitudinal wave in a epicenter of the earthquake is located at a distance
medium, the following quantities also propagate in of about
the same direction :
(1) 25 km (2) 250 km
(1) Energy, Momentum and Mass
(3) 2500 km (4) 5000 km
(2) Energy
12. The equation of a simple harmonic wave is given by
(3) Energy and Mass
(4) Energy and Linear Momentum p
y = 3 sin (50 t – x), where x and y are in metres
8. A wave of frequency 500 Hz travels between X and 2
Y and travel a distance of 600 m in 2 seconds and t is in seconds. The ratio of maximum particle
velocity to the wave velocity is :-
between X and Y. How many wavelength are there
in distance XY : 2 3
(1) 3p (2) p (3) 2p (4) p
(1) 1000 (2) 300 (3) 180 (4) 2000 3 2

236 E
Pre-Medical : Physics
13. A wave in a string has an amplitude of 2cm. The 19. A man standing on a cliff claps his hand and hears
wave travels in the + ve direction of x axis with a its echo after one second. If the sound in reflected
speed of 128 m/s an d it is no ted th at
from another mountain then the distance between
5 complete waves fit in 4 m length of the string.
the man & reflection points is Vsound = 340 m/sec.
The equation describing the wave is :-
(1) y = (0.02) m sin (7.85x – 1005t) (1) 680 m (2) 340 m
(2) y = (0.02) m sin (7.85x + 1005t) (3) 170 m (4) 85 m
(3) y = (0.02) m sin (15.7x – 2010t) 20. If at some point the amplitude of the sound
(4) y = (0.02)m sin (15.7x + 2010t) becomes double and the frequency becomes one
14. The velocities of sound at the same pressure in two fourth then at that point the intensity of sound will
monoatomic gases of densities r1 and r2 are v1 and be :-
r1 v (1) Become double
v2 respectively. If = 4 , then the value of 1
r2 v2 (2) Be half
is :
(3) Become one fourth
1 1
(1) (2) (3) 2 (4) 4 (4) Remain unchanged
4 2
21. What is your observation when two source are
15. A sound is produced in water and moves towards emitting sound with frequency 499 Hz & 501 Hz:
surface of water and some sound moves in air
(1) Frequency of 500 Hz is heard with change in
velocity of sound in water is 1450 m/s and that in intensity takes place twice.
air is 330 m/s. When sound moves from water to
(2) Frequency of 500 Hz is heard with change in
air then the effect on frequency f and wave length
intensity takes place Once.
l will be:
(3) Frequency of 2Hz is heard with change in intensity
(1) f and l will remain same
takes place Once.
(2) f will remain same but l will increase
(4) Frequency of 2Hz is heard with change in intensity
(3) f will remain same but l will decrease
takes place twice.
(4) f will increase and l will decrease
22. 16 tuning forks are arranged in increasing order
16. If um is the velocity of sound in moist air and ud is of frequency. Any two consecutive tuning forks when
the velocity of sound in dry air then : sounded together produce 8 beats per second. If
(1) um < ud (2) um > ud the freqency of last tuning fork is twice that of first
(3) ud >> um (4) um = ud then the frequency of first tuning fork is –
17. The equation of a wave on a string of linear density (1) 60 (2) 80 (3) 100 (4) 120
0.04 kg m–1 is given by
23. Frequency of tuning fork A is 256 Hz. It produces
four beats/sec with tuning fork B. When wax is
é æ t x öù
y = 0.02(m) sin ê2p ç - ÷ú. applied at tuning fork B then 6 beats/sec are
ë è 0.04(s) 0.50(m) ø û
heard. Frequency of B is :
The tension in the string is : (1) 252 (2) 260 Hz
(1) 6.25 N (2) 4.0 N (3) (1) & (2) both (4) 264
(3) 12.5 N (4) 0.5 N 24. Frequency of tuning fork A is 256 Hz. It produces
18. A uniform rope of mass 0.1 kg and length 2.5 m four beats/sec with tuning fork B. When wax is
hangs from ceiling. The speed of transverse wave applied at tuning fork B then 6 beats/s are heard.
in the rope at upper end and at a point 0.5 m By reducing little amount of wax 4 beats/s are
distance from lower end will be : heard. Frequency of B is :
(1) 5 m/s, 2.24 m/s (2) 10 m/s, 3.23 m/s (1) 250 Hz (2) 260 Hz
(3) 7.5 m/s, 1.2 m/s (4) 2.25 m/s, 5 m/s (3) 252 Hz (4) 256 Hz
E 237
Pre-Medical : Physics
25. A tuning fork produces 4 beats/sec. with another 31. An organ pipe closed at one end has fundamental
tuning fork B of frequency 288 Hz. If fork is loaded frequency of 1500 Hz. The maximum number of
overtones generated by this pipe which a normal
with little wax no. of beats per sec decreases. The
person can hear is
frequency of the fork A, before loading is
(1) 14 (2) 13 (3) 6 (4) 9
(1) 290 Hz (2) 288 Hz
32. A wave y = 10 sin (ax + bt) is reflected from a dense
(3) 292 Hz (4) 284 Hz medium at an origin. If 81% of energy is reflected
26. Two sources of sound placed close to each other, then the equation of reflected wave is:
are emitting progressive waves given by (1) y = –8.1 sin (ax - bt) (2) y = 8.1 sin (ax+bt)

y1 = 4 sin 500pt and y2 = 5 sin 508pt (3) y = –9 sin (bt - ax) (4) y = 10 sin (ax - bt)

An observer located near these two sources will 33. If the air column in a pipe which is closed at one
hear :- end, is in resonance with a vibrating tuning fork of
(1) 8 beats per second with intensity ratio 81 : 1 frequency 260 Hz, then the length of the air column
between waxing and waning is :
(2) 4 beats per second with intensity ratio 81 : 1 (1) 35.7 cm (2) 31.7 cm
between waxing and waning (3) 12.5 cm (4) 62.5 cm
(3) 4 beats per second with intensity ratio 25 : 16 34. An open resonating tube has fundamental
between waxing and waning frequency of n. When half of its length is dipped
(4) 8 beats per second with intensity ratio 25 : 16 into water, then its fundamental frequency will be:
between waxing and waning
(1) n (2) n/2 (3) 2n (4) 3/2 n.
27. Two waves of wave length 2 m and 2.02 m
35. For a certain organ pipe three successive resonable
respectively moving with the same velocity
frequencies are observed at 425, 595 and 765 Hz
and superimpose to produce 2 beats per second.
respectively. Taking the speed of sound in air to be
The velocity of the waves is:
340 m/sec (i) whether the pipe is closed end or open
(1) 400.0 m/s (2) 402 m/s
end (ii) determine the length of pipe.
(3) 404 m/s (4) 406 m/s
(1) closed end, 1 m (2) open end, 1 m
28. A tube closed at one end and containing air
(3) closed end, 2m (4) open end, 1 m
produces, when excited, the fundamental note of
frequency 512 Hz. If the tube is open at both 36. A cylindrical tube (L = 120 cm.) is resonant with
ends,the fundamental frequency that can be excited a tuning fork of frequency 330 Hz. If it is filling by
is (in Hz) water then to get resonance minimum length of
(1) 1024 (2) 512 (3) 256 (4) 128 water column is (Vair = 330 m/s)
29. An air column in pipe,which is closed at one end (1) 45 cm. (2) 60 cm.
will be in resonance with a vibrating tuning fork of (3) 25 cm. (4) 20 cm.
frequency 264 Hz if the length of the column in cm 37. The second overtone of an open organ pipe has the
is : [v = 330 m/s] same frequency as the first overtone of a closed pipe
(1) 31.25 (2) 62.50 (3) 110 (4) 125 50 cm long. The length of the open pipe will be
30. A hollow metallic tube of length L and closed at one (1) 25 cm (2) 200 cm
end produce resonance with a tuning fork of (3) 50 cm (4) 100 cm
frequency n . The entire tube is then heated 38. The two nearest harmonics of a tube closed at one
carefully so that at equilbrium temperature its end and open at other end are 460 Hz and
length changes by l . If the change in velocity V of 500 Hz. What is the fundamental frequency of the
sound is v, the resonance will now be produced by system?
tuning fork of frequency.
(1) 20 Hz (2) 30 Hz
(1) (V +v) / [4(L + l)] (2) (V +v) / [4(L - l)]
(3) 40 Hz (4) 10 Hz
(3) (V -v) / [4(L + l)] (4) (V -v) / [4(L - l)]

238 E
Pre-Medical : Physics
39. A tuning fork is used to produce resonance in a glass 44. Two identical piano wires, kept under the same
tension T have a fundamental frequency of
tube. The length of the air column in this tube can
600 Hz. The fractional increase in the tension of
be adjusted by a variable piston. At room one of the wires which will lead to occurrence of
temperature of 27°C two successiv resonances are 6 beats/s when both the wires oscillate together
produced at 40 cm and 93 cm column length. If the would be :-
(1) 0.01 (2) 0.02 (3) 0.03 (4) 0.04
frequency of the tuning fork is 320 Hz, the velocity
45. A source of unknown frequency gives 4 beats/s,
of sound in air at 27°C is :-
when sounded with a source of known frequency
(1) 330 m/s (2) 339 m/s 250 Hz. The second harmonic of the source of
unknown frequency gives five beats per second,
(3) 350 m/s (4) 300 m/s
when sounded with a source of frequency 513 Hz.
40. An air column, closed at one end and open at the The unknown frequency is
other, resonates with a tuning fork when the smallest (1) 260 Hz (2) 254 Hz (3) 246 Hz (4) 240 Hz
length of the column is 40 cm. The next larger length 46. If n1, n2 and n3 are the fundamental frequencies of
of the column resonating with the same tuning fork three segments into which a string is divided, then
is : the original fundamental frequency n of the string
(1) 66.7 cm (2) 80 cm is given by :-

(3) 120 cm (4) 150 cm 1 1 1 1


(1) n = n + n + n
1 2 3
41. A wave of frequency 100 Hz travels along a string
towards its fixed end. When this wave travels back, 1 1 1 1
(2) = + +
after reflection, a node is formed at a distance of n n1 n2 n3
10 cm from the fixed end. The speed of the wave
(3) n = n 1 + n2 + n 3
(incident and reflected) is :
(4) n = n1 + n2 + n3
(1) 5 m/s (2) 10 m/s (3) 20 m/s (4) 40 m/s
47. A string is stretched between two fixed points
42. A second harmonic has to generated in a string of
separated by 75.0 cm. It is observed to have
length l stretched between two rigid supports. The
resonant frequencies of 420 Hz and 315 Hz. There
points where the string has to be plucked and
are no other resonant frequencies between these
touched are –
two. The lowest resonant frequencies for this string
l 3l is :-
(1) Pluck at touch at
2 4 (1) 105 Hz (2) 155 Hz

l l (3) 205 Hz (4) 10.5 Hz


(2) Pluck at touch at 48. The tension in a stretched string fixed at both ends
2 4
is changed by 2%, the fundamental frequency is
l 3l found to get changed by 15 Hz. Select the incorrect
(3) Pluck at touch at
4 4 statement
(1) Wavelength of the string of fundamental
l l
(4) Pluck at touch at frequency does not change
4 2
(2) Velocity of propagation of wave changes by 2%
43. A tuning fork of frequency 512 Hz makes 4 beats (3) Velocity of propagation of wave changes by 1%
per second with the vibrating string of a piano. The
(4) Original frequency is 1500 Hz
beat frequency decreases to 2 beats per seconds
when the tension in the piano string is slightly 49. A string is rigided by two ends and its equation is
increased. The frequency of the piano string before given by y = cos2pt sin2px
increasing the tension was : Then minimum length of string is
(1) 508 Hz (2) 510 Hz (3) 514 Hz (4) 516 Hz (1) 1m (2) 1/2m (3) 5m (4) 2p m

E 239
Pre-Medical : Physics
50. A wave represented by the equation y = a cos 56. A uniform rope of length L and mass m hangs
(wt – kx) is superposed by another wave to form a vertically from a rigid support. A block of mass
2m is attached to the free end of the rope.
stationary wave such that the point x = 0 is a node.
A transverse pulse of wavelength l1 is produced
The equation for other wave is – at the lower end of the rope. The wavelength of
(1) y = a sin (wt + kx) (2) y = – a cos (wt – kx) the pulse when it reaches the top of the rope is
l2 . The ratio l2 /l1 is :
(3) y = – a cos (wt + kx) (4) y = – a sin (wt – kx)
51. A stretched string is 1 m long. Its mass per unit 3
(1) 1 (2)
length is 0.5 g/m. It is stretched with a force of 2

20 N. It plucked at a distance of 25 cm from one (3) 2 (4) 3


end. The frequency of note emitted by it will be: 57. A source and an observer moves away from each
(1) 400 Hz (2) 300 Hz other, with a velocity of 15 m/s with respect to
ground. If observer finds the frequency of sound
(3) 200 Hz (4) 100 Hz
coming from source as 1950 Hz. Then actual
52. Two wires are fixed in a sonometer. Their tensions frequency of source will be
are in the ratio 8:1. The lengths are in the ratio (velocity of sound = 340 m/s) :
36:35. The diameters are in the ratio 4:1. (1) 1785 Hz (2) 1968 Hz
Densities of the materials are in the ratio 1:2. If the (3) 1950 Hz (4) 2130 Hz
higher frequency in the setting is 360 Hz, the beat 58. Sound source of frequnecy 170 Hz is placed near
frequency when the two wires sounded together is: a wall. A man walking from the source towards the
(1) 8 (2) 5 (3) 10 (4) 6 wall finds, that there is periodic rise and fall of sound
53. A sonometer wire resonates with a given tuning fork inte nsity. If the s peed o f sound i n air is
forming standing waves with five antinodes 340 m/s, then the distance separating the two
between the two bridges when a mass of 9kg is adjacent portions of minimum intensity is:
suspended from the wire. When this mass is (1) (1/2) m (2) (3/2) m
replaced by mass M, the wire resonates with the
(3) 1 m (4) 2 m
same tuning fork forming three antinodes for the
59. A whistle revolves in a circle with angular speed
same positions of the bridges. Then find the value
w = 20 rad/sec using a string of length 50 cm. If
of square root of M.
the frequency of sound from the whistle is 385 Hz,
(1) 5 (2) 10 (3) 25 (4) None then what is the minimum frequency heard by an
54. When a guitar string is sounded with a 440 Hz tuning observer which is far away from the centre:
(Vsound = 340 m/s)
fork a beat frequency of 5 Hz is heard. If the
(1) 385 Hz (2) 374 Hz
experiment is repeated with a tuning fork of 436
(3) 394 Hz (4) 333 Hz
Hz the beat frequency is 9 Hz. The string frequency
60. A source of frequency 200 Hz is moving towards
(Hz) is – an observer with a velocity equal to the sound
(1) 445 (2) 435 (3) 429 (4) 448 velocity V. If observer also moves away from the
55. A string of linear mass density 4 g/cm is vibrating source with same velocity then apparent frequency
according to equation :- heard by observer will be :
(1) 50 Hz (2) 160 Hz
æ 4p ö
y = A sin(240pt) cos ç x÷ (3) 150 Hz (4) 200 Hz
è 5 ø
61. A bus is moving with a velocity of 5 m/s towards
where x is in centimeters.
a huge wall. The driver sounds a horn of frequency
Find the tension in the string
165Hz. If the speed of sound in air is 335 m/s, No.
(1) 3.6 N (2) 36 N of beats heared by a passenger on bus will be–
(3) 7.2 N (4) 72 N (1) 6 (2) 5 (3) 3 (4) 4

240 E
Pre-Medical : Physics
62. A body is walking away from a wall towards an 66. Doppler effect for light differs from that for sound
in regards that :
observer at a speed of 1 m/s and blows a whistle
(1) the relative frequency shift is smaller for light
whose frequency is 680 Hz. The number of beats than for sound.
heard by the observer per second is :- (2) the velocity addition valid for sound is not true
(velocity of sound in air = 340 m/s) for light waves.
(3) velocity of light is very large as compared to
(1) 4 (2) 8 (3) 2 (4) zero sound.
63. A siren emitting a sound of frequency 900 Hz moves (4) light waves are electromagnetic waves but
sound waves are mechanical.
away from an observer towards a cliff at a speed
67. An observer moves towards a stationary source of
of 30ms–1. Then, the frequency of sound that the sound with a speed 1/5th of the speed of sound.
observer hears in the echo reflected from the cliff The wavelength and frequency of the source are
l and f respectively. The apparent frequency and
is :
wavelength recorded by the observer are
(Take velocity of sound in air = 330 ms–1) respectively :–
(1) 930 Hz (2) 960 Hz (1) 1.2f, 1.2l (2) 1.2f, l
(3) f, 1.2l (4) 0.8f, 0.8l
(3) 990 Hz (4) 1000 Hz
68. An astronomical object is moving with such a
64. The wavelength of the light received from a galaxy is speed that red shift of 1nm is observed in
0.4% greater than the wave length on the earth then wavelength of 600 nm of wave received from it,
the velocity of galaxy relative to the earth will be: the speed of wave is :-
(1) 1.2x107 m/sec (2) 1.2 x 106 m/sec (1) 5× 105 m/s (2) 4 × 105 m/s
(3) 1.2 x 10 m/sec
5
(4) 1.2 x 104 m/sec (3) 3 × 10 m/s
5
(4) 2 × 105 m/s
65. Doppler effect for sound depends upon the
relative motion of source and listener and it also
depends upon that which one of these is in
motion. Whereas in doppler effect for light it only
depends upon the relative motion of the source of
light and observer. The reason for it is :
(1) Einstein's mass energy relation
(2) Einstein's theory of relativity
(3) Photo electric effect
(4) none of above

ANSWER KEY
Que. 1 2 3 4 5 6 7 8 9 10 11 12 13 14 15
Ans. 3 3 2 2 1 3 4 1 2 2 3 4 1 2 3
Que. 16 17 18 19 20 21 22 23 24 25 26 27 28 29 30
Ans. 2 1 1 3 3 1 4 3 2 3 2 3 1 1 1
Que. 31 32 33 34 35 36 37 38 39 40 41 42 43 44 45
Ans. 3 3 2 1 1 1 4 1 2 3 3 4 1 2 2
Que. 46 47 48 49 50 51 52 53 54 55 56 57 58 59 60
Ans. 1 1 2 2 3 3 3 1 1 1 2 4 3 2 4
Que. 61 62 63 64 65 66 67 68
Ans. 2 1 3 2 2 2 2 1

E 241
Pre-Medical : Physics

IMPORTANT NOTES

242 E
Pre-Medical : Physics
BASIC MATHEMATICS USED IN PHYSICS AND VECTORS

1 1 1
1. 1+ + + ... using G.P.
2 4 8 6 5
11. 1 Þ cos q =
1 q 6
a + ar + ar2... arn where a = 1 & r =
2 5
dy
a 1 12. y = sin(4x – 3) Þ = 4 cos (4x - 3)
sum = = =2 dx
1-r 1 -1/2
13. 3x – 2y + 4 = 0 Þ 2y = 3x + 4
dx ln (2x - 3) 3 4 3
2. ò 2x - 3
=
2
+c y=
2
x+
2
Þ y= +
2
x+2

æ hö
-2
æ 2h ö 3
3. by binomial expainsion ç 1 + ÷ = ç1 - ÷ slope = +m = & c = +2
è R ø è R ø 2
14. y = x2 represent
if h << R
2 3 5
4. 2x2 – 3x + 5 = 0 Þ x - x+ =0
2 2
-b
sum or product of root for ax2 + bx + c = 0 is 15. Continously increasing slope.
a
c
and product of root is B
a D
C
5. ( 0.996)1 / 4 = (1 – 0.004)1/4
also for (1 + x)n and |x| <<< 1 16. v = 4t2 – 2t
(1 + x)n = (1 + nx)
dV
æ 0.004 ö a= = 8t - 2
(1 – 0.004) = ç1 - = 0.999 dt
4 ÷ø
1/4
è
a = represent straight line
cos 4x
6. ò sin 4x dx = -
4
+c

7. f = 2t2 – 3t + 4
tanq=8
df
e=- = –4t + 3 at t=2
dt c=-2

e = –8 + 3= – 5 units 17. for minima

3 dy d2 y
8. sin 480° = sin (540 – 60º) = sin (60º) = = 0 and >0
2 dx dx2
so graph should be
dy
9. y = x sin x Þ = x (cos x) + 1 sin x
dx or

dy
= x cos x + sin x
dx 18. y = 2x – 4x2
y = –4x2 + 2x
ò cos y = x(–4x + 2)
2
10. qdq
y = 0 at x = 0
cos2 q – sin 2 q = cos 2q Þ 2cos2 q – 1 = cos 2q x = 1/2
also –4x2 represent downward parabola
1 + cos2q
cos2 q =
2 1 g 1
20. Q f= \ f2 µ
2p l l
æ 1 + cos 2q ö q sin 2q
ò cos2 q dq = ò ç
è 2 ÷ dq = +
ø 2 4
+c

E 243
Pre-Medical : Physics
1 Solving (i), (ii) and (iii) Þ F1 = 6N, F2 = 10N

22. v av =
ò =òvdt
0
(2t + 3)dt
=
(t 2
+ 3t)10
= 4m / s 33.
r r r
P = Q = R and P + Q = R
ò dt
1
(t)10
ò 0
dt
R Q
1
23. As K= mv 2 ³ 0 so 9–x2 > 0 Þ –3 < x < 3
2 q1 = 60°
q

é3 4 ù P
24. 3 cos q + 4 sin q = 5 ê cos q + sin q ú r r r
ë5 5 û If P = Q = R and P + Q + R = 0

= 5[sin a cos q + cos a sin q] = 5 sin(q + a) Q

-1 æ 3 ö
Therefore A = 5 and a = sin ç ÷
è5ø R
ur ur q2 = 120°
120° P
26. F1 – F2 £ F 1 + F 2 £ F1 + F2

F 3 must lie between F 1 – F 2 £ F 3 £ F 1 + F 2


34. If q and q' are the angles made by resultant
to produce zero resultant.
velocities of first and second ball respectively from
the x-axis, then
29. B R
vy 3
tan q = = = 3 or q = 60°
vx 1
q
A v 'y 2
and tan q' = = = 1 or q' = 45°
R B/ 2 1 v 'x 2
sinq = = = Þ q = 45°
B B 2 Angle between the paths of the balls
f = q – q' = 60° – 45° = 15°
d 35. Let q is the angle between the vectors
30.
\ A2 = A2 + A2 + 2AA cos q
-c
uur uuur uuuur 1
31. Dr = OB - OA which gives cos q = – or q = 120°
2
uur 36. The second's needle gets rotated by 90° in
Dr = l2 + l2 - 2l2 cos q = 2l2 (1 - cos q)
15 seconds,
æqö
2l2 ´ 2sin2 q / 2 = 2l sin çè 2 ÷ø so Dv = v 2 + v 2 - 2vv cos90°

32. F1 + F2 = 16 ...(i) 2p p 2
= 2v = 2wr = 2 ´ ´1 = cm / s
60 30
8= F12 + F22 + 2F1F2 cos q
B sin q
37. tan q / 2 = \A=B
Þ F + F + 2F1F2 cos q = 64
2 2
...(ii) A + B cos q
1 2
39. The forces are drawn along the sides of a square as
R shown in the figure. It is clear from the figure that
F2 3P
R = (2P)2 + (2P)2 = 2 2 P

q a = 90° and it makes angle 4P 2P


2P a
F1
a = 180° + 45°
F2 sin q 2P P
tan a = = tan 90° = ¥
F1 + F2 cos q = 225° with x-axis R

F2cosq = –F1 ...(iii)

244 E
Pre-Medical : Physics
ALLEN
r r r r
40. |V1 + V2 | =|V1 - V2 | 1 1 2 1
cos a = ,cos b = ,cos g = =
2 2 2 2
or V12 + V22 + 2V1 V2 cos q = V12 + V22 - 2V1 V2 cos q
48. If three unit vectors are along the same direction, then
or cos q = 0 \ q = 90º their resultant will be 3. The resultant may be zero
r when each one has angle 120° from other.
41. If A is the required vector, then
49. If A, B and C are the magnitudes of three vectors,
ˆ + (3iˆ + 6ˆj - 7k)
A + (i - 5ˆj + 2k) ˆ = i or ˆj or kˆ then for their resultant to be zero.
r A – B £ C £ A+B
\ A = -3iˆ - ˆj + 5kˆ

42. For parallel vectors 50. The dot product of 5kˆ with 4iˆ + 3j,6i
ˆ ˆ and 3iˆ + 4ˆj

A1 A 2 A 3 is zero, so these are perpendicular vectors to 5kˆ


= =
B1 B2 B3
51. Let eastern line be taken as x-axis, northern as y-axis
r
ˆ BA and vertical upward as z-axis. Let the velocity v makes
43. The required vector is BA =
A angle a, b and g with x, y and z-axis respectively, then
a = 60° , g = 60°
(3iˆ + 4j)
ˆ
= 72 + 242 ´ = 15i + 20j we have cos2 a + cos2 b + cos2 g = 1
32 + 42
240° 1
or cos260 + cos2b + cos260 = 1 or cos b =
44. x 2
r
A \ v = v cos ai + v cos bˆj + v cos gkˆ

A
Ax = A cos 240º = - é1 1 ˆ 1 ˆù
2 = 20 ê ˆi + j + k ú = 10iˆ + 10 2jˆ + 10kˆ
ë2 2 2 û
A
and Ay = A sin 240º = – 3
r
2 52. ( 2i$ + 3j$ + 8k$ ) .( 4j$ - 4iˆ + ak$ ) = 0
45. Let b = (iˆ + ˆj)
–8 + 12 + 8a = 0 Þ 8a = – 4
r
The component of ar along b
a = –1/2
rrö
æ a.b ur uur
a cos q b̂ = çç b ÷÷ b̂ 53. P.Q = 0 Þ ( ai$ + a$j + 3k$ ) . ( ai$ - 2j$ - k$ ) = 0
è ø
a2 – 2a – 3 = 0 Þ (a – 3) (a + 1) = 0 Þ a = 3, –1
(2iˆ + 3j)
ˆ (iˆ + ˆj) (iˆ + ˆj) r r
= 54. A ×B = 0
2 2 2 2
1 +1 1 +1
wt wt
cos wt cos - sin wt sin =0
2 ´ 1 + 3 ´ 1 (iˆ + ˆj) 5 ˆ ˆ 2 2
= = (i + j)
2 2 2 æ wt ö
cos ç wt + = 0 Þ cos 3wt = 0
è 2 ÷ø 2
x - comp 2
46. Angle with y-axis Þ tanq = =
y - comp 3 3wt p p
Þ = Þ t=
2 2 3w
æ2ö
Þ q = tan–1 ç ÷ r r r r
uur
è3ø
55. ( )(
A+B . A-B = 0 )
47. A = $i + $j + 2k$ r r r r
Þ A2 – A . B + B . A – B2 = 0
Ax Ay Az r r r r
cos a = ,cos b = ,cos g = Þ A = B (Q A . B = B . A )
A A A

E 245
Pre-Medical : Physics
ur uur uur ur uur
uur ur 64. R = (A + B
ˆ ) + (A - Bˆ ) Þ R = 2A
A×B 1+1 2 2
56. cos q = = = = ur uur
AB 3× 2 6 3 Thus R and A are in same direction.

1 ˆ ˆ
(3iˆ + 4k).j
sin q = 1 - cos 2 q = 65. Projection = =0
3 ĵ
r r
æ 1 ö
-1 66. If q is the angle between A1 and A 2 , then
Þ q = sin ç ÷
è 3ø
A 2 = A12 + A 22 + 2A1 A 2 cos q
r r
57. A = akˆ , B = bjˆ or 32 = 22 + 32 + 2 × 2 × 3 cosq
r r \ cos q = –1/3
A ´ B = ab(kˆ ´ ˆj) r r r r
Now, (A1 + 2A2 ) (3A1 - 4A 2 )
= ab (–i) = ab (along west) r r r r r r r r
= 3A1.A1 + 6A 2 .A1 - 4A1 .A 2 - 8A2 .A2
r r r r r
58. A ´ B = 0 Þ A||B = 3 × 22 + 2 × 2 × 3 × (–1/3) – 8 × 32 = –64
r r r r r
B ´ C = 0 Þ B||C
r r
thus A and C may be parallel
r r
59. (A ´ B) will be perpendicular to the plane of
r r
A and B
uur ur uur ur
61. A ´ B = 3 A.B Þ AB sinq = 3 AB cosq

Þ tanq = 3 Þ q = 60°

R= A 2 + B2 - 2AB cos 60° = (A + B – AB)


2 2 ½

1 r r
62. Area of parallelogram = d ´ d2
2 1
r r
d1 and d2 are diagonals.

B sin q
63. tan a = = tan 90° = ¥
A + B cos q

thus, A + B cos q = 0 Þ cosq = –A/B

246 E
Pre-Medical : Physics
UNITS, DIMENSION & MEASUREMENTS

1. Time period, T = kSarbrc 7. Applying dimensional analysis


M0L0T1 = [MT–2]a [L]b [ML–3]c SµEVT
a b c

= [M]a+c [L]b–3c [T]–2a 1 0 –2 1 2 –2 a 1 –1 b 1 c


[M L T ] = k [M L T ] [L T ] [T ]
\ –2a = 1 or a = –1/2 1 0 –2 a 2a –2a b –b c
Also a + c = 0 or c = 1/2 [M L T ] = k [M L T ] [L T ] [T ]
1 0 –2 a 2a+b –2a–b+c
and b – 3c = 0 or b = 3c = 3/2 [M L T ] = k[M] [L] [T]
Comparision
rr 3
Thus T = k 2a + b = 0 –2 = –2a – b + c
S a =1
b = –2 –2 = –2(1) + 2 + c
N - m3 c = –2
2. SI units of A º units of (Ex3) =
C So the dimensional formula for surface tension will
1 –2 –2
be [E V T ]
æNö 1 æVö 1 V
SI units of B º units of (E/y) º ç ÷ = ç ÷ = 2 Alternate solution :
èCøm èmøm m
Surface energy
Surface Tension =
Area
æNö 1 N
SI units of C º units of (E/z2) º ç ÷ 2 = 2 [E]
C
è ø m m -C [Surface tension] =
–2 –2
= [E V T ]
[V × T]2
8. nc µ [hx ry rz]
é A ù é F / x ù é 3/2 ù 3/ 2
3. ê B ú = ê F / x2 ú = ë x û = L [L1T-1] µ [M1 L–1 T–1]x [M1 L–3 ]y [L1]z
ë û êë úû
[L1T–1] µ [Mx+y] [L–x-3y+z] [T–x]
éxù taking comparision on both size
4. ê g ú is dimensionless
ë û x + y = 0, –x – 3y + z = 1, –x = –1
Þ x = 1, y = –1, z = –1
éb ù
but [ax] = [b] so [x] = ê a ú
ë û c
é e2 ù
a b
é b ù é ag ù 9. [L] = [c] [G] ê ú
Therefore ê ú or ê ú is a dimensionless ë 4pe0 û
ë ag û ë b û
combined
c
[L] = [LT–1]a [M–1L3T–2]b éë M L3 T -2 ùû
1 é 1 ù -1
5. = cÞê ú = LT
m0 Î0 êë m 0 Î0 úû [L] = La+3b+3c M–b+c T–a–2b–2c

Energy a + 3b + 3c =1
6. Planck's constant h =
Frequency
–b + c = 0
éML2 T –2 ù a + 2b + 2c = 0
ë û
h= = [ML2T–1]
é T –1 ù
ë û On solving,

Gm1m2
F= 1 1
r2 a=–2,b= ,c=
2 2
Fr 2
\ Gravitational constant G =
m1m 2 1
1 é e2 ù 2
[M1 L1 T –2 ][L2 ] \ L = 2 êG. ú
= = [M–1L3T–2] c êë 4p e0 úû
[M2 ]

E 247
Pre-Medical : Physics
10. The dimensions of Plank constant = [ML2T–1]
DX DM DL DT
Energy = [ML2T–2] 21. =a +b +c = aa% + bb% + cg%
X M L T
11. The work done = force × displacement
\ unit u1 = Fs DX
× 100 = (aa + bb + cg)%
and u2 = 4F × 4s = 16u1 X
12. n1u1 = n2u2 2.63 + 2.56 + 2.42 + 2.71 + 2.80
22. Mean value a =
3 3 5
u1 éM ù éL ù é 1 ù é5 ù
\ n2 = n1 = 8 ê 1 ú ê 2 ú = 8 ê ú ê ú = 50
u2 a = 2.624
ë M2 û ë L1 û ë 20 û ë 1 û
absolute error in various readings are
13. n2 = n1 Da1 = 2.624 – 2.63 = –.006
1 2
é M1 ù é L1 ù é T1 ù
-2 2
1 æ1ö æ1ö
-2
Da2 = 2.624 – 2.56 = .064
ê ú ê ú ê ú = 1´ ´ç ÷ ´ç ÷ Da3 = 2.624 – 2.42 = .204
ë M2 û ë L 2 û ë T2 û a èbø è gø
Da4 = 2.624 – 2.71 = –.086
= g 2 / ab2 Da5 = 2.624 – 2.80 = –.176

14. According to the rules of significant figures Da1 + Da2 + Da 3 + Da 4 + Da5


Da = = 0.11 sec
0.007 m2 has one significant figures. 5
2.64 × 1024 kg has three significant figures.
0.0006032 m2 has four significant figures. pD2 l
23. QV=
4
6.3200 J has five significant figures.
15. Density of material DV 2DD Dl æ 2 ( 0.01) 0.1 ö
\ = + =ç + ÷ ´ 100
V D l è 4 5 ø
g 4 ´ 1000 100g
=4 =
cm3 100 (10cm)3 = 2.5% (Approx)

xy2
(100g) 24. e= Þ e µ x1y2z–1/3
= 40 10z1 / 3
(10cm)3
De æ Dx Dy 1 Dz ö
1 100 Þ ´ 100 = ç +2 + ÷ ´ 100
16. The percentage error = ´ = 0.8% e è x y 3 z ø
5 25
17. The mean value of refractive index, 1
= 2 % + (2 × 1 %) + ( × 3 %)
3
1.34 + 1.38 + 1.32 + 1.36 = 2% + 2% + 1 % = 5 %
m= = 1.35
4
M M
and 25. r= = 2
V pr l
(1.35 - 1.34) + (1.35 - 1.38) + (1.35 - 1.32) + (1.35 - 1.36)
Dm= Dr DM 2Dr Dl
4 = + + (For maximum error)
r M r l
= 0.02
DM 0.003
Dm 0.02 ´ 100 = ´ 100 = 1%,
Thus ´ 100 = ´ 100 = 1.48% M 0.3
m 1.35
Dr .005
1 2 ´ 100 = ´ 100 = 1%
19. h = gt (for free fall) r 0.5
2
Dl 0.06
2h Dg Dh 2Dt ´ 100 = ´ 100 = 1%
g= 2 Þ = + = e1 + 2e2 l 6
t g h t
20. Large number of readings will reduce the random Dr
× 100 = 4%
errror. r

248 E
Pre-Medical : Physics
28. One main scale division, 1 M.S.D. = x cm
V 10
26. R= = = 5W
I 2 (n - 1)x
One vernier scale division, 1 V.S.D. =
n
DR DV DI
Also, ´ 100 = ´ 100 + ´ 100 Least count = 1 M.S.D. – 1 V.S.D.
R V I
nx - nx + x x
= = cm.
0.5 0.2 n n
= ´ 100 + ´ 100 = 15%
10 2 29. Reading of screw gauge
Thus R = 5 ± 15% W = MSR + VSR × LC + zero error

l = 0.5 cm + 25 × 0.001 cm + 0.004 cm


4 p2 l
27. T = 2p \ g=
g T2 = 0.529 cm

Dg é Dl DT ù
and ´ 100 = ê +2 ´ 100
g ë l T úû

é 0.1 0.1 ù
= ê +2 ´ 100
ë 100 2 ´ 100 úû

= 0.2 %

E 249
Pre-Medical : Physics
KINEMATICS
1. The average speed
u2
8. 0 = u2 – 2a s Þ s=
2x 3x 2a
+
5 5 5v1 v 2
= = . s1 u12 u2 1
2x / 5 3x / 5 3v1 + 2v 2 \ = 2 = =
+
v1 v2 s2 u 2 ( 4u )2 16
2. Average speed is given by 9. For first 1m of fall,

2v 0 ( v1 + v 2 ) 1 2 2
v av = 1= gt1 , \ t1 =
2v 0 + v1 + v 2 2 g
For 2m of fall,
2 ´ 3 ( 4.5 + 7.5 )
= = 4 m/s 1 2 4
(2 ´ 3 ) + 4.5 + 7.5 2= gt , \ t= ,
2 g
dx dx
3. V= a x Þ = a x Þ = adt 4 2 2
dt x \ t 2 = t - t1 = - = ( 2 - 1)
x t g g g
Integrating, ò
x =0
x -1/ 2 dx = ò a dt
t =0
For 3 m of fall,
1 2 2
2 x = at Þ x = at/2 3= gt , \ t= 3 ,
2 g
Put this value of x in the original given eqn.
2
\ t3 = t - t2 = ( 3 - 2 ) .
V = a x = a(a.t/2) = a t/2 2 g
10. If h is the height of the building, then
\ Vµt
v max
v 2A = v 2 + 2gh

v max v max b t t and v 2B = ( - v ) + 2gh


2
= 2 = 1
4. a= t ; b= t \ a v max t2 ....(i) Clearly vA = vB.
1 2
t1 2
ævö
V2max = 2 a x1 V2max = 2bx2 11. 2
ç ÷ = v - 2g ´ 3
è2ø
2
2
Vmax Vmax x1 b
x1 = x2 = = ...(ii) \ v = 8g .
2a 2b x2 a
If h is the further height, then
x1 t1 x1 t 1 b
Now x = t So = =
x2 t2 a ævö
2
2 2
0 = ç ÷ - 2gh
5. For uniformly accelerated motion, the x must be è2ø
quadratic in t. So,
v 2 8g
x-a \ h= = =1m.
t2 = or x = a + bt2 8g 8g
b
1 2
1 1 12. h = ut1 - gt1
s1 = a (10 ) and s = a (20) = 4s1
2 2
6. 2
2 2
\ s2 = s – s1 = 3s1 1 2
Also h = ut2 -
gt2
7. Velocity changes from v1 to v2 in time t1 + t2, so 2
v 2 - v1 After simplify above equations, we get
a= .
t1 + t 2 1
h= gt1 t2 .
2
v3 - v 2
Similarly a= a1 a2
t2 + t3 13. 0+ ( 2 ´ 5 - 1) = 0 + [2 × 3 – 1]
2 2
æ v 2 - v1 ö æ v 3 - v 2 ö
Thus ç ÷=ç ÷ a1 5
è t1 + t2 ø è t 2 + t 3 ø \ = .
a2 9

250 E
Pre-Medical : Physics
1 Distance travelled from t = 0 to t = 2 sec
14. h1 = ´ 10 ´ 52 = 125 m
2 1 1
´ 1´ 8 + ´1 ´ 8 = 4 + 4 = 8 m
=
2 2
1
h2 = ´ 10 ´ 32 = 45 m 22. The height h is covered in time interval t = 1 s to
2
\ h = h1 – h2 = 80 m. t = 2s or t = 5s to t = 6s.
v t At maximum height t = 3.5 sec.
dv
15.
dt
= at or ò dv = ò ( at ) dt
0 0
h between t5 & t6 is
1 2 1
h= a ( 6 – 3.5 ) – a(5 – 3.5)2
at 2 2 2
\ v = u+
2 1 1 2
= ´ 7.5(6.25) – (7.5)(1.5) = 15 m
v A tan 30° 1 / 3 1 2 2
16. = = =
v B tan 60° 3 3 u
23. t asc =
17. Effective acceleration in ascending lift = (g + a) g t-t1 t-t1

2h 2 ´ 9.5 \ time for upward journey P


t= = t1
g+a 32 + 6 from this particular point

2 ´ 9.5 1 éu ù
t= = sec = (t asc - t1 ) = ê - t1 ú
38 2 ëg û
18. The maximum acceleration will occur in the duration
30 s to 40 s. So éu ù
\ total time = 2 ê - t1 ú
v 2 - v 1 60 - 20 ëg û
a= = = 4 m/s 2 .
t 2 - t1 40 - 30 24. Let downwards direction is +ve
r
19. It is the s - t graph of a body projected upward. 1 2
It has uniform acceleration downward. h = – ut1 + gt ...... (1)
2 1
vT 1 2
20. The distance, s = h = ut2 + gt ....... (2)
2 2 2
Multiplying eqn. (1) by t2 and eqn. (2) by t1
v v
Also, m = , \ T= .
T m 1 2
ht2 = –ut1t2 + gt t ........ (3)
2
2 1 2
v
Now s= .
2m 1 2
ht1 = ut2t1 + gt t ....... (4)
21. Area of (v–t) curve = displacements 2 21
x = 2 + 8t – 4t2 adding eqn. (3) & (4)
v 8 0 -8
t 0 1 2 1 2 1 2
ht1 + ht2 = gt1 t2 + gt t
2 2 21
dx 1
v= = 8 - 8t
dt Þ h(t1 + t 2 ) = gt1t 2 (t1 + t 2 )
2
v(m/s)
1
Þ h= gt t ......... (5)
8 2 12
for free fall when u = 0, then
1 2
2 h= gt ....... (6)
t(s) 2
0 1
From eqn. (5) & (6)
1 2 1
gt = gt t Þ t = t1 t 2
–8 2 2 12

E 251
Pre-Medical : Physics
25. v = b x–4n 36. Area under a-t graph gives the change in velocity
during given time interval.
dv 1
so = – 4nb x–4n–1 \ umax = × 5 × 6 = 15 m/s
dx 2
Since initial velocity = 0
dv
Now a = v = (bx–4n) (–4nb x–4n–1) \ Maximum speed of the particle = 15 m/s
dx
37. If v1 and v2 are the velocities, then
Þ a = –4nb2 x–8n–1
(v1 – v2) × 20 = (100 + 100) ....(i)
and (v1 + v2) × 10 = (100 + 100) ....(ii)
1 dx 1
26. x= Þv= = - After solving above equations, we get
t + 10 dt ( t + 10 )2
v1 = 15 m/s and v2 = 5 m/s.
38. vA = u – gt and vB = gt.
dv 2
Acceleration, a = = Þ aµ (velocity)3/2
dt ( t + 10 )3 vA – vB = (u – gt) – gt = u – 2gt.
39. If u is the velocity of projection,
28. x = 50 + 12 t – t3
then 0 = u2 – 2g(4h)
y
dx \ u= 8gh
v= = 0 + 12 - 3t2
dt
1 2
Now y = gt ...(i)
for v=0 r 12 – 3 t2 = 0 2 u h–y
r t2 = 4 r t = 2 sec 1 2
and h – y = ut - gt ...(ii)
Þ xt = 0 = 50 m 2
From above equations, we have
xt = 2 S = 50 + 12 × 2 – (2)3 = 66
distance travelled = xt = 2 – xt = 0 = 66 – 50 = 16 m h h h
t= = =
u 8gh 8g
ds
29. Instantaneous velocity = = slope of s–t curve. 40. For maximum velocity, a = 0
dt
and so, 0 = b – cx or x = b/c.
a
(2n - 1)
v x
30. sn = u + dv
2 Now, v
dx
= b - cx Þ ò vdv = ò ( b - cx )dx
0 0
120 a
= 0 + (12 - 1) v2
cx b c (b / c)
2 2
b
100 2 = bx - = b´ - or v =
2 2 c 2 c
6´2
a= 1 2 2s
5 ´ 11 41. s= ft1 ; \ t1 =
2 f
a = 0.218 m/s2
2s
x - xi 0-0 v max = ft1 = f = 2 fs .
32. Average velocity = f = =0 f
t t
1
Thus 5s = éét + 3t1 ) + t ûù ´ v max
33. Area of (v–t) curve = displacement (height) 2 ëë
1
=
1
´ 120 ´ 1000 = 60,000m = 60km or 5s = (2t + 3t1 ) ´ v max
2 2
34. Acceleration is always downward i.e. positive so 1æ 2s ö
or 5s = ç 2t + 3 ÷ ´ 2 fs
slope of v – t curve will always be positive 2è f ø
Hence correct option (3) 1 2
\ s= ft
2

252 E
Pre-Medical : Physics
1 ® ® ®
42. u= = 4 km/h 47. v rm = v r - v m
1/4 vm=v
®
v rm = -ujˆ - viˆ

vr=u
r

5km/h v
v
u=4 km/h r tanq =
u q
v

\ v = 52 - 42 = 3 km/h æ vö q u
r q = tan–1 ç ÷
r
è uø
43. v m = 2iˆ + 3jˆ m/s
r 48. The velocity upstream is (4 – 2) km/hr and
v rm = -4jˆ m/s downstream is (4 + 2) km/hr.
r r r
v rm = v r - v m
2 2 4
\ Total time taken = + = h
r
(
-4ˆj = v r - 2iˆ + 3jˆ ) 2 6 3

r = 80 minutes
v r = 2i - ˆj
49. Let v = speed of buses travelling between A and B
Now for downward motion
r r r The relative speed of bus going from A to B w.r.t.
( )
v rm = v r - v m = 2iˆ - ˆj + 2iˆ + 3ˆj = 2iˆ - ˆj + 2iˆ + 3jˆ
cyclist = (v – 20) km hr–1
r r
v rm = 4iˆ + 2jˆ Þ v rm = 20 = 2 5 m/s and relative speed of bus goint from B to A w.r.t.
44. vC = 20 km/hr EAST cyclist = (v + 20) km hr–1

vTC = 20 3 km/hr NORTH in time T, distance covered by bus = vT


r r r when bus and cyclist are in same direction then
v TC = v T - v C
r r r
v T = v TC + v C vT
= 18 min ....(1)
r v - 20
v T = 20 3jˆ + 20iˆ
r when bus and cyclist are in opposite direction then
v T = 1200 + 400 = 40 m/s
vT
20 3 = 6 min ....(2)
tan q = v + 20
20

tan q = 3 v + 20 18
q = 60° dividing eq (1) by eq (2) = =3
v - 20 6
d d
45. t= = Þ v = 40 km/h
v + v cos90 v
d 40 ´ T
46. t= putting value v in eq (1) = 18 min
40 - 20
v - v cos120

d d
= 40
v v Þ T = 18 min Þ T = 9 min
v- 20
2 2

2d
=
v

E 253
Pre-Medical : Physics
51. At t = 0 let the man's position be the origin.
\ xp = 0 l l t t
t= = = 12
The bus door is then at xB = 6.0 m. V1 + V2 l l t1 + t2
+
The equation of motion for the man is t1 t 2
Distance covered by person in time t to access the
57. At the highest point of trajectory, the acceleration
door is xp = 4t
is equal to g.
Distance covered by bus in time t is
u2 sin ( 2 ´ 15° ) u 2
1 58. R1 = =
x B = ´1.2t2 = 0.6t2 g 2g
2
and also u2 sin ( 2 ´ 45° ) u 2
and R2 = =
xp – xB = 6 g g
4t – 0.6t2 = 6 \ R2 = 2R1 = 2 × 1.5 = 3 km
t = 2.3 s & 4.4 sec. 1
The man shall access the door at 2.3 sec. 59. 1960 = ´ 9.8 ´ t2
2
B 10 \ t = 20 s

10 10 2 æ 5 ö
Now AB = ut = ç 600 ´ ÷ ´ 20 = 3333 m.
52. è 18 ø
O 60. The vertical components of the velocities must be
A equal so.
r
v BA = 102 + 102 = 10 2 kmph v2 1
v1sin 30° = v2 or =
v1 2
distance OB = 100 cos45°= 50 2 km 61. The boy velocity = horizontal velocity of the ball
Time taken to reach the shortest distance between = u cos q.
62. Let u is the velocity of projection, then
50 2 50 2 u2
A&B= r =
R max == d or u = gd
v BA 10 2 g
t = 5 hrs. Let h is the height upto which ball rise, then
53. For two particles to collide, the direction of the u 2 gd d
relative velocity of one with respect to other should 0 = u2 – 2gh or h= = =
2g 2g 2
be directed towards the relative position of the other 63. uy = u sin q
particle
1 2 1
r r y = uy t - gt or 5 = (25 sin q) × 2 – ´ 10 ´ 22
r1 - r2 2 2
i.e. rr - rr ® direction of relative position of 1 w.r.t. 2.
1 2
1
\ sin q = or q = 30°.
r r 2
v 2 - v1
& vr - vr ® direction of velocity of 2 w.r.t. 1 2u x u y
2 ´ 10 ´ 20
2 1
64. R= = = 40 m
g 10
so for collision of A & B
65. Time of motion,
r r r r
r1 - r2 v 2 - v1 x 80 8
r r = r r t= = = s
r1 - r2 v 2 - v1 u x 30 3
54. xP(t) = at + bt2 xQ(t) = ft + t2
2u y
vP = a + 2bt vQ = f + 2t t =
as vP = vQ g
f -a
a + 2bt = f + 2t Þ t = 8 10
2(1 - b) uy = ´
3 2
55. V1 ® velocity of Priya
V2 ® velocity of escalator 40
uy = m/s
l ® distance 3

254 E
Pre-Medical : Physics
72. R is same at an angle q and (90° - q)
u 2 sin 45° u 2 u2
66. H= = Þ R= = 4H
2g 4g g 2u sin q 2u sin(90 - q) 2u cos q
t1 = & t2 = =
g g g

67. DP = 2 mu sin q 2u sin q 2u cos q u2 2 sin q cos q 2


t1 t 2 = ´ = ´
g g g g
u 2 sin2 q
68. H1 = 2R
2g r t1 t 2 = r t1t2 µ R
g

u 2 sin2 ( p / 2 - q ) u 2 cos2 q u2
and H2 = = 73. Rmax =
2g 2g g

u2 sin2 q u 2 cos2 q
H1H2 = ´
2g 2g
Rmax
( u 2sin q cos q )
2
2
R2
2
= = 2 é u2 ù
area = p R = pê ú
2
16g 16 max
ëgû
\ R = 4 H1 H 2 . 2 ´ 20
74. t= = 2 sec
g
r
69. v = aiˆ + bjˆ r u cos q = a and u sin q = b 4
x = vxt Þ vx = = 2m/s
2
R = 2Hmax
75. vx = 10iˆ
b u 2 sin2q 2u 2 sin2 q
tan q = also = vy = -10jˆ t = 1.5 sec. a = - gjˆ = -10jˆ
a g 2g
r
( u y ) = -10jˆ – ( -10jˆ ) × (1.5)
Þ tanq = 2
r
Þ u y = 5 ĵ
b
\ =2 Þ b = 2a r
a u = 10iˆ + 5ˆj Þ ur = 5 5 m/s
70. For same range q1 + q2 = 90°
76. u x1 t + u x2 t = x
p
q1 = rad = 60°
3 é u ù
ê cos 30o + u cos 60o ú t = x
ë 3 û
\ q2 = 90 ° – q1 = 30°

u2 sin2 60° u2 ´ 3 u 2 y1 é u 3 uù x
y1 = = r = ê ´ + út = x r t=
2g 2g ´ 4 8g 3 ëê 3 2 2 úû u

r r r
u2 sin2 30° u2 ´ 1 y1 77. v = u + at = ( 3iˆ + 4ˆj ) + ( 0.4iˆ + 0.3jˆ ) ´ 10
y2 = = =
2g 2g ´ 4 3 = 7iˆ + 7jˆ
4H 4
71. R = 2H r tan q = = =2 or v = 72 + 72 = 7 2 unit
R 2
1
2v2 78. x= ´ 6 ´ 42 = 48 m
R= sin q cos q 2
g
2 5 1
and y= ´ 8 ´ 42 = 64 m
2
2v 2 1 4v 2 q 2
R= ´ ´ =
g 5 5 5g 1
\ s= x 2 + y 2 = 482 + 64 2 = 80 m
r
u = 0, a = const. so path is straight line

E 255
Pre-Medical : Physics
r
79. R = 2sin (2pt) î + 2 cos 2pt ĵ ds
83. v = At + Bt2 Þ = At + Bt 2
dt
r
r dR
v= = 4 p cos 2 p t ˆi – 4p sin 2pt ˆj s 3
Þ ò ds = ò (At + Bt )dt
2
dt
0 2
r
v = 4 p 2 m/s A 2 B 5A 19B
Þs= (3 - 22 ) + (33 - 23 ) = +
r 2 3 2 3
80. r = cos wt xˆ + sin wt yˆ
r
r dr
Þ v= = -w sin wt xˆ + w cos wt yˆ
dt
r
r dv r
Þ a= = -w2 cos wt xˆ - w2 sin wt yˆ = -w2 r
dt
r
a is directed towards the origin.
rr r r
Also r.v = 0 hence r ^ v
82. 0 < t < 1s : velocity increases from 0 to 12 m/s
1 < t < 2s : velocity decreases from 12 to 0 m/s
but car continues to move forward
2 < t < 3s : since field strength is same
Þ s am e a cc e l er a t i o n \ car's velo cit y
increases from 0 to 12 m/s
v (m/s)
12

0 3 t (s)
1 2
–12

Distance travelled in first second


æ u + v ö æ 0 + 12 ö
S=ç ÷t = ç ÷ (1) = 6m
è 2 ø è 2 ø
18m
Total distance = 9 m so average speed = = 6m/ s
3s
6m
Displacement = 3m so average velocity = = 2 m/s
3s

256 E
Pre-Medical : Physics

NLM AND FRICTION

1. Forces from ground causes horse motion. dm


2. Weight = mg 9. F = v
dt
\ as g = 0, w = 0 F = 1 ×5
ma = 5
F
3. a=
m 5
a =
2
F a = 2.5 m/s2
v2 - 0 = 2 ´d
m 10. Mg = n × 2mV

1 1 × 9.8 = 10 × 2 × 0.05 × V
\ vµ
m
9.8
4. Dp = 2 mv cosq V= = 9.8 m/s
1
dp dm
F= = (2v cos q)
dt dt 100t2
11. I= ò Fdt = 500t -
2

é10 cos 60° ù 2


12. a=ê ú = 1m / s
ë 2+3 û
The mass crossing an area in t would be and T = ma = 2 × 1 = 2 N.
13. For 8 kg block,
v × t × a × d = m ...(1)
For small element
8kg
dm
\ = vad \ F = 2av d cosq
2 ////////////////////////////////////////////////

dt ///////////////////////////////////////////////

r r
5. For constant velocity, a = 0, or F=0
r r r
Thus F1 + F2 + F3 = 0
r r r
or (
F3 = - F1 + F2 )
r
ë ( ) (
F3 = - é 3iˆ + 2jˆ - 4kˆ + -5iˆ + 8ˆj - 3kˆ ù
û ) T = 0.5 × 8 g
and for bucket, 2T = mg
\ m = 8 kg
( ) (
= - -2iˆ + 10jˆ - 7kˆ = 2iˆ - 10jˆ + 7kˆ N .) mass of sand added = 7 kg
r 14. T2 + ( T + mg )
2
6. a = -12sin 30°ˆi - 12cos 30°ˆj = -6iˆ - 6 3jˆ . F=
By Newton's law, we have
r T
(
20i + F = 2 ´ -6iˆ - 6 3ˆj )
r
\ F = -32iˆ - 21jˆ N .
7. J = M(vf – vi)
or 50 × 5 + 75 × 5 = 16(vf – 0)
\ vf = 39 m/s. T=Mg
mg
0.2 ´ 20
8. F= = 8N
( Mg ) + ( Mg + mg )
2 2
0.5 =

E 257
Pre-Medical : Physics
22. The system, as a whole, will fall towards ground under
a=
(10 - 8 ) g = g m / s2
15. gravity. The spring will neither be compressed nor
10 + 8 9
stretched regradless of the values of m1 and m2.
Tension in the string
T = m(g – a) = 4(g – g/9) 23. N = m(g – a) = 0.5(10 – 2) = 4N

8 320
= 4´ g= N
9 9 m ma
24.
F cos 60° F F a
16. ax = = = q mg
M 2 ´ 10 20
According to question sinq = 1/x (1 in x)
F
Thus 3 = ( 2 + 1 ) ´
20
1
\ F = 20 N. So tan q =
2
x –1 N
17. Acceleration of blocks

q
os
ac
k To keep the block stationary

m
ma
m q
in
////////////////////////////////// gs
relative to the inclined plane m
///////////////////////////////////

mgcosq
mg + masinq
mgsinq = macosq
M
g
a = gtanq Þa= 2
Mg x –1
a=
m+M 25. Increased
Tension in the string/spring
26. F = 10 × 4 = 40N
mMg
T = ma =
m+M
40N 20kg
200N
T mMg
\ Extension in the string, x = =
k k (m + M)

æ P ö 160
18. a =ç ÷
èM+mø a= = 8m/s2
20
m
M P
27. In case (b) entire tension = 2mg, hence accelera-
The force exerted by rope on the block tion is more.
æ MP ö 28. Let horizontal velocity of block is u
F = Ma = ç ÷
èM+mø

Mg sin q g æp ö
a= = sin q u cos ç - q ÷ = v
19. 2
(M + M) 2 è ø

Mg
\ T = Ma = sin q
2 v
\ u=
20. mg – T = ma sin q
3mg
or mg - = ma 29. Let the block moves upward with velcity
4
\ a = g/4 m/s2 u cosq = V
R
21. T cosq = W ; T sinq = R ; tan q =
W v
\ u=
T2 = R2 + W2 cos q
r r r
also as for equilibrium R + T + W = 0

258 E
Pre-Medical : Physics
31. 2m1m2 (g + g) 4m1m2 g
37° 53° 37. T= =
m1 + m2 m1 + m2
T1
37° O 53° In terms of w
T 4w1 w 2
200 T = w +w
1 2

In equilibrium Acc to Lami's thearum


F P Q
200 T1 38. m 3m 5m
=
sin (180 - 90 ) sin ( 90 + 53 ) [sin(90+q) = cosq]

200 T1 Let a be the common acceleration of the system


= T1 = 120
1 cos53
F F
Kx = 120 Þ K × 4/100 = 120 \ a= = ....(i)
m + 3m + 5m 9m
K = 3000N/m
32. T1 – mg = m × 2 \ T1 = m × 12 \ P = (3m + 5m)a
T2 = mg T2 = m × 10
16 = 8ma [Q P = 16N(Given)]
mg – T3 = m × 4 T3 = m × 6
T1 : T2 : T3 = 6 : 5 : 3 2
a= ....(ii)
m
F
33. a=
m1 + m2 + m3 Substituting this value of a in eqn(i), we get
F = 18 N
For m2 to be at rest wrt m3
m2g = m1a
P' Q'
m 3m 5m F
æ F ö
m2g = m1 ç ÷
m
è 1 + m + m 3 ø
2 P' = ma = 2N (Q a = 2/m)

æ m2 ö Q' = (3m + m)a = 4ma = 4 × 2 N = 8 N


F = (m1 + m2 + m3) ç g÷
è m1 ø * None of the given option is correct.
34. Tension in string
2m1m2 g 2 ´ 2 ´ 1g Dp 2mv sin 30° 2 ´ 0.5 ´ 48 1
T= = 39. Favg = = = ´ = 48N
m1 + m2 3 t t 1/ 2 2

Thrust on pulley 40. during up the incline and down the incline constant
8 forces respectively would be used.
FThrust =g
3 41. Change in momentum,
\ Reading of spring balance should be less than
3kg–wt Dp = ò Fdt
35. Unchanged, both have same accleration always.
= Area of F-t graph
36. In (A) T = kx1 = 2g
g 12 1
In (B) T = kx2 = 3g – 3 × = g =( × 2 × 6) – (3 × 2) + (2 × 3)
5 5 2
g 4 = 6 N-s
In (C) T = kx3 = 2g – 2 × = g
3 3
uuur uuur mV
x1 5x 2 3x 3 42. Impulse = Dp = m DV = m(V cos 60°) =
= = 2
2 12 4

E 259
Pre-Medical : Physics
43. Before cutting the string :-
g
4mg T
50. m ( ma ) ³ mg a³
m

A B 51. Coefficient of sliding friction has no dimension


52. FBD w.r.t. wedge
mg T 3mg
\ T = 3mg m ac N
os q
After cutting the string :-
4mg q

ma mg
A B si n
q

mg 3mg mg q
4mg - mg
aA = = 3g
m macosq = mgsinq Þ a = g tan q
3mg
aB = =g
3m 53. a
T
45. fL = msN mk m1
= 0.6 × 20 g = 120 N mk = m1g
120 - 0.2 ´ 20g T
Now a= = 4m / s 2
20
46. flim = µN = 0.4(25 – 9) = 6.4 N
m2 a
External force is 6 N and so block will not move.
So frictional force = 6.0 N.
m2g
47. 10 = mgsin30°
\ m = 2 kg
For the motion of both blocks
48. Q cos q + mg = N
m2g – T = m2a
P + Q sinq = mN
(m2 - mk m1 )g
T – mk m1g = m1a Þ a = m1 + m2
P + Q sin q
\ m=
Q cos q + mg For the block of mass 'm2'

é m - mk m1 ù
m2 g - T = m2 ê 2 úg
49. T ë m1 + m2 û
12N 30°
é m - mk m1 ù
T = m2 g - ê 2 ú m2 g
ë m1 + m2 û
W
é m + mk m1 ù
= m2 g ê 1 ú
ë m1 + m2 û
T cos30° = 12 ...(i)
T sin30° = w ...(ii) m1m2 (1 + mk )g
Þ T= m 1 + m2
tan30° = w/12

1
w = 12 × = 6.92 N
3

260 E
Pre-Medical : Physics
54. 55. Let the length of incline is d
Case I : for rough incline plane
N3
F g – mg æ1 – m ö
a ar = gsin45° – mgcos45° = = ç ÷g
2 è 2 ø
N2 T2
T2 °
60 60° ° 2d
sin m 3g os
60 time taken to slide down (tr) =
N1 T1 gc ar
m 3

T1
0°60°
s in6m 2g o s6
0° Case II : For smooth incline plane
g gc
m 2
m 2

60° g
° ° as = gsin45° =
60 m 1g 60
sin os 2
g 60° gc
m 1
m1

2d
Here, m1 = 2.0 kg, m2 = 4.0 kg, m3 = 6.0 kg, Þ ts = as
F = 120 N
Acc to question
Let a be common acceleration of the system.
nts = tr Þ n2ts2 = tr2
The equation of motion of block 1 is
T1 – m1gsin60° = m1a ....(i) æ 2d ö 2d
n2 ç =
ç g/ 2 ÷÷ æ 1 – m ö
The equation of motion of block 2 is è ø
ç ÷g
è 2 ø
T2 – T1 – m2gsin60° = m2a ....(ii)
1
The equation of motion of block 3 is n2 =
1–m
F – T2 – m3gsin60° = m3a ....(iii) 1
Þ m=1–
Adding (i), (ii) and (iii), we get n2
F – (m1 + m2 + m3) g sin 60° = (m1 + m2 + m3)a 56. mg sinq = m mgcosq
sinq = mcosq ...(i)
F - (m1 + m2 + m3 )g sin 60°
a= when pushed up
m1 + m 2 + m 3
acceleration (a) = –(gsinq + mgcosq)
From equation (i) = –(gsinq + gsinq) = –2gsinq
T1 = m1gsin 60° + m1a \ v 2 - v 20 = 2(a) ´ s
m1 [F – (m1 + m2 + m3 )g sin 60°]
= + m1 g sin 60° Final velocity v = 0
m1 + m2 + m3
–v02 = 2as Þ –v02 = 2(–2gsinq)s
m1F 2 ´ 120
T1 = = = 20N v 20
m1 + m2 + m3 12 \ s=
2(2g sin q)
From equation (iii) 57. a10 = a4
T2 = F – m3gsin60° –m3a F - 12 12
=
10 4
æ F – (m1 + m2 + m3 )g sin 60° ö
T2 = F – m3gsin60° – m3 ç ÷
è m1 + m2 + m3 ø F - 12
=3
(m1 + m2 )F 10
T2 = m + m + m = 60 N F – 12 = 30
1 2 3

F = 42 N

E 261
Pre-Medical : Physics
58. 10g = m (10 + m)g
10 = m (10 + m)
10 = 0.5 (10 + m)
10 + m = 20
m = 10 kg
59. Limiting friction between B and ground
= 0.4(5 + 10) (10) = 60N
Þ acceleration of B is zero.
Therefore acceleration of block A

50 - 0.8 (5) (10)


aA = = 2 m / s2
5
F
60. a= for block A
7
F
2 ´ = 0.6 ´ 2 ´ 10 Þ F = 42N
7

61. T = M2g = m ( M1 + m ) g

\ M2 = m ( M1 + m )

6 = 0.4(4 + m) = 11 kg

262 E
Pre-Medical : Physics
WORK, ENERGY & POWER
r r r
1. d = r2 - r1 t2 t
9. s= v=
r r 4 2
ˆ
Þ d = ˆi + ˆj - kˆ & F = (ˆi + 2ˆj + 3k)
rr t = 0, u = 0
So W = F.d = 0
2. N = Mg + F sinq t = 2, v = 1

F c o sq 1
q \ WD = DKE = × 6 × 1 = 3J
N 2

F sin q 10. Work will be zero as force is perpendicular to


displacement.
mg
11. WD is independent of path for conservative forces.
r
FNet = µ k(mg + F sinq) r r r
12. s = rf - ri = 4iˆ - ˆj + 3kˆ
Work = –µ k(mg + Fsinq) s
rr
3.
4.
Work done = Area under F – d curve
dW = kx2dx cos60°
( )
W = F.s = 2iˆ + 3jˆ . é4i - ˆj + 3kˆ ù = 8 - 3 = 5J
ë û
13. As we are pulling the bucket with constant velocity
\ WD =
k x2 2
2 x1 ò
x dx =
k 3
6
(
x2 - x13 ) and leakage is at constant rate. We can take
5. x = 3t – 4t2 + t3 average mass

dx æ 15 + 9 ö
v= = 3 - 8t + 3t2 W = mavggh = ç ÷ × 10 × 15 = 1800J
dt è 2 ø
1 1
WD = mv 24 - mv 02 = DkE 14. Since force is constant so work done is path
2 2
independent. Hence W1 = W2
1 æ 30 ö 15. For conservative force in a closed loop W = 0
÷ é(19) – (3) ùû = 5.285 J
2 2
WD = ç
2 è 1000 ø ë WDPQ + WDQR + WDRP = 0
8 + 2 + WDRP = 0
5

6. ò
WD = (3x2 + 2x - 7)dx
0
\ WDPR = 10 J

DK 2 Dp Dp 1 DK
16. K µ p2 Þ K = p So p = 2 = 2%
K
= 125 + 25 – 35 = 115 J
p2 1
7. Work will be zero as angle between force & 17. Ek = Þ E K ´ p = constant
2m
displacement is 90°
\ graph is rectangular hyperbola
8. For the block T 18. Kµp2

Mg – T = M(g/4) So if p become ‘n’ times & K becomes n 2 times

a 19. pµ K
3
Þ T = Mg
4 p1 K1
Mg Þ p = Þ p2 = 2p1 as K2 = 2K1
2 K2
3 3
So, Work = Mg(h) cos180° = – Mgh Þ p2 = 1.41 p1
4 4
So momentum will increase by 41.4%

E 263
Pre-Medical : Physics
20. Work = Change in kinetic energy
A B
24. U= 12
-
1 r r6
= Ef – Ei = m(vf2 – vi2)
2
dU
1 2 2
= 0 at Equlibrium
W = (2)(0 - 20 ) Þ W = – 400 J dr
2
21. Till x = 2m, area under the curve F – d is zero so
-12A ( -6)B 6 é -2A ù
W.D. is zero therefore KE remains same at x = 2m, \ - = 0; ê + Bú = 0
r 13
r 7
r7 ë r6 û
v = 4 m/s
Force = –4 N, mass of body = 2 kg
1/ 6
æ 2A ö
-4N r=ç ÷
\ acceleration (a) = = –2 m/s2. è B ø
2kg

This reduces velocity. \ In Eq U is given by


Now v = u + 2as.
2 2

\ v2 = (4)2 + 2 × (–2) = (16 – 4) = 12 A B B2 B2 B2


U= - = - =-
at x = 3 m and onwards. æ 2A ö
2 2A 4A 2A 4A
ç B ÷ B
1 è ø
\ Kinetic energy = × 2 × 12 = 12 J
2
22. Work done = mgh 25. 20cm
m = mass of hanging part A

B
M æ 2L ö 2M
= .ç ÷ =
L è 3ø 3
2L
the height by which the COM riased 3
æ 2L ö 1 L
= çè ÷ø . = v=0 Work done by gravitational force
3 2 3

2M L 2MgL = force ×component of displacement along force


WD = mgh = g. =
3 3 9
æ 2 ö æ 20 ö
20xy = mg(R) = ç ÷ ´ (9.8) ´ ç
23. Given : U = è 1000 ø è 100 ÷ø
z
For a conservative field
r r = 392 × 10–5 J = 3.92 mJ
F = -ÑU
r ¶ ˆ ¶ ˆ ¶ 1 2U
Where, Ñ = ˆi + j +k 26. U= mv 2 \ m=
¶x ¶y ¶k 2 v2

éˆ ¶U ˆ ¶U ˆ ¶U ù 27. Energy dissipated = kinetic energy – potential enrgy


r
\ F =– ê i ¶x + j ¶y + k ¶z ú
ë û 1
ÞE= mv2 – mgh
2
éˆ ¶ æ 20xy ö ˆ ¶ æ 20xy ö ˆ ¶ æ 20xy ö ù
= – ê i x ç z ÷ + j y ç z ÷ + k z ç z ÷ú
ë ¶ è ø ¶ è ø ¶ è øû
1
ÞE= × 0.5 × (14)2 – (0.5) (9.8) (8.0)
2
é æ 20y ö æ 20x ö ˆ æ 20xy ö ˆ ù
= – êˆi ç ÷+ç ÷ j + ç - 2 ÷ kú
Þ E = (49 – 39.2) J Þ E = 9.8 J.
ë è z ø è z ø è z ø û

æ 20y ö ˆ æ 20x ö ˆ 20xy ˆ KE1 m1gh 4 1


= -ç ÷i -ç ÷j+ 2 k 28. = = =
è z ø è z ø z KE2 m2 gh 8 2

264 E
Pre-Medical : Physics

-A WD1
29. Force = 35. P1 =
R2 T1

WD2 P1 2 7 7
-A
R
P2 = \ P = 3 ´ 10 = 15
\ Potential energy = -ò FdR = T2
¥ R 2

36. For the block moving in upward direction


1A mv 2 T – 10g = 10a Þ T = 10(g + g/2) = 150 N
K.E. = by Fcentripetal =
2R R 1 æ gö 2
s= ç ÷ (4) = 40 m
2 è 2ø
-A
T.E. =
2R W T.s 150 ´ 40
P= = = = 1500W
t t 4
30. Work done by the gravity (Wg) = mgh
= 10–3 × 10 × 103 = 10 J æ dv ö
37. P = mav Þ P = m ç v ÷ .v
è dx ø
By work–energy theorem = Wg + Wres = DKE
mv 3
Þ mv2dv = Pdx Þ = px Þ v µ x1 / 3
1 3
10 + Wres = ´ 10-3 ´ (50)2
2 dm
38. Amount of water flowing per unit time =Avr
dt
Wres = –8.75 J

1 1 v = velocity of flow, A is area of cross–section,


31. ´ 0.5 ´ (1.5)2 = ´ 50 ´ x2
2 2 r = density of liquid
To get n times water in the same time,
0.5 ´ (1.5)2
= x2 x = 0.15m
50 æ dm ö ¢ dm
32. WD is +ve by gravity and –ve by spring on body. çè dt ÷ø = n dt Þ A v'r = nAvr Þ v'=nv

1 2 vdm dm ' dm
\ WD = mg(h + d) – kd F= Þ F' = v' = n2 v = n2F
2 dt dt dt
To gets n times water, force must be increased n2
33. times.
39. Power = 100 × 10 × 100 = 100 kW
Fsinq
M (i) F
40.
x q v
Fcosq
M (ii)
rr
Power = F.v = Fv cos q
By conservation of energy
dM
TE at (i) = Mgx 41. Here m =
dl
1 2 d(KE)
TE at (ii) = kx
2 So, Rate of KE =
dt
1 2 2Mg d(KE) 1 dM 2
Mgx = kx = x = = v
2 k dt 2 dt
Also,
34. T = kx for spring
dM dM dl dM dl
= . = . = mv
2 2 dt dt dl dl dt
1 2 1 T T
Energy = kx = k 2 = d(KE) 1 1
2 2 k 2k = (mv)(v 2 ) = mv 3
dt 2 2

E 265
Pre-Medical : Physics
47. F.v = P0
dv
42. P = Fv = mav Þ k = mv
dt dv
m ´ v = P0
dt
By integrating the equation
v 2 P0 t t
= Þ vµ µ t1 / 2
k 2 m m
Þ ò v dv = ò m dt dv
a= µ t -1 / 2
dt
v2 k 2k 48. Pressure = 75 mm Hg
Þ = t Þ v= t
2 m m

dV 10 ´ 10-3
dv 2k æ 1 - 12 ö Pumping rate = = m3/s
a= = t ÷ dt 60
dt m çè 2 ø

dV dV
æ 1 ö 2k mk Power of heart = P. = rgh ´
F = ma = m ç ÷ Þ F= dt dt
è 2 ø mt 2t

43. P = F.v = ma.v


10 ´ 10-3
= (13.6 ×103 kg/m3) (10) ×(0.075) ×
v1 v1 60
a= t &v=0+ t t
1 1

13.6 ´ 10 ´ 0.075
æ v1 ö æ v1 ö mv 2 = = 1.70 watt
So P = m ç t ÷ ç t t÷ Þ P = 2 1 t 6
è 1øè 1 ø t1

44. Force against which work done is r r


dv
49. F = 2tiˆ + 3t 2 ˆj Þ m = 2tiˆ + 3t2ˆj {m = 1 kg}
1 dt
F = mg sinq = 4 × 9.8 × = 0.98 N r
40 v
r
t

ò dv = ò (2tiˆ + 3t ˆj)dt Þ vr = t2ˆi + t3ˆj


2
speed v = 40 m/s Þ
0 0

for 50% efficiency required power = 2 (F×v) r


Power = F.vr = (2t3 + 3t5)W
45. Mass of water = 2238 × 10 × 10
–3 3

= 2238 kg
\ Energy = 2238 × 10 × 10 = mgh
2238 ´ 30 ´ 10
\ = 1 × 750 (T is time)
T

2238 ´ 10 ´ 30
\ T= second = 15 min.
750

v mv
46. a= & F = ma = t1
t1

1 2 1 v 2
s= at Þ s = t
2 2 t1

2
1 v 2
W = F.s = m 2 t
2 t1

266 E
Pre-Medical : Physics
CIRCULAR MOTION

1. (q) = 2t3 + 0.5 8. Speed is constant


dq at = 0
w= = 6t2
dt
v2
at t = 2 s a cp = = constant
r
w = 6 × 4 = 24 rad/s
a net = constant
Dq 2p p
2. w= Þ w= = rad/s
Dt 60 30
r r r 9.
3. v = w´ r T
i j k
r F (centrifugal)
v = 3 -4 1
5 -6 6 w
r dv
v = -18iˆ - 13jˆ + 2kˆ m/s 10. a cp =
v2
Þ at = = a Þ a net = a2t + acp2
R dt
B
S 11. fnet = centripetal force

q
4. A
O

q = wt
r uuur uuur
S = OB - OA mv 2
T=
r
S = R 2 + R 2 + 2R 2 cos( p - wt)
v2
æ wt ö 12. v = 72 km/h Þ = tan q
S = 2R sin ç ÷ rg
è 2 ø
72 ´ 5 400
v2 4 4 v= = 20 m/s Þ tan q =
5. aC = = 2 Þ v = 2 18 20 ´ 10
r r r
tan q = 2 Þ q = tan –12
2
Þ v=
r 13.

2m
Momentum p = mv = l
r

w
a v

v by COME
6.
aC
1
mv 2 = mgl + O Þ v = 2gl
2
7. (FC)heavier = (FC)lighter
14. Using centre of Mass
2 2
2mV m(nV)
Þ = Þ n2 = 4 Þ n = 2 mw2 l
r 2r Fcp = mw2r =
2

E 267
Pre-Medical : Physics

15. 2h 2 ´ 2L L
t= = =2
g g g
q
Tcosq T L
\ horizontal range x = vt = gL ´ 2 = 2L
g

Tsinq v2
21. tan q =
mg rg
Tsinq = mv2/r by ­ speed by 10% speed becomes 1.1v
Tcosq = mg
v2 (1.1v)2
v2 \ =
tan q = rg r 'g
rg
100 \ r' = (1.1)2r = 24.2 m
tan q = =1
10 ´ 10 A B C
q = 45° 22.
TA TB TC
16. by COME
v = 2gl sin q angular velocity w is same for all.

a T = a 2cp + a t2 TC = mw2 (3l)

v 2 2gl sin q TB = TC + mw2 (2l) = mw2(5l)


a cp = =
r l
TA = TB + mw2 (l) = mw2 (6l)
= 2gsinq
and at = gcosq
\ TC : TB : TA ::3:5:6
( g cos q ) + (2gsin q)
2 2
a net = 23. At top most point speed of the body may be zero,
because rod will support the body their
= g 1 + 3 sin 2 q
17. to complete circle v0=0
1
5 \ mv 2 = 0 + mg(2l )
v= 5gr by COME h³ r 2
2

v = 4gl
b h m
18. h tan q ; sin q = v
b r r
q 24. v = vjˆ and u = uiˆ
y v
h v 2 By COME between A and B x
\ = O l
b Rg B
1 1 l
mv 2 + mgl = mu 2 u
2 2 2
v b A
\ h=
Rg \ v= u2 - 2gl
19. T cos q = mg r r r
T Dv = v - u = u2 - 2gl ˆj - uiˆ
q q
T sin q = mrw2 2
mrw r
|Dv|= ( u2 - 2gl )2 + u2 = 2(u 2 - gl)
rw 2 mg
\ tan q = 25. In Balancing condition
g
Fcentritugal £ Friction force
20. For looping the loop minimum velocity at top point
mrw2 £ µmg
v= gL µg

time taken by particle w2

268 E
Pre-Medical : Physics
Mv 2
26. fr = FCP =
R
fr But fr < µMg
So v2 < µ sRg

vmax = m s Rg

27. By using work-energy theorem, W=DKE

æ1 2ö
1 ç mv ÷
Þ (mat) (4pR) = mv Þ at = è 2
2 ø
2 4 pmR

8 ´ 10 –4
Þ at = = 0.1 m/s2
4 ´ 3.14 ´ 10 ´ 10 –3 ´ 6.4 ´ 10-2
OR

1 1 æ 10 ö 2
mv 2 = KE Þ ç v = 8 × 10–4
2 2 è 1000 ÷ø
Þ v2 = 16 × 10–2 Þ v = 4 × 10–1 = 0.4 m/s
Now,
v2 = u2 + 2ats (s = 4pR)

16 æ 22 6.4 ö
Þ = 02 + 2a t ç 4 ´ ´
100 è 7 100 ÷ø

16 7 ´ 100
Þ at = ´ = 0.1 m/s2
100 8 ´ 22 ´ 6.4

v2
28. = tan(f - q)
rg

tan f - tan q
= = ( ms = tan q )
1 - tan f tan q

m s - tan q
Þ v = rg 1 -m tan q (ms = tanf)
s

OR
Check by dimensions.

v2
29. Centripetal acceleration = = a cos 30°
R

3
Þ v= aR cos 30° = 7.5 ´ 5 ´ = 5.7m/s
2

E 269
Pre-Medical : Physics
COLLISION & CENTER OF MASS
1. m1 = M (Solid sphere) 12aiˆ + 8ajˆ 4a ˆ
= = 2aiˆ + j
6 3
rx1 = 0 ry1 = a
H 20
5. hcm = = = 5 cm
Hollow sphere m2 = M 4 4
L

ò x ( kx dx )
2
rx2 = 0 ry2 = 0
x cm =
ò x dm = 0

6. ò dm
L

ò kx dx
2
Disc. m3 = M
0

rx3 = a ry3 = 0 L

ò x dx
3

L4 3 3L
m1rx1 + m2 rx2 + m 3rx3 = 0
= ´ =
rx cm = 0 + 0 + aM a L
4 L3 4
= =
ò x dx
2
m1 + m2 + m 3 3M 3 0

m1ry1 + m2 ry2 + m 3ry3 x=O x=L


rycm = Ma + 0 + 0 a
= = 7. Area of
m1 + m2 + m 3 3M 3
x cross-section = 1
dx
So co-ordinate of mass of system will be

ò xdm = ò x (r x /L ) Adx = 3L
L L
2 2
4
æa aö
0 3L
x cm = 0 0
=
ç 3,3 ÷
ò dm ò (r x /L ) Adx 4L
L L 3
è ø
2 2 4
0 0 0

r r r 8. Centre of mass of solid hemisphere from its base


r m r + m2 r2 + m3 r3
2. rcm = 1 1
m1 + m2 + m3
3R 3×24
is = = = 9 cm
r 8 8
rcm = 0
r r r 9. If external force is zero there will be no shift in COM.
Þ m1 r1 + m2 r2 + m3 r3 = 0 r r
r m1a1 + m2a2
10. a cm =
r r r r m1 + m2
r m r + m2 r2 + m3 r3 + m4 r4
r 'cm = 1 1
m1 + m2 + m3 + m4 æ 15 - 5 ö g r g r g
a1 = ç ÷ g= , a1 = - ˆj , a2 = ˆj
è 15 + 5 ø 2 2 2
ˆ ˆ ˆ 5
(ˆi + 2jˆ + 3kˆ ) = 0 +14+a2(i ++ 32j++43k) Þa=
2
1
(-15 + 5)g -10 -5
a cm =2 = = m / s2
r r 20 4 2
r m r + m2 r2 5 (5iˆ + 4ˆj) + 10 (4iˆ + 3ˆj)
3. rcm = 1 1 =
m1 + m2 5 + 10 (3m - m) g
11. a= g=
3m + m 2
65iˆ + 50jˆ 13iˆ + 10jˆ
= = r r
15 3 r 3ma1 + ma2 m
a cm =
4. m 3m + m 3m

Both mass have same magnitude of accleration but


(a,3a) r r
in opposite direction a1 = -a2 = a Let
2m 3m

(a,a) (3a,a) æ 3m - m ö g g
a cm = ç ´ =
r r r è 4m ÷ø 2 4
r m1 r1 + m2 r2 + m3 r3
rcm =
m1 + m2 + m3 12. Centre of mass is towards heavier mass and bottom
piece has more mass in comparison to the handle
m(aiˆ + 3aj)
ˆ + 2m(aiˆ + aj)
ˆ + 3m (3aiˆ + aj)
ˆ
= piece.
m + 2m + 3m

270 E
Pre-Medical : Physics
Let mass of circular plate be = M
12R
M
13. Mass per unit area =
p(28)2

Initial distance between their centers = 12 R M 9M


Mass of part removed = 2
´ p(21)2 =
p(28) 16

R 2R 7M
Mass of remaining part =
16

At time of collision the distance between their 7M 9M


Now ´x = ´7
centers = 3R 16 16
So total distance travelled by both=12R–3R = 9R x = 9 cm
Since the bodies move under mutual forces, center
The centre of mass of the remaining portion from
of mass will remain stationary so
the centre of plate is 9 cm.
m1x1 = m2x2 18. By COLM
mx = 5m(9R –x) r r r
m1 v1 + m2 v 2 + m 3 v 3 = 0
x = 45R – 5x 1 1 1
6x = 45R m1 = kg , m2 = kg , m3 = kg
4 4 2
also v1 = 30iˆ m/s , v 2 = 30jˆ m/s
45R
x= r r
6 r æ m v + m2 v 2 ö
v3 = - ç 1 1 ÷ v 3 = 15 2 m/s
è m3 ø
x = 7.5 R 19. By COLM
(M - m)u
Mu = mu + Mv Þ v =
m1 x1 - m2 x2 M
14. x CM =
m1 - m2 20. M1 × 8v = M2v
4 3 4 æ 4 3 3ö
m1 =
3
pR r , m2 = pa 3 r
3 M2 = 8M1 çQ M = 3 pr r,M µ r ÷
è ø
x1 = 0, x2 = b
- ba 3 r1 1
putting value x CM = \ r2 = 2r1 Þ =
R 3 - a3 r2 2
4m ´ 0 - m a/2 ma a 21. Let plank shifted by x then
15. YCM = =- =- DxCM = 0, as there is no external force on the system.
4m - m 2 ( 3m ) 6
(M + m) xplank – (m) (L – xplank) = 0
m1 x1 - m2 x2 pR 2
16. x CM = , m = pR 2
r , m = r mL
m1 - m2 1 2
4 xplank =
M+m
R
also x1 = 0, x 2 = - 22. Let plank moved by x in right side.
2
R but Dxcm = 0
putting value x CM = 150(x) + 50(10 + x) = 100 (10 – x)
6
3x + (10 + x) = 20 – 2x
5
17. 6x = 10 Þ x = m
3
28cm
1 1 2
21cm 23. mv = MV ; MV2 = kd
2 2
O2 x 7cm
O O2
kd2 M kd2 d
\V= \v= = kM .
M m M m

E 271
Pre-Medical : Physics
24. mv = (M + m)vƒ (1 + 0.5 ) m -6
æ 1 - 0.5 ö
æ m ö

2m ÷ 6m + 2m
( )
vƒ = ç è ø
÷v
èm+Mø 0.5 ´ 6 1.5
= + ( -6 ) = 1.5 – 4.5 = –3 m/s
25. 2 2
32. Momontum is conserved in all collision.
33. By COLM
5m mu + 0 = mv1 + mv2 ..... (1)

2kg v1 v 2 - v1
0.1 m e= ..... (2)
u-0
10gm v2
2kg Þ v2 – v1 = eu ..... (3)
500 m/s from (1)
Applying momentum conservation u = v1 + v2 ..... (4)
putting (4) in (1)
10 10
´ 500 + 0 = 2 ´ v 1 + ´ v2
1000 1000 v2 – v1 = e(v1 + v2)
Þ v2 – ev2 = v1 + ev1
Þ 5 = 2v1 + 0.01v2 ......(1)
Applying work energy theorem for block v1 1 - e
=
W = DKE v2 1 + e

1 34. Let a ball fall from a height (h) and let it touch the
Þ 2 × 10 × 0.1 = × 2 × v12 ground with a velocity v taking time (t) to reach the
2
ground.
Þ v1 = 2 = 1.4 m/s Let v1, v2, v3....... be the velocities immediately
Putting the value of v1 in equation (1) after first, second, third......collisions with the ground.
5 = 2 × 1.4 + 0.01 v2 Þ v2 = 220 m/s Height Attained by the Ball After the 'n'th Rebound
m v - m2 v 2 10 ´ 10 - 20 ´ 15
26. V= 1 1 =
m1 + m 2 10 + 20 v1 = ev Þ 2gh1 = e 2gh Þ h1 = e2h,
100 - 300 200 20
= =- =- = -6.6 m/s
30 30 3 v2 = e2v Þ 2gh2 = e2 2gh , h2 = e4h.
27. Velocities get inter changed.
u1 = 6 m/s ; u2 = 0
\ v1 = 0 ; v2 = 6 m/s Similarly hn = e2n h
28. Since both bodies are identical and collision is elastic.
Therefore velocities will be interchanged after
collision. 35. 2
eh 4
vA = –3 m/s and vB = 5 m/s h eh

16
29. e2 = Þ e = 4/5
25
30. if 60% is lost
Total E = mgh Total distance = h + 2e2h + 2e4h + ....
40
Remaining E = mgh = h + 2e2h(1 + e2 + e4+...)
100
40
So mgH = mgh Þ H = 4 m 2 1
100 = h + 2e h ´
31. u1 = 6 m/s, u2 = –6 m/s, e = 0.5 1 - e2

v1 =
m1 - em2
u1 +
(1 + e ) m2 u
2
m1 + m2 m1 + m2 é 2e2 ù æ 1 + e2 ö
= h ê 1 + 2ú
= çç 2 ÷
÷
h
ëê 1 - e ûú è 1 - e ø

272 E
Pre-Medical : Physics
36. According to the law of conservation of momentum 40. Case-I
r r r r
m1 u1 + m2 u2 = m1 v1 + m2 v 2 ......(i)
m2 m1 v
r r 2v m1
Here u1 , u2 = Initial velocities of m1 and m2 px
B m2 q
r r A
v 1 , v 2 = Final velocities of m1 and m2 before after
r r py
Change in velocity of m1 = v 1 - u1
r r
Change in velocity of m2 = v 2 - u2 px = m2 2v
r r r r p y = m2 v
m1 v1 - m1 u1 = -m2 v 2 + m2 u 2

r r r r
or m1 ( v1 - u1 ) = m2 ( - v 2 + u2 ) py 1
\ tan q = =-
r r px 2
v1 - u1 m2
or - vr + ur = m
2 2 1 Case-II
changes in velocity of m1 m 2
\ changes in velocity of m = m py
2 1

37. It is not possible that after collision one ball moves m2 m1


similarly 2v m2 q px
along the original line of motion while the other ball m1
B A
moves along some angle (a) with original line.The v after
px = m2v before
momentum perpendicular to original line of motion
cannot be conserved in this situation. p y = - m2 v
Initial momentum along perpendicular direction =
zero py 1
\ tan q = =
px 2
Final momentum along perpendicualr direction
= m2v2 sina. Hence momentum is not conserved. 41. By COLM
Hence the situation is physically impossible. v
m
hf 1.8 3 9m/s 30°
e= = = 0.6 = m m
38. hi 5 5 30°
rest
m
v
3 2
loss in velocity = 1 - = m × 9 = 2[mv cos30°]
5 5
v = 3 3m/s
39. Max. compression happen when both have same
speed
\ 2 × 10 + 5 × 3 = 7 × v

35
\ v= = 5 m/s
7

Energy conservation

1 1 1 1
´ 2 ´ 102 + ´ 5 ´ 32 = ´ 7 ´ 52 + 1120x2
2 2 2 2
Þ x2 = 0.0625
\ x = 0.25 m
E 273
Pre-Medical : Physics
ROTATIONAL MOTION
X
ML2 MR 2 MR2 L2 R 2 10.
1. + = Þ = ÞL= 3 R 1
12 4 2 12 4

MR 2 2 3
2. I = Idisc + IMasses = + 4mR2 r r
2
3. As disc is lying in the x-z plane, so applying X'
perpendicular axis theorem. Ix + Iz = Iy Ixx' = I1 + I2 + I3
80 + Iz = 100 Þ Iz = 100 – 80 = 20 kg m2 2 2 æ2 2 2ö æ2 2 2 ö
4. as I = MR2 or I a R2 = 3 mr + ç 3 mr + mr ÷ + ç 3 mr + mr ÷
è ø è ø
so graph between I and R will be a parabola. Þ Ixx' = 2mr2 + 2mr2 = 4mr2
5. I = m(a) + m(a) + m(0) + m(2a) Þ I = 6ma
2 2 2 2 2
11. given Isolid sphere = Ihollow sphere
6. I = MK2 Þ I = 10(0.2)2 = 0.4 kg-m2
2 2 2 2 r12 5
2 Þ Mr1 = Mr2 Þ 2 =
7. I2 = ml 5 3 r2 3
12
90° r1
Þ = 5: 3
r2
2
I1 = ml
12
12.
D P
M
ml2 ml2 ml2
So I = I1 + I2 = + = R
12 12 6 O O'

8. l/2
B
D' Q

Applying parallel axis theorem


A l IPQ = IC + Md2
C
MR 2 3
2 Þ IPQ = + M(R)2 = MR2
ælö 5 2 2 2
I = IA + IB + IC Þ I = 0 + m ç ÷ + m l = m l
2
è2ø 4 13. The moment of inertia is minimum about FH because
mass distribution is at minimum distance from FH.
9. A E D

A F H
l/2 l O

B G C
1 2 1
14. as, K= Iw = (I ´ w) ´ w
Moment of inertia of rad A about given axis, IA 2 2
so, IA = IC + Md2 1 2K
Þ K = Lw or L=
2 w
2
Ml2 ælö Ml2 Ml2
IA = + Mç ÷ = + L 2 K 2 w1 1 1
12 è2ø 12 4 so, = ´ = ´
L1 K1 w2 2 2
L2 1 L
Ml2 4 Þ = or L2 =
Þ IA = so, I = 4 ´ I A = Ml2 L1 4 4
3 3
274 E
Pre-Medical : Physics
20. The ladder touches the wall at A and the normal
w0
reaction is denoted by R. The whole system is in
15. equilibrium.Take moments of forces about B.
x P A
m1 m2 R
(L-x)

The position of point P on rod through which the Wall


G 6m
axis should pass so that the work required to set the
S
rod rotating with minimum angular velocity w0 is their
centre of mass 500 N
B F 4m
m 2L Ground
so m1x = m2 (L–x) Þ x = m + m
1 2 4
R × 6 = 500 ×
r
Torque rt = rr ´ F
3
17.
500 ´ 4 1000
or R= = or R = 111 N.
ˆi ˆj kˆ 6´3 9
r
\ t = -2 0.5 1 21. Angular acceleration will be more if M.I will be less.
2 0 -3 r r
( t = Ia )

r ˆ ˆ - 1]
t = i [ -1.5 - 0] - ˆj[6 - 2] + k[0 Change in angular momentum
22. Torque =
time
r
(
t = -1.5iˆ - 4jˆ - kˆ N - m ) t=
L 2 - L1 4A 0 - A0
=
t 4
18. 3A 0
Þ t=
4
l/2
\ \\\\\\\\\\\\\
23.
A
mg

q
l ml2
tA = mg ´
\\\\\\\\\ \\\\\\\
Torque, and IA =
2 3
as, tA = IA × a Increasing angle causes equilibrium.
24. (i) Given, the centre of mass is closer is closer to
mgl
tA 3g B than A
or a= = 22 Þ a=
IA ml 2l \ rA > rB
3 r
Torque rt = rr ´ F A B
19. Here a = 2 revolutions/s2 = 4p rad/s2 rA rB
For rotational equilibrium
1 1 25 tA = tB
I= MR2 = (50) (0.5)2 = Kg-m2 Mg
2 2 4 TArA = TBrB
As t = Ia so TR = Ia
TA rB
(ii) TArA = TBrB Þ =
æ 25 ö TB rA
Ia çè 4 ÷ø
( 4p ) \ rA > rB Þ TB > TA
Þ T= = N = 50p N = 157 N
R (0.5) 25. For same tension in cable, more counter torque
will be provided in (A) Þ Pattern A is more sturdy

E 275
Pre-Medical : Physics

60 31. Before After


26. f = 60 rpm = rps = 1 rps; I = 2 kg-m2 w w'
60

w = w0 + at Þ 0 = 2p + a(60)

2p p Applying conservation of angular momentum :-


a= - =- \ (Retardation)
60 30 I1w1 = I2w'
Mr2 × w = (Mr2 + 4mr2)w'
p p
t = Ia Þ t =2´ = Mr2 w Mw
30 15 so, w' = =
r 2 (M + 4m) M + 4m
27. By torque balancing about B 32. As the angular momentum is conserved
NA(d) = W(d–x) \ I1w1 = I2w2 or mr23w = m(r/2)2w2
w2 = 12w.
W(d - x)
Þ NA =
d 33. v =1m/s (w.r.t. ground)

NA NB
w0

A B
x
d–x
W MI of platform = 200 kg-m2,
MI of man = mR2 = 200 kg-m2
28. Particle at periphery will have both radial and
For system (platform + man) by using COAM
tangential acceleration
at = Ra = 0.5 × 2 = 1 m/s2 50 ´ 1 ´ 2 1
Iw0 = mvR Þ w0 = = rad/s
200 2
w = w0 + at = 0 + 2 × 2 = 4 rad/sec
Angular velocity of man w.r.t. platform
ac = w2R = (4)2 × 0.5 = 16 × 0.5 = 8 m/s2
v 1 1
= + w0 = + = 1 rad/s
atotal = a 2p + a c2 = 12 + 82 » 8 m/s2 R 2 2
Time taken by the man to complete one revolution
*In this question we have assumed the point
2p rad
to be located at periphery of the disc. = = 2p s
1 rad / s
29. t = Ia
r é v 2 sin2 45° ù
RF = mR2a 34. L = mvr^ = m(vcos45°) ê ú
êë 2g úû
30 N mv 3
=
4 2g

35. J = Iw where ‘I’ is constant. So graph will be straight line.


F 30 mv 2
a= = = 20 rad/s 2 36. Centripetal force, FC = ...(i)
mR 50 r

100 and angular momentum, L = mvr
L
2p so, v=
30. w1 = = p rad/s. mr
2 puting value of v in (i),
By conservation of Angular momentum m æ L ö
2
FC = ´
I1w1 = I2w2 r çè mr ÷ø

p m L2 L2
Þ 100 p = [100 + 50(2) ]w Þ w =
2 Þ FC = ´ 2 2 =
3 r m r mr 3
276 E
Pre-Medical : Physics
37. On loosing atmosphere, moment of inertia of earth 1 mL2
so, K= ´ ´ (4p2 ´ n2 )
decreases so, w increases and time period T 2 3
decreases. 2
Þ K = mL2 p2 n2
3
ì 2p ü
íQ T = ý
î wþ L2A L2B
44. KA = KB Þ =
2I A 2IB

A B As IB > IA So, L2A < L2B Þ LA < LB


38.
1 2 1 2
45. Esphere = Is w = ´ MR2 ´ w2
2 2 5
Initially as ant move towards axis I decreases so w
increases. 1 1 MR 2
Edisc = I(2w)2 = ´ ´ 4w2
After crossing axis I increases and w decreases. 2 2 2

so, first w increases then w decreases. E sphere 1


=
39. Applying conservation of angular momentum :- E disc 5
I1w
I1w = (I1 + I2)w' so, w' = w1 + w2
(I1 + I2 ) 46. COAM : Iw1 + Iw2 = 2Iw Þ w =
2
40. As the string is pulled downwards, tension in the
string, always pass through point O. 1 2 1 2
(K.E.)i = Iw1 + Iw2
So, torque of tension about O will be zero and 2 2
angular momentum of object about O will remain
2
constant. 1 æ w + w2 ö
(K.E.) f = ´ 2Iw2 = I ç 1 ÷
L2 O r 2 è 2 ø
also, K =
2I
I
1 1 Loss in K.E. = (K.E.)i – (K.E)f = ( w1 - w2 )2
Þ Kµ µ { \ L is constant} 4
I mr 2
1 2
K 47. W = loss in KE = Iw µ I
so, if r becomes 2r then K becomes 2
4

dL 2
41. text = 0 Þ = 0 Þ L = constant IA = MR 2 = 0.4MR 2
dt 5

1
IB = MR 2 = 0.5MR 2
42. Rotational kinetic energy of a body, 2
1 2 IC = MR2
Krot = Iw
2 \ WC > WB > WA
2K rot
or, Iw2 = 2Krot or w = ....(i) K2
I
K Rotation R2 K2 1
Angular momentum of a body L = Iw 48. = For disc, = ,
K Total K2 R 2
2
1+ 2
2K rot R
or, L = I = 2IK rot [From (i)]
I
43. Frequency of rotation = n Hz 1
so, w = 2pn so, fraction = 2 =1 : 3
1
1 2 1+
and kinetic energy, K= Iw 2
2

E 277
Pre-Medical : Physics
49. Different parts of the rolloing wheel move with the 54. Disc
same linear and angular speed. The magnitude of
the linear velocity of the points at the extermities
of the horizontal diameter of the wheel is equal to L h
5 2 ms–1
q

Applying conservation of energy,


5 m/s
1 æ K2 ö
mgh = mv 2 çç 1 + 2 ÷÷
2 è R ø
5Ö2 m/s
5 m/s
K2 1
also, h = L sinq and 2
=
R 2
1 2h æ K2 ö
50. As we know, t = çç 1 + 2 ÷÷ 1 æ 1ö
sin q g è R ø so, mgL sin q = mv 2 ç 1 + ÷
2 è 2ø

K2 2 K2 1 4gL sin q
For sphere, = & For disc, = v=
R 2
5 R 2
2 on solving,
3

1 2h æ 2ö 2L
1+ ÷ 55.
t sphere sin q g çè 5ø 7 t1 = time for sliding =
as
so, = = 5 = 14 : 15
t disc 1 2h æ 1ö 3
1 + 2
sin q g çè 2 ÷ø 2L
t2 = time for rolling =
aR
51. vµ a
Þ t1 : t2 = aR : as
v a g sin q g sin q
= & aR = =
vR aR I 2 mg sin q g sin q 1
1+ aR = = = as
mR 2 I 2 2
m+ 2
R
v
a = g sinq Þ vR = Þ t1 : t2 = 1 : 2
2
56. Solid cylinder
1 2
52. Translational KE = mv = E
2

æ h
1 K2 ö
Total KE = mv
2
çç 1 + 2 ÷÷
2 è R ø
v
1 2
= mv (1 + 1) = 2E Applying conservation of energy :-
2
mgh = KTotal
K Rotation
53. Given, ´ 100 = 50% 1 æ K2 ö
K Total so mgh = mv 2 ç 1 + 2 ÷
2 ç R ÷ø
è
K2 K2
1 æ 1ö
R 2 ´ 100 = 50 Þ R2 = 1 mgh = mv 2 ç 1 + ÷ on solving, v = 4 gh
Þ 2 è 2ø 3
K2 K2 2
1+ 2 1+ 2
R R 57. For rolling motion on an inclined plain acceleration

g sin q
K2 is given by a R =
so, = 1, so it is a ring. æ K2 ö
R2 ç1 + 2 ÷
è R ø
278 E
Pre-Medical : Physics
58. Applying conservation of energy.
1 æ K2 ö
mv 2 çç 1 + 2 ÷÷ = mgh
2 è R ø
1 2æ 1ö ìï K2 1 üï
Þ mv ç 1 + ÷ = mgh í Q for disc = ý
2 è 2 ø îï R 2 2 ïþ
3v 2
Þ h=
4g
59. At maximum compression of spring, total kinetic
energy of rolling cylinder will be converted into
potential energy of the spring.
1 æ K2 ö 1
so, mv 2 çç1 + 2 ÷÷ = kx2
2 è R ø 2
1 é 1ù 1
Þ 12 ´ (4)2 ê1 + ú = ´ 200 ´ x2
2 ë 2û 2
on solving, we get x = 1.2 m
g sin q
60. acceleration =
K2
1+ 2
R
K2 1
for disc ; = = 0.5
R2 2
K2 2
for sphere ; = = 0.4
R2 5
Þ a(sphere) > a(disc) \ sphere reaches first

E 279
Pre-Medical : Physics
GRAVITATION
3. More work is done against the gravitational pull of
G (1) (1) 6.67 ´ 10 -11 the earth.
1. F1 = F2 = = = 1.67 ´ 10 -9
( 0.2 )2 0.04
4. Action and reaction force are equal in magnitude.
y
2 M Earth
Moon
P
5.
x (D–x)
F2
M x At point 'p' for gravitational field to be zero
1 M F1 3
field due to earth = field due to moon
r
Fnet = F1 (ˆi ) + F2 (ˆj ) = F (ˆi + ˆj ) = 1.67 ´ 10 -9 (ˆi + ˆj ) N
GMe GMm 81Mm Mm
Þ 2
= 2 Þ 2
=
x (D - x) x (D - x)2
m 2m
x 9D
F1 F2 Þ = 9 Þ 9(D – x) = x Þ x =
D-x 10
2.
m
GMe
F4 F3 6. g=
R2e
4m 3m
G(0.1)Me 0.4GMe
g mars = = = 0.4 g
(0.5) 2 R2e R 2e
Gmm Gm2m
F1 = , F2 =
(a / 2 ) (a / 2 )
2 2

2h 2h 2h
7. t= =1 sec; t ' = = 6 = 6 sec
g g' g
Gm3m Gm4m
F3 = , F4 =
(a / 2 ) (a / 2 )
2 2

g
8. gheight = g/4 = 2
So resultant of forces will be : æ hö
çè 1 + R ÷ø
e

Þ h = Re = 6400 km
90°
9. Weight is due to gravitational pull of earth which
F' = F4–F2 F'' = F3 –F1 1
varies with distance as . Weight becomes zero
r2
Gm4m Gm2m Gm3m Gmm when gravitational pull of both planets become equal
F'= - , F''= -
(a / 2 ) ( a / 2 ) (a / 2 ) ( a / 2 )
2 2 2 2
& opposite After that its weight again increase to
240 N due to mars.

2Gm2 2Gm2
2h ö æ 32 ´ 2 ö
F' =
a2 /2
, F'' =
a2 /2 10. geff = g æç 1 - = g ç1 - = g(1 – 0.01)
è R ø÷
è 6400 ÷ø

4Gm2 4Gm2 geff = 0.99g ms–2


F' = , F'' =
a2 a2 11. Inside the earth g µ r and outside the earth
Now they are at 90°
1

Gm2 r2
So resultant force = 4 2 2
a
280 E
Pre-Medical : Physics

GME Dg d 1
12. g= = D gh
R 2E g 2
where ME and RE is the mass and radius of the earth
respectively. 18. S2 is correct because whatever be the g, the same
force is acting on both the pans. Using a spring
g 2
ME = RE balance, the value of g is greater at the pole.
G
Therefore mg at the pole is greater. S4 is correct.
S2 and S4 are correct.
g
13. As g' = 2
æ hö GM GM p 4GMp
ç1 + R ÷ 19. g= = =
è ø R 2 2
D2p
2
æ Dp ö
g g æ hö çè 2 ÷ø
= Þ ç1 + R ÷ = 2
2 æ hö
2 è ø
ç1 + R ÷
è ø
æ GM ö
20. g = ç 3e ÷ r for 0 < r £ Re Þ g µ r
è Re ø
Þ
h
R
= 2 -1 Þ h =R ( 2 -1 )
GMe 1
g= for r ³ Re Þ gµ
14. g¢ = g – Rew2cos2l Þ g' < g r2 r2
except for l = 90° i.e. for poles.
ms
21. m's = & G' = 6 G
15. There is no atmosphere on the moon, means no 6
buoyant force in upward direction so it will fall down
G 'ME 6GM E
with acceleration g/6. gE = 2
= = 6g
R R2

GM \ Raindrops will fall faster, Walking on the ground


16. As g= 2 would become more difficult, Time period of a
R
simple pendulum on the earth would decrease.
Dg -2DR
%= %
g R GM GM
22. Ig = – , V=- ,
R 2
R
Dg
% = -2(-1)% = +2%
g V=IgR=6× 8 × 106 = 4.8 × 107

17. m decreased by 1% for h height GM GM 3GM


23. V=- - =-
-2 (a / 2) a a
é hù
gh = g ê1 + ú
ë Rû
M
Dgh -2h M
=
g R a/2

for depth.

é dù 24. Gravitational potential


gd = g ê1 - ú
ë Rû -GM
V= for r > R
r
Dgd - d -GM
= (here d = h) V= for r Î [0, R]
g R R
Potential remains constant inside the hollow sphere.
Dgh 2Dgd
=
g g

E 281
Pre-Medical : Physics
–GM
25. V= = –5.4 × 107..... (1) 2GMe
R+h 31. Escape velocity v = is independent of
R
GM angle at projection
and g = =6 ..... (2)
(R + h)2
dividing (1) and (2) M
32. As v esc µ
R
5.4 ´ 107
Þ R +h =
6 v2 M2 R1
Hence =
Þ R + h = 9000 km so h = 2600 km v1 M1 R2

GM em GM m m G(81M m )m GM m m
26. U= - - =- - v2 1 1
R r R r = 100 = 10 ´ = 5
v1 4 2
æ 81 1ö
= -GmM m ç + v2 = 11.2 × 5 = 56 km/sec
è R r ÷ø
27. To escape from the earth total energy of the body GMe m 1 GMe m
33. - + mK 2 Ve2 = -
should be zero KE+ PE = 0 R 2 r

1 GMm 1 2 2 æ 1 1ö
mv 2 - = 0 Þ KE = mgR e 2
K Ve = GMe ç - ÷
èR rø
2 5R e min
5

2GMe
28. Vescape = 2gR but Ve =
R
V1 g1R1
1 2 2GMe æ 1 1ö
V2 = g2 R 2
= pq K = GMe ç - ÷
2 R è R rø

2GM K2 1 1 1 1 k 2 1 - k2 R
29. Escape velocity = = c = speed of light = - Þ = - = Þr =
R R R r r R R R 1 - K2

2GM 2 ´ 6.6 ´ 10-11 ´ 5.98 ´ 1024 2GM 2G æ 4 3 ö


R= = = 10–2 m 34. Escape Velocity = = . pR r ÷ µ R r
c2 (3 ´ 108 )2 R R çè 3 ø

30. when particle falls from infinity


ve
PEi + KEi = PEf + KEf \ Ratio, =1: 3 3
vp
-GMm 1
0+0= + mv12 1 C
R 2 35. Fµ m ;
F= m
r r
2GM This force will provide the required centripetal force
v1 =
R Therefore
when particle falls from a height
C C
PE1 + KEi = PEf + kEf mw2 r = m ;
w2 =
r mr m +1
2p
GMm GMm 1 T= Þ T µ r ( m +1) /2
- +0=– + mv22 w
11R R 2
GMm
10 2GM 36. Q total mechanical energy E = – 2r
´ = v2
11 R E1 m1 r2 3 4r 12
v1 11 \ E = m × r = × =
= 2 2 1 1 r 1
v2 10

282 E
Pre-Medical : Physics
GMm æ 1 1 ö GMm
37. W = E2 – E1 = ç - = v 0 = 60 ´ 106 m/s
2 è 2R 3R ÷ø 12R
v0 = 7.76 × 103 m/s = 7.76 km/s
38. Because its period is equal to the period of rotation
43. The gravitational force on projectile of mass m at
of the earth about its axis so it will remain in the
a distance r from a planet of mass 4M is zero
same place with respect to the earth.

GMm GMm r 6R–r


39. TE1 = – =-
2(R + R) 4R
R m R
A
GMm GMm
TE2 = – =- 4M
2(R + 7R) 16R 6R 9M

TE1 4
= G(4M)m G(9M)m
TE2 1 \ =
r2 ( 6R - r )2

or 4(6R–r)2 = 9r2 or 2(6R – r) = 3r or r = 2.4 R.


GMm
-
KE1 4R 4
= = 44. First law of motion i.e. law of inertia.
KE2 GMm 1
- 45. As T2 µ r3
16R
3 3
T2 æ r2 ö
2
æ 9R ö 2
=ç ÷ =ç ÷ = 27
GMm T1 è r1 ø è R ø
40. K.E. = Þ Kinetic energies are unequal
2r
R
T2 = 27 ´ 2p
g
2pr 3/ 2
T= Þ Time period are equal 46. For system to be bounded
GM
Net energy i.e. (KE + PE) must be negative

P.E. = –
GMm
Þ Potential energies are unequal 47. T = 2p
r3
Þ T = 2p
( R + h )3
r GM GM
1/ 3
é GMT2 ù
GM Þ h=ê 2 ú
-R
v= Þ Orbital speeds are equal ë 4p û
r
4 p2 3
48. T2 = r
GM
GM
41. Speed of satellite V = 2/ 3
r æ T1 ö r1
çè T ÷ø = r [\ T1 = T2]
2 2

VB rA 4R r1
Þ = = = 2 Þ VB = (3V)(2) = 6V
VA rB R Þ r =1
2

42. For the satellite revolving around earth GM


49. Orbital velocity of satellite, n = = n0
R
GMe GMe g Re
v0 = = = GM 2GM 2
(Re + h) æ hö h = = n0
R e ç1 + ÷ 1+ R 3R 3 .
è Re ø Re \ n' = R+
2

substituting the values

E 283
Pre-Medical : Physics
54. For satellite S moving elliptical orbit around the earth
GMe m
50. Total energy = - net force will be towards centre of the earth.
2(R + h)
(like centripetal force in circular motion)
GM e g0 R 2 55. Apply angular momentum conservation law
Q g0 = Þ Me = mv1r1 = mv2r2
R2 G
Þ 60 × 1.6 × 1012 = v2 × 8 × 1012
mg0 R2 v2 = 12 m/sec
\ Energy = -
2(R + h) 56. Since the angular momentum of the satellite about
the earth is conserved.
4p2 3 r r
51. T2 = a
GM \ L a = LP
3
4p2 æ rp + rA ö p2 58. vB
T =
2
GM
çè
2
÷ø =
2GM
( rp + rA )
3
rB
vC
rA
mvr = constant Þ vr = constant rC
S
\ vArA = vprp
\ rA > rp \ vA < vp vA
52. According to Kepler's law As ^ = mvr = constant
and rC > rB > rA
A SCD A SAB 2A SAB A SAB so vA > vB > vC
Þ = Þ = Þ t1 = 2t2
t1 t2 t1 t2

53. From conservation o f angular momentum

v1 r2
mv1r1 = mv2r2 Þ =
v2 r1

284 E
Pre-Medical : Physics
PROPERTIES OF MATTER & FLUID MECHANICS

Breaking stress 15.8 ´ 108 8. Potential energy per unit volume is


1. l max = = = 20 km
rg 7.9 ´ 103 ´ 10
1 (Stress)2 U1 F12 A22 r24
mv 2 = ´ Þ U = 2 ´ 2 = 4 = 16 : 1
2. Maximum force in the string is Fmax =mg + 2 Y 2 A1 F2 r1
l
Q v = 2gl , Fmax = 3 mg F YDL
9. =
A L
Fl 3mgl 3 ´ 40 ´ 5
Dl = = = = 1.5 mm YA
YA YA 2 ´ 1011 ´ 2 ´ 10 -6 F= × DL
L
1
3. Energy U = (stress)2 volume F
2Y µA
DL
F 40
Stress = = = 105 N/m2, more the slope, more will be area.
A 4 ´ 10-4
Volume = Al
F
1
A Fl
U= ´ (105 )2 ´ 4 ´ 10 -4 ´ 0.2 = 4 µJ 10. Y= Þ Dl =
2 ´ 1011 Dl AY
4. Dl µ l2 l
V
7mm /1m 1 But V = Al so A = (V = volume)
5. Slope = 2
= Þ Y = 1 ´ 109 N / m2 l
7000kN / m Y
Fl 2
L Therefore Dl = µ l2
6. Total weight at height from its lower end = WT VY
4
L Fl V
WT = weight suspended + weight of of wire 11. Y= Q V = Al so l =
4 ADl A

W YADl YA2 Dl
WT = W1 + So F = = µ A2
4 l V
F W
Stress = = T 2
Area S F1 æ A1 ö
=
F2 çè A 2 ÷ø
æ Wö
ç W1 + 4 ÷ 2
è ø 2F æ A ö 1
Stress = Þ =ç ÷ =
S F2 è 2A ø 4

Þ F2 = 8F
7. From graph

F æ DV ö 0.004
Slope = 12. ç- V ÷ =
Dl è ø 100

FL
also Y = æ DV ö 0.004
ADl DP = K × ç - ÷ = 2100 × 106 ×
è V ø 100

F A = 84 kPa
Slope = =Y
Dl L
Volume Stress
13. Bulk modulus K =
Þ more slope more Y Volume Strain

So YA > YB rgh 103 ´ 9.8 ´ 400


K= = = 39.2 ´ 109 N / m2
DV / V 0.01 /100

E 285
Pre-Medical : Physics
(Fb)w = (Fb)oil
14. DP , DV 3DR
B= = rwVdg = roilVd g
DV V R
-
V V V 4
(1) g = roil g Þ roil = gm / cc
3 4 3
DP DR P
B= Þ - = (DP = P) 21. Due to upthrust on the block, its apparent wt. will
- 3D R R 3B
reduce & hence the reading of spring
R
balance A will be less then 2 kg. But since the liquid
15. Y = 2h (1 + s)
in the beaker exerts buoyant force on the block, the
2.4h = 2h (1 + s)
s = 0.2 block will also exert reaction force on the liquid in
the downward direction and the reading of balance
FA PA A A rA gh A A A
16. = = =1 B will increase due to this additional force.
FB PB A B rB ghB A B
22. Liquid with low density float over high density.
17. Hydraulic lift, hydraulic jack, hydraulic press works
on Pascal's law. 23. When coin is removed the downward force an block
decrease, which will reduce the volume submerged
18. The pressure at any level is the same in both arms
also the removing volume will filled by water so h ¯.
of the U-tube. That is the pressure at the interface
sides are the same. 24. WAPP = W – FB (FB = force of buoyant)
i.e. P1 = P2 WAPP = Vrg – Vr1g
or roil g(d + l) = rwater gl
é r ù
= Vrg ê1 - 1 ú
roil ´ 9.8 ´ (0.0125 + 0.135) = 1000 ´ 9.8 ´ 0.135 ë rû

1000 ´ 9.8 ´ 0.135 é r1 ù


\ roil = WAPP = W ê1 - ú
9.8 ´ (0.0125 + 0.135) ë rû
135
= = 915 kg/m3 25. For wooden block rBlock < rWater
0.1475
Fb So FB > Mg, hence it will rise up with an constant
acceleration.
x 26. In floating condition
W = Th
(10–x)
19. Let V1 volume immersed in lower liquid then
Vrg = (V–V1)r1g + V1r2g
V(r – r1) = V1(r2 – r1)
mg
Fb = mg V1 r - r1 r1 - r
= =
rLVdg = rBVBg V r2 - r1 r1 - r2
(1) (10) (10) (10 – x) = 800 27. P1 = P2
x = 2 cm
F1 F 10 F
Outside volume = 2 × 10 × 10 = 200 cm3 = 2 Þ 2 = 22
A1 A 2 pr1 pr2
(Fb)w (Fb)oil F2 = 90 N
28. Writing equation of continuity for the tube & the
Water Oil holes
20.
r Atube vtube = nAhole vhole
mg mg
Vd = V/3 Vd = V/4 vR 2
pR2 × v = npr2v' r v' =
nr 2
286 E
Pre-Medical : Physics
R Shear stress F / A mgl
29. 39. h = = =
Strain Rate V / l AV

Av = constant l is thickness of film

VR 2 0.01 ´ 9.8 ´ 0.3 ´ 10 -3


pR V = npr v1 Þ v 1 =
2 2 h= = 3.45 ´ 10-3 Pa - s
nr2 0.1 ´ 0.085
30. Rate of flow = Av = same everywhere in pipe
v hAv
40. Fv = hA Þ l=
l Fv
3800 ´ 10 -3 m 3 1
Þ v= ´
60s p(6.4 ´ 10 -2 ) 2 0.01 ´ 10 -1 ´ 1 ´ 4
l= = 2 mm
2
Þ v = 5 m/s
2 r 2 rg
41. vT = (r of air neglect)
9 h
31.
u1 = 5 m/s 2 r 2rg 2 (10 -1 )2 ´ 1.47 ´ 980
h= = ´
h = 3.75 m A1 = 10–4 m2 9 vT 9 2.1
u2
= 1.52 poise

u22 = u12 + 2gh 43. Final velocity is terminal velocity which is


independent from 'h'
2
u = 25 + 2 ´ 10 ´ 3.75
2
2r 2 (r - s )
vT = g
u2 = 10 m/s 9 h
A1u1 = A 2u2 Þ A 2 = 5 ´ 10-5 m2 2r 2 (r - s )
44. vT = g
9 h
1
32. F = ( DP)A =
2
(
r v 22 - v12 A ) vT = 0.2 m/s for gold
r = 19.5 kg/m3 (gold)
1
(
F = ´ 1.2 (40)2 - 0 ´ 250
2
) s = 1.5 kg /m3
r2
= 2.4 ´ 10 N upwards
5 From above data we can find value of
n
33. AV = constant putting in 2nd case for silver
If A ¯ then speed ­ and pressure ¯ r2 = 10.5 kg/m3
1
34. F = DPA =
2
(
r v 22 - v 12 A ) v T = 0.1m / s

2 r 2 (r - s )
45. vT =
1
F = r éê ( ) - v 2 ùú A = 2rv 2 A
2
5v 9 h
2 ë û
v1 (r -s1 )h2
35. For maximum range of y = h =
v2 (r -s2 )h1
36. Volume flowing per second = Av
V1 [r - sgly ]hwater
v2 =
= A 2gh = 10-4 2 ´ 10 ´ 5 = 10–3 m3/s [r - swater ] hgly

v2 = 3.98 × 10–4 cm/s


37. Scent sprayer is based on Bernoulli's theorem.
38. A1v1 = A2v2 46. By using A1v1 = A2v2
we have 80(3) = 40(v)
L2 ´ 2gy = pR 2 ´ 2g4y
L Þ v = 6 m/s
R=
2p

E 287
Pre-Medical : Physics
47. 2Tl = W
4T
60. Pexcess =
W 1.5 ´ 10 -2 r
T= = = 0.0125 N / m
2l 2 ´ 0.6
4T 4T
rgh = Þ 103 × 10 × 4 × 10–3 =
49. Elastic membrane is formed on the surface of water r 16 ´ 10-3
due to surface tension. This help's spider & insects T = 160 dyne/cm
to move and run on the surface of water.
4T
61. P1– P2 =
50. Detergent decreases the oil-water surface tension r
and helps in removing dirty greasy stains. 4T 4T 4T
Þ – =
51. W.D. = 2 × 4pT r - r ( 2
2
2
1 ) r1 r2 r

If initial volume = V r1r2


Þr=
r2 - r1
then final volume = 8V then radius will become 2R
r1r2 4´5
So (
W.D. = 2 ´ 4 pT (2R) 2 - R 2 = 24 pR 2 T ) r = r -r =
2 1 5-4
cm. = 20 cm.

SE i (n)1 / 3 (2)1 / 3 62. Excess pressense inside a bubble is


52. = =
SE f 1 1
4T
53. W.D. = 2TDA PEx =
r
= 2 × 3 × 10–2 × (30 – 25) × 10–4
= 30 × 10–6 J 1
PEx µ
R2
54. D E = 4pR2 T n
1/ 3
(
-1 ) r
2 1 R1
r1 < r2
D2 (R2 > R1)
= 4p T [(27)1/3 –1] = 2p D2 T So, (P1 > P2)
4
55. W = TDA 63. For hemispherical
W = TA R=r

R
as DA = A i.e. cosq = 1 qC
r
qC
56. (Pex)1 = 2(Pex)2 q = 0° PC

4T æ 4T ö r1 1 as Rcosq = r r
R = cos q
= 2ç r ÷ Þ =
r1 è 2 ø r2 2 in capillary tube
3 3
V1 æ r1 ö æ1ö 1 2T cos q
V2
= ç ÷ = ç ÷ = 64. h=
r
è 2ø è2ø 8 rgr
57.
cos q
As r, h, T are same, = constant
58. For soap film we have two free surface r
W = T × 2DA
cos q1 cos q2 cos q3
= 3×10–2× 2[4 × 5 – 4 × 4] × 10–4 Þ = =
r1 r2 r3
= 24 × 10–6 J
As r1 > r2 > r3
59. W = T(2DA) { DA = (20 – 14) cm2 }
Þ cos q1 > cos q2 > cos q3 Þ q1 < q2 < q3
-4
W 1.5 ´ 10
Þ T= = = 0.125 Nm -1 As water rises so q must be acute
2DA 2 ´ 6 ´ 10-4
So, 0 £ q1 < q2 < q3 < p/2

288 E
Pre-Medical : Physics
69. Q geff = 0

Then height = 50 cm
65. 37° 60°
l1 l2 70. When water rise upto a height h then mass of liquid
rise m = (pr2h)r
h = l1cos37° h = l2cos60° Q total mass be located at centre of mass.
Height will remain same
l1cos37° = l2cos60° h h
then potential energy U = mg = (pr2Hr)g
1 2 2
l1 4 = l2
5 2
by Zurin law r × h = constant
l1 5
= \ U = constant
l2 8
71. mg = (2pr)Tcosq
2T cos qc
66. h = Q qC = 0°
rrg 6.28 × 10–4 = 2p(r) × (5× 10–2) × 1

2T 2 ´ 75 ´ 10-3 r = 2 × 10–13 m
r= = = 0.5 mm
rhg 103 ´ 0.03 ´ 10
67. m µ r
m1 r1
= Þ m2 = 2M
m2 r2

pP r 4
68. Rate flow Q =
8hl

2l
l 2r
r
P1
P2
In series combination rate flow is same
Q1 = Q2

pP1r 4 pP2 (2r)4


=
8hl 8h(2l)

P1 8
=
P2 1

E 289
Pre-Medical : Physics
THERMAL PHYSICS
1. Pyrometer is used to measure a temperature of
Fl Fl F
sun by using radiation 11. Y= = =
ADl AlDTa AaDT
5 5 C
2. C= F - ´ 32 1011 ´ 10-3 ´ l ´ 10 -6 ´ 100
9 9 F= = 104 N
F l
y = mx – C 12. We known that, when rod is freely expand,
a stress produce = 0
3. Slope of line AB

DC 100 - 0 100 5
= = = =
DF 212 - 32 180 9
4. For any temperature scale

Reading –ice point


= constant
Steam point–ice point stress a strain
Solve for the two thermometer. so strain = 0
x - MP °C
5. = DV
BP - MP 100 13. gV =
VDT
x - 39 39
r = r x = 117 °W From PT4 = constant & PV = mRT
239 - 39 100
DV DT DV 5
6. aRod > aFrame, then rod may touch ground. Þ V µ T5 Þ =5 Þ = = gV
V T VDT T
7. TFFT
8. Thermal expansion is like a photographic 1 æ1 ö V2
14. ´ ç mV 2 ÷ = msdq Þ dq =
enlargement. 2 è2 ø 4s

9. Dl = laDT 15. KE got converted into heat energy


m L
Dl Þ L = mgh or h =
a= 5 5g
lDT
1
16. KE = (10 ´ 10-3 ) (20)2 = 2J
1 2
a1 = = 10-2
1 ´ 100 Now mL = 50% of KE
1J
2 Þ m= Þ m = 0.003 gm
a2 = = 2 ´ 10-2 80 ´ 4.2 J / gm
100
mgh
3 17. = mL
a3 = = 4 ´ 10-2 2
1.5 ´ 50
2L 2 ´ 3.4 ´ 105
4 Þh = = = 68 km.
a4 = = 8 ´ 10-2 (maximum) g 10
2.5 ´ 20
18. Q = mLf + ms Dq + mLv
10. Dl 1 = Dl 2 = 1 × 80 + 1 × 1 × 100 + 1 × 536 = 716 cal
19. Heat required for vapourisation = Rate × time
l 1a 1 DT = l 2a2 DT = 42 × 103 × (30–20) = mL = 5 × L Þ L = 84 J/K
1
l 1a 1 = l 2 a2 20. Q1 = msDT = 10 × × 40 = 200 ® (S)
2
Q2 = mL = 10 × 80 ® (P)
l1 l1 / l2 a2 / a1 a2
Þ = = = Q3 = msDT = 10 × 1 × 100 = 1000 ® (Q)
l 1 + l 2 l 1 / l 2 + 1 a2 / a1 + 1 a1 + a2
Q4 = mL = 10 × 540 = 5400 ® (R)
290 E
Pre-Medical : Physics
Q msDT
21. P= ÞP= 1 1
t t 27. n= So, PV = RT
2 2
m
P = 2100 J/s ; = 20 g/s 28. According to ideal gas equation
t
rRT rRT
T1 = 10 °C T2 = ? P= Þ Mw =
Mw P
æ 2100 ö cal M A rA TA PB
ç ÷ = 20 × 1 (T2 – 10) = . . æ 1ö
è 4.2 ø s so MB rB TB PA = (1.5) (1) çè 2 ÷ø
T2 = 35 °C
MA 5
22. Let q be the final common temperature. Further, Þ M =4
B
let sc and sh be the average heat capacities of the
cold and hot (initially) bodies respectively (where sc P RT
29. = (Ideal gas equation)
< sh given) r Mw
From, principle of calorimetry,
heat lost = heat gained PM w P ´ (mN A ) Pm
Þ r = RT = kN T =
sh(100°C – q) = scq A kT

P1 V1 P2 V2 P1 T2
sh 100°C 30. = T Þ V2 = P ´ T ´ V1
\ q= ´ 100°C = T1 2 2 1
(s h + s c ) æ sc ö
çè 1 + s ÷ø 1 270
´
h V2 = × 500 m3 = 900 m3
0.5 300
Q sc / sh < 1 \ 1 + sc / sh < 2 31. Closed container r V = constant
DP DT P
100°C = r T= × DT
\ q> or q > 50°C P T DP
2
100
OR T= × 1 K = 250 K
0.4
Body at 100°C has more heat capacity then body M
at 0°C so final temperature must be greater than 32. PV = RT
MW
50°C.
V, MW,R are constant
23. Heat lost from steam = Heat gained by water
P1 P
m × 540 + m × 1 × 20 = 1400 × 1 (80 – 16) = 2
M1 T1 M2 T2
1400 ´ 64 M1 P1 T2 P ´ 300
Þ m= = 160gm = =
560 M2 P2 T1 (P / 2) ´ 330
24. At 1kg ice at –10°C + 4.4 kg of water at 30°C M1 600 20
Heat gain = Heat loss = =
M2 330 11
1 11 11 77
1000 ´ ´ 10 + 1000 ´ 80 + 1000 (T – 0) M2 = M1 = ´ 28 =
2 20 20 5
= 4.4 × 1000 × 1 × (30 – T) 77
Leaked amount = M1 – M2 = 28 –
Þ 5.4 T = 4.4 × 30 – 85 5
140 - 77 63
Þ T = 8.7°C = = g
5 5
25. 2kg ice at – 20°C + 5 kg water at 20°C
P1 P
Qgain = Qlost 33. V = constant r = 2
M1 T1 M2 T2
1 P2 M1 T1
2´ ´ 20 + M ´ 80 = 5 ´ 1 ´ 20 ´
2 Final mass M2 =
T2 P1
M = 1 kg
Water = 5 + 1 = 6 kg P 2 6g ´ 400
M2 = ´ =4g
26. No. of molecules will be same 300 P
as PV= nRT (all have same moles) Leak out mass = M1 – M2 = 6 – 4 = 2 g

E 291
Pre-Medical : Physics
34. ni = nf (final pressure will be common) 43. v1 = 2u, v2 = 10u, v3 = 11u (here, N=3)
P0 V0 P0 V0 PV0 PV0
+ = RT + RT v12 + v 22 + v 23
RT0 RT0 0 Find vrms =
N
2P0 T
Þ P = (T + T ) v1 + v 2 + v 3
0 vmean =
N
35. µi = µf µ ® moles
by solving rms speed exceeds the mean speed by
36. P2V = constant
2
about u.
æ nRT ö
ç ÷ V = constant Þ T2 µ V 44. Even power will not give zero value.
è V ø
So, V­ Þ T­ Þ dU = Å v12 + v 22 + v 23 + v 24 + v 25
45. v r.m.s =
mRT 5
37. PV = mRT Þ P=
V
22 + 32 + 42 + 5 2 + 6 2
mRT = = 4.24 m/s
PV 2/3
=C Þ ´ V2/3 = C 5
V
TV–1/3 = C Þ T µ V1/ 3 T
46. v rms µ
T­ Þ V­ M.W.

PV P (v rms )H = (v rms )O2


38. T=
mR
T 273 + 47
æ PV ö = Þ T = 20K
Pç ÷=C 2 32
è mR ø
1 T
P2V = C Þ P2 µ V 47. < V > µ
V MW
39. < VH > (M W )He 4
P B A = = =2
< VHe > (M W )H 1

T
48. v rms µ
C Mw
V
A ® B Þ P constant & T¯ So V¯ v' T ' Mw
than = ´
A ® B Þ P constant & T¯ So V¯ v Mw T

B ® C Þ T constant & P¯ So V­
2T M
C ® A Þ V constant & P­ So T­ v' = ´ w ´v
Mw 2 T
40. A ® B isobaric (P = constant) and V­ Þ T­
C ® D isochoric (P µ T) and P¯ Þ T¯ v' = 2v
P A B
v 500 5
C 49. vµ T Þ 200 = 400
Þ v = 200
4
m/s

D
T = 100 5

nRT 1 æ 1 ö
=ç P Þ slope µ
1 50. v = v xˆi + v yˆj + v z kˆ
41. P= or ÷
V V è nRT ø T

Þ T C > TB > T A 3KT


v=
m
1
42. P= ´ r ´ v 2rms
3 51. vrms > vavg > vmp

292 E
Pre-Medical : Physics
53. U = U1 + U2 62. For 1 mole of O2 gas
CP – CV = 32b = R
= m 1 C v1 T + m 2 C v 2 T For 1 mole of H2 gas
5 3 CP – CV = 2a = R
= 2´ RT + 4 ´ RT Þ 2a = 32b Þ a = 16b
2 2
= 5RT + 6RT = 11RT R 5R
63. = 0.4 Þ C V =
Cv 2
f
54. KE = NKT = same So, It is diatomic gas
2
64. 1+2
g=
1 f
NT = constant Þ Ta
N
µ1f1 + µ 2 f2
f f=
and KE = PV = same µ1 + f2
2
65. P = aV2 Þ PV–2 = constant
V = const. then pressure is also constant.
and T' = T/2 R
C = CV + ; x = –2
3 1- x
55. KE = PV
2 5R R 17R
ÞC= + =
KE 3 2 3 6
So energy density = = P
V 2 66. CV = f/2 R
56. Ui = Uf and f = 3 + 3 {Rotational + Translational}
Let final temperature is T then Þ CV = 3R
3 3 3 3 1
n1 kT1 + n2 kT2 + n3 kT3 = ( n1 + n2 + n3 ) kT fR + R
CP C V + R 2 f+2
2 2 2 2 67. g= = = =
CV CV 1 f
fR
n1 T1 + n2 T2 + n3 T3 2
hence T = n1 + n2 + n3 2
Þ gf – f = 2 Þ f=
1 g -1
58. Mean free path lm = 68.
x
PV = constant (Polytropic process)
2pd2n
where d = diameter of molecule Heat capacity in polytropic process is given by
n = molecular density
é R ù
1 êC = CV + 1 - x ú
Þ lm µ 2 ë û
r
5
1 Given that PV = constant Þ x = 5 ...(1)
59. Number of collision a Vrms a
MW 3
also gas is monoatomic so C V = R ...(2)
Number of collision by a molecule of He : O2 2
1 1
: = 2 2 :1 3 R 3 R 5
4 32 by formula R+ C= = R- = R
2 1-5 2 4 4
60. specific heat of an ideal gas is independent on
70. For polytropic process
temperature
mRDT
æ aT2 ö c W=
61. çP + ÷ V = RT + b 1- x
è V ø
Here x=–2
aT2 mR(T2 - T1 )
P+ = (RT + b)V - c
V W=
1 - ( -2)
P = (RT + b)V–c – aT2V–1 ........... (i) 1
P = AVm – BVn (given) .............(ii) = R(T2 - T1 )
3
By comparing
m = –c, n = –1
E 293
Pre-Medical : Physics
71. According to FLOT 80. for iaf DU = Q – W = 30 cal
for fi Q = DU + W
Q = W + DU Q =(–DUiaf) + W
Þ Q = –30 – 30 = –60 cal
Isometric V = Constant DV = 0 81. A®B
Q = DU Q = 200 J, W = 0, DU = 200 J
B®C
72. W= ò PdV = Area under P-V Q = 600 J, W = PB (VC–VA) = 240, DU = 360J
1 A®C
Þ W= ´ (4 + 2) × (5 – 1) × 10–3 J
2 DU = DUA®B + DUB®C Þ DU = 560 J
{1m 3 = 1000 L} 82. Path acb
W = 12 × 10–3 J
DU = Q – W = 200 – 80
73. W = Area of ellipse = pab
DU = 120 J
æ P2 - P1 ö æ V2 - V1 ö Path adb
= pç ÷ç ÷
è 2 øè 2 ø W = Q – DU = 144 – 120
74. = 24 J
P
3P 0
C B 83. P = const.
w æ1 -1ö
2P 0 O =ç ÷ ´ 100%
Q è g ø
P0 D
A 84. For adiabatic process
V0 2V 0 V
P1–g Tg = constant
æ -g ö æ g ö
Work = + Area of AOD and – Area of BCO ç 1-g ÷ ç g -1 ÷
P µ Tè ø
µ Tè ø

1 1 as per question P2 µ TC or P µ TC/2


= P0 V0 - P0 V0 = zero on comparing
2 2
C g 5/3 5
= = = ÞC=5
f P V - P1 V1 2 g -1 5 / 3 -1 2
75. DU = µCVDT = (P V – P1V1) = 2 2
2 2 2 g -1 85. TV g -1 = constant Þ (300 K) (V)5/3–1 = Tf(8V)5/3–1

76. Change in internal energy is same and 300 K


Þ Tf = = 75 K
WA > WB So, QA > QB (8)2 / 3
77. dQ = µCpdT {P = constant} 86. PVg = constant
g
1æ5 ö 5 æ Vi ö
Q = ç R ÷ (T2 - T1 ) = K B N A (T2 - T1 ) P = P0 ç ÷ = P0 (27)4 / 3 = 81P0
1
4è2 ø 8 è Vf ø

78. Q1 = DU1 + W1 87. PVg = P1 (V/2)g


P1 = P(2)g = 21.4 P
Q1 = 16 – 20 = –4 kJ
Q
88. Q = W + DU[ W = ]
Q2 = DU2 + W2 4
Q
W2 = Q2 – DU2 (Q DU1 = DU2 ) Q = + DU C = Molar specific heat
4
= 9 – (–4) = 13 kJ 3Q 3 3
DU = Þ µCvDT = µCDT Þ Cv = C
79. AB ® Isothermal ® P­ses, V¯ses Þ dU = 0, 4 4 4
dW = (–)
4 4æ f ö
BC ® Isochoric ® T­ses Þ dU = (+), dW = 0 C = C v = ç R ÷ [For diatomic gas f = 5]
3 3è2 ø
CD ® Isothermal ® P¯ses, V¯ses Þ dU = 0,
dW = (+) 4 5 10
C= ´ R= R
DA ® Isochoric ® T¯ses Þ dU = (–), dW = 0 3 2 3

294 E
Pre-Medical : Physics
89. At constant temperature 93. Process (1) ® volume constant ® Isochroic
PV = constant Process (2) ® adiabatic
On differentiation Process (3) ® Temperature constant ® Isothermal
VdP + PdV = 0 Process (4) ® Pressure constant ® Isobaric
b 94. Slopeadiabatic µ g
dP P dV V
=- Þ =- Slope of 1 < Slope of 2
dV V dP P
g1 < g2 and gmono > gdia
æ dV ö
-ç ÷
è dP ø = 1 So, 2 is monoatomic & 1 is diatomic
b= P
V P
1
1 1 95. B ® Adiabatic ® P µ V ­ P ¯ (more steep)
b= Hence b µ Vg
P P
No heat exchange during AD process
Q mC P DT
90. = (as P is constant)
W mRDT
1
A ® Isothermal ® P µ V ­ P ¯ (less steep)
V
CP CP C / CV g
= = = P =
R CP - C V C P / C V g - 1 (C,D) ® positive slope

91. Process A ® B 96.


P = constant, V¯, W = –ve Þ work done on the gas
and T µ V ; T¯, DU = –ve Þ change in internal energy ¯
Q = DU + W = –ve ie. heat is lost
Process B ® C, V = constant ; W=0
and P µ T ; P¯, T¯ ; change in internal energy ¯
Q = DU
Q = –ve ® Heat loss P1 > P2 , final pressure in vessel (1) is more than
Process C ® D, P = constant vessel (2)
V­, WD = +ve
W1 > W2 W µ Area
and V µT; DT­, DU = change in internal energy ­
Q = DU + W
so, In first vessel, both pressure and work done
Q = +ve ® Heat gained
are more.
Process D ® A
T = constant DU = 0 P
V¯, W = –ve ; WD on the gas
adiabatic
Q = W + DU
Q = –ve ® Heat loss 97.
isothermal
92. I ® Isothermal expansion from V ® 3V (T = constant)
II ® compressed at constant pressure from O
V0 2V0 V
3V ® V
Wext = negative of area with volume-axis
P W(adiabatic) > W(isothermal)
A
98. Free expansion as Pext = 0 Þ W = 0

B 99. Heat cannot be converted in 100% work, but


reverse is true.
V
V 3V
E 295
Pre-Medical : Physics
T2 W 106. h1 = 50 % r T2 = 500 K
100. h = 1 - =
T1 Q
h2 = 60 % r T'2 = ?
300 W
1- =
900 3 ´ 106
æ T ö
2 W h = ç 1 - 2 ÷ ´ 100%
= è T1 ø
3 3 ´ 106
W = 2 × 106 Cal = 2 × 4.2 × 106 J = 8.4 × 106 J
æ Q2 ö 50 500
101. W = Q1 – Q2 = Q1 ç 1 - Q ÷ Þ =1-
è 1 ø 100 T1
æ T ö Q
Þ 800 J = Q1 ç 1 - 2 ÷ = 1 Þ Q1 = 1600 J
è T1 ø 2 Þ T1 = 1000 K ......(i)

æ T1 - T2 ö
for h=60%
102. h = ç T ÷ ´ 100
è 1 ø
60 T'
where T1 = 373, T2 = 273 = 1 - 2 Þ T2' = 400 K
100 1000
100
Þ h= ´ 100 = 26.8%
373 107. 300 K
Q1
T2 W
103. 0.4 = 1 – T ...(1)
1
Q2
æ 50 ö T2 270 K
and 0.4 ç 1 + ÷ =1- ...(2)
è 100 ø T1 + DT
æ Q1 ö
from (1) & (2) W = Q1 – Q2 or W = Q2 ç - 1÷
è Q2 ø
T2
Þ T1 = = 500K
0.6 æT ö æ 300 ö
or W = 360 ç 1 - 1 ÷ = 360 ç - 1 ÷ = 40 J
T
è 2 ø è 270 ø
T2
and T1 + DT = = 750K Þ DT = 250 K Power = W per second = 40 W
0.4
TC
104. W = Q1 – Q2 108. COP =
TH – TC
æ Q2 ö æ T2 ö
Þ W = Q1 ç1 - Q ÷ = Q1 ç 1 - T ÷ T2 270
è 1 ø è 1 ø 109. C.O.P = T - T = 30 = 9
1 2

æ 300 ö Heat transfer per second 180


or W = 3000 ç 1 - 900 ÷ Kcal Wattage= =
è ø C.O.P 9
or W = 2000 × 103 × 4.2 J = 20 watt
or W = 8.4 × 106 J 110. Coefficient of performance of refrigerator
TL
COP =
T1 1 T1 TH – TL
105. h = 1 - T = 2 Þ T2 = 2 Where TL ® lower Temperature
2
& TH ® Higher Temperature
TL
T2 - 100 2 So, 5 =
h1 = 1 - = TH – TL
T1 3
6 6
Þ 3(T2 – 100) = 2T1 Þ T1 = 600 K Þ TH = TL = (253) = 303.6 K
5 5

296 E
Pre-Medical : Physics
Q2 T2 (T - 100) T - 0 T - 50
111. b = W = T – T (Where Q2 is heat removed) 120. + + =0
1 2
L L L
3KA KA 2KA
300 ´ 4.2 277 Þ 3(T – 100) + T + 2 (T – 50) = 0
Þ =
W 303 – 277
400 200
Þ W = 118.25 joule Þ T= = °C
6 3
W 118.25 joule
Þ Power = = = 118.25 watt. 121. (9K)(A)(100 - f) K(A)( f - 20)
t 1sec =
18 6
112. Heat delivered = Q1 900 – 9f = 3(f – 20)
900 + 60
Q2 Q1 - W Q1 T2 Þ f= = 80°C
COP(b) = W = W = W - 1 = T - T 12
1 2

122.
Q1 t2 + 273 t1 + 273 2R R
Þ W =1+ t - t = t - t
1 2 1 2

Q2 1 - h K1 A1 + K 2 A 2
113. b = = Ke = (For parallel combination)
W h A1 + A 2

Þ
Q2 1 - 0.05
= = 19 ( )
K1 pR 2 + K 2 é p ( 2R ) - pR 2 ù
ë
2

û
20 0.05 =
4pR 2
Þ Q2 = 19 × 20 = 380 J
114. A®B ; dW = 0 ; K1 + 3K 2
Ke =
3 3 4
Q = DU = (2p0v0 – p0v0) = p0v0
2 2 L eq L1 L2
B®C ; W = 2p0v0; 123. K A = K A + K A
eq 1 2
3
DU = (4p0v0 – 2p0v0) = 3p0v0 ; Q = 5p0v0 d1 + d2 d d k1k2 (d1 + d2 )
2 = 1 + 2
C®D ; W = 0; DU = –3p0v0 ; Q = –3p0v0 K eq A k1 A k2 A Þ Keq = d k + d k
1 2 2 1
3 5
D®A ; W = –p0v0; DU = – p0v0 ; Q = – p0v0 L L L 5L
2 2 124. R i = + + =
(200)A (400)A (200A) (400)A
so, Wcycle = p0v0 and
Now they are in parallel
heat from source = (Q)+ = 6.5 p0v0
1 L
Wcycle p0 v 0 Rf = =
(200)A (400)A (200)A (800) A
115. h = ´ 100 = ´ 100 Þ h = 15% + +
Q+ 6.5p0 v 0 L L L
116. Metal has high thermal conductivity. Rf 1 Hf
Þ = so, = 10 Þ H f = 400W
L L L L K 4 R i 10 Hi
117. As R = Þ 1 = 2 r 1 = 1 =
KA K1 A K 2 A L 2 K2 9
1 1 1
l l 125. In parallel R = R + R
118. Heat resistance R = ÞRµ eq 1 2
kA A
R is minimum for option IV
K eq (2A) K1 A K 2 A
So conduction of heat will be more. = +
DQ l l l
119. = same
Dt K1 + K 2
K eq =
KA ( 20 - 10) 2KA (10 - q) 2
So =
l l
Þ 2q =10 Þ q = 5 °C

E 297
Pre-Medical : Physics
K eq. A ( q1 - q2 ) t 135. Good absorber is a good emitter, according to
126. Q = Kirchhoff's law.
L
For fig. (a) ; 1
136. l m µ (wein's law)
T
Keq = HM of K1 & K2
137. E = s ( 400) ´ 8 ´ 4
4
Keq = K
KA + KA E ' s ( 800) 4 ´ 4 ´ 2
for fig. (b) ; K eq. = =K
A+A
4
(AM of K1 & K2) E æ 1ö
= ´4 Þ E' = 4E
Q Q1 = Q2 E ' çè 2 ÷ø
K(A) K(2A) 138. E µ T4
Þ (q1 - q2 )t1 = (q1 - q2 )t 2 4
2L L æT ö
Given t1 = 4 min. E2 = E1 ç 2 ÷
è T1 ø
q1 – q2 = 100 r t2 = 1 min.
4
127. Convection is due to density difference æ 1000 ö
E2 = 7 ç ÷ = 112units
128. Convection heat the air molecules which move è 500 ø
upward, while high density molecule comes down 139. P µ r2 T4
for low density (pressure) 2 4
P1 æ r1 ö æ T1 ö
4 4 Þ =
P2 çè r2 ÷ø çè T2 ÷ø
129. P = sAT Þ P µ T
l m1 T1 = l m 2 T2
P2 = 1800 watt
3 140. R1 µ (600)4 – (200)4
Þ l 0 T = l 0 T2
5
R2 µ (400)4 – (200)4
5
Þ T2 = T R 2 (16 + 4 )(16 - 4 ) 20 ´ 12
3 =
4 R1 ( 36 + 4 )(36 - 4 ) = 40 ´ 32
æ 5 ö 625
\ P2 = C ´ ç T ÷ = ´P 3
è3 ø 81 R2 = R
16
625
\n = 141. R H µ Area ( A ) , so same for both
81
130. l ­ T ¯
dq 1 R F1 r2 r2 1 ´ 2
wavelength of blue colour is lesser than red so the 142. R F = µ Þ R = r r = 2 ´1 = 1:1
temperature of blue star is more. dt rr F2 1 1

1 b nm
131. l m µ Þ lm = 143. Newton's law of cooling is used to calculate the
T T B
specific heat of liquid .
c
lm = 144. In accordance with newton's law of cooling.
nm
nm µ temperature dq
T 145. We know = = -k (q - q0 )
4 4
dt
A1 E1 æ T1 ö æ lm2 ö
132. = ç ÷ =ç ÷ after integration
A2 E2 è T2 ø è lm1 ø loge(q – q0) = –kt + c
Spectural energy distribution curver for (IBB) 146. According to newton's law of cooling
dq
133. l m1 T1 = l m2 T2 µ (qavg - q0 )
dt
2597 K × 12000Å = 4800Å × T2
80 - 60 æ 80 + 60 ö
T2 = 6492.5 K = 6219.5 °C =kç - 30 ÷
10 min è 2 ø
134. According to wein's law T¯ ,lm­
60 - T æ 60 + T ö
= kç - 30 ÷
so l Re d > l Blue Þ TRe d < TBlue 10 min è 2 ø
60 - T T
Tb > Ta Þ = Þ T = 48°C
20 80
298 E
Pre-Medical : Physics

90 - 70 é 90 + 70 ù
147. = kê - 20 ú
5 ë 2 û

60 - 30 é 60 + 30 ù
= kê - 20 ú
t ë 2 û

20t 60
= Þ t = 18 min
5 ´ 30 25
148. Newton's laws of cooling

T1 - T2 æ T + T2 ö
=kç 1 - T÷
t è 2 ø

3T - 2T æ 5T - 2T ö T æ 3T ö
= kç ÷ Þ 10 = k ç 2 ÷ ...(i)
10 è 2 ø è ø

2T - T ' æ 2T + T ' ö 2T - T ' = k æ T ' ö ...(ii)


= kç - T÷Þ ç2÷
10 è 2 ø 10 è ø

3
By solving (i) and (ii) T ' = T
2

s.4pr 2 T 4 sr 2 T 4
149. =
4pR 2 R2
150. Kirchhoff's law :-
el
= El
al

E 299
Pre-Medical : Physics
OSCILLATIONS (SHM)

1. Force constant t
K = mw 2
P t=0
8.
K = m(2pn)2 –A –A/2 O A
K = m4p n 2 2
Total length in one cycle = 4A
2
d x ækö
2. = -ç ÷ x 5 5A
dt 2
èmø So, × (4A) =
8 2
3. a + 16 p2x = 0
Þ Particle travels from 0 to P Þ Df = 210°
2 p 2p
Þ w2 = 16 p2 Þ T = = = 0.5 s T 7T
w 4p t = × 210° =
360° 12
4. S.H.M. is periodic function of unique frequency and 9. For first particle x = A/2 Þ q1 = 30°
finite amplitude.
For second particle
Tæp ö x = –A/2 Þ q2 = 2p – 30°
5. t=0 t = ç Angle ÷
4è2 ø
Df = q2 – q1 = 300 ° or 60°
p
phase will develop when particle move from (as phase difference of f or 2p – f is same)
6
mean position towards amplitude position. It mean 1 1
particle move in + × direction 10. f= Þ f = = 25 Hz
Time period 0.04
x = a sin wt
11. vmax = wA = 0.4
x = a sin q Amax = A = 10 cm
p a So, w = 4 rad/sec
x = asin = asin30° =
6 2
6. Time required between mean to half of amplitude 2p p
and T = = second
w 2
æ aö T
çè ÷ø position in t1 is = 12. Vmax = Aw
2 12
æ aö T 13. In one complete oscillation the displacement is zero,
Time required between çè ÷ø to a position is t2 =
2 6 so average velocity is also zero.
T
t1 12 1 14. x = Asin(wt+f)
= = = 1:2
t2 T 2
Q At t = 1 second x=0
6
7. x = Asinwt 0 = Asin(w×1+f) So f = –w

2p Now V = Awcos(wt+f)
If t = 1 second then x = A sin ´1
T
1
2p A At t = 2 second V= m/s
4
or x I = A sin =
8 2 1
So = A × w cos (w × 2 – w)
but in two second it will reach to extreme position, 4
so distance travelled in next second 1
= Awcosw
4
A ( 2 - 1)A
x II = A - =
2 2 by putting values

A/ 2 1 1 2p 2p 3
= = A´ cos So A = m
ratio of distance 4 6 6 2p
( 2 - 1)A / 2 2 -1
300 E
Pre-Medical : Physics
15. y1 = 0.1sin(100pt + p/3) 23. Amplitude A = 3 cm
æ pö When particle is at x = 2 cm ,
Þ V1 = 0.1 × 100p cos ç 100pt + ÷ its |velocity| = |acceleration|
è 3ø
and y2 = 0.1cos100pt
A 2 - x2
Þ V2 = –0.1 × 100 p sin100pt i.e., w A 2 - x 2 = w2x Þ w =
x
or V2 = 10 p sin (100pt + p)
2p æ 3 ö 3p
é p pù T= = 2p ç ÷ =
Now f12 = f1 – f2 = ê100pt + + ú - éë100pt + p ùû w è4ø 2
ë 3 2û
1
p 24. PE = (TE)
= – rad. 4
6
16. Amplitude = same
1 2 1 æ 1 2ö A
Df = const Þ T = same kx = ç kA ÷ Þ x = ±
2 4è2 ø 2
So, vmax = wA = same
17. r r 1 2
|a|= w2 |x| Þ 20 = w2(5) 25. Umax = kx
2
So, w = 2 rad/s.
18. x = 1 cm a = 1 cm/s2 1 = 1 k(0.2) 2 Þ k = 50
2
V = 1 cm/s
m 4 2p 2
a = w2x Þ w2 =1 Þ T = 2p and T = 2p Þ T = 2p = sec
k 50 5
19. a = –w2x ,
So graph is straight line with negative slope dU -d
26. F=- = (8x 2 ) Þ -16x
20. x = 5.0 cos(2pt + p) = –5.0 cos2pt dx dx

dx (F) = –16(–2) = 32 Dyne.


Velocity = = 10p sin2pt
dt
1
27. K max = KA 2
2
d2 x
Acceleration = = 20p2 cos 2pt 1 1
dt2 (KE)P = K max = K(A2 - x 2 )
2 2
\ At t = 1.5 sec,
A
x = –5.0 cos3p = –5.0 (–1) = 5 Þ x = or q = 45°
2
dx
Velocity = = 10p(zero) = 0 T
dt So, time = × 45° = T/8
360°
28. T.E. = mgh + (K.E.)max
d2 x
Acceleration = = 20p2 ( -1) = -20p2 9 = 5 + (K.E.)max
dt 2
(K.E.)max = 4 J This is KE at mean position
21. amax = –w2A
1
a max w2 (500)2 1 29. KE = KA2 cos2 wt
So = 2
= = 2
a 'max (w ') (5000)2 100
22. a = w2A 1
Q PEmax = TE = KA2 = K0
2
a a 2p 2pb
b = wA \ =wÞ = ÞT=
b b T a
1
30. <E> = <V> = KA2
4

E 301
Pre-Medical : Physics
31. Energy at rest position is 3J this is potential energy
due to height, mgh = 3J. So N l long = (N + 1) l short

Mean K.E. = 4 J Þ N 1.21 = (N + 1) 1


So Maximum K.E. = 2(Mean K.E.) = 8J Þ 1.1 N = N + 1 Þ N = 10
T.E. = mgh + (K.E.)max = 3 + 8 = 11 J r
38. | a | = w2x

1 R 84.6 Þ 16 = w2 4 Þ w2 = 4
32. T = 2p Þ T = 2p = min
æ 1 1ö 2g 2 2p
gç + ÷ =2
è R Rø Þ w=2Þ Þ T=ps
T
or T = 59.8 min
1 K/2
39. f1 = {Keq = K/2}
l 2p m
33. T ' = 2p
geff
1 2K / 3 ì 2K × K 2K ü
f2 = íK eq = = ý
l 3 2p m î 2K + K 3 þ
T ' = 2p = T
g + g/3 2
f1 3
34. A = a Þ x = a/2 Þ q = 30° =
f2 2
–a O a/2 a
k1 k2
40. Vmax1 = × A1 Þ Vmax2 = A2
p 3a m m
So, v = vmax cos30° =
T
A1 k2
35. mg Since Vmax1 = Vmax2 Þ =
sin A2 k1
a
gs
O ina m
1
41. t1 = 2p
k1
mg
sin
mgcosa a
a
m
t 2 = 2p
k2
When pendulum is observed from vehicle frame
m
t = 2p {k eq = k1 + k 2 }
L k1 + k 2
geff = gcosa Þ T = 2p
g cos a
t1 t2
T' 1.44 Solving t = t12 + t 22
36. Tµ l Þ =
T 1
42. In series combination
Þ T' = 1.2T Þ T' = 72 sec

1.21 1 1 1 1 K 1K 2
37. T1 = 2p Þ T2 = 2p = + Þ KS =
g g K S K1 K 2 K1 + K 2
Þ T1 = 1.1 T2 or 10T1 = 11 T2
This means 10 oscillation of larger pendulum will 43. Both springs are connected in parallel
take same time as 11 oscillation of samller
pendulum and after that they will be in plane. so Keq = 2k + k = 3k
OR
1 3k
They will be in same phase when long pendulum frequency n =
2p m
completes N and short pendulum completes (N +1)
oscillation.

302 E
Pre-Medical : Physics
44. Effective value of g is zero in artificial satellite. 49. amax = w2A = g {when N = 0}

g 10
m Þ A = 2 = = 1m
45. T = 2p w p2
k
and Vmax = wA = 3.14 m/s
m 50. ­y = Kt2
3 = 2p ...(1)
k
d2 y
ay = = 2K = 2 m/s2 ay
m+2 dt 2
5 = 2p ...(2)
k So, geff = g + ay = 12 m/s2
(1)2 9 m 18 9 l l
Þ = Þ m= = T1 = 2p = 2p
(2) 2
25 m + 2 16 8 g 10
46. In 20 seconds the amplitude becomes 1/3
l l T2 12 6
1 1 1 T2 = 2p = 2p So, 12 = =
So in 40 seconds the amplitude = ´ = of geff 12 T2 10 5
3 3 9
initial.

47. Amplitude of vibrations remain constant in case of


free, maintained & forced vibrations.

Re
48. Tst = 2p = 84.6 min
g

Re
Tma = 2p = 84.6 min
g

Re Re
TSP = 2p g = 2p = 84.6 min
eff g

Re
Tis = 2p = 84.6 min
g

E 303
Pre-Medical : Physics
WAVE MOTION & DOPPLER'S EFFECT

é æ t x ö pù d d
5. y = 4 sin ê p ç - ÷ + ú compare this equation 11. - = 4 ´ 60
ë è 5 9 ø 6û vP v S

by
d d
y = a sin (wt–kx) Þ - = 240
4.5 8
p p
w= k= d = 2468.6 km » 2500 k
5 9
p
T l 12. y = 3sin (50 t – x)
a=4 =5 =9 2
2 2

T = 10 s, l = 18 cm ¶y p p
Particle velocity = = 3 æç ´ 50ö÷ cos (50t–x)
7. Wave is a disturbance, which carries energy and
¶t è2 ø 2
momentum without transport of mass of matter. Maximum particle velocity = 75 p m/s

600 w 50
8. Velocity of wave = = 300 m/s Wave velocity v = = = 50 m/s
2 k 1

3 75p 3
Frequency = 500 Hz, Wavelength l = m Required ratio = = p
5 50 2

OR
600
Number of wavelength = = 1000 (v P )max. Aw p 3p
35
= = kA = ´ 3 =
Vmax. w/ k 2 2
9. w = 2pn
wµn (2p = const) 2p 2p
13. k= = = 7.85
l 4/5
1 1 w
wµ n=
l l
w
v= Þ w = vk = 128 ´ 7.85 = 1005
Þ wµn
n k

and moves in + x direction


10.
v1 r2 1 1
B 14. = = =
A v2 r1 4 2
A B
15. frequency does not depends upon medium,
velocity of sound decreases when it goes from wa-
ter to air. v = fl (Q f = const.)

from figure A is moving down & B is moving up and so l also decreases


the phase at A is greater the phase at B. 16. Velocity of sound

gP
v= as rd > rm \ um > ud
r

304 E
Pre-Medical : Physics
24. A(256 Hz)
æ 1 ö 4beats 4beats
T w T ç ÷ T
17. v= Þ = Þ è 0.04 ø =
m K m æ 1 ö 0.04 (B) 252 (B) 260
ç ÷
è 0.50 ø On waxing On waxing

250 Hz 250 Hz
0.50 0.50 T 50
Þ ´ = \ T= × 0.5 = 6.25 N
0.04 0.04 0.04 4
By removing By removing
same amount litle amount
T 0.1
18. v= , T = 0.1 × 10 = 1N, m = 252 Hz 252 Hz
m 2.5

Velocity at upper point v = \ frequency of B = 260 Hz


1 ´ 25
v = 5 m/s
25.
Now velocity at 0.5 m distance from lower point -

T 1 1 1
v= Þ T= 2.5 × 0.5 = 5 N, m = 25
m
After loading wax, frequency of A decrease. So beats
1 25 frequency will also decrease.
v= ´ = 5 = 2.24 m/s
5 1 Ans. would be 292
19. Let distance between cliff and mountain be d
508 p 500p
d d 26. Number of beats = -
1= + Þ d = 170m 2p 2p
340 340
= 254 – 200 = 4
d
340
distance = = 170m
v=340 m/s 2 Imax æ a1 + a 2 ö æ4+5ö
2
81
and =ç ÷= ç4-5÷ = 1
Imin è a1 - a 2 ø è ø
1
20. I= rva2 w2 Þ I µ n2a2
2 27. Number of beats D n = n1 – n2

499 + 501 v v
21. Average frequency = = 500Hz v v
2 2= - Þ 2 = -
l1 l 2 2 2.02
Beat frequency = 501 – 499 = 2Hz
22. n, n + 8 ............ n + 120 v
2 = Þ v = 404
202
n + 120 = 2n , n = 120

23. B A B 28. For closed pipe

v
n= = 512
4l
For open pipe
252 256 260
beats beats v
n= = 1024
increases decreases 2l

So answer is 252, and 260 is also when more wax n = 1024 Hz.
is applied on B.

E 305
Pre-Medical : Physics
35. 425 : 595 : 765
v
29. n= 5 : 7 : 9
4l
n = 264 Hz ratio of frequency is odd

v = 330 m/s pipe ® closed end.


Then 425 is frequency of 2nd overtone

l = 31.25 m 5v 5 ´ 340
= 425 Þ = 425 , l = 1m
4l 4l
v
30. n=
4l v 330
36. l= = = 1m = 100 cm
By increasing temperature, n 330
velocity of sound & length both for first resonance length of air column
increases, so L
l
l= = 25 cm
V+v 4
n=
4 (L + l)
3l
second resonance l 2 = = 75 cm
4
v
31. = fundamental frequency = 1500 Hz
4l 5l
third resonance l 3 = = 125 cm which is not
Normal person can listen frequency 20,000 Hz. 4

1500x = 20,000 , x = 13 (No. of Harmonic) possible


minimum length of water column
Then overtones and frequency in case of closed
organ pipe, ratio of frequency = 120 – 75 = 45 cm
37. For second overtone (3rd harmonic) in open organ
1, 3, 5, 7, 9, 11, 13
pipe,
I II III IV V VI
Then 6 overtones. 3l 2l 0
= l0 Þ l=
2 3
v v
32. n1 = , n2 = for first overtone (3rd harmonic) in closed organ pipe,
4l1 4l 2
3l 4l C 4L
l1 = 0.75 m l2 = 0.770 m = lC Þ l= =
4 3 3

væ 1 1 ö
n1–n2=3 Þ - = 3 Þ v =346.5 m/s 2l 0 4L
4 çè l1 l 2 ÷ø So, = Þ l 0 = 2L
3 3

v 38. Difference between any two consecutive frequencies


33. = 260, l = 31.7 cm
4l 2v
of COP = = 500 - 460 = 40 Hz
34. For open organ pipe 4l

v l/2 v
n1 = l Þ = 20Hz
2l 4l

So fundamental frequency = 20 Hz
v 2v v
for COP n2 = = Þ n2 = , n1 = n2 39. V = 2f(l2 – l1)
4(l 2) 4l 2l
= 2(320) × (0.93 – 0.40)
= (640) × (0.53) = 339.2 m/s

306 E
Pre-Medical : Physics
47. All harmonic frequencies are obtained for string
40. First minimum resonating length for closed organ
fixed at both ends.
l \ r0 = 420 Hz – 315 Hz = 105 Hz
pipe = = 40 cm
4
1 T
3l 48. n=
= 120 cm 2L m
\ Next larger length of air column =
4
nµ T
l
41. n = 100 Hz., Given = 10 cm
2 n1 T
=
n2 1.02T

n T 1
N = =
N n + 15 1.02T 1.02

n = 1500 Hz
v = n l = 100 × 0.2 v = 20 m/ s
42. Plucking distance Dv 1 DT Dv
and v µ T Þ = Þ ´ 100 = 1%
v 2 T v
l l
x= = so (2) is incorrect
2p 4 N
49. y = cos 2pt sin 2px, k = 2p
touching distance = l/2
43. The frequency of the piano string may be 508 or l = 1m
516 Hz. So, minimum length of string = l/2 = 0.5 m

As frequency µ Tension so answer will be


508 Hz.

Dn 1 DT DT æ Dn ö
44. nµ T Þ n = 2 T Þ T = 2 çè n ÷ø
50. y = a cos (wt – kx)

æ 6 ö x = 0 is a node
=2 çè = 0.02
600 ÷ø So rigid end reflection
45. Let unknown frequancy be n y = a cos (wt + kx + p)
Case-I n can be 250 – 4, 250 + 4 y = –a cos (wt + kx)
[cos (180 +q) = –cosq]
246 Hz or 254 Hz
51.
Case-II 2n can be 513 – 5, 513 + 5

508 518
\ n can be , 0.25m
2 2

i.e. 254 , 259 p T 2 20 ´ 103


n= = = 200 Hz
2l m 2l 0.5
254 Hz satisfy both condition.
46. Total length of string l = l1 + l2 + l3
n1 l 2 T1 r22 d2
52. = ´ ´
1 n2 l 1 T2 r12 d1
But frequency µ
length
n1 35 35 ´ n2 35 ´ 360
n2 36 Þ 1
= n = = = 350
1 1 1 1 36 36
so = + +
n n1 n2 n3
beats frequency = n2 – n1 = 10 Hz

E 307
Pre-Medical : Physics
59. w = 20 rad/s ,v = wr = 20 × 0.50= 10 m/s
5 9 3 M
53. v= = ´
4l m 4l m
æ 340 ö
Minimum frequency = 385 × ç ÷
è 340 + 10 ø
M =5

Hence M =5 340
= 385 × = 374 Hz
350
54. When sounded with 440 Hz,
v v
Frequency of guitar = 440 + 5, 440 – 5 60. S O
When sounded with 436 Hz,
There is no relative motion between source and
Frequency of guitar = 436 + 9, 436 – 9 observer so no doppler effect.
so frequency of guitar = 445 Hz
61. n = 165 Hz, vs = vo= 5 m/s, v = 335 m/s

w 240p
55. v= = = 300 ´ 10-2 = 3 æ v + vo ö æ 340 ö
K é 4p ù n' = n ç v - v ÷ Þ n' = 165 × ç ÷ Hz
êë 5 ´ 10-2 úû è s ø è 330 ø

n' = 170 Hz then number of beats


-3
T 4 ´ 10 (170 – 165) = 5
v= Q m= = 4 ´ 10 -1
m 10-2
OR

T 2v 0
3= Þ 9 ´ 0.4 = T Dn = n
0.4 v - v0

T = 3.6 N
2nv
56. T1 = m2g T2 62. b = Dn =
Vsound
T2 = (m1 + m2)g m1
L
Velocity µ T
T1 Observer
m2 63. Source
Þlµ T Wall
15m/s

l1 T1 l2 m1 + m2 3
Þ l = Þ = =
2 T2 l1 m2 2 frequency at wall is n¢

æ 340-15 ö v
n¢ = n0
57. 1950 = n × ç ÷ v - vs
è 340+15 ø

1950 ´ 355 330 330 ´ 900


n= = 2130 Hz n¢ = (900) = = 990 Hz
325 330 – 30 300

Since the observer and wall are stationary so


v 340
58. l= = =m frequency of echo heard by observer will also be 838
n 170
Hz.
l = 2m
v Dl 0.4
\ Distance between 2 portions of minimum 64. Dl = l Þ l = 100
c
intensity

l 0.4 v
= =1m \ = \ v = 1.2 × 106 m/s
2 100 3 ´ 108

308 E
Pre-Medical : Physics
65. As speed of light is independent of relative motioin
b/w source and observer.

æ vö
æ v + v0 ö çv+5÷
67. n' = nç ÷ = fç ÷ Þ n' = 1.2 f
è v ø è v ø

Only observer move. There will be no change in


the l.

v v
68. Dl = l 0 Þ 1nm = 600nm
c c

Þ v = 5 × 105 m/s

E 309

You might also like